Sei sulla pagina 1di 688

Cardiac Electrophysiology

Andrea Natale Amin Al-Ahmad


Paul J. Wang John P. DiMarco
(Editors)

Cardiac
Electrophysiology
Clinical Case Review
Editors
Andrea Natale Paul J. Wang
Texas Cardiac Arrhythmia Institute School of Medicine
St. Davids Medical Center Stanford University
1015 East 32nd Street 300 Pasteur Drive
Suite 516, H-2146,
Austin, TX 78705 Stanford, CA 94305
USA USA

Amin Al-Ahmad John P. DiMarco


School of Medicine Cardiovascular Division
Stanford University University of Virginia Health System
300 Pasteur Drive 1215 Lee Street
H-2146, Charlottesville, VA 22908
Stanford, CA 94305 USA
USA

ISBN 978-1-84996-389-3 e-ISBN 978-1-84996-390-9


DOI 10.1007/978-1-84996-390-9
Springer London Dordrecht Heidelberg New York

British Library Cataloguing in Publication Data


A catalogue record for this book is available from the British Library

Library of Congress Control Number: 2010937972

Springer-Verlag London Limited 2011

Apart from any fair dealing for the purposes of research or private study, or criticism or review, as permitted under the
Copyright, Designs and Patents Act 1988, this publication may only be reproduced, stored or transmitted, in any form
or by any means, with the prior permission in writing of the publishers, or in the case of reprographic reproduction in
accordance with the terms of licenses issued by the Copyright Licensing Agency. Enquiries concerning reproduction
outside those terms should be sent to the publishers.

The use of registered names, trademarks, etc., in this publication does not imply, even in the absence of a specific
statement, that such names are exempt from the relevant laws and regulations and therefore free for general use.

Product liability: The publisher can give no guarantee for information about drug dosage and application thereof con-
tained in this book. In every individual case the respective user must check its accuracy by consulting other pharma-
ceutical literature.

Cover design: eStudioCalamar, Figueres/Berlin

Printed on acid-free paper

Springer is part of Springer Science+Business Media (www.springer.com)


To our wives and families, for the constant support and patience.

Preface

As the field of cardiac electrophysiology continues to evolve and advance, we continue to be


challenged to educate new generations of cardiac electrophysiologists with the basics as well
as the latest advances in the field.
While there are many outstanding books that provide an in-depth review of topics in elec-
trophysiology, there are few case-based books that comprehensively cover clinical electro-
physiology, devices, and ablation. Case reviews offer a simple, yet effective way in teaching
important concepts, offering insight into both the basic pathophysiology of a problem as well
as the clinical reasoning that leads to a solution.
In Cardiac Electrophysiology: Clinical Case Review we sought to put together the most
comprehensive case-based review of electrophysiology that would appeal to all students of the
field whether they are fellows, allied professionals, or practicing electrophysiologists.
In Cardiac Electrophysiology: Clinical Case Review we are fortunate to have many true
experts in the field contributing cases that they have encountered and summarizing the impor-
tant learning objectives in a succinct way. The book covers clinical electrophysiology, device
troubleshooting and analysis, as well as intracardiac electrogram analysis and ablation.
It is our sincere hope that the readers of Cardiac Electrophysiology: Clinical Case Review
will find the cases useful as a review of electrophysiology or in their day-to-day interactions
with patients.

Stanford CA, USA Amin Al-Ahmad


Charlottesville VA, USA Paul J. Wang
Cleveland OH, USA John P. DiMarco

vii

Contents

Section 1 Ablation

Case 1 . . . . . . . . . . . . . . . . . . . . . . . . . . . . . . . . . . . . . . . . . . . . . . . . . . . . . . . . . . . . . . . 3
Amin Al-Ahmad

Case 2 . . . . . . . . . . . . . . . . . . . . . . . . . . . . . . . . . . . . . . . . . . . . . . . . . . . . . . . . . . . . . . . 7
Michel Haissaguerre

Case 3 . . . . . . . . . . . . . . . . . . . . . . . . . . . . . . . . . . . . . . . . . . . . . . . . . . . . . . . . . . . . . . . 11
Amin Al-Ahmad

Case 4 . . . . . . . . . . . . . . . . . . . . . . . . . . . . . . . . . . . . . . . . . . . . . . . . . . . . . . . . . . . . . . . 15
Michel Haissaguerre

Case 5 . . . . . . . . . . . . . . . . . . . . . . . . . . . . . . . . . . . . . . . . . . . . . . . . . . . . . . . . . . . . . . . 19
Anurag Gupta and Amin Al-Ahmad

Case 6 . . . . . . . . . . . . . . . . . . . . . . . . . . . . . . . . . . . . . . . . . . . . . . . . . . . . . . . . . . . . . . . 23
Michel Haissaguerre

Case 7 . . . . . . . . . . . . . . . . . . . . . . . . . . . . . . . . . . . . . . . . . . . . . . . . . . . . . . . . . . . . . . . 29
Michel Haissaguerre

Case 8 . . . . . . . . . . . . . . . . . . . . . . . . . . . . . . . . . . . . . . . . . . . . . . . . . . . . . . . . . . . . . . . 35
Michel Haissaguerre

Case 9 . . . . . . . . . . . . . . . . . . . . . . . . . . . . . . . . . . . . . . . . . . . . . . . . . . . . . . . . . . . . . . . 39
Alberto Diaz, Dimpi Patel, William R. Lewis, and Andrea Natale

Case 10 . . . . . . . . . . . . . . . . . . . . . . . . . . . . . . . . . . . . . . . . . . . . . . . . . . . . . . . . . . . . . . 41
Antonio Rossillo, Sakis Themistoclakis, Aldo Bonso, Andrea Corrado,
andAntonio Raviele

Case 11 . . . . . . . . . . . . . . . . . . . . . . . . . . . . . . . . . . . . . . . . . . . . . . . . . . . . . . . . . . . . . . 47
Yash Y. Lokhandwala, Anoop K. Gupta, and Ranjan K. Thakur

ix
x Contents

Case 12 . . . . . . . . . . . . . . . . . . . . . . . . . . . . . . . . . . . . . . . . . . . . . . . . . . . . . . . . . . . . . . 51
Eric Buch, Shiro Nakahara, Marmar Vaseghi, Noel G. Boyle,
and Kalyanam Shivkumar

Case 13 . . . . . . . . . . . . . . . . . . . . . . . . . . . . . . . . . . . . . . . . . . . . . . . . . . . . . . . . . . . . . . 53
Bradley P. Knight

Case 14 . . . . . . . . . . . . . . . . . . . . . . . . . . . . . . . . . . . . . . . . . . . . . . . . . . . . . . . . . . . . . . 57
M. Eyman Mortada, Jasbir S. Sra, and Masood Akhtar

Case 15 . . . . . . . . . . . . . . . . . . . . . . . . . . . . . . . . . . . . . . . . . . . . . . . . . . . . . . . . . . . . . . 63
Eric Buch, Shiro Nakahara, Marmar Vaseghi, Noel G. Boyle,
and Kalyanam Shivkumar

Case 16 . . . . . . . . . . . . . . . . . . . . . . . . . . . . . . . . . . . . . . . . . . . . . . . . . . . . . . . . . . . . . . 65
Antonio Rossillo, Sakis Themistoclakis, Aldo Bonso, Andrea Corrado,
and Antonio Raviele

Case 17 . . . . . . . . . . . . . . . . . . . . . . . . . . . . . . . . . . . . . . . . . . . . . . . . . . . . . . . . . . . . . . 69
Bradley P. Knight

Case 18 . . . . . . . . . . . . . . . . . . . . . . . . . . . . . . . . . . . . . . . . . . . . . . . . . . . . . . . . . . . . . . 71
Antonio Rossillo, Sakis Themistoclakis, Aldo Bonso, Andrea Corrado,
and Antonio Raviele

Case 19 . . . . . . . . . . . . . . . . . . . . . . . . . . . . . . . . . . . . . . . . . . . . . . . . . . . . . . . . . . . . . . 81
Yash Y. Lokhandwala, Anoop K. Gupta, and Ranjan K. Thakur

Case 20 . . . . . . . . . . . . . . . . . . . . . . . . . . . . . . . . . . . . . . . . . . . . . . . . . . . . . . . . . . . . . . 83
Marmar Vaseghi, Noel G. Boyle, and Kalyanam Shivkumar

Case 21 . . . . . . . . . . . . . . . . . . . . . . . . . . . . . . . . . . . . . . . . . . . . . . . . . . . . . . . . . . . . . . 85
Bradley P. Knight

Case 22 . . . . . . . . . . . . . . . . . . . . . . . . . . . . . . . . . . . . . . . . . . . . . . . . . . . . . . . . . . . . . . 87
Antonio Rossillo, Sakis Themistoclakis, Aldo Bonso, Andrea Corrado,
and Antonio Raviele

Case 23 . . . . . . . . . . . . . . . . . . . . . . . . . . . . . . . . . . . . . . . . . . . . . . . . . . . . . . . . . . . . . . 95
Yash Y. Lokhandwala, Anoop K. Gupta, and Ranjan K. Thakur

Case 24 . . . . . . . . . . . . . . . . . . . . . . . . . . . . . . . . . . . . . . . . . . . . . . . . . . . . . . . . . . . . . . 101
Marmar Vaseghi, Noel G. Boyle, and Kalyanam Shivkumar

Case 25 . . . . . . . . . . . . . . . . . . . . . . . . . . . . . . . . . . . . . . . . . . . . . . . . . . . . . . . . . . . . . . 103
Bradley P. Knight

Case 26 . . . . . . . . . . . . . . . . . . . . . . . . . . . . . . . . . . . . . . . . . . . . . . . . . . . . . . . . . . . . . . 107
Richard H. Hongo and Andrea Natale
Contents xi

Case 27 . . . . . . . . . . . . . . . . . . . . . . . . . . . . . . . . . . . . . . . . . . . . . . . . . . . . . . . . . . . . . . 111
M. Eyman Mortada, Jasbir S. Sra, and Masood Akhtar

Case 28 . . . . . . . . . . . . . . . . . . . . . . . . . . . . . . . . . . . . . . . . . . . . . . . . . . . . . . . . . . . . . . 115
Yash Y. Lokhandwala, Anoop K. Gupta, and Ranjan K. Thakur

Case 29 . . . . . . . . . . . . . . . . . . . . . . . . . . . . . . . . . . . . . . . . . . . . . . . . . . . . . . . . . . . . . . 123
Eric Buch, Shiro Nakahara, Marmar Vaseghi, Noel G. Boyle,
and Kalyanam Shivkumar

Case 30 . . . . . . . . . . . . . . . . . . . . . . . . . . . . . . . . . . . . . . . . . . . . . . . . . . . . . . . . . . . . . . 125
Luis C. Senz and Miguel A. Vacca

Case 31 . . . . . . . . . . . . . . . . . . . . . . . . . . . . . . . . . . . . . . . . . . . . . . . . . . . . . . . . . . . . . . 133
Bradley P. Knight

Case 32 . . . . . . . . . . . . . . . . . . . . . . . . . . . . . . . . . . . . . . . . . . . . . . . . . . . . . . . . . . . . . . 135
Luigi Di Biase, Rodney P. Horton, and Andrea Natale

Case 33 . . . . . . . . . . . . . . . . . . . . . . . . . . . . . . . . . . . . . . . . . . . . . . . . . . . . . . . . . . . . . . 139
M. Eyman Mortada, Jasbir S. Sra, and Masood Akhtar

Case 34 . . . . . . . . . . . . . . . . . . . . . . . . . . . . . . . . . . . . . . . . . . . . . . . . . . . . . . . . . . . . . . 143
Antonio Rossillo, Sakis Themistoclakis, Aldo Bonso, Andrea Corrado,
and Antonio Raviele

Case 35 . . . . . . . . . . . . . . . . . . . . . . . . . . . . . . . . . . . . . . . . . . . . . . . . . . . . . . . . . . . . . . 147
Yash Y. Lokhandwala, Anoop K. Gupta, and Ranjan K. Thakur

Case 36 . . . . . . . . . . . . . . . . . . . . . . . . . . . . . . . . . . . . . . . . . . . . . . . . . . . . . . . . . . . . . . 151
Eric Buch, Shiro Nakahara, Marmar Vaseghi, Noel G. Boyle,
and Kalyanam Shivkumar

Case 37 . . . . . . . . . . . . . . . . . . . . . . . . . . . . . . . . . . . . . . . . . . . . . . . . . . . . . . . . . . . . . . 153
Bradley P. Knight

Case 38 . . . . . . . . . . . . . . . . . . . . . . . . . . . . . . . . . . . . . . . . . . . . . . . . . . . . . . . . . . . . . . 157
Luigi Di Biase, Rodney P. Horton, and Andrea Natale

Case 39 . . . . . . . . . . . . . . . . . . . . . . . . . . . . . . . . . . . . . . . . . . . . . . . . . . . . . . . . . . . . . . 159
M. Eyman Mortada, Jasbir S. Sra, and Masood Akhtar

Case 40 . . . . . . . . . . . . . . . . . . . . . . . . . . . . . . . . . . . . . . . . . . . . . . . . . . . . . . . . . . . . . . 163
Antonio Rossillo, Sakis Themistoclakis, Aldo Bonso, Andrea Corrado,
and Antonio Raviele

Case 41 . . . . . . . . . . . . . . . . . . . . . . . . . . . . . . . . . . . . . . . . . . . . . . . . . . . . . . . . . . . . . . 181
Yash Y. Lokhandwala, Anoop K. Gupta, and Ranjan K. Thakur
xii Contents

Case 42 . . . . . . . . . . . . . . . . . . . . . . . . . . . . . . . . . . . . . . . . . . . . . . . . . . . . . . . . . . . . . . 185
Eric Buch, Shiro Nakahara, Marmar Vaseghi, Noel G. Boyle,
and Kalyanam Shivkumar

Case 43 . . . . . . . . . . . . . . . . . . . . . . . . . . . . . . . . . . . . . . . . . . . . . . . . . . . . . . . . . . . . . . 189
Bradley P. Knight

Case 44 . . . . . . . . . . . . . . . . . . . . . . . . . . . . . . . . . . . . . . . . . . . . . . . . . . . . . . . . . . . . . . 191
Antonio Rossillo, Sakis Themistoclakis, Aldo Bonso, Andrea Corrado,
and Antonio Raviele

Case 45 . . . . . . . . . . . . . . . . . . . . . . . . . . . . . . . . . . . . . . . . . . . . . . . . . . . . . . . . . . . . . . 195
Yash Y. Lokhandwala, Anoop K. Gupta, and Ranjan K. Thakur

Case 46 . . . . . . . . . . . . . . . . . . . . . . . . . . . . . . . . . . . . . . . . . . . . . . . . . . . . . . . . . . . . . . 199
Eric Buch, Shiro Nakahara, Marmar Vaseghi, Noel G. Boyle,
and Kalyanam Shivkumar

Case 47 . . . . . . . . . . . . . . . . . . . . . . . . . . . . . . . . . . . . . . . . . . . . . . . . . . . . . . . . . . . . . . 201
Bradley P. Knight

Case 48 . . . . . . . . . . . . . . . . . . . . . . . . . . . . . . . . . . . . . . . . . . . . . . . . . . . . . . . . . . . . . . 203
Antonio Rossillo, Sakis Themistoclakis, Aldo Bonso,
Andrea Corrado, and Antonio Raviele

Case 49 . . . . . . . . . . . . . . . . . . . . . . . . . . . . . . . . . . . . . . . . . . . . . . . . . . . . . . . . . . . . . . 217
Eric Buch, Shiro Nakahara, Marmar Vaseghi, Noel G. Boyle,
and Kalyanam Shivkumar

Case 50 . . . . . . . . . . . . . . . . . . . . . . . . . . . . . . . . . . . . . . . . . . . . . . . . . . . . . . . . . . . . . . 221
Bradley P. Knight

Case 51 . . . . . . . . . . . . . . . . . . . . . . . . . . . . . . . . . . . . . . . . . . . . . . . . . . . . . . . . . . . . . . 223
Yash Y. Lokhandwala, Anoop K. Gupta, and Ranjan K. Thakur

Case 52 . . . . . . . . . . . . . . . . . . . . . . . . . . . . . . . . . . . . . . . . . . . . . . . . . . . . . . . . . . . . . . 229
Eric Buch, Shiro Nakahara, Marmar Vaseghi, Noel G. Boyle,
and Kalyanam Shivkumar

Case 53 . . . . . . . . . . . . . . . . . . . . . . . . . . . . . . . . . . . . . . . . . . . . . . . . . . . . . . . . . . . . . . 233
Bradley P. Knight

Case 54 . . . . . . . . . . . . . . . . . . . . . . . . . . . . . . . . . . . . . . . . . . . . . . . . . . . . . . . . . . . . . . 235
Yash Y. Lokhandwala, Anoop K. Gupta, and Ranjan K. Thakur

Case 55 . . . . . . . . . . . . . . . . . . . . . . . . . . . . . . . . . . . . . . . . . . . . . . . . . . . . . . . . . . . . . . 241
Eric Buch, Shiro Nakahara, Marmar Vaseghi, Noel G. Boyle, and Kalyanam Shivkumar

Case 56 . . . . . . . . . . . . . . . . . . . . . . . . . . . . . . . . . . . . . . . . . . . . . . . . . . . . . . . . . . . . . . 245
Bradley P. Knight
Contents xiii

Case 57 . . . . . . . . . . . . . . . . . . . . . . . . . . . . . . . . . . . . . . . . . . . . . . . . . . . . . . . . . . . . . . 247
Yash Y. Lokhandwala, Anoop K. Gupta, and Ranjan K. Thakur

Case 58 . . . . . . . . . . . . . . . . . . . . . . . . . . . . . . . . . . . . . . . . . . . . . . . . . . . . . . . . . . . . . . 253
Roopinder Sandhu, Dimpi Patel, William R. Lewis, and Andrea Natale

Case 59 . . . . . . . . . . . . . . . . . . . . . . . . . . . . . . . . . . . . . . . . . . . . . . . . . . . . . . . . . . . . . . 263
M. Eyman Mortada, Jasbir S. Sra, and Masood Akhtar

Case 60 . . . . . . . . . . . . . . . . . . . . . . . . . . . . . . . . . . . . . . . . . . . . . . . . . . . . . . . . . . . . . . 267
Matthew D. Hutchinson

Case 61 . . . . . . . . . . . . . . . . . . . . . . . . . . . . . . . . . . . . . . . . . . . . . . . . . . . . . . . . . . . . . . 271
Ronald Lo, Henry H. Hsia, and Amin Al-Ahmad

Case 62 . . . . . . . . . . . . . . . . . . . . . . . . . . . . . . . . . . . . . . . . . . . . . . . . . . . . . . . . . . . . . . 275
Richard H. Hongo and Andrea Natale

Case 63 . . . . . . . . . . . . . . . . . . . . . . . . . . . . . . . . . . . . . . . . . . . . . . . . . . . . . . . . . . . . . . 279
David J. Callans

Case 64 . . . . . . . . . . . . . . . . . . . . . . . . . . . . . . . . . . . . . . . . . . . . . . . . . . . . . . . . . . . . . . 283
J. David Burkhardt, Dimpi Patel, and Andrea Natale

Case 65 . . . . . . . . . . . . . . . . . . . . . . . . . . . . . . . . . . . . . . . . . . . . . . . . . . . . . . . . . . . . . . 287
Matthew D. Hutchinson

Case 66 . . . . . . . . . . . . . . . . . . . . . . . . . . . . . . . . . . . . . . . . . . . . . . . . . . . . . . . . . . . . . . 291
Matthew D. Hutchinson

Case 67 . . . . . . . . . . . . . . . . . . . . . . . . . . . . . . . . . . . . . . . . . . . . . . . . . . . . . . . . . . . . . . 295
Matthew D. Hutchinson

Case 68 . . . . . . . . . . . . . . . . . . . . . . . . . . . . . . . . . . . . . . . . . . . . . . . . . . . . . . . . . . . . . . 299
Matthew D. Hutchinson

Case 69 . . . . . . . . . . . . . . . . . . . . . . . . . . . . . . . . . . . . . . . . . . . . . . . . . . . . . . . . . . . . . . 303
Matthew D. Hutchinson

Case 70 . . . . . . . . . . . . . . . . . . . . . . . . . . . . . . . . . . . . . . . . . . . . . . . . . . . . . . . . . . . . . . 307
Matthew D. Hutchinson

Case 71 . . . . . . . . . . . . . . . . . . . . . . . . . . . . . . . . . . . . . . . . . . . . . . . . . . . . . . . . . . . . . . 311
Matthew D. Hutchinson

Section 2 Devices

Case 72 . . . . . . . . . . . . . . . . . . . . . . . . . . . . . . . . . . . . . . . . . . . . . . . . . . . . . . . . . . . . . . 317
Amin Al-Ahmad and Paul J. Wang
xiv Contents

Case 73 . . . . . . . . . . . . . . . . . . . . . . . . . . . . . . . . . . . . . . . . . . . . . . . . . . . . . . . . . . . . . . 319
Amin Al-Ahmad and Paul J. Wang

Case 74 . . . . . . . . . . . . . . . . . . . . . . . . . . . . . . . . . . . . . . . . . . . . . . . . . . . . . . . . . . . . . . 321
Kenneth A. Ellenbogen

Case 75 . . . . . . . . . . . . . . . . . . . . . . . . . . . . . . . . . . . . . . . . . . . . . . . . . . . . . . . . . . . . . . 325
Nora Goldschlager

Case 76 . . . . . . . . . . . . . . . . . . . . . . . . . . . . . . . . . . . . . . . . . . . . . . . . . . . . . . . . . . . . . . 327
Fred M. Kusumoto, Jennifer Crain, and Nora Goldschlager

Case 77 . . . . . . . . . . . . . . . . . . . . . . . . . . . . . . . . . . . . . . . . . . . . . . . . . . . . . . . . . . . . . . 331
Gregory M. Marcus and Nora Goldschlager

Case 78 . . . . . . . . . . . . . . . . . . . . . . . . . . . . . . . . . . . . . . . . . . . . . . . . . . . . . . . . . . . . . . 335
Amin Al-Ahmad and Paul J. Wang

Case 79 . . . . . . . . . . . . . . . . . . . . . . . . . . . . . . . . . . . . . . . . . . . . . . . . . . . . . . . . . . . . . . 337
Kenneth A. Ellenbogen and Rod Bolanos

Case 80 . . . . . . . . . . . . . . . . . . . . . . . . . . . . . . . . . . . . . . . . . . . . . . . . . . . . . . . . . . . . . . 341
Nora Goldschlager

Case 81 . . . . . . . . . . . . . . . . . . . . . . . . . . . . . . . . . . . . . . . . . . . . . . . . . . . . . . . . . . . . . . 343
Fred M. Kusumoto, Jennifer Crain, and Nora Goldschlager

Case 82 . . . . . . . . . . . . . . . . . . . . . . . . . . . . . . . . . . . . . . . . . . . . . . . . . . . . . . . . . . . . . . 345
Amin Al-Ahmad and Paul J. Wang

Case 83 . . . . . . . . . . . . . . . . . . . . . . . . . . . . . . . . . . . . . . . . . . . . . . . . . . . . . . . . . . . . . . 347
Kenneth A. Ellenbogen

Case 84 . . . . . . . . . . . . . . . . . . . . . . . . . . . . . . . . . . . . . . . . . . . . . . . . . . . . . . . . . . . . . . 349
Nora Goldschlager

Case 85 . . . . . . . . . . . . . . . . . . . . . . . . . . . . . . . . . . . . . . . . . . . . . . . . . . . . . . . . . . . . . . 351
Anurag Gupta and Amin Al-Ahmad

Case 86 . . . . . . . . . . . . . . . . . . . . . . . . . . . . . . . . . . . . . . . . . . . . . . . . . . . . . . . . . . . . . . 353
Fred M. Kusumoto, Jennifer Crain, and Nora Goldschlager

Case 87 . . . . . . . . . . . . . . . . . . . . . . . . . . . . . . . . . . . . . . . . . . . . . . . . . . . . . . . . . . . . . . 357
Kenneth A. Ellenbogen

Case 88 . . . . . . . . . . . . . . . . . . . . . . . . . . . . . . . . . . . . . . . . . . . . . . . . . . . . . . . . . . . . . . 361
Nora Goldschlager

Case 89 . . . . . . . . . . . . . . . . . . . . . . . . . . . . . . . . . . . . . . . . . . . . . . . . . . . . . . . . . . . . . . 363
Kenneth A. Ellenbogen, Rod Bolanos, and Mark A. Wood
Contents xv

Case 90 . . . . . . . . . . . . . . . . . . . . . . . . . . . . . . . . . . . . . . . . . . . . . . . . . . . . . . . . . . . . . . 369
Fred M. Kusumoto, Jennifer Crain, and Nora Goldschlager

Case 91 . . . . . . . . . . . . . . . . . . . . . . . . . . . . . . . . . . . . . . . . . . . . . . . . . . . . . . . . . . . . . . 371
M. Eyman Mortada, Jasbir S. Sra, and Masood Akhtar

Case 92 . . . . . . . . . . . . . . . . . . . . . . . . . . . . . . . . . . . . . . . . . . . . . . . . . . . . . . . . . . . . . . 373
Amin Al-Ahmad

Case 93 . . . . . . . . . . . . . . . . . . . . . . . . . . . . . . . . . . . . . . . . . . . . . . . . . . . . . . . . . . . . . . 375
M. Eyman Mortada, Jasbir S. Sra, and Masood Akhtar

Case 94 . . . . . . . . . . . . . . . . . . . . . . . . . . . . . . . . . . . . . . . . . . . . . . . . . . . . . . . . . . . . . . 379
Kenneth A. Ellenbogen and Rod Bolanos

Case 95 . . . . . . . . . . . . . . . . . . . . . . . . . . . . . . . . . . . . . . . . . . . . . . . . . . . . . . . . . . . . . . 383
Byron K. Lee

Case 96 . . . . . . . . . . . . . . . . . . . . . . . . . . . . . . . . . . . . . . . . . . . . . . . . . . . . . . . . . . . . . . 387
Amin Al-Ahmad and Paul J. Wang

Case 97 . . . . . . . . . . . . . . . . . . . . . . . . . . . . . . . . . . . . . . . . . . . . . . . . . . . . . . . . . . . . . . 389
Kenneth A. Ellenbogen and Rod Bolanos

Case 98 . . . . . . . . . . . . . . . . . . . . . . . . . . . . . . . . . . . . . . . . . . . . . . . . . . . . . . . . . . . . . . 391
Kenneth A. Ellenbogen

Case 99 . . . . . . . . . . . . . . . . . . . . . . . . . . . . . . . . . . . . . . . . . . . . . . . . . . . . . . . . . . . . . . 395
Amin Al-Ahmad and Paul J. Wang

Case 100 . . . . . . . . . . . . . . . . . . . . . . . . . . . . . . . . . . . . . . . . . . . . . . . . . . . . . . . . . . . . . 399


Kenneth A. Ellenbogen

Case 101 . . . . . . . . . . . . . . . . . . . . . . . . . . . . . . . . . . . . . . . . . . . . . . . . . . . . . . . . . . . . . 403


Amin Al-Ahmad and Paul J. Wang

Case 102 . . . . . . . . . . . . . . . . . . . . . . . . . . . . . . . . . . . . . . . . . . . . . . . . . . . . . . . . . . . . . 405


Kenneth A. Ellenbogen

Case 103 . . . . . . . . . . . . . . . . . . . . . . . . . . . . . . . . . . . . . . . . . . . . . . . . . . . . . . . . . . . . . 407


Amin Al-Ahmad and Paul J. Wang

Case 104 . . . . . . . . . . . . . . . . . . . . . . . . . . . . . . . . . . . . . . . . . . . . . . . . . . . . . . . . . . . . . 411


Kenneth A. Ellenbogen

Case 105 . . . . . . . . . . . . . . . . . . . . . . . . . . . . . . . . . . . . . . . . . . . . . . . . . . . . . . . . . . . . . 415


Kenneth A. Ellenbogen and Rod Bolanos

Case 106 . . . . . . . . . . . . . . . . . . . . . . . . . . . . . . . . . . . . . . . . . . . . . . . . . . . . . . . . . . . . . 419


Kenneth A. Ellenbogen
xvi Contents

Case 107 . . . . . . . . . . . . . . . . . . . . . . . . . . . . . . . . . . . . . . . . . . . . . . . . . . . . . . . . . . . . . 423


Paul A. Friedman and Charles D. Swerdlow

Case 108 . . . . . . . . . . . . . . . . . . . . . . . . . . . . . . . . . . . . . . . . . . . . . . . . . . . . . . . . . . . . . 427


Paul A. Friedman and Charles D. Swerdlow

Case 109 . . . . . . . . . . . . . . . . . . . . . . . . . . . . . . . . . . . . . . . . . . . . . . . . . . . . . . . . . . . . . 431


Paul A. Friedman and Charles D. Swerdlow

Case 110 . . . . . . . . . . . . . . . . . . . . . . . . . . . . . . . . . . . . . . . . . . . . . . . . . . . . . . . . . . . . . 433


Paul A. Friedman and Charles D. Swerdlow

Case 111 . . . . . . . . . . . . . . . . . . . . . . . . . . . . . . . . . . . . . . . . . . . . . . . . . . . . . . . . . . . . . 439


Paul A. Friedman and Charles D. Swerdlow

Case 112 . . . . . . . . . . . . . . . . . . . . . . . . . . . . . . . . . . . . . . . . . . . . . . . . . . . . . . . . . . . . . 441


Paul A. Friedman and Charles D. Swerdlow

Case 113 . . . . . . . . . . . . . . . . . . . . . . . . . . . . . . . . . . . . . . . . . . . . . . . . . . . . . . . . . . . . . 443


Paul A. Friedman and Charles D. Swerdlow

Case 114 . . . . . . . . . . . . . . . . . . . . . . . . . . . . . . . . . . . . . . . . . . . . . . . . . . . . . . . . . . . . . 447


Paul A. Friedman and Charles D. Swerdlow

Case 115 . . . . . . . . . . . . . . . . . . . . . . . . . . . . . . . . . . . . . . . . . . . . . . . . . . . . . . . . . . . . . 451


Paul A. Friedman and Charles D. Swerdlow

Case 116 . . . . . . . . . . . . . . . . . . . . . . . . . . . . . . . . . . . . . . . . . . . . . . . . . . . . . . . . . . . . . 453


Paul A. Friedman and Charles D. Swerdlow

Case 117 . . . . . . . . . . . . . . . . . . . . . . . . . . . . . . . . . . . . . . . . . . . . . . . . . . . . . . . . . . . . . 455


Paul A. Friedman and Charles D. Swerdlow

Case 118 . . . . . . . . . . . . . . . . . . . . . . . . . . . . . . . . . . . . . . . . . . . . . . . . . . . . . . . . . . . . . 459


Paul A. Friedman and Charles D. Swerdlow

Case 119 . . . . . . . . . . . . . . . . . . . . . . . . . . . . . . . . . . . . . . . . . . . . . . . . . . . . . . . . . . . . . 465


M. Eyman Mortada, Jasbir S. Sra, and Masood Akhtar

Case 120 . . . . . . . . . . . . . . . . . . . . . . . . . . . . . . . . . . . . . . . . . . . . . . . . . . . . . . . . . . . . . 469


Paul A. Friedman and Charles D. Swerdlow

Case 121 . . . . . . . . . . . . . . . . . . . . . . . . . . . . . . . . . . . . . . . . . . . . . . . . . . . . . . . . . . . . . 471


Kenneth A. Ellenbogen

Case 122 . . . . . . . . . . . . . . . . . . . . . . . . . . . . . . . . . . . . . . . . . . . . . . . . . . . . . . . . . . . . . 473


Anurag Gupta

Case 123 . . . . . . . . . . . . . . . . . . . . . . . . . . . . . . . . . . . . . . . . . . . . . . . . . . . . . . . . . . . . . 477


Ronald Lo, Amin Al-Ahmad, and Paul J. Wang
Contents xvii

Case 124 . . . . . . . . . . . . . . . . . . . . . . . . . . . . . . . . . . . . . . . . . . . . . . . . . . . . . . . . . . . . . 481


Ronald Lo, Amin Al-Ahmad, and Paul J. Wang

Case 125 . . . . . . . . . . . . . . . . . . . . . . . . . . . . . . . . . . . . . . . . . . . . . . . . . . . . . . . . . . . . . 483


Ronald Lo, Amin Al-Ahmad, and Paul J. Wang

Section 3 Clinical Cases

Case 126 . . . . . . . . . . . . . . . . . . . . . . . . . . . . . . . . . . . . . . . . . . . . . . . . . . . . . . . . . . . . . 487


Mehmet K. Aktas, Abrar H. Shah, and James P. Daubert

Case 127 . . . . . . . . . . . . . . . . . . . . . . . . . . . . . . . . . . . . . . . . . . . . . . . . . . . . . . . . . . . . . 491


Loren P. Budge and John P. DiMarco

Case 128 . . . . . . . . . . . . . . . . . . . . . . . . . . . . . . . . . . . . . . . . . . . . . . . . . . . . . . . . . . . . . 495


David J. Callans

Case 129 . . . . . . . . . . . . . . . . . . . . . . . . . . . . . . . . . . . . . . . . . . . . . . . . . . . . . . . . . . . . . 497


Andrew E. Darby and John P. DiMarco

Case 130 . . . . . . . . . . . . . . . . . . . . . . . . . . . . . . . . . . . . . . . . . . . . . . . . . . . . . . . . . . . . . 501


Thomas J. Sawyer, Burr W. Hall, and James P. Daubert

Case 131 . . . . . . . . . . . . . . . . . . . . . . . . . . . . . . . . . . . . . . . . . . . . . . . . . . . . . . . . . . . . . 505


John P. DiMarco

Case 132 . . . . . . . . . . . . . . . . . . . . . . . . . . . . . . . . . . . . . . . . . . . . . . . . . . . . . . . . . . . . . 509


John P. DiMarco

Case 133 . . . . . . . . . . . . . . . . . . . . . . . . . . . . . . . . . . . . . . . . . . . . . . . . . . . . . . . . . . . . . 511


John P. DiMarco

Case 134 . . . . . . . . . . . . . . . . . . . . . . . . . . . . . . . . . . . . . . . . . . . . . . . . . . . . . . . . . . . . . 513


John P. DiMarco

Case 135 . . . . . . . . . . . . . . . . . . . . . . . . . . . . . . . . . . . . . . . . . . . . . . . . . . . . . . . . . . . . . 517


John P. DiMarco

Case 136 . . . . . . . . . . . . . . . . . . . . . . . . . . . . . . . . . . . . . . . . . . . . . . . . . . . . . . . . . . . . . 521


John P. DiMarco

Case 137 . . . . . . . . . . . . . . . . . . . . . . . . . . . . . . . . . . . . . . . . . . . . . . . . . . . . . . . . . . . . . 523


John P. DiMarco

Case 138 . . . . . . . . . . . . . . . . . . . . . . . . . . . . . . . . . . . . . . . . . . . . . . . . . . . . . . . . . . . . . 527


John P. DiMarco

Case 139 . . . . . . . . . . . . . . . . . . . . . . . . . . . . . . . . . . . . . . . . . . . . . . . . . . . . . . . . . . . . . 529


John P. DiMarco
xviii Contents

Case 140 . . . . . . . . . . . . . . . . . . . . . . . . . . . . . . . . . . . . . . . . . . . . . . . . . . . . . . . . . . . . . 531


John P. DiMarco

Case 141 . . . . . . . . . . . . . . . . . . . . . . . . . . . . . . . . . . . . . . . . . . . . . . . . . . . . . . . . . . . . . 535


John P. DiMarco

Case 142 . . . . . . . . . . . . . . . . . . . . . . . . . . . . . . . . . . . . . . . . . . . . . . . . . . . . . . . . . . . . . 539


John P. DiMarco

Case 143 . . . . . . . . . . . . . . . . . . . . . . . . . . . . . . . . . . . . . . . . . . . . . . . . . . . . . . . . . . . . . 545


John P. DiMarco

Case 144 . . . . . . . . . . . . . . . . . . . . . . . . . . . . . . . . . . . . . . . . . . . . . . . . . . . . . . . . . . . . . 551


John P. DiMarco

Case 145 . . . . . . . . . . . . . . . . . . . . . . . . . . . . . . . . . . . . . . . . . . . . . . . . . . . . . . . . . . . . . 555


John P. DiMarco

Case 146 . . . . . . . . . . . . . . . . . . . . . . . . . . . . . . . . . . . . . . . . . . . . . . . . . . . . . . . . . . . . . 559


Brett A. Faulknier, David T. Huang, and James P. Daubert

Case 147 . . . . . . . . . . . . . . . . . . . . . . . . . . . . . . . . . . . . . . . . . . . . . . . . . . . . . . . . . . . . . 565


Stefan H. Hohnloser and Joachim R. Ehrlich

Case 148 . . . . . . . . . . . . . . . . . . . . . . . . . . . . . . . . . . . . . . . . . . . . . . . . . . . . . . . . . . . . . 567


Joachim Ehrlich and Stefan H. Hohnloser

Case 149 . . . . . . . . . . . . . . . . . . . . . . . . . . . . . . . . . . . . . . . . . . . . . . . . . . . . . . . . . . . . . 571


Bradley P. Knight

Case 150 . . . . . . . . . . . . . . . . . . . . . . . . . . . . . . . . . . . . . . . . . . . . . . . . . . . . . . . . . . . . . 575


Bradley P. Knight

Case 151 . . . . . . . . . . . . . . . . . . . . . . . . . . . . . . . . . . . . . . . . . . . . . . . . . . . . . . . . . . . . . 577


Bradley P. Knight

Case 152 . . . . . . . . . . . . . . . . . . . . . . . . . . . . . . . . . . . . . . . . . . . . . . . . . . . . . . . . . . . . . 579


Andrew D. Krahn

Case 153 . . . . . . . . . . . . . . . . . . . . . . . . . . . . . . . . . . . . . . . . . . . . . . . . . . . . . . . . . . . . . 583


Byron K. Lee, Melvin M. Scheinman, and Zian H. Tseng

Case 154 . . . . . . . . . . . . . . . . . . . . . . . . . . . . . . . . . . . . . . . . . . . . . . . . . . . . . . . . . . . . . 585


Srijoy Mahapatra

Case 155 . . . . . . . . . . . . . . . . . . . . . . . . . . . . . . . . . . . . . . . . . . . . . . . . . . . . . . . . . . . . . 589


Pamela K. Mason

Case 156 . . . . . . . . . . . . . . . . . . . . . . . . . . . . . . . . . . . . . . . . . . . . . . . . . . . . . . . . . . . . . 593


Pamela K. Mason
Contents xix

Case 157 . . . . . . . . . . . . . . . . . . . . . . . . . . . . . . . . . . . . . . . . . . . . . . . . . . . . . . . . . . . . . 595


Pamela K. Mason

Case 158 . . . . . . . . . . . . . . . . . . . . . . . . . . . . . . . . . . . . . . . . . . . . . . . . . . . . . . . . . . . . . 597


Pamela K. Mason

Case 159 . . . . . . . . . . . . . . . . . . . . . . . . . . . . . . . . . . . . . . . . . . . . . . . . . . . . . . . . . . . . . 599


Lisa G. Umphrey and John Paul Mounsey

Case 160 . . . . . . . . . . . . . . . . . . . . . . . . . . . . . . . . . . . . . . . . . . . . . . . . . . . . . . . . . . . . . 603


James A. Reiffel

Case 161 . . . . . . . . . . . . . . . . . . . . . . . . . . . . . . . . . . . . . . . . . . . . . . . . . . . . . . . . . . . . . 607


James A. Reiffel

Case 162 . . . . . . . . . . . . . . . . . . . . . . . . . . . . . . . . . . . . . . . . . . . . . . . . . . . . . . . . . . . . . 611


Jens Seiler and William G. Stevenson

Case 163 . . . . . . . . . . . . . . . . . . . . . . . . . . . . . . . . . . . . . . . . . . . . . . . . . . . . . . . . . . . . . 617


Jens Seiler and William G. Stevenson

Case 164 . . . . . . . . . . . . . . . . . . . . . . . . . . . . . . . . . . . . . . . . . . . . . . . . . . . . . . . . . . . . . 625


Jens Seiler and William G. Stevenson

Case 165 . . . . . . . . . . . . . . . . . . . . . . . . . . . . . . . . . . . . . . . . . . . . . . . . . . . . . . . . . . . . . 631


J. Jason West and John Paul Mounsey

Case 166 . . . . . . . . . . . . . . . . . . . . . . . . . . . . . . . . . . . . . . . . . . . . . . . . . . . . . . . . . . . . . 635


Darren Traub, James P. Daubert, and Spencer Rosero

Case 167 . . . . . . . . . . . . . . . . . . . . . . . . . . . . . . . . . . . . . . . . . . . . . . . . . . . . . . . . . . . . . 639


Vikas P. Kuriachan and George D. Veenhuyzen

Case 168 . . . . . . . . . . . . . . . . . . . . . . . . . . . . . . . . . . . . . . . . . . . . . . . . . . . . . . . . . . . . . 643


Vikas P. Kuriachan and George D. Veenhuyzen

Case 169 . . . . . . . . . . . . . . . . . . . . . . . . . . . . . . . . . . . . . . . . . . . . . . . . . . . . . . . . . . . . . 647


Jeffrey D. Booker and George D. Veenhuyzen

Case 170 . . . . . . . . . . . . . . . . . . . . . . . . . . . . . . . . . . . . . . . . . . . . . . . . . . . . . . . . . . . . . 649


M. Eyman Mortada, Jasbir S. Sra, and Masood Akhtar

Case 171 . . . . . . . . . . . . . . . . . . . . . . . . . . . . . . . . . . . . . . . . . . . . . . . . . . . . . . . . . . . . . 653


M. Eyman Mortada, Jasbir S. Sra, and Masood Akhtar

Index . . . . . . . . . . . . . . . . . . . . . . . . . . . . . . . . . . . . . . . . . . . . . . . . . . . . . . . . . . . . . . . . 655

Section
I
Ablation

Case 1

Amin Al-Ahmad

Case Summary What maneuvers would be important to elucidate the


diagnosis during electrophysiology study and ablation?
A 72-year-old male with a history of aortic sinus of valsalva
rupture and repair 30 years prior presents with palpitations.
The patient reports having symptoms of shortness of breath Case Discussion
and fatigue with palpitations. He is currently taking only
aspirin. He has had cardioversions in the past for this arrhyth-
This tachycardia may represent a focal atrial tachycardia
mia and would like to consider ablation. He does not want to
given the isoelectric segment on the 12-lead ECG. However,
take anti-arrhythmic medications.
in patients with prior cardiac surgery, atrial flutters should be
His 12-lead ECG (Fig. 1.1) shows a 2:1 atrial tachyar-
considered. The possibility that this tachycardia is typical
rhythmia at a rate of approximately 180 bpm. The P-waves in
isthmus-dependent atrial flutter should be explored using
the inferior leads are negative; however there is an isoelectric
entrainment pacing at the tricuspid valve (TV) to inferior
segment between P-waves.

Fig. 1.1 Twelve-lead ECG showing atrial arrhythmia. Note the negative atrial deflections in the inferior leads and the isoelectric segment
between them

A. Al-Ahmad
School of Medicine, Stanford University, 300 Pasteur Drive, H-2146,
Stanford, CA 94305

A. Natale et al. (eds.), Cardiac Electrophysiology, 3


DOI: 10.1007/978-1-84996-390-9_1, Springer-Verlag London Limited 2011
4 A. Al-Ahmad

vena cava (IVC) isthmus. Reverse typical atrial flutter would In this case, entrainment at the TV-IVC isthmus demon-
be unlikely given the morphology of the flutter waves. Scar- strated a near perfect PPI and ablation at the TV-IVC isthmus
related atrial flutters should also be considered. Ablation of terminated the atrial flutter (Figs.1.2 and 1.3). The isoelectric
all potential atrial flutter circuits in patients post cardiac sur- segment between flutter waves likely represents slow atrial
gery may reduce the likelihood of recurrence.1 conduction in atrial flutter rather than atrial tachycardia.

200ms
I

aVF

V6

hRA p

HIS p

HIS d

CS 9,10

CS 7,8

CS 5,6

CS 3,4

CS 1,2

RV p

ABL
PPI TCL
ABL d

Stim 4
12:22:16 PM 12:22:17 PM

Fig.1.2 Pacing using the ablation catheter positioned at the tricuspid valve-inferior vena cava (TV-IVC) isthmus. Concealed entrainment and a
near perfect post-pacing interval are noted
Case 1 5

500ms

aVF

hRA p

HIS p

HIS d

CS 9,10

CS 7,8

CS 5,6

CS 3,4

CS 1,2

RV p

ABL

ABL d

Stim 4
1:01:47 PM 1:01:48 PM 1:01:49 PM 1:01:50 PM 1:01:51 PM

Fig.1.3 Application of RF energy in the TV-IVC area leads to termination of the arrhythmia

Reference

1. Verma A, Marrouche NF, Seshadri N, Schweikert RA, Bhargava M,


Burkhardt JD, Kilicaslan F, Cummings J, Saliba W, Natale A.
Importance of ablating all potential right atrial flutter circuits in
postcardiac surgery patients. J Am Coll Cardiol. July 2004;44(2):
409414.

Case 2

Michel Haissaguerre

Case Summary AF always started following monomorphic atrial ectopy


(Fig.2.1).
A 47-year-old male with a 4-year history of symptomatic, A decapolar catheter (Xtrem, ELA Medical, Le-Plessis-
drug-resistant lone paroxysmal atrial fibrillation (AF) was Robinson, France) was inserted inside the coronary sinus
referred for a first ablation procedure. He suffered from (CS) while the ablation catheter (Thermocool Biosense
daily AF episodes that lasted a maximum of 8 h. Episodes of Webster, Diamond Bar, CA) and a decapolar circumferential

II

III

aVR

aVL

aVF

V1

V2

V3

V4

V5

V6

Fig.2.1 Twelve-lead ECG. Sinus rhythm and short coupling atrial ectopies (with functional left bundle branch block [LBBB])

M. Haissaguerre
Department of Cardiology and Electrophysiology, CHU de Bordeaux,
Hospital Cardiologique de Haut Leveque, Avenue de Magellan, Pessac
33604, France

A. Natale et al. (eds.), Cardiac Electrophysiology, 7


DOI: 10.1007/978-1-84996-390-9_2, Springer-Verlag London Limited 2011
8 M. Haissaguerre

catheter (Lasso, Biosense Webster, Diamond Bar, CA) were the adjacent LA and is synchronous with the second half of
introduced through a long sheath in the left atrium (LA). The the P-wave (in the right PVs it should be the first part of the
circumferential catheter was placed in the left superior pul- P-wave). The second potential reflects local activity from
monary vein (LSPV) and recorded venous and atrial poten- the PV striated musculature (black star). When ectopy
tials (Fig.2.2). occurs in the PV (Fig.2.2, second complex), there is a rever-
What mechanism is illustrated and what action is sal of the described activation sequence, with the PV poten-
required? tial preceding the atrial potential. This pattern of reverse
activation in a dead-end structure during ectopic triggered
AF evidence for the arrhythmogenic potential of that PV.
Mapping of the earliest site of activity during ectopy allows
Case Discussion identification of discrete sites inside the vein, while the atrial
exit site is dependent on the anatomy of the PV-LA connect-
Figure2.1 shows a recording of atrial ectopics with a short ing fascicles.
coupling interval (also note the functional left bundle branch Given that arrhythmia recurrence can occur from either
block [LBBB]). The P-wave morphology during ectopy the pulmonary vein that is active at the time of the procedure
is flat in the lateral leads, positive in the inferior leads and or any other PV, complete electrical isolation of all PVs has
in lead V1, suggesting they originate from the LSPV. to be carried out with a series of coalescent RF applications
Endocardial tracings from the LSPV (Fig. 2.2) show two using a dedicated PV circumferential catheter to help with
separated potentials during sinus rhythm (Fig.2.2, first com- mapping. Ablation is performed outside the vein (within 12
plex). The first potential (white star) represents activation of cm of the PV ostia) for right PVs and for the posterior part

II

V1

PV 1-2

PV 2-3

PV 3-4

PV 4-5

PV 5-6

PV 6-7

PV 7-8

PV 8-9

PV 9-10

CS 1-2
CS 3-4
CS 5-6
CS 7-8
CS 9-10

Fig. 2.2 Endocardial tracings from the left superior pulmonary vein complex), there existed the presence of two separated potentials (one
(LSPV). Recording from a decapolar circumferential catheter inserted atrium, white star followed by one venous, black star). During ectopy
inside the LSPV (PV 12 to PV 910) and a decapolar catheter inserted (second complex), reversal of the described activation sequence with
into the coronary sinus (CS 12 to CS 910). During sinus rhythm (first the PV potential preceding the atrial potential occurred
Case 2 9

of the left PVs; however, due to the ridge between the left segmental or circumferential ablation with a continuous cir-
pulmonary veins and the left atrial appendage (LAA), cath- cular lesion can be performed depending on the operators
eter stability is an issue for ablation of the anterior aspect of preference. When doing continuous circumferential lesions,
the left PVs, and ablation is often within 1 mm of the it is unusual to achieve PV isolation without further ablation
veins. targeting the earliest PV activity or sites of reverse PV polar-
In this case, ablation was started at the low anterior LSPV ity as recorded on the circumferential mapping catheter, as in
(pole 5) where the earliest activity was located and where a Fig.2.2, indicating a residual anatomical connection on the
reverse in PV polarity was observed, both criteria pointing at line of ablation.
a local anatomical connection (Fig.2.3, panel A). Ablation at After ablation, nine attempts at induction (with bursts up
this point delayed the venous potentials (Fig.2.3, panel B), to a cycle length of 200 ms) at three different places (CS and
and a second anatomical breakthrough was subsequently both appendages) could not induce sustained arrhythmia,
ablated at the upper part of the vein (pole 1). The ectopic predicting a favorable clinical outcome.
beats stopped (Fig.2.3, panel C) and the venous potentials This case illustrates a typical ablation of paroxysmal AF
became dissociated (Fig.2.4).Ablation of the left inferior PV where it was clearly demonstrated that the arrhythmogenic
(LIPV) was performed in the same way. For the right veins, ectopic beats triggering AF originated from the LSPV.

A B C
I

II

V1

PV 1-2

PV 2-3
PV 3-4

PV 4-5
PV 5-6
PV 6-7
PV 7-8
PV 8-9
PV 9-10

CS 1-2
CS 3-4
CS 5-6
CS 7-8
CS 9-10

Fig.2.3 Endocardial tracings recorded during sinus rhythm from the 910). During ablation targeting the earliest venous potential (black
left superior pulmonary (LSPV) vein. Recording from a decapolar cir- star), progressive slowing of the conduction to the vein (panel A and B)
cumferential catheter inserted inside the LSPV (PV 12 to PV 910) to the complete block (panel C)
and a decapolar catheter inserted into the coronary sinus (CS 12 to CS
10 M. Haissaguerre

17 18 19 20 21 22 23 24 25 26
I

II

V1

PV 1-2
PV 2-3
PV 3-4
PV 4-5
PV 5-6
PV 6-7
PV 7-8
PV 8-9
PV 9-10

CS 1-2
CS 3-4
CS 5-6
CS 7-8
CS 9-10

Fig.2.4 Endocardial tracings recorded during sinus rhythm from the and a decapolar catheter inserted into the coronary sinus (CS 12 to CS
left superior pulmonary vein (LSPV). Recording from a decapolar cir- 910). Dissociation of the venous potential (black star) with a slow
cumferential catheter inserted inside the LSPV (PV 12 to PV 910) automatic activity

Bibliography Hassaguerre M, Hocini M, Sanders P, etal. Localized sources main-


taining atrial fibrillation organized by prior ablation. Circulation.
2006;113(5):616-625.
Hassaguerre M, Shah DC, Jas P, et al. Electrophysiological break- Sanders P, Hocini M, Jais P, etal. Characterization of focal atrial tachy-
throughs from the left atrium to the pulmonary veins. Circulation. cardia using high-density mapping. J Am Coll Cardiol. 2005
2000;102:2463-2465. December 6;46(11):2088-2099.
Haissaguerre M, Sanders P, Hocini M, etal. Catheter ablation of long- Mohamed U, Skanes AC, Gula LJ, etal. A novel pacing maneuver to
lasting persistent atrial fibrillation: critical structures for termina- localize focal atrial tachycardia. J Cardiovasc Electrophys.
tion. J Cardiovasc Electrophys. 2005;16(11):1125-1137. 2007;18(1):1-6.
Case 3

Amin Al-Ahmad

Case Summary arrhythmic medications does not seem to be an attractive


option given his multiple co-morbid conditions. And despite
better rate control he remains symptomatic and is taken to
A 51-year-old male with a history of pulmonary fibrosis and a
the electrophysiology suite.
single lung transplant is found to have atrial flutter on routine
Where is this tachycardia origin, and what pacing maneu-
follow-up. Initially he is unaware of this rhythm, but later recalls
vers may help determine the best area to ablate?
an increase in fatigue over the previous week. He is referred for
evaluation by an electrophysiologist. A Holter monitor shows
his heart rate to be approximately 90 on average. He has periods
of normal sinus rhythm and periods of atrial flutter. A 12-lead
ECG during atrial flutter is shown in Fig.3.1. Case Discussion
Over the next few months his atrial flutter becomes per-
sistent and he continues to complain of fatigue. He is talking Atrial flutter can be common immediately after lung trans-
multiple medications post transplant to prevent rejection and plantation. Canine models have suggested a substrate for
has some renal insufficiency. Drug therapy with anti- atrial flutter in the left atrium due to suture lines. In addition,

Fig.3.1 Twelve-lead ECG of atrial flutter. Note the peaked and strongly positive flutter waves in lead V1

A. Al-Ahmad
School of Medicine, Stanford University, 300 Pasteur Drive, H-2146,
Stanford, CA 94305

A. Natale et al. (eds.), Cardiac Electrophysiology, 11


DOI: 10.1007/978-1-84996-390-9_3, Springer-Verlag London Limited 2011
12 A. Al-Ahmad

right-sided atrial flutters may also develop as these patients determine if the flutter is left-sided or right-sided. Entrainment
can commonly have right atrial dilation due to potentially can further distinguish the critical isthmus for the atrial flut-
long-standing increased right-sided pressures. ter and help guide ablation.
To differentiate right- and left-sided atrial flutter, the flut- In this case, entrainment on the right side clearly demon-
ter wave morphology can be useful. In this case a strongly strated that the flutter was not right-sided (Fig.3.2). Entrainment
positive flutter wave in the anterior precordial leads suggests in the left atrium near the right inferior pulmonary vein was
that the flutter is left-sided. In addition, entrainment from near perfect. Ablation in that area to create a line to the mitral
areas on the right side can be done quickly and can help valve caused the flutter to terminate (Figs.3.3 and 3.4).

I 200ms

II
V1
V6 S

RA 9,10
RA 7,8
RA 5,6
RA 3,4 PPI TCL
RA 1,2
CS 9,10
CS 7,8
CS 5,6
CS 3,4
CS 1,2
HIS p
HIS m C

HIS d
ABL
ABL d S1
S1 S1
S1 S1
Stim 3 9:42:27 AM 9:42:28 AM

Fig.3.2 Pacing in the right atrium shows a long post-pacing interval. This was the case from multiple areas in the right atrium
Case 3 13

I 200ms

II
V1
V6
RA 9,10
RA 7,8
RA 5,6
RA 3,4
RA 1,2
CS 9,10
CS 7,8
CS 5,6 T

CS 3,4
CS 1,2

ABL

ABL d PPI
S1 TCL

S1 S1
Stim 4 12:55:47 PM 12:55:48 PM 12:55:49 PM

Fig.3.3 Pacing in the left atrium near the right inferior pulmonary vein yields a near-perfect PPI

Bibliography

Nielsen TD, Bahnson T, Davis RD, Palmer SM. Atrial fibrillation after
pulmonary transplant. Chest. August 2004;126(2):496-500.

Fig.3.4 Right anterior oblique view of the ablation catheter in the left
atrium near the right inferior pulmonary vein. CS coronary sinus, ICE
intracardiac echo, ABL ablation catheter

Case 4

Michel Haissaguerre

Case Summary electrograms (CFAEs) and finally performing linear lesions.


Any resulting atrial tachycardias (ATs) are mapped and
A 56-year-old male was referred for ablation of persistent ablated. In this case, following PV isolation there was pro-
drug-resistant atrial fibrillation (AF). He had a 17-year history longation of the AFCL to 165 and 159 ms at left and right
of AF, which had been persistent for the last 12 months. The appendages, respectively. A NavX fractionation map of the
atrial fibrillation cycle length (AFCL) measured during elec- LA was performed using a 20-pole high-density mapping
trophysiological study was 154, 158, and 152 ms on surface catheter. CFAEs were distributed throughout the LA and the
ECG, left and right atrial appendages, respectively (Fig.4.1). CS, as shown in Fig.4.2.
Our approach in persistent AF is to first isolate the pulmo- In cases like this, which CFAE should be targeted, and
nary veins (PVs), before targeting complex fractionated atrial what is the value in measuring AFCL?

II

V1

LAA

RAA
Fig.4.1 Recording from left
(LAA) and right (RAA) atrial
appendages, and the coronary CSd
sinus (CS 12 to 910). While
fractionation is present in the
coronary sinus, preventing an
easy measurement of the cycle
length, potentials at the top of
both appendages are almost
always of high voltage and CSp
unambiguous

M. Haissaguerre
Department of Cardiology and Electrophysiology, CHU de Bordeaux,
Hospital Cardiologique de Haut Leveque, Avenue de Magellan, Pessac
33604, France

A. Natale et al. (eds.), Cardiac Electrophysiology, 15


DOI: 10.1007/978-1-84996-390-9_4, Springer-Verlag London Limited 2011
16 M. Haissaguerre

7CFE Mean Map continuous duration). AFCL of less than 140 ms is associ-
ated with AF termination in less than 69%, while a higher
AFCL like in our case is associated with more than 89% of
AF termination. Furthermore, the impact of ablation of
each region during electrogram-based ablation can be
quantified in both atria by AFCL monitoring. After each
step of ablation, a gradual prolongation of AFCL is usu-
ally observed (a change in AFCL greater than 6 ms is con-
sidered significant). Conversion to sinus rhythm or AT
occurs when AFCL reaches between 180 and 200 ms and,
120 mS
conversely, rarely occurs when the AFCL is shorter
65 mS
thanthis.
Concerning the appropriate sequence of ablation during
the electrogram-based part of the procedure, the main issue
0 mS is to distinguish active from passive patterns, which still
remains one of the major obstacles to minimal effective abla-
Fig.4.2 NavX (EnSite System, St. Jude Medical, MN, USA) fraction-
ation map of the left atrium and coronary sinus after isolation of the
tion of persistent AF. This is particularly striking when there
four pulmonary veins showing ubiquitous distribution of complex frac- are multiple CFAEs distributed throughout the atrium, as in
tionated electrograms this case. From a purely anatomic perspective, in addition to
PVs, ablation at structures annexed to the LA, namely the
interface between the inferior LA and the CS and the base of
Case Discussion the LAA, have been shown to have the greatest impact on the
AF, as measured by AFCL.1 The endpoint of ablation during
Atrial electrograms in persistent AF are complex and cannot electrogram-based ablation remains imprecise; however, the
be reliably evaluated in terms of CL except in either right or organization and slowing of local potentials by ablation
left appendage. We routinely place a catheter in the right seems for us to be preferable to complete elimination of local
atrial appendage (RAA) and LAA to measure simultane- potentials.
ously the left and right AFCLs at the start of the procedure In this case, following PV isolation, ablation started along
(Fig.4.1). Using custom analysis software (Bard EP, Lowell, the inferior LA where continuous activity was recorded
MA, USA), the mean AFCL for the selected window is cal- (Fig.4.3, panel A). Ablation at the endocardial interface of
culated. The electrogram annotation is then verified manu- the CS aims at interrupting the muscular fascicles connecting
ally online to ensure accuracy (Fig.4.1). the LA and the CS and organizing the chaotic activity
The electrograms measured at the top of both append- recorded within the CS. This resulted in a prolongation of the
ages are almost always discrete and of high amplitude, AFCL to 171 ms in the LAA and 168 ms in the RAA.
thereby facilitating unambiguous automatic annotation. In The following step consisted of the ablation at the poste-
the absence of this software, the mean AFCL can be easily rior part of the LAA, where consistent distal-to-proximal
estimated by measuring the total duration required for 1030 electrograms suggesting centrifugal activation were recorded
cycles (or longer) and then dividing that number by the (Fig. 4.3, panel B); this resulted in a prolongation of both
number of cycles. Often, the AFCL is also reasonably esti- AFCLs to 176 ms. Ablation was performed along the roof of
mated from V1 on the 12-lead ECG. Following PV isolation the LA where almost continuous electrograms were recorded
and throughout the procedure, the circumferential mapping (Fig.4.3, panel C), and the AFCL was prolonged to 188 and
catheter is placed within the RAA, and the ablation catheter 183 ms in the right and left appendages, respectively.
in the LAA, and the AFCL may then be assessed simultane- Finally, ablation of continuous electrical activity at the
ously to measure the impact of ablation in both chambers. anteroseptal LA resulted in restoration of sinus rhythm
Initial AFCL has been shown to be the strongest predic- (Fig.4.4). During sinus rhythm, PV isolation was confirmed
tor of success for AF ablation (AF of less than 5 years and the roofline was completed.
Case 4 17

Fig.4.3 Recording of the ablation catheter (RF) and the coronary sinus consistent distal-to-proximal fractionated electrograms. Panel C:
(CS 12 to 910). Panel A: Ablation along the inferior left atrium tar- Ablation was then performed along the roof targeting almost continu-
geting continuous electrical activity. Panel B: Ablation at the posterior ous fractionated electrograms
part of the left atrial appendage targeting centrifugal activation with
18 M. Haissaguerre

I
I

II

II
V1

V1

RFp

CS 1-2

RFd
CS 3-4

CS 1-2 CS 5-6

CS 3-4

CS 7-8
CS 5-6

CS 7-8
CS 9-10

CS 9-10

Fig.4.4 Recording of the ablation catheter (RF) and the coronary sinus (CS 12 to 910). Ablation of fractionated electrical activity at the
anterseptal LA (left panel) resulted in direct restoration of sinus rhythm (right panel, black star)

Bibliography Hassaguerre M, Hocini M, Sanders P, etal. Localized sources main-


taining atrial fibrillation organized by prior ablation. Circulation.
2006;113(5):616-625.
Hassaguerre M, Shah DC, Jas P, et al. Electrophysiological break- Mohamed U, Skanes AC, Gula LJ, etal. A novel pacing maneuver to
throughs from the left atrium to the pulmonary veins. Circulation. localize focal atrial tachycardia. J Cardiovasc Electrophys. 2007;
2000;102:2463-2465. 18(1):1-6.
Haissaguerre M, Sanders P, Hocini M, etal. Catheter ablation of long- Sanders P, Hocini M, Jais P, etal. Characterization of focal atrial tachy-
lasting persistent atrial fibrillation: critical structures for termina- cardia using high-density mapping. J Am Coll Cardiol. 6 December
tion. J Cardiovasc Electrophys. 2005;16(11):1125-1137. 2005;46(11):2088-2099.
Case 5

Anurag Gupta and Amin Al-Ahmad

Case Summary activation sequence (Fig.5.2). Can this patient be treated with
a single application of radiofrequency energy? What maneu-
vers may be helpful in determining this?
A 21-year-old male with WolfParkinsonWhite syndrome is
referred for electrophysiology study given worsening episodes
of palpitations. A 12-lead electrocardiogram (ECG) during
sinus rhythm shows pre-excitation (Fig.5.1). Diagnostic elec- Case Discussion
trode catheters are then placed for recording in the His bundle
area and right ventricle, as well as a 20-pole Halo catheter for Analysis of the delta waves) on the 12-lead ECG during
recording in the right atrium adjacent to the tricuspid valve sinus rhythm (Fig.5.1), including R/S <1 in V2, negative D
annulus. Ventricular pacing shows the retrograde atrial in III, positive D in V1, and negative D in aVF, is suggestive

Fig.5.1 Twelve-lead ECG during sinus rhythm showing ventricular pre-excitation

A. Gupta (*) and A. Al-Ahmad


Cardiac Electrophysiology Service, Division of Cardiology,
Department of Medicine, Stanford University Hospital and Clinics,
300 Pasteur Drive, Room H2146, Stanford, CA
e-mail: agupta@stanfordalumni.org; aalahmad@stanford.edu

A. Natale et al. (eds.), Cardiac Electrophysiology, 19


DOI: 10.1007/978-1-84996-390-9_5, Springer-Verlag London Limited 2011
20 A. Gupta and A. Al-Ahmad

Fig.5.2 Ventricular pacing from


the RV apex. Note the change in
the retrograde activation
sequence. In the first beat,
earliest atrial activation is at pole
RA 9,10. However, in the
following beat, atrial activation at
pole 9,10 is late, whereas atrial
activation recorded from the His
bundle catheter is earliest. TV
tricuspid valve, CS coronary
sinus, RA right atrium, HBE His
bundle electrogram, P proximal
electrode pair, M middle
electrode pair, D distal electrode
pair, RVA right ventricular apex

Fig.5.3 Induced SVT. Note the


change in the retrograde atrial
activation. The SVT continues at
the same rate. Is this evidence for
another accessory pathway?

of a right posterior/right posterolateral accessory pathway. tricuspid valve depicted as a clockface, the SVT is termi-
During electrophysiology study, initiation of orthodromic nated and pre-excitation is eliminated. However, a second
AVRT is observed at the end of atrial extrastimuli testing; the supraventricular tachycardia is subsequently induced with
retrograde atrial activation is observed to change spontane- atrial extrastimuli testing. The second SVT has a concentric
ously during tachycardia (Fig.5.3). A VPC during the period retrograde activation pattern consistent with the other retro-
that the His bundle is refractory advances the following atrial grade pattern seen intermittently during the initial SVT.
beat and resets the tachycardia. Para-Hisian pacing is performed (Fig.5.4).Ventricular pac-
With RF application near Halo RA 9,10 at approximately ing next to the His bundle and proximal to the right bundle
8:00 with reference to the left anterior oblique view with the branch at high output directly captures the His bundle, thus
Case 5 21

Fig.5.4 Para-Hisian pacing is


performed. The first QRS
complex obtained with high-
output pacing is narrow, while
the second QRS complex
obtained with low-output pacing
is wide with LBBB morphology
and longer activation time of RV
apex, which is indicative of the
loss of His bundle capture. While
earliest retrograde activation
occurs at the proximal His bundle
electrogram in both cases,
conduction over an accessory
pathway is indicated by minimal
change in stimulus to atrial
interval and in atrial activation
sequence

activating the ventricles through the HisPurkinje system Bibliography


with resultant narrow QRS complex. Ventricular pacing at
low output does not capture the HisPurkinje system, as evi- Chiang CE, Chen SA, Teo WS, etal. An accurate stepwise electrocar-
denced by left bundle branch block (LBBB) QRS morphol- diographic algorithm for localization of accessory pathways in
ogy with later activation of the RV apex. However, despite patients with WolffParkinsonWhite syndrome from a compre-
hensive analysis of delta waves and R/S ratio during sinus rhythm.
loss of His bundle capture, there is minimal change in the
Am J Cardiol. 1995;76:40-46.
stimulus to atrial interval and activation sequence, which is Hirao K, Otomo K, Wang X, etal. Para-Hisian pacing: a new method
indicative of another retrograde accessory pathway. RF for differentiating retrograde conduction over an accessory AV
application of an anteroseptal pathway terminated the tachy- pathway from conduction over the AV node. Circulation.
1996;94:1027-1035.
cardia, and no further SVT was inducible.

Case 6

Michel Haissaguerre

Case Summary time period. Mapping of such organized AF can be per-


formed conventionally and allows in most cases to determi-
nate the atrial area from which the centrifugal activation
A 62-year-old male was referred for a second ablation of per-
emanates. Then, the area is closely mapped to find either a
sistent atrial fibrillation (AF). He had a 5-year history of AF,
discrete site of early activity (focal point), or a small circuit
which had been persistent for the previous 5 months. The
(where early and late cannot, therefore, be defined) rep-
first procedure consisted of pulmonary vein (PV) isolation,
resenting localized re-entry.
ablation of the inferior left atrium (LA) and coronary sinus
In our case, global activation demonstrated a consistent
(CS), electrogram-based ablation at the posterior and ante-
septal-to-lateral activation sequence anteriorly and posteriorly
rior LA and a linear lesion at the LA roof. This terminated
(Fig.6.2, panel A), a high-to-low activation sequence in the
the AF to an intermediate atrial tachycardia (AT), which was
anterior (Fig.6.3, panel A) and septal LA (Fig.6.2, panel A),
subsequently ablated anterior to the ostium of the left atrial
and a low-to-high sequence in the posterior LA (Fig. 6.3,
appendage (LAA) with subsequent restoration of sinus
panel B). This activation sequence was compatible with cen-
rhythm.
trifugal activation from a localized source located anterior
Unfortunately, a few days later, he had a recurrence,
and septal to the roofline in the presence of roofline block.
which was considered to be AF (Fig.6.1). There was rela-
(Amacro re-entrant AT was excluded by the irregularity of
tively organized atrial activity in V1, and the atrial cycle
the cycle length.)
length was 230 ms with slight irregularity and variability in
The ablation catheter was placed at the high anteroseptal
the F-wave morphology. A quadripolar catheter was placed
LA (Fig.6.4) where the electrograms showed a significant
into the CS and this demonstrated a consistent proximal-to-
conduction delay between the distal and proximal bipoles.
distal activation pattern during arrhythmia. A quadripolar
Furthermore, a pause in electrical activity confirmed that the
cooled-tip ablation catheter was then inserted into the LA.
first electrogram to show variation was the distal electrode
LA endocardial activation was sequentially mapped and
appeared to be organized most of the time (Figs.6.26.4).
What inferences can be made from Figs. 6.16.4 with
I
relevance to the mechanism of atrial arrhythmia?
II

III
AVR
Case Discussion
AVL
AVF
Organized AF may be defined as AF displaying irregular
VI
atrial cycle lengths with beat-to-beat variations of 20 ms,
V2
but with discrete atrial complexes having a consistent activa-
tion sequence for (arbitrarily) 75% of the time over a 10-min V3
V4
V5
V6
M. Haissaguerre
Department of Cardiology and Electrophysiology,
CHU de Bordeaux, Hospital Cardiologique de Haut Leveque, Fig. 6.1 Electrocardiogram of the atrial arrhythmia with relatively
Avenue de Magellan, Pessac 33604, France organized atrial activity in V1 despite slight irregularity

A. Natale et al. (eds.), Cardiac Electrophysiology, 23


DOI: 10.1007/978-1-84996-390-9_6, Springer-Verlag London Limited 2011
24 M. Haissaguerre

Fig.6.2 Schematic representation


of the LA including the left A B
septum and the left atrial
appendage (LAA), after the first
atrial fibrillation ablation. LAA
Representation of previous RPV LPV

Septum
encirclement of the right (RPV)
and left (LPV) pulmonary veins
(continuous lines), and the
previous ablation of the roofline
(dotted line). Electrograms on
distal (RFd) and proximal (RFp)
poles of the ablation at the mouth CS
of the left appendage (panel A),
and at the left septum (panel B).
Recording of V1 and electrogram
on distal coronary sinus (CS) V1

RFd

RFp

CSd

100 ms

pair of the mapping catheter, while the other electrograms A localized source may be a discrete point or a small area.
followed with the same activation sequence. During organized AF, when atrial activation mapping con-
After these observations, other electrograms were recor verges towards the origin of activity, a small area displaying
ded within 1015 mm of the original site, and the whole span centrifugal activation and including the localized source can
of the cardiac cycle was mapped in this area: the initial part be analyzed. This area may include a discrete point harbor-
of the circuit, the second part of the circuit, and throughout ing less than 75% of the cycle length (CL) which allows
the circuit (Fig.6.5). These findings were compatible with a tracking of the earliest activity and where local electrograms
localized re-entrant circuit with centrifugal activation of the display centrifugal activation with or without one-to-one
LA, except for the posterior wall because of the roofline conduction to the adjacent atrium (i.e., the same or faster
block. Ablation at this site slowed and terminated the tachy- cycle length of the source compared to the atrial fibrillation
cardia. Conduction block over the roofline was confirmed in cycle length [AFCL], respectively). In the majority of cases,
sinus rhythm. the source is not a discrete point but a small area of local-
Localized sources perpetuating AF have been described ized re-entry, as in this case. This area of earliest activation
during ablation procedures as persistent fibrillatory activity displays electrical activity covering 75100% of the cycle
confined within isolated LA areas after restoration of sinus length, suggesting a localized small circuit. By mapping with
rhythm. These sources play an important role in AF main- a conventional quadripolar ablation catheter, local potentials
tenance, in addition to pulmonary vein (PV) activity and will display either continuous activity spanning most of the
re-entrant loops. After pulmonary vein isolation (PVI) and CL or temporally alternating potentials between distal and
linear lesions, ablation of these sources remains the last proximal bipoles (depending on the re-entrant circuit size
step of substrate elimination and results in AT conversion and properties). By mapping with a high-density 20-pole
or restoration of sinus rhythm. catheter, local activity will span most or all of the CL.
Case 6 25

Fig.6.3 Same graphic as


Fig.6.2, with the addition of an A B
anterior wall. Ablation catheter
positioned at the anterior (panel A)
and posterior (panel B) left
atrium

V1

RFd

RFp

CSd
100 ms

Indeed, irregularity was >15% of the CL (inset ECG) most frequent AT mechanism, representing 71% of focal ATs
suggesting, in fact, focal AT. Furthermore, post pacing (and 37% of total ATs). The preferential regions for focal
interval (PPI) at the posterior wall was short (+20 ms), ATs are the PVLA junction, left septum, and the mouth of
but the anterior PPI was very long (+200 ms), ruling out the LAA.
macro re-entrant roof-dependant AT. Since the roofline Focal ATs are generally localized to sites targeted during
was previously blocked, it suggested a focal origin from electrogram-based AF ablation. Injury or oedema to the atrial
the high posterior LA. Mapping this area close to the roof tissue induced by RF applications could generate the sub-
line revealed very low voltage local activity spanning strate for further arrhythmia by creating an anchoring point
most of the CL at three different spots very close to each potentially able to maintain re-entry. Therefore the mecha-
other (Fig. 6.3, POST 13). This voltage was only 0.04 nism of AT after AF ablation may be different to that of
mV, which can be easily hidden in case of electrical noise. spontaneous AT. The presence of pre-existing LA scar (which
Ablation of a very low voltage continuous activity site may be idiopathic or related to underlying structural heart
located in between the three described spots terminated disease) may also result in local slow conduction areas pre-
the tachycardia (Fig. 6.4). disposing to re-entry.
This case emphasizes the importance of localized Mapping and ablation of focal or macro re-entrant AT is a
re-entrant AT in the context of prior AF ablation. In this crucial step in the AF ablation process, and often represents
context, 53% of AT are focal, and localized re-entries are the the difference between procedural success and failure.
26 M. Haissaguerre

Fig.6.4 Same graphic as Fig.6.2. Ablation catheter positioned


a nterior and septal to the roofline

LAA
RPV LPV

Septum
CS

V1

RFd

0, 100 mV
RFd

RFp

CSd

100 ms

V1

0, 100 mV
RFd
RFd1

LAA
POST
RFd2
RPVs LPVs

Fig.6.5 Scanning the area


within 1015 mm of the site
um

showed in Fig.6.4: Electrograms


pt
Se

show activation compatible with RFd3


the entire circuit (initial part in CS
RF2, second part in RF3 and
throughout the circuit in RF1).
These data are most compatible
with left atrial activation coming CSd
centrifugally from a small
re-entrant source central to the
excursion of the catheter in this
small area 100 ms
Case 6 27

Bibliography Haissaguerre M, Sanders P, Hocini M, etal. Catheter ablation of long-


lasting persistent atrial fibrillation: critical structures for termina-
tion. J Cardiovasc Electrophys. 2005;16(11):1125-1137.
Hassaguerre M, Hocini M, Sanders P, etal. Localized sources main- Sanders P, Hocini M, Jais P, etal. Characterization of focal atrial tachy-
taining atrial fibrillation organized by prior ablation. Circulation. cardia using high-density mapping. J Am Coll Cardiol. 6 December
2006;113(5):616-625. 2005;46(11):2088-2099.
Hassaguerre M, Shah DC, Jas P, et al. Electrophysiological break- Mohamed U, Skanes AC, Gula LJ, etal. A novel pacing maneuver to
throughs from the left atrium to the pulmonary veins. Circulation. localize focal atrial tachycardia. J Cardiovasc Electrophys.
2000;102:2463-2465. 2007;18(1):1-6.

Case 7

Michel Haissaguerre

Case Summary and ablation of the AT (Fig.7.2). Entrainment manoeuvres


performed during the arrhythmia point towards a focal origin
by demonstrating long return cycle lengths around the mitral
A 67-year-old male with an 8-year history of persistent atrial
annulus, the roofline or the tricuspid annulus except at sites
fibrillation (AF) and three external direct current (DC) car-
adjacent to the focus (by definition, less than two segments).
dioversions was referred for ablation. The inital procedure
For localized re-entry, local activity spanning all of the AT
consisted of pulmonary vein (PV) isolation, extensive abla-
CL can be demonstrated at the area of earliest activity. For
tion based on fractionated electrograms in the left atrium
focal point AT, an area of early activity may exhibit mid-
(LA) and the right atrium (RA), and linear lesions at the roof
diastolic potential.
and the left mitral isthmus. AF was not terminated and an
These tracings illustrate a case with prior extensive LA
external cardioversion was required. Linear lesions (left
ablation including the left mitral isthmus line. Conduction
atrial roofline and mitral isthmus line) were completed dur-
around the mitral annulus was compatible with a perimitral
ing sinus rhythm.
circuit (lateral to septal anteriorly and septal-to-lateral poste-
Early recurrence of AT necessitated a second ablation pro-
riorly). The PPI was <30 ms at the anterior annulus, but
cedure one month later. A decapolar catheter was placed into
>30ms at the CS, which made a macro re-entrant peri-mitral
the CS and showed a proximal-to-distal activation pattern.
circuit unlikely and suggested a focal AT located lateral to
Aquadripolar cooled-tip ablation catheter was then inserted
the complete left mitral isthmus line. Mapping was per-
into the LA. AT cycle length was 260 ms with 10% irregular-
formed in this area where a site displaying most of the CL
ity. PV isolation was confirmed. Extensive anterior scar was
was found at the anterior mouth of the LAA (Fig.7.1); how-
noted, and anterior endocardial activation was difficult to
ever, the PPI was 100 ms at this site, suggesting it was far
assess, but it appeared to be septal to lateral. Post pacing
from the origin of the AT.
interval (PPI) at the RA was long (+130 ms), PPI at the pos-
A second site displaying most of the CL with an activa-
terior LA was long (+120 ms). PPI in the LAA was long (+60
tion gradient on the electrogram recording and a short PPI
ms), while PPI at the anterior LA was good (+10 ms). The
(10 ms) was mapped on the anterior LA (Fig.7.3). A three-
anterior part of the LAA showed almost continuous activity
dimensional reconstruction of this spot with activation
(Fig.7.1).
mapping confirmed that the activity spanned all the cycle
Can this site be considered for ablation?
lengths in a very localized region (Fig. 7.4). Ablation at
this site restored sinus rhythm, and a local double potential
was visualized after ablation (Fig. 7.5, arrows) in sinus
Case Discussion rhythm.
This case illustrates how useful the PPI can be in difficult
In the case of extensive scarring or ablation, mapping of AT cases. During entrainment, attention must be paid to avoid
can be challenging with the practical algorithm. In this situ- induction of AF at short CLs and one must be aware of the
ation, three methods can be used to facilitate identification possibility of conversion to another AT.

M. Haissaguerre
Department of Cardiology and Electrophysiology, CHU de Bordeaux,
Hospital Cardiologique de Haut Leveque, Avenue de Magellan,
Pessac 33604, France

A. Natale et al. (eds.), Cardiac Electrophysiology, 29


DOI: 10.1007/978-1-84996-390-9_7, Springer-Verlag London Limited 2011
30 M. Haissaguerre

I 55 56

II

V1

RFd

RFp

CS 1-2

CS 3-4

CS 5-6

CS 7-8

CS 9-10

Fig.7.1 Recordings of the radiofrequency RF ablation catheter and a decapolar catheter inserted into the coronary sinus (CS 12 to 910). Site of
high-voltage continuous electrograms displaying almost all the cycle length

PPI to assess number of segment(s) involved

2 segments >2 segments


FOCAL MACRO RE-ENTRY

EGM spanning all the Activation compatible with:


AT cycle length in the Perimitral macro re-entry
involved segment(s) Roof dependant
Mid diastolic potentials macro re-entry
Fig.7.2 Approach to atrial tachycardia in the Peritricuspid macro re-entry
context of atrial fibrillation (AF), in the case of More complex macro re-entry
extensive scar or ablation. See text for details
Case 7 31

I 29 30

II

V1

RFd

RFp

CS 1-2

CS 3-4

CS 5-6

CS 7-8

CS 9-10

Fig.7.3 Recordings of the RF ablation catheter and a decapolar catheter inserted into the coronary sinus (CS 12 to 910). Site of complex frac-
tionated electrograms with a gradient of activation between the proximal and distal bipoles (black arrows)
32 M. Haissaguerre

Fig.7.4 NavX (EnSite System,


St. Jude Medical, MN, USA)
activation map (antero-posterior
view) showing local activity
spanning all the cycle lengths in
a very localized region below the
left atrial appendage

27 28
I

II

V1

RFd

154 ms

CS 1-2

CS 3-4
Fig.7.5 Recordings of the RF
ablation catheter and a decapolar CS 5-6
catheter inserted into the
coronary sinus (CS 12 to 910).
CS 7-8
After restoration of sinus rhythm,
local double potential visualized
after ablation CS 9-10
Case 7 33

Bibliography Hassaguerre M, Hocini M, Sanders P, etal. Localized sources main-


taining atrial fibrillation organized by prior ablation. Circulation.
2006;113(5):616-625.
Hassaguerre M, Shah DC, Jas P, et al. Electrophysiological break- Mohamed U, Skanes AC, Gula LJ, etal. A novel pacing maneuver to
throughs from the left atrium to the pulmonary veins. Circulation. localize focal atrial tachycardia. J Cardiovasc Electrophys.
2000;102:2463-2465. 2007;18(1):1-6.
Haissaguerre M, Sanders P, Hocini M, etal. Catheter ablation of long- Sanders P, Hocini M, Jais P, etal. Characterization of focal atrial tachy-
lasting persistent atrial fibrillation: critical structures for termina- cardia using high-density mapping. J Am Coll Cardiol. 2005
tion. J Cardiovasc Electrophys. 2005;16(11):1125-1137. December 6;46(11):2088-2099.

Case 8

Michel Haissaguerre

Case Summary required external cardioversion to restore sinus rhythm.


During sinus rhythm, both linear lesions were assessed and
A 54-year-old female with persistent atrial fibrillation (AF) completed.
for 12 months, a 10-year history of AF, a dilated left atrium Four months later, the woman developed an AT and she
(LA) (54 mm diameter), and having failed five external car- therefore had a second ablation procedure. A decapolar cath-
dioversions and amiodarone, was referred for ablation. eter was placed into the coronary sinus (CS) and demon-
Baseline AF cycle length (CL) before ablation was 135 ms in strated a colliding activation pattern in the proximal CS. A
the left atrial appendage (LAA) and 132 ms in the right atrial quadripolar cooled-tip ablation catheter was then inserted
appendage (RAA). The first procedure consisted of PV isola- into the LA. AT cycle length was 271 ms with irregularity
tion, extensive ablation based on electrograms in the LA, and reaching 22% of the mean AT cycle length. Pulmonary vein
linear lesions at the roof and the left mitral isthmus. Then isolation (PVI) was confirmed, and the endocardial activa-
electrogram-based ablation in the RA resulted in a transient tion was determined by moving the ablation catheter sequen-
conversion to atrial tachycardia (AT). Early recurrence of AF tially in the anterior LA (Fig.8.1, panel A, low anterior; and

I A I B I C I D
II II II II
V1 V1 V1 V1

Abl Abl Abl


Abl

CS CS CS CS

B C

D
A

Fig.8.1 Location of different recordings in the anterior and posterior left atrium, high and low

M. Haissaguerre
Department of Cardiology and Electrophysiology, CHU de Bordeaux,
Hospital Cardiologique de Haut Leveque, Avenue de Magellan,
Pessac, 33604, France

A. Natale et al. (eds.), Cardiac Electrophysiology, 35


DOI: 10.1007/978-1-84996-390-9_8, Springer-Verlag London Limited 2011
36 M. Haissaguerre

Fig. 8.1, panel B, high anterior) and in the posterior LA 2. Macro re-entry is first investigated by assessing the
(Fig.8.1, panel C, high posterior; and Fig.8.1, panel D, low activation in the LA, in order to determine the likeli-
posterior). Activation was ascending in the anterior wall hood of perimitral, roof-dependent, or peritricuspid
(Fig. 8.1, panels A and B) and descending at the posterior circuit (entrainment maneuvers are used to confirm the
wall (Fig.8.1, panels C and D) with the activation sequence diagnosis)
covering all the AT cycle length. 3. In the case of the absence of macro re-entrant AT, focal
What inferences can be made from Fig.8.1 with relevance point/area AT is determined by tracking the earliest area
to the mechanism of AT, and what maneuver has to be done of activity (entrainment maneuvers can be used to evalu-
to confirm the diagnosis? ate the site of origin). In the case of localized re-entry,
long-duration fractionated potentials spanning most or all
of the CL are tracked. Otherwise, the earliest activity is
Case Discussion targeted
4. In the case of nonconsistency, more complex tachycardia
First, for the purposes of describing the approach to ATs aris- are investigated, or a change in AT (mechanical or due to
ing in the context of catheter ablation of AF, the following stimulation) is searched
definitions are employed: macro re-entry is defined as a cir-
In our case, the activation front was consistent with a roof-
cuit involving more than three atrial segments, usually greater
dependent macro-re-entry (from A to D): ascending the
than 2 cm in diameter and where more than 75% of the circuit
anterior wall (solid line, A to B) and descending the posterior
is mapped. Focal point tachycardia is defined as centrifugal
wall (dashed line, C to D). The mapped activity spanned the
activation originating from a discrete site and includes auto-
entire CL (potentially compatible with roof dependent AT),
maticity, triggered activity, and re-entrant mechanisms where
but slow conduction in the anteroseptal area (A) could be
<75% of the cycle can be mapped in the chamber of interest.
misleading.
Localized re-entry constitutes a circuit involving one or
Indeed, irregularity was >15% of the CL (inset ECG),
two adjacent segments, usually smaller than 2 cm in diameter
suggesting in fact focal AT. Furthermore, PPI at the posterior
and spanning more than 75% of the CL within the involved
wall was short (+20 ms), but the anterior PPI was very long
segment(s).
(+200 ms), ruling out macro re-entrant roof-dependant AT.
A diagnostic algorithm has been developed by our group
Because the roof line was previously blocked, it suggested a
to allow easy and accurate mapping of AT arising in the con-
focal origin from the high posterior LA. Mapping this area
text of prior AF ablation (Fig. 8.2). After confirmation of
close to the roof line revealed very low voltage local activity
PVI, it consists of
spanning most of the CL at three different spots very close to
1. Assessment of AT cycle length variability: if >15%, sug- each other (Fig. 8.3, POST 1 to 3). This voltage was only
gestive of focal AT and if <15%, not of discriminating 0.04 mV, which can be easily hidden in case of electrical
value (CL variability is calculated by dividing the CL noise. Ablation of a very low voltage continuous activity site
range by the mean CL averaged over 30 cycles) located in between the three described spots terminated the
tachycardia (Fig.8.4).
This case emphasizes the importance of localized
CL Irregularity re-entrant AT in the context of prior AF ablation.1 In this
Yes No
>15% context, 53% of AT are focal and localized re-entries are the
most frequent AT mechanism, representing 71% of focal
Focal Focal or macroreentry ATs (and 37% of total ATs). The preferential regions for
focal ATs are the PVLA junction, left septum, and the
mouth of the LAA. Focal AT are generally localized to sites
Map earliest region Activation compatible with: targeted during electrogram-based AF ablation. Injury or
Perimitral macro re-entry edema to the atrial tissue induced by RF applications could
Roof dependant macro re-entry
Peritricuspid macro re-entry generate the substrate for further arrhythmia by creating an
Localized re-entry anchoring point potentially able to maintain re-entry.
Focal point AT Therefore the mechanism of AT after AF ablation may be
No ~100% cycle different to that of spontaneous AT. The presence of pre-
Entrainment
existing LA scar (which may be idiopathic or related to
Yes underlying structural heart disease) may also result in local
Macro re-entry
slow conduction areas predisposing to re-entry.
Mapping and ablation of focal or macro re-entrant AT is a
Fig.8.2 Practical approach to atrial tachycardia (AT) in the context of crucial step in the AF ablation process, and often represents
atrial fibrillation (AF) ablation. See text for details the difference between procedural success and failure.
Case 8 37

Fig.8.3 Antero-posterior fluoroscopic


view of the left atrium (LA). Mapping of
three close posterior spots (post 13). Local I
activity (within red rectangles) spanning II
the entire CL (pink bars)
V1

Post 1

Post 2

Post 3

CS

A
I

II

V1
0.05 mV

Abl

CS

18
19 20 21 B
I

II

V1

CS 1-2

CS 3-4

CS 5-6

CS 7-8

CS 9-10

Fig.8.4 Antero-posterior fluoroscopic view of the left atrium (LA) with a (<0.05 mV) signal spanning all of the CL. Panel B: Change of the atrial
decapolar catheter placed into the coronary sinus (CS). Panel A: Ablation tachycardia (AT) (black star) during radiofrequency application at this pos-
catheter (Abl) placed in the posterior LA and recording a very low voltage terior wall
38 M. Haissaguerre

Bibliography Hassaguerre M, Hocini M, Sanders P, etal. Localized sources main-


taining atrial fibrillation organized by prior ablation. Circulation.
2006;113(5):616-625.
Hassaguerre M, Shah DC, Jas P, et al. Electrophysiological break- Mohamed U, Skanes AC, Gula LJ, etal. A novel pacing maneuver to
throughs from the left atrium to the pulmonary veins. Circulation. localize focal atrial tachycardia. J Cardiovasc Electrophys.
2000;102:2463-2465. 2007;18(1):1-6.
Haissaguerre M, Sanders P, Hocini M, etal. Catheter ablation of long- Sanders P, Hocini M, Jais P, etal. Characterization of focal atrial tachy-
lasting persistent atrial fibrillation: critical structures for termina- cardia using high-density mapping. J Am Coll Cardiol. 2005
tion. J Cardiovasc Electrophys. 2005;16(11):1125-1137. December 6;46(11):2088-2099.
Case 9

Alberto Diaz, Dimpi Patel, William R. Lewis, and Andrea Natale

Case Summary Following ablation, isoproterenol was given up to a dose of


20 mcg/kg/min. Following that, the PVs were mapped again to
The patient is a 55-year-old male with a history of hyperten- check for electrical isolation. The EGMs below were observed
sion, left ventricular hypertrophy, and paroxysmal atrial fibril- placing the Lasso catheter in the RSPV and the LSPV.
lation (AF) since 2001; both Flecainide and Sotalol had failed. Based on the recordings, what would you consider?
He underwent AF ablation in August 2007. He was placed
a) Ablate the left superior PV
on Sotalol after radiofrequency ablation (RFA). A month
b) Ablate the right superior PV
later he had recurrent palpitations that required electrical car-
c) Ablte both PVS
dioversion. His arrhythmia was exacerbated after trying to
d) No additional ablation
come off the Sotalol. He was evaluated for a repeat ablation.
His echocardiogram showed a normal left ventricular ejec-
tion fraction (LVEF), normal left atrial size, no valvular
abnormalities. His adenosine stress test showed no evidence Case Discussion
of ischemia with a LVEF of 74%.
He was brought to the EP lab. A catheter was placed in the Both left and right PVs showed dissociated firing, which
CS via the internal jugular vein; two transseptal sheaths were confirms both entry and exit blocks. During isoproternol
used to advance the ablation catheter and the Lasso catheter. infusion, AF started inside the right upper PV without
The right and left superior pulmonary veins (RSPV and affecting the atrial chambers, which remained in sinus
LSPV) appeared to have recovered and were re-ablated rhythm. This confirms that there is no reason for additional
(Figs.9.1 and 9.2.). ablation.

A. Diaz ()
Heart and Vascular Department, Metro Health Medical Center,
2500 Metro Health Drive, Cleveland, OH, 44109
e-mail: adiaz@nohc.com
D. Patel
St. Davids, Texas Cardiac Arrhythmia Institute, 1015 E. 32nd St,
#516, OH Austin, TX 78705
W.R. Lewis
Clinical Cardiology, Case Western Reserve University, Heart and
Vascular Center, MetroHealth Medical Center, 2500 MetroHealth
Drive, Suite H322, Cleveland, OH 44109
A. Natale
Texas Cardiac Arrhythmia Institute, St. Davids Medical Center,
1015 East 32nd Street, Suite 516, Austin, TX 78705

A. Natale et al. (eds.), Cardiac Electrophysiology, 39


DOI: 10.1007/978-1-84996-390-9_9, Springer-Verlag London Limited 2011
40 A. Diaz et al.

Fig.9.1 Lasso in the left superior pulmonary vein (LSPV)

Fig.9.2 Lasso in the right superior pulmonary vein (RSPV)


Case 10

Antonio Rossillo, Sakis Themistoclakis, Aldo Bonso,


Andrea Corrado, andAntonio Raviele

Case Summary After examining Figs. 10.110.3, what is the likely


diagnosis?
The patient is a 13-year-old male with prior 8-year
historyof palpitations during exercise. No structural heart
disease. The transesophageal EP study easily demonstrated Case Discussion
the induction of sustained SVT during minimal effort
(AA300ms). This case demonstrates a narrow complex tachycardia that is
The echocardiography evaluation was normal and 24 not 1:1; with A > V. This excludes AVRT and makes AVNRT
h-ECG Holter documented two episodes of SVT which less likely unless there is block below the lower common
began after a physiologic increase in heart rate. pathway (see Figs.10.410.8).

Fig.10.1 12-lead resting ECG (paper speed 25 mm/s) showing normal sinus rhythm with a ventricular rate of 66 bpm and a normal PR interval
(136 ms) and a QRS width of 100 ms

A. Rossillo (), S. Themistoclakis, A. Bonso,


A. Corrado, and A. Raviele
Ospedale dellAngelo, Mestre, Venice, Italy

A. Natale et al. (eds.), Cardiac Electrophysiology, 41


DOI: 10.1007/978-1-84996-390-9_10, Springer-Verlag London Limited 2011
42 A. Rossillo et al.

Fig.10.2 Panel A: 12-lead ECG taken during atrial tachycardia (paper speed 25 mm/s). Panel B: a detail of lead D2, aVL, and V1
Case 10 43

Fig.10.3 Intracardiac record-


ings taken during the electro-
physiology study (paper speed
100 mm/s) revealing SVT with a
cycle length of 300 ms. Four
surface ECG leads (I, aVF, V1,
V6), three bipolar recordings
from the His bundle region
(distal= HIS D, intermediate =
HIS I, and proximal = HIS P),
two bipolar recordings from the
coronary sinus (CS prox =
proximal coronary sinus and CS
dist = distal coronary sinus), and
the unipolar (MC U-CATH) and
the distal bipolar recording of the
mapping catheter (MC D)

Fig.10.4 The electroanatomical mapping of the tachycardia showed


the origin of the arrhythmia coming from the His bundle region (the
red area in the LAO view). The orange tags show the area where it
was possible to record the His bundle potentials
44 A. Rossillo et al.

Fig.10.5 Electroanatomical mapping with CARTO: detailed remap of the area of interest; the orange tag shows the His bundle potential and the
blue tag shows the site of earliest activation during tachycardia

Fig.10.6 Electroanatomical mapping with CARTO: detailed


remap of the area of interest; the orange tag reflects the His bundle
potential, the blue tag shows the site of earliest activation during
tachycardia, and the red tag shows where a single RF lesion is
applied
Case 10 45

Fig.10.7 Intracardiac record-


ings taken at the ablation site
(paper speed 100 mm/s). Same
display as that shown in
Fig.10.3. A atrium, Vventricle

Fig.10.8 Intracardiac
recordings taken at the ablation
site (paper speed 100 mm/s)
during RF application. A single
RF application of 30 s with
titration of the power starting
from 30 W resulted in sinus
rhythm restoration. RF energy
was stopped as soon as the His
bundle potential appeared on the
ablation catheter. Same display
as that shown in Fig.10.3. A
atrium, V ventricle, H His bundle
46 A. Rossillo et al.

Bibliography Lai LP, Lin GL, Chen TF, etal. Clinical electrophysiological character-
istics and radiofrequency catheter ablation of atrial tachicardia near
the apex of Kochs triangle. PACE. 1998;21:367-374.
Chen SA, Chiang CE, Yang CJ, Cheng CC, et al. Sustained atrialtachy- Lesh MD, Van Hare GF, Epstein LM, etal. Radiofrequency Catheter
cardia in adult patients. Electrophysiological characteristics, phar- ablation of atrial arrhythmias: results and mechanisms. Circulation.
macological response, possible mechanisms, and effects of 1994;89:1074-1089.
radiofrequency ablation. Circulation. 1994;90:1262-1278. Pappone C, Stabile G, De Simone A, et al. Role of catheter-induced
Frey B, Kreiner G, Gwechenberger M, Gossinger H. Ablation of atrial mechanical trauma in localisation of target sites of radiofrequency
tachycardia originating from the vicinity of the atrioventricular ablation of automatic atrial tachycardia. J Am Coll Cardiol.
node: significance of mapping both sides of the interatrial septum. 1996;27:1090-1097.
J Am Coll Cardiol. 2001;38:394-400. Poty H, Saudi N, Haissaguerre M, etal. Radiofrequency catheter abla-
Kalman JM, Olgin JE, Karch MR, etal. Cristal tachycardias: origin of tion of atrial tachycardias. Am Heart J. 1996;131:481-489.
right atrial tachycardias from the crista terminalis identified by intrac- Tang CW, Scheinmann MM, Van Hare GF, etal. Use of P wave configu-
ardiac echocardiography. J Am Coll Cardiol. 1998;31:451-459. ration during atrial tachycardia to predict site of origin. J Am Coll
Kay GN, Chong F, Epstein AE, Dailey SM, Plumb VJ. Radiofrequency Cardiol. 1995;26:1315-1324.
ablation for treatment of primary atrial tachycardias. J AM Coll Tracy CM, Swartz JF, Fletcher RD, etal. Radiofrequency catheter abla-
Cardiol. 1993;21:901-909. tion of ectopic atrial tachycardia using paced activation sequence
Knight BP, Zivin A, Souza J, etal. A technique for the rapid diagnosis mapping. J Am Coll Cardiol. 1993;21:910-917.
of atrial tachycardia in the electrophysiology laboratory. J Am Coll
Cardiol. 1999;33:775-781.
Case 11

Yash Y. Lokhandwala, Anoop K. Gupta, and Ranjan K. Thakur

Case Summary Case Discussion

A 17-year-old male with a normal heart has an electrocardio- The first two complexes show earliest A in CS 1-2 and in
gram (ECG) showing intermittent pre-excitation suggestive RFD; after this the coronary sinus (CS) activation reverses,
of left free-wall accessory pathway (AP) and supraventricu- but the A remains earliest in Radiofrequency Distal (RFD),
lar tachycardia (SVT). suggesting that the orthodromic atrioventricular reentrant
The induced tachycardia is a narrow QRS regular tachy- tachycardia (AVRT) is mediated by a right free-wall AP with
cardia (Fig.11.1) with ST depression in inferior leads. a bystander left-lateral AP.
The patient is taken to the electrophysiological (EP) lab. Ablation of the right-sided AP resulted in success.
In Fig.11.2 the ablation catheter is positioned at the lateral
tricuspid annulus (9 oclock). Where is the site of the suc-
cessful ablation?

Y.Y. Lokhandwala ()
KEM Hospital, Parel, Mumbai, India
A.K. Gupta
Apollo Hospital, Ahmedabad, India
R.K. Thakur
Thoracic and Cardiovascular Institute Sparrow Health System,
Michigan State University 405 West Greenlawn, Suite 400,
Lansing, MI 48910

A. Natale et al. (eds.), Cardiac Electrophysiology, 47


DOI: 10.1007/978-1-84996-390-9_11, Springer-Verlag London Limited 2011
48 Y.Y. Lokhandwala et al.

2008021

II

III

AVR

AVL

AVF

V1

V2

V3

V4

V5

V6

10 mm/mV 25 mm/s

Fig.11.1 The electrocardiogram (ECG) shows induced tachycardia


Case 11 49

2008021
I
AVF
V1
V6

CS910

CS78

CS56

CS34

CS12

HISD

HISP

RFD

RFP

10 mm/mV 200 mm/s

Fig.11.2 Intracardiac recordings. CS 1-2 is distal. The ablation catheter (RFD) is placed along the lateral tricuspid annulus

Case 12

Eric Buch, Shiro Nakahara, Marmar Vaseghi, Noel G. Boyle,


and Kalyanam Shivkumar

Case Summary performed during flutter demonstrated a long post-pacing


interval of 442 ms. Noteworthy was that during entrainment
pacing from the high right atrial (HRA) catheter, 2:1 block of
A 59-year-old male with ischemic cardiomyopathy who
right-to-left atrial conduction was observed without termi-
underwent heart transplantation three years earlier was
nating the tachycardia (Fig. 12.1, panel C). Where is the
repeatedly hospitalized for atrial flutter (AFL) (Fig. 12.1,
likely circuit of this flutter?
panel A) refractory to rate control and antiarrhythmic drugs.
Left ventricular ejection fraction was normal by echocar-
diography. He was referred for electrophysiology study and
ablation. Case Discussion
Intracardiac recordings showed AFL with tachycardia
cycle length 268 ms and earliest left atrial activation from the Based on Fig.12.1, which shows the left atrium dissociated
distal coronary sinus (CS), consistent with a left atrial circuit from the flutter circuit during tachycardia, left AFL was
(Fig. 12.1, panel B). Entrainment pacing from the mid-CS excluded from the differential diagnosis. Electroanatomic

A B C

Fig.12.1 Cardiac tracings


recorded during the clinical atrial
flutter (AFL). Panel A: Surface
12-lead ECG recorded during
AFL. Panel B: Intracardiac
tracings and ECG leads II, V2,
and V5 showing earliest left
atrial activation in the distal CS.
Panel C: Intracardiac tracings
and ECG leads II, V2, and V5
showing entrainment pacing
from the HRA, and 2:1 block of
the right-to-left atrial conduction.
CS coronary sinus, HRA high
right atrium, CS (HB) d,p the
distal and proximal electrode
pairs of the coronary sinus (His
bundle) catheter

E. Buch (*), S. Nakahara, M. Vaseghi,


N.G. Boyle, and K. Shivkumar
UCLA Cardiac Arrhythmia Center,
David Geffen School of Medicine at UCLA,
10833 Le Conte Ave, A2-237 CHS,
Los Angeles, CA 90095-1679
e-mail: ebuch@mednet.ucla.edu

A. Natale et al. (eds.), Cardiac Electrophysiology, 51


DOI: 10.1007/978-1-84996-390-9_12, Springer-Verlag London Limited 2011
52 E. Buch et al.

mapping of the right atrium (RA) showed large areas of low- occurs over multiple septal pathways, including Bachmanns
voltage scar (colored gray in Fig.12.2). Activation mapping bundle, the foramen ovale, and CS.1 In this case, due to scar-
revealed a macro-reentrant AT utilizing a channel in the lat- ring over a large part of the RA, conduction through
eral wall of the RA (Fig.12.2). A line of ablation along the Bachmanns bundle was more rapid than the lower septal
lateral RA wall, shown in Fig.12.2, successfully terminated sites, resulting in distal to proximal of LA activation as seen
the tachycardia. It is known that conduction from RA to LA in the CS catheter.

200 ms

Fig.12.2 Electroanatomic map 0 ms


of the right atrium during the
clinical AFL. Red areas indicate
earliest endocardial activation;
200 ms
orange, yellow, green, blue, and Conduction via
purple indicate progressively Bachmanns bundle
delayed activation. The scar is
shown in gray (<0.1 mV). White
tags indicate the ablation sites.
The black solid arrows show the
activation sequence within the
tachycardia circuit, and the black
dotted arrows show bystander
activation of right atrium as well
as left atrium via Bachmanns
bundle. TA tricuspid annulus, RA
right atrium, AFL atrial flutter CS catheter

Reference

1. Roithinger FX, Cheng J, SippensGroenewegen A, etal. Use of elec-


troanatomic mapping to delineate transseptal atrial conduction in
humans. Circulation. 1999;100:1791-1797.
Case 13

Bradley P. Knight

Case Summary is VA linking. This observation strongly suggests that the


atrial depolarization is dependent on the ventricular depolar-
ization, meaning that the atrial rhythm is dependent on VA
A 54-year-old female with mental retardation presented to
conduction. This should not be present with an atrial tachy-
the emergency department with a recurrent supraventricular
cardia, because the first beat of the tachycardia should have
tachycardia (SVT) at 180 bpm. An electrophysiology (EP)
no relationship to the prior QRS, which was a result of con-
study was performed. The baseline rhythm was sinus rhythm
duction of the atrial extrastimulus to the ventricle.
without ventricular preexcitation. Her clinical tachycardia
This observation can be useful, but it is important to rec-
was induced when an atrial extrastimulus was delivered with
ognize that it is not absolute and should be reproducible.
a coupling interval of 340 ms during a drive train with a cycle
There is always a small chance that the first VA interval will
length (CL) of 550 ms (Fig.13.1). There was a one-to-one
be the same or similar as that during tachycardia by chance
relationship between the atrium and the ventricle. The ven-
alone. Indeed, in this case the tachycardia was actually a left
triculoatrial (VA) interval associated with the first beat of the
atrial septal tachycardia. Figure 13.2 shows a spontaneous
tachycardia is nearly identical to the remaining VA intervals.
initiation of the same tachycardia that is shown in Fig.13.1
Can atrial tachycardia be excluded?
by a premature atrial beat during sinus rhythm. This event
shows that at times the tachycardia does not have a constant
VA interval and occasionally displays Wenckebach AV block.
Case Discussion

When a tachycardia with a constant VA interval is induced


with an atrial premature beat and the first VA interval is the
same as the subsequent VA intervals, it can be said that there

B.P. Knight
Division of Cardiology, Northwestern Medical Center, 676 N.
St. Clair, Suite 600, Chicago, IL 60611

A. Natale et al. (eds.), Cardiac Electrophysiology, 53


DOI: 10.1007/978-1-84996-390-9_13, Springer-Verlag London Limited 2011
54 B.P. Knight

I
ARRHYTHMIA INDUCTION RA_d S1 = 550 S2 = 340
PRINT REVIEW FIT

II 190 360 190 550 550 420 360 350 380 360

V1

V6

HRA p 550 550 340 1520 360

ABL d

ABL m

ABL p

RVA p 540 550 550 400 370 730 360

Stim 550 550 550 340

University of Chicago

Fig. 13.1 Induction of a supraventricular tachycardia with a single atrium (HRA), the ablation catheter (ABL) positioned at the His bundle
atrial extrastimulus. Shown are surface electrograms from leads I, II, location, and the right ventricular apex (RVA)
V1, and V6, and the intracardiac electrograms from the high right
Case 13 55

PRINT REVIEW FIT


I

II 610 610 610 440 410 400 410 490

V1

V6

HRA p 610 610 300 790 370 360 370

ABL d

ABL m

ABL p

RVA p 620 610 600 430 420 390 400

Stim

University of Chicago

Fig.13.2 A different induction of the same supraventricular tachycardia shown in figure 13.1 showing a variable ventriculo-atrial relationship.
The format is the same as in Figure 13.1

Case 14

M. Eyman Mortada, Jasbir S. Sra, and Masood Akhtar

Case Summary After pacing from the right ventricle, the QRS narrows
down to a normal duration (72 ms). The electrical alternans
remains the same. However, the CL of the tachycardia accel-
A 27-year-old male complained of multiple episodes of
erates from 259 to 240 ms. This indicates the presence of an
palpitations. His echocardiogram (ECG) and baseline ECG
accessory pathway (AP) ipsilateral to the bundle branch
were within normal limits. His palpitations were captured
block (right), which is part of the electrical circuit of the
with the 12-lead ECG shown in Fig.14.1.
tachycardia. Hence the tachycardia is due to orthodromic
In the electrophysiology (EP) lab, the same tachycardia
AV reentry with aberrancy. The aberrancy disappears due to
was induced. After pacing from the right ventricle during the
early activation of the right bundle by pacing from the right
tachycardia shown in Fig.14.1, the rhythm changed to the
ventricle, leading to a break-in-the-link phenomenon and
ECG shown in Fig.14.2.
early recovery of the right bundle, in readiness for antero-
During the ventricular programmed stimulation, the results
grade activation from the next beat, causing narrow QRS
shown in Fig.14.3 were observed. The tachycardia was initiated
morphology.1
in the lab with the atrial programmed stimulation shown in
The narrow complex tachycardia (Fig. 14.2) has an incom-
Fig.14.4. Two ventricular extrastimuli were performed from the
plete RBBB with QRS at 80 ms and a rate of 240 beats per
right ventricular apex during the tachycardia, showing the results
minute. The P-wave is in the ST-T segment, suggestive of
in Fig. 14.5. Three ventricular extrastimuli were performed
short RP tachycardia. The most likely diagnosis is either
during the tachycardia, which showed the results in Fig.14.6.
AP-mediated tachycardia or atrial tachycardia. Though, one
What can be excluded? What is the diagnosis?
cannot exclude AV-nodal reentry tachycardia or para-Hisian
ventricular tachycardia with one-to-one VA conduction
Case Discussion but this phenomenon is very rare.
During ventricular pacing (Fig. 14.3), the VA conduction
was retrograde over the AV node during the first two beats.
The initial presentation of this healthy young male is a wide
When a ventricular extrastimuli was introduced (third beat),
complex tachycardia with right bundle branch block
the VA conduction continued retrogradely over the AV node
(RBBB) morphology and right axis deviation. The duration
with a longer VA duration, due to the decremental property
of the QRS is 145 ms. It is difficult to assess if there are
of the AV node rather than to a jumping phenomenon (dual
P-waves. If the small deflection between the two QRS com-
physiology). The fourth ventricular beat is either a ventricu-
plexes is a P-wave, there is an AV association. Interestingly,
lar repetitive response or a bundle branch reentry beat, which
there is an obvious electrical alternans present, occurring
does not result in retrograde atrial activation. The absence of
simultaneously with a CL alternans. These findings favor
VA conduction over an AP during ventricular pacing and at
supraventricular tachycardia (SVT) rather than ventricular
the time of AV nodal block makes it less likely that
tachycardia (VT) or atrial flutter (AFL). However, we
AP-mediated tachycardia is the cause but it does not
cannot exclude any of the possibilities.
exclude the possibility.
Some APs have multiple fibers at their ventricular inser-
tion sites where they might interact during the electrical
propagation, leading to block and prevention of retrograde
M.E. Mortada (*), J.S. Sra, and M. Akhtar conduction over the AP.2, 3 The possibility that this VT is
Department of Electrophysiology, Aurora Cardiovascular Services,
para-Hisian in origin is less likely, due to the weak retrograde
Aurora Sinai/Aurora St. Lukes Medical Centers, University of Wisconsin
School of Medicine and Public Health, 2801 W. Kinnickinnic River conduction over the AV node. Therefore, none of the findings
Parkway, #777, Milwaukee, WI 53215 in Fig. 14.3 help define the cause of the tachycardia.

A. Natale et al. (eds.), Cardiac Electrophysiology, 57


DOI: 10.1007/978-1-84996-390-9_14, Springer-Verlag London Limited 2011
58 M.E. Mortada et al.

Fig.14.1 The patients palpitations were captured with this 12-lead ECG

Fig.14.2 The same tachycardia was induced in the electrophysiology lab. After pacing from the right ventricle during the tachycardia, the
patients rhythm changed, as shown in this ECG
Case 14 59

Fig. 14.3 Tracing of patients response to ventricular programmed 9,10) to distal coronary sinus (CS 1, 2), His bundle recording and right
stimulation with one ventricular extrastimuli. The recording in this fig- ventricular recording. HRA: high right atrium; CS: coronary sinus;
ure and other figures are (from top to bottom): surface ECG with leads HISp: proximal His, HISm: middle His; HISd, distal His, RVa: apical
I, II and V1, high right atrial recording, proximal coronary sinus (CS right ventricle

Fig.14.4 Tracing demonstrating the initiation of the tachycardia with atrial programmed stimulation
60 M.E. Mortada et al.

Fig.14.5 Two ventricular extrastimuli during the tachycardia

Fig.14.6 Three ventricular extrastimuli during the tachycardia


Case 14 61

One extrastimuli from the high right atrium (HRA) AP-mediated tachycardia cannot be excluded, because
induced tachycardia with right bundle branch block mor- the distance from the right ventricular apex to the assumed
phology and right axis deviation (Fig. 14.4). The activation AP, which should be on the left ventricular free wall in
sequence of the tachycardia is: left atrium (LA) to right this scenario, far from the tachycardia circuit. Therefore,
atrium (RA), then His, followed by ventricular activation pacing with one or two extrastimuli, from the right ven-
(VA). The most likely cause for this rhythm is left atrial tricular apex, may not enter and entrain the tachycardia
tachycardia or orthodromic AV reentry tachycardia (AVRT) circuit.
via the left free-wall AP. Three ventricular extrastimuli were performed at cycle
Antidromic AVRT, in which the electrical activation lengths of 210 ms to entrain the tachycardia (cycle length,
should be V-H-A, is ruled out in this case due to the sequence 245 ms), which showed VAV response, thus atrial tachycar-
of the electrical activation (A-H-V) and the fact that the dia was excluded. The post-pacing interval was 330 ms, thus
tachycardia is initiated by antegrade activation over the AV the pacing site is near to the circuit and the diagnosis is
node. Induction of VT via extrastimuli at the high right orthodromic AVRT via the left ventricular free-wall AP.
atrium (HRA) can be seen in bundle branch reentry or fas-
cicular ventricular tachycardia.
Atrial eccentric activation usually does not rule out
AVNRT. Six to eight percent of AVNRTs display eccentric
atrial activation.4,5 However, the eccentric activation occurs References
with earliest atrial activation coming from an area not more
than 4 mm from the AV node. Hence, AVNRT can be ruled 1. Lehmann MH, Denker S, Mahmud R, Addas A, Akhtar M. Linking:
out in this case because the earliest atrial activation was at a dynamic electrophysiologic phenomenon in macroreentry cir-
cuits. Circulation 1985;71:254-265.
CS 1-2 (left atrial free wall), which is more than 4 mm away 2. Rordorf R, Vicentini A, Petracci B, Landolina M. Intermittent rate-
from the AV node. dependent retrograde conduction over a concealed atrioventricular
A study of the ventricular extrastimuli present during accessory pathway: what is the mechanism? Europace 2008;Dec
supraventricular tachycardia (SVT) at the time of His refrac- 3(epub ahead of print).
3. Bai R, Tritto M, Di Biase L, Salerno-Uriarte JA. Pacing site and
toriness is sometimes helpful in defining the cause of the bradycardia dependent retrograde conduction block over an atrio-
tachycardia. In this case (Fig. 14.5), two extrastimuli are ventricular accessory pathway. Europace 2006;8:438-442.
seen. The first was late and fused with the QRS of the tachy- 4. Ong MG, Lee PC, Tai CT, Lin YJ, Hsieh MH, Chen YJ, Lee KT,
cardia. The second came a few milliseconds prior to the His Tsao HM, Kuo JY, Chang SL, Chen SA. The electrophysiologic
characteristics of atrioventricular nodal reentry tachycardia with
activation (His refractory period). If the next atrial activation eccentric retrograde activation. Int J Cardiol 2007; 120:115-122.
occurred earlier, the presence of AP could be confirmed. 5. Katritsis DG, Ellenbogen KA. Eccentric retrograde atrial activation
However, the atrial activation did not change, as occurs in in atrioventricular nodal reentrant tachycardia. Heart Rhythm
both atrial tachycardia and AVNRT. 2005;2:1394-1395.

Case 15

Eric Buch, Shiro Nakahara, Marmar Vaseghi, Noel G. Boyle,


and Kalyanam Shivkumar

Case Summary Case Discussion

A 75-year-old male with history of orthotopic heart trans- Electroanatomic mapping demonstrated recurrent cavotricuspid
plantation with a cavo-caval anastomosis initially developed isthmus dependent atrial flutter (Fig.15.1) and an ablation line
atrial flutter 13 years post-transplantation with an episode of from the inferior vena cava to the tricuspid valve terminated the
mild rejection. He was successfully treated for rejection and tachycardia. He has had no recurrence in 3 years of follow-up.
a right atrial cavotricuspid isthmus dependent atrial flutter Atrial flutter is the most common arrhythmia in post-cardiac
was ablated. Two years later, presented with palpitations and transplant patients. Of all supraventricular tachycardias occur-
shortness of breath and was found to have recurrent atrial ring in stable transplant patients without evidence of rejection
flutter. Cardiac catheterization and biopsy did not show evi- or vasculopathy, cavotricuspid isthmus dependent atrial flutter
dence of transplant coronary artery vasculpathy or acute is most common. Furthermore, the type of anastomosis at the
rejection. What is the most likely mechanism of atrial flutter time of transplant, whether cavo-caval or atrio-atrial, does not
in this patient? affect the incidence of atrial flutter in these patients.

E. Buch (*), S. Nakahara, M. Vaseghi,


N.G. Boyle, and K. Shivkumar
UCLA Cardiac Arrhythmia Center,
David Geffen School of Medicine at UCLA,
10833 Le Conte Ave, A2-237 CHS,
Los Angeles, CA 90095-1679
e-mail: ebuch@mednet.ucla.edu

A. Natale et al. (eds.), Cardiac Electrophysiology, 63


DOI: 10.1007/978-1-84996-390-9_15, Springer-Verlag London Limited 2011
64 E. Buch et al.

Fig.15.1 (Panel A) Activation map: this patient, despite a cavo-caval placed in the right atrium with bipole D10 placed along the lateral wall
anastomosis, developed atrial flutter. The activation map demonstrated and D1 along the interatrial septum, showing propagation from D1 to
a right atrial flutter with the wave-front propagating superiorly along D10 in the right atrium. D duodecapolar, LAO left atrium, RAO right
the interatrial septum, around the annulus, and inferiorly along the lat- atrium, CS coronary sinus
eral wall of the right atrium. (Panel B): A duodecapolar catheter was
Case 16

Antonio Rossillo, Sakis Themistoclakis, Aldo Bonso, Andrea Corrado,


and Antonio Raviele

Case Summary Case Discussion

The patient is a 27-year-old women with a history of palpita- This patient has a normal 12-lead ECG. The induced narrow
tions. No heart disease was documented. Event recorder complex tachycardia is regular. The possibilities include
documented sustained SVT. Examine Figs.16.116.3. What AVNRT, AT, and AVRT. The atrial activation sequence dur-
are the likely possibilities? ing SVT is eccentric indicating either a left-sided AT or a
left-sided AP. In this case a left-sided AP was successfully
ablated (Figs.16.416.6).

Fig.16.1 Twelve-lead resting ECG (paper speed 25 mm/s) showing normal sinus rhythm

A. Rossillo (*), S. Themistoclakis, A. Bonso, A. Corrado


and A. Raviele
Ospedale dellAngelo, Mestre, Venice, Italy

A. Natale et al. (eds.), Cardiac Electrophysiology, 65


DOI: 10.1007/978-1-84996-390-9_16, Springer-Verlag London Limited 2011
66 A. Rossillo et al.

Fig.16.2 Twelve-lead ECG taken during SVT (paper speed 25 mm/s)

Fig.16.3 Intracardiac recordings taken during the electrophysiology region (HIS PROX, MED, and DIST), five bipolar recordings from the
study (paper speed 100 mm/s). Five surface ECG leads (II, III, aVF, V1, coronary sinus (CS), and a bipolar recording from the distal mapping
V6), two bipolar recordings from the pacing catheter placed in right catheter (MAP D). AVRT (cycle length 408 ms) with earliest activation
atrium or ventricle (HRA), three bipolar recordings from the His bundle on CS 1-2. A atrium, V ventricle, H His
Case 16 67

Fig.16.4 Intracardiac recordings taken during RF energy delivery (paper speed 100 mm/s). Same display as shown in Fig.16.3. The red arrow
showed termination of the tachycardia. A atrium, V ventricle, H His

Fig.16.5 Intracardiac recordings taken during RF energy delivery (paper speed 12.5 mm/s). Same display as shown in Fig.16.3. The red arrows
point at the restoration of sinus rhythm after 5 s of RF delivery
68 A. Rossillo et al.

Fig.16.6 Intracardiac recordings (paper speed 100 mm/s) taken during programmed ventricular stimulation showing the absence of VA conduc-
tion (paper speed 100 mm/s). Same display as shown in Fig.16.3. A atrium, V ventricle

Bibliography Jackman WM, Wang XZ, Friday KJ, etal. Catheter ablation of acces-
sory atrioventricular pathways (Wolff-Parkinson-White syndrome)
by radiofrequency current. N Engl J Med. 1991;324:1605-1611.
Calkins H, Langberg J, Sousa J, etal. Radiofrequency catheter ablation Scheinman MM, Wang YS, Van Hare GF, Lesh MD. Electrocardio
of accessory atrioventricular connections in 250 patients. graphic and electrophysiologic characteristics of anterior, midsep-
Abbreviated therapeutic approach to Wolff-Parkinson-White syn- tal and right anterior free wall accessory pathways. J Am Coll
drome. Circulation. 1992;85:1337-1346. Cardiol. 1992;20:1220-1229.
Hassaguerre M, Dartigues JF, Warin JF, Le Metayer P, Montserrat P,
Salamon R. Electrogram patterns predictive of successful catheter
ablation of accessory pathways. Value of unipolar recording mode.
Circulation. 1991;84:188-202.
Case 17

Bradley P. Knight

Case Summary
A 14-year-old male underwent an electrophysiology proce-
dure for recurrent left atrial tachycardia (AT). A prior attempt
at ablation was unsuccessful. After transeptal catheteriza-
tion, an electroanatomic map of the left atrium (LA) and left
atrial appendage (LAA) was created during AT. The mecha-
nism of the tachycardia did not appear to be macroreentrant.
The origin of the tachycardia appeared to be the left atrial
appendage. However, the electroanatomic map showed a
relatively broad area of early activation and failed to identify
a clear focus of origin. What additional strategy could be
used to increase the likelihood of successful ablation? Fig.17.1 Fluoroscopic view during ablation of a left atrial appendage
tachycardia. The view is left anterior oblique. A coronary sinus elec-
trode catheter can be seen placed from a superior approach. An ablation
catheter and the 20-electrode circular mapping catheter positioned in
the appendage, placed via double transseptal catheterization, can also
Case Discussion be seen

A multielectrode circular mapping catheter, which is usually


used to map pulmonary vein (PV) ostia, was subsequently complex tachycardias. However, it is important to recog-
placed in the LAA (Fig. 17.1). Using the bipolar signals from nize the limitations of such systems, and to occasionally
this catheter, the LAA was explored until the earliest activa- confirm the findings using conventional mapping tech-
tion site was identified. Figure17.2 shows the earliest atrial niques. In this case, the complex structure of the LAA and
activation recorded from bipole 1314. The ablation catheter its trabeculations proved to be a challenge for the electro-
was then positioned adjacent to the earliest bipole and radiof- anatomic contact mapping system to reconstruct the cham-
requency current was delivered. The tachycardia terminated ber accurately enough to provide a clear ablation target. The
and was noninducible. circular mapping catheter may have also distended the
Advanced computerized three-dimensional (3D) map- appendage and moved trabeculations that may have inter-
ping systems have greatly improved the ability to tackle fered with localization.

B.P. Knight
Division of Cardiology, Northwestern Medical Center,
676 N. St. Clair, Suite 600, Chicago, IL 60611

A. Natale et al. (eds.), Cardiac Electrophysiology, 69


DOI: 10.1007/978-1-84996-390-9_17, Springer-Verlag London Limited 2011
70 B.P. Knight

Fig. 17.2 This tracing was recorded during ablation of a left atrial 20-electrode circular mapping catheter positioned in the appendage, and
appendage tachycardia. Shown are the surface tracings from leads I and the octapolar electrode catheter placed in the coronary sinus. Note termi-
V5, and the bipolar intracardiac electrograms from the ablation catheter, nation of the atrial tachycardia during delivery of radiofrequency current
Case 18

Antonio Rossillo, Sakis Themistoclakis, Aldo Bonso, Andrea Corrado,


and Antonio Raviele

Case Summary Case Discussion

The patient is a 73-year-old female with a long history of Although the patient is elderly, the ECG shows a regular nar-
palpitations and hypertension. Echo reveals normal LVEF row complex tachycardia. Atrial tachycardia, AVNRT, and
with an LA diameter of 38 mm. The transesophageal EP AVRT should all be considered. In this case, no visible P waves
study revealed SVT, which was induced with minimum are seen during the SVT. This suggests either AVNRT or AT
effort (Figs.18.1 and 18.2). What is the likely mechanism of with a long PR interval. In this case, AVNRT was induced in
the SVT? the EP lab and successfully ablated (Figs.18.318.16).

Fig.18.1 The 12-lead resting


ECG (paper speed 25 mm/s)
showed sinus rhythm with a
ventricular rate of 80 bpm a short
PR interval (102 ms) and a QRS
width of 80 ms

A. Rossillo (*), S. Themistoclakis, A. Bonso, A. Corrado,


and A. Raviele
Ospedale dellAngelo, Mestre, Venice, Italy

A. Natale et al. (eds.), Cardiac Electrophysiology, 71


DOI: 10.1007/978-1-84996-390-9_18, Springer-Verlag London Limited 2011
72 A. Rossillo et al.

Fig.18.2 12-lead ECG taken


during SVT (paper speed
25 mm/s) with a heart rate of
186 bpm

Fig.18.3 Intracardiac recordings


taken at baseline during the
electrophysiology study (paper
speed 200 mm/s). Four surface
ECG leads (I, aVF, V1, V6), one
bipolar recording from the high
right atrium (HRA), three bipolar
recordings from the His bundle
region (distal = HIS D, interme-
diate = HIS I and proximal = HIS
P), two bipolar recordings from
the coronary sinus (CS prox =
proximal coronary sinus and CS
dist = distal coronary sinus), and
the distal bipolar recording of the
mapping catheter (MC D). A
atrium, V ventricle, H His bundle
Case 18 73

Fig.18.4 (a, b). Intracardiac a


recordings taken during AVN RT Stim A2 Mestre Studio SAP100 S: 36: 45: 408
programmed atrial stimulation 200 mm/s 100 ms
11
(paper speed 200 mm/s). Same
display that is shown in Fig.18.3.
2avF
(a) With a coupling interval of 198 ms
410 ms the AH interval is 188 ms
and (b) with a coupling interval 3V1

of 280 ms it suddenly increased


to 307 ms (ERP of the fast 4V6
pathway with a jump of 120 ms)
A atrium, V ventricle, H His 5hRA
bundle V
6His D
A H
7His I

8His P

9CS Prox

10CS dist

11MC D B1

15STIM A2
S1 S2

b
AVN RT Stim A2 Mestre Studio jump S: 36: 58: 779

11 200 mm/s 100 ms

2avF

3V1
307 ms

4V6

5hRA
V

6His D
A H
7His I

8His P

9CS Prox

10CS dist

11MC D B1

15STIM A2
S2
S1
74 A. Rossillo et al.

Fig.18.5 Intracardiac record-


ings taken during programmed
atrial stimulation with two beats
with retrograde conduction
through the fast pathway
(S slow pathway and F fast
pathway) (paper speed
100 mm/s). Same display as
that shown in Fig.18.3

Fig.18.6 Intracardiac recordings


taken during programmed atrial
stimulation (paper speed
200 mm/s) showing the atrial
ERP with a coupling interval of
200 ms. Same display as that
shown in Fig.18.3. A atrium,
V ventricle, H His bundle
Case 18 75

Fig.18.7 Intracardiac recordings


taken during continuous atrial
stimulation (paper speed
100 mm/s) with induction of
non-sustained AVNRT. A atrium,
V ventricle, H His bundle

Fig.18.8 Intracardiac recordings


taken during programmed atrial
stimulation with an infusion of
isoproterenol IV (paper speed
100 mm/s) showing induction of
AVNRT (cycle length 300 ms).
Same display as shown in
Fig.1.19.3. A atrium, V ventricle,
H His bundle
76 A. Rossillo et al.

Fig.18.9 Intracardiac record-


ings (paper speed 200 mm/s).
Same display as shown in
Fig.18.3. Pace mapping of the
anteroseptal region of Kochs
triangle with a stim-H interval of
78 ms. St pacing, A atrium, V
ventricle, H His bundle

Fig.18.10 Intracardiac
recordings (paper speed
200 mm/s). Same display as
Fig.1.19-3. Pace mapping of the
midseptal region of Kochs
triangle with a stim-H interval of
81 ms. St pacing, A atrium, V
ventricle, H His bundle
Case 18 77

Fig.18.11 Intracardiac
recordings (paper speed 200
mm/s). Same display as shown in
Fig.18.3. Pace mapping of the
posteroseptal region of Kochs
triangle with a stim-H interval of
106 ms. St pacing, A atrium, V
ventricle, H His bundle

Fig.18.12 Intracardiac
recordings (paper speed
200 mm/s). Same display as
shown in Fig.18.3. Ablation site
slow pathway potential
78 A. Rossillo et al.

Fig.18.13 Intracardiac
recordings during radiofrequency
delivery (paper speed 100 mm/s)
with induction of slow junctional
beats in fusion with a sinus beat.
Same display as shown in
Fig.18.3

Fig.18.14 Intracardiac
recordings taken during
programmed atrial stimulation
with infusion of isoproterenol IV
(paper speed 100 mm/s) with a
very slow anterograde conduction
(AH interval 280 ms). Same
display as shown in Fig.18.3. A
atrium, V ventricle, H His bundle
Case 18 79

Fig.18.15 Intracardiac
recordings taken during
continuous atrial stimulation
with an infusion of isoproterenol
IV (paper speed 100 mm/s)
showing the Wenckebach point
and without induction of
tachycardia. Same display as
shown in Fig.18.3. A atrium,
V ventricle, H His bundle

Fig.18.16 RAO and LAO of


mapping catheter at the ablation
site in the posteroseptal part of
Kochs triangle RAO LAO

Bibliography guide application of radiofrequency energy. Circulation. 1992;85:


2162-2175.
Hassaguerre M, Jas P, Shah DC, etal. Analysis of electrophysiological
Delise P, Gianfranchi L, Paparella N, etal. Clinical usefulness of slow activity in Kochs triangle relevant to ablation of the slow AV nodal
pathway ablation in patients with both paroxysmal atrioventricular pathway. Pacing Clin Electrophysiol. 1997;20:2470-2481.
nodal reentrant tachycardia and atrial fibrillation. Am J Cardiol. Jackman WM, Beckman KJ, McClelland JH, et al. Treatment of
1997;79:1421-1423. supraventricular tachycardia due to atrioventricular nodal reentry,
Delise P, Sitta N, Bonso A, et al. Pace mapping of Kochs triangle by radiofrequency catheter ablation of slow-pathway conduction.
reduces risk of atrioventricular block during ablation of atrioven- NEngl J Med. 1992;327:313-318.
tricular nodal reentrant tachycardia. J Cardiovasc Electrophysiol. Natale A, Greenfield RA, Geiger MJ, et al. Safety of slow pathway
2005;16:30-35. ablation in patients with long PR interval: further evidence of fast
Haissaguerre M, Gaita F, Fischer B, etal. Elimination of atrioventricu- and slow pathway interaction. Pacing Clin Electrophysiol. 1997;20:
lar nodal reentrant tachycardia using discrete slow potentials to 1698-1703.

Case 19

Yash Y. Lokhandwala, Anoop K. Gupta, and Ranjan K. Thakur

Case Summary CL. The bundle branch block has a typical morphology and
R > S in V6. Hence, this is SVT, not VT.
The diagnosis of SVT confirmed, note that the HV inter-
A 50-year-old female with a normal heart presents with a
val is normal. The first two complexes show A-waves in
wide complex tachycardia (Fig.19.1). The patient is taken to
the coronary sinus (CS) simultaneous with the QRS, sug-
the electrophysiology (EP) lab (Fig.19.2). What clues to the
gesting that this is most likely slow/fast atrioventricular
diagnosis are present in the electrocardiogram (ECG)? Is this
nodal reentrant tachycardia (AVNRT). Then, two prema-
supraventricular tachycardia (SVT) or ventricular tachycar-
ture atrial complexes (PACs) are delivered from CS 5-6.
dia (VT)? What is the mechanism of the tachycardia?
The first PAC does not reset the tachycardia. The second
PAC conducts with a longer AH interval, and subsequently
the tachycardia has a longer CL, but the retrograde atrial
Case Discussion activation remains the same, suggesting the presence of a
different slow pathway.
The tachycardia has right bundle branch block (RBBB) There is baseline RBBB. The patient has slowfast
morphology with QRS axis +90. There is a sudden change AVNRT with two slow pathways, participating in AVNRT;
in cycle length (CL), but it is not a multiple of the shorter hence, she has two CLs.

Y.Y. Lokhandwala (*)


KEM Hospital, Parel, Mumbai, India
A.K. Gupta
Apollo Hospital, Ahmedabad, India
R.K. Thakur
Thoracic and Cardiovascular Institute, Sparrow Health System,
Michigan State University, 405 West Greenlawn, Suite 400,
Lansing, MI 48910

A. Natale et al. (eds.), Cardiac Electrophysiology, 81


DOI: 10.1007/978-1-84996-390-9_19, Springer-Verlag London Limited 2011
82 Y.Y. Lokhandwala et al.

2008070

II

III

AVR

AVL

AVF

V1

V2

V3

V4

V5

V6

10 mm/mV 25 mm/s

Fig.19.1 Electrocardiogram (ECG) of wide complex tachycardia

2008070

AVF

V1

V6

CS910

CS78

CS56

CS34

CS12

HISD

HISP

Fig.19.2 Intracardiac recordings. CS 1-2 is distal


Case 20

Marmar Vaseghi, Noel G. Boyle, and Kalyanam Shivkumar

Case Summary Fig.20.1. Duodecapolar bipoles 69 were positioned more


posteriorly at the native atrium. What type of AFL is demon-
strated by the intracardiac duodecapolar ECGs? Is there acti-
A 21-year-old female with a history of viral cardiomyopathy
vation of the native atrium? What is the rate of the native
status post orthotopic heart transplantation (OHT) in June
atrial rhythm?
2002 presented with recurrent palpitations three years post
transplant. She had received a right atrioatrial anastomosis
at the time of transplantation. She was found to have an atrial
flutter (AFL) on surface electrocardiogram (ECG) that
proved refractory to medications. Her cardiac catheterization Case Discussion
and biopsy showed no evidence of transplant coronary artery
vasculopathy or acute rejection, respectively. She was The duodecapolar recordings from the donor atrium showed
referred for ablation of her AFL.Fluoroscopic position of the a typical tricuspid annulus dependent counterclockwise AFL;
duodecapolar catheter in the right anterior oblique (RAO) Fig. 20.2 (top panels). This is the most common type of
view and the corresponding ECG recordings are shown in supraventricular tachycardia (SVT) in stable OHT patients.1

A B
I
AVF
200ms
V1
V6
D10
D9
Fig.20.1 Panel A:
D8
Fluoroscopic RAO view shows
D7
the position of the duodecapolar
D6
catheter. Panel B: Intracardiac
D5
activation sequence. CS
D4
coronary sinus, D1D10
D3
duodecapolar bipoles 110,
D2
DDC duodecapolar catheter, RV
D1
right ventricle, LV left ventricle

M. Vaseghi (*), N.G. Boyle, and K. Shivkumar


UCLA Cardiac Arrhythmia Center,
David Geffen School of Medicine at UCLA,
10833 Le Conte Ave, A2-237 CHS,
Los Angeles, CA 90095-1679
e-mail: mvaseghi@mednet.ucla.edu

A. Natale et al. (eds.), Cardiac Electrophysiology, 83


DOI: 10.1007/978-1-84996-390-9_20, Springer-Verlag London Limited 2011
84 M. Vaseghi et al.

Fig.20.2 Panel A: Fluoroscopic A B


RAO view shows the position I
of the duodecapolar catheter AVF
with the distal bipoles (D1D3) 200ms
V1
along the tricuspid valve isthmus
V6
and the proximal electrode (D10)
D10
along the superior aspect of the
D9
interatrial septum. Panel B:
D8
Intracardiac activation sequence
D7
in the donor atrium confirms
counterclockwise activation. D6
CS coronary sinus, D1D10 D5
duodecapolar bipoles 110, D4
DDC duodecapolar catheter, RV D3
right ventricle, LV left ventricle D2
D1

Fig.20.3 Electroanatomic maps


of atrio-atrial anastamosis (gray
areas identify native atrial
tissue). The flutter wavefront
involves the donor atrium only
and is propagating in a counter-
clockwise fashion along the
tricuspid annulus

Poles D6D9 are in contact with the native atrial tissue, Reference
which is not in flutter, and is in a dissociated regular brady-
cardic rhythm of 40 bpm (beats per minute) (arrow), separate 1. Vaseghi M, Boyle NG, Kedia R, etal. Supraventricular tachycardia
from the tachycardia. A critical isthmus is noted on the intrac- following orthotopic heart transplantation. J Am Coll Cardiol.
ardiac ECGs at duodecopolar position D5. The activationmap 2008;51:2241-2249.
is shown in Fig.20.3. An inferior vena cava (IVC) tricuspid
isthmus ablation line terminated the donor atrial tachycardia.
No ablation was performed on the native atrium.
Case 21

Bradley P. Knight

Case Summary earliest atrial activation can be seen in the proximal CS at


electrode pair CS 3-4. The QRS complex, however, normal-
izes during the second part of the recording.
A young man underwent an electrophysiology (EP) proce-
An important measurement to make whenever a SVT is
dure for recurrent supraventricular tachycardia (SVT). The
associated with both a normal QRS complex and a bundle
recording obtained after induction of the tachycardia is
branch block is the VA interval. The reason for this is that
shown in Fig. 21.1. Surface intracardiac recordings from the
when the tachycardia mechanism is orthodromic reentrant
His bundle electrogram (HBE), mapping catheter (Map),
tachycardia (ORT), and a rate-related bundle branch block
coronary sinus (CS), and right ventricle (RV) are shown.
develops on the same side as the accessory pathway (AP),
What is the mechanism of the tachycardia?
the reentrant circuit will increase in length (due to transeptal
conduction) and results in a longer VA interval during the
ipsilateral bundle branch block.
Case Discussion In this case, the VA interval is clearly longer during the
LBBB compared to when the QRS complex normalizes. This
The initial part of the tracing shows a wide complex tachycar- is diagnostic for ORT using a left-sided AP. This case is also
dia with a cycle length (CL) of 320 ms. There is a His-bundle interesting, because the tachycardia CL does not change sig-
recording before the QRS complex, which has a typical left nificantly during the LBBB, despite an increase in the VA
bundle branch block (LBBB) pattern. Therefore, the tachy- interval. This is because there is a compensatory decrease in
cardia is consistent with a supraventricular mechanism. The the AH interval during the LBBB.

B.P. Knight
Division of Cardiology, Northwestern Medical Center, 676 N.
St. Clair, Suite 600, Chicago, IL 60611

A. Natale et al. (eds.), Cardiac Electrophysiology, 85


DOI: 10.1007/978-1-84996-390-9_21, Springer-Verlag London Limited 2011
86 B.P. Knight

II

III

V1

V5

HBE d

HBE p

Map d

Map p

CS 1-2

CS 3-4

CS 5-6
CS 7-8

CS 9-10
RV d

Stim

Fig.21.1 This tracing was obtained during an ablation procedure for depicts the onset of the surface QRS complex in each case and the
supraventricular tachycardia. Note the change in the ventricular atrial arrow points to the earliest intracardiac atrial activation
interval when the wide-QRS complex normalizes. The vertical line
Case 22

Antonio Rossillo, Sakis Themistoclakis, Aldo Bonso,


Andrea Corrado, and Antonio Raviele

Case Summary Case Discussion

The patient was a 34-year-old female with a history of pal- In this case, catheter manipulation induced AVNRT. Ventricular
pitations. No heart disease was documented. Two months pacing demonstrated retrograde conduction via a left-sided
prior to the ablation the arrhythmia episodes became more accessory pathway. AVRT was also easily induced and was con-
frequent. After examining Figs.22.122.4, will ablation at sistent with the clinical tachycardia. Ablation of the left-sided AP
a single site eliminate the patients palpitations? and the slow pathway were performed (see Figs.22.522.11).

II

III

aVR

aVL

aVF

V1

V2

V3

V4

V5

V6

Fig.22.1 Twelve-lead resting ECG (paper speed 25 mm/s) showing short PQ interval with ventricular pre-exitation with an early transition of R
wave between lead V1 and V2 and negative wave in leads III and aVF

A. Rossillo (), S. Themistoclakis, A. Bonso, A. Corrado,


and A. Raviele
Ospedale dellAngelo, Mestre, Venice, Italy

A. Natale et al. (eds.), Cardiac Electrophysiology, 87


DOI: 10.1007/978-1-84996-390-9_22, Springer-Verlag London Limited 2011
88 A. Rossillo et al.

Fig.22.2 Intracardiac recordings at baseline during the electrophysiol- atrium or ventricle (HRA), three bipolar recordings from the His bundle
ogy study (paper speed 100 mm/s). Four surface ECG leads (I, aVF, V1, region (HIS PROX, MED, and DIST), and five bipolar recordings from
V6), two bipolar recordings from the pacing catheter placed in right the coronary sinus (CS). A Atrium, V Ventricle, H His bundle

Fig.22.3 Intracardiac recordings taken during SVT induced with catheter manipulation (paper speed 100 mm/s, cycle length 330 ms). Same
display as that shown in Fig.22.2. A Atrium, V Ventricle, H His bundle
Case 22 89

Fig.22.4 Intracardiac recordings taken during programmed ventricular stimulation with induction of SVT (paper speed 100 mm/s, cycle length
290 ms) with earliest activation on CS 3. Same display as that shown in Fig.22.2. A Atrium, V Ventricle

Fig. 22.5 Intracardiac recordings taken during AVRT (paper speed A spontaneous ventricular ectopic beat reduced the cycle length of the
100 mm/s, cycle length 264 ms). Same display as that shown in Fig.22.2 tachycardia and the anterograde conduction after the ectopic beat is
and a bipolar recording from the distal mapping catheter (ABL dist). through nodal decremental fibers
90 A. Rossillo et al.

Fig.22.6 Intracardiac recordings taken during sinus rhythm (paper speed 100 mm/s). Same display as that shown in Fig.22.5. Ablation site

III
aVF
V1
V6
HRA 1-2
HRA 3-4
HIS PROX
HIS MED
HIS DIST
CS 1
CS 2
CS 3
CS 4
CS 5
ABL dist

Fig.22.7 Intracardiac recordings taken during RF energy delivery (paper speed 100 mm/s). Same display as that shown in Fig.22.5. The red
arrows show the absence of the ventricular pre-excitation
Case 22 91

Fig.22.8 Intracardiac recordings taken during RF energy delivery (paper speed 25 mm/s). Same display as that shown in Fig.22.5. The red
arrows show the absence of ventricular pre-excitation

III
aVF
V1
V6
HRA 1-2
HRA 3-4
HIS PROX
HIS MED
HIS DIST
CS 1
CS 2
CS 3
CS 4
CS 5
ABL dist

V A V A

Fig.22.9 Intracardiac recordings (paper speed 100 mm/s) taken during programmed ventricular stimulation showing a decremental VA conduc-
tion (paper speed 100 mm/s). Same display as that shown in Fig.22.5. A Atrium, V Ventricle
92 A. Rossillo et al.

Fig.22.10 Intracardiac recordings (paper speed 100 mm/s) taken during programmed atrial stimulation with two extrastimuli showing the ERP
of the AV node (paper speed 100 mm/s). Same display as that shown in Fig.22.5. A Atrium, V Ventricle

II

III

aVR

aVL

aVF

V1

V2

V3

V4

V5

V6

Fig.22.11 12-lead resting ECG (paper speed 25 mm/s) showing normal PQ interval with the absence of ventricular pre-excitation with negative
T waves in the inferior leads
Case 22 93

Bibliography ablation of accessory pathways. Value of unipolar recording mode.


Circulation. 1991;84:188-202.
Jackman WM, Wang XZ, Friday KJ, etal. Catheter ablation of acces-
Calkins H, Langberg J, Sousa J, etal. Radiofrequency catheter ablation sory atrioventricular pathways (Wolff-Parkinson-White syndrome)
of accessory atrioventricular connections in 250 patients. by radiofrequency current. N Engl J Med. 1991;324:1605-1611.
Abbreviated therapeutic approach to Wolff-Parkinson-White syn- Scheinman MM, Wang YS, Van Hare GF, Lesh MD. Electrocardio
drome. Circulation. 1992;85:1337-1346. graphic and electrophysiologic characteristics of anterior, midseptal
Hassaguerre M, Dartigues JF, Warin JF, Le Metayer P, Montserrat P, and right anterior free wall accessory pathways. J Am Coll Cardiol.
Salamon R. Electrogram patterns predictive of successful catheter 1992;20:1220-1229.

Case 23

Yash Y. Lokhandwala, Anoop K. Gupta, and Ranjan K. Thakur

Case Summary Case Discussion

The patient is a 58-year-old male with a normal heart. The The ventriculo-atrial (VA) conduction pattern and VA time
resting electrocardiogram (ECG) is normal. He has a narrow are different in the two beats in Fig.23.1. The first retrograde
QRS tachycardia. He is taken to the electrophysiology (EP) A is bracketed, earliest in coronary sinus (CS) 3-4 and 5-6;
lab; see Figs.23.1 and 23.2. What is the likely mechanism of the second retrograde A is also bracketed, but earliest in CS
this tachycardia? 5-6 and 7-8. Figure23.2 shows para-Hisian pacing. The VA

Fig.23.1 Ventricular pacing

Y.Y. Lokhandwala (*)


KEM Hospital, Parel, Mumbai, India
A.K. Gupta
Apollo Hospital, Ahmedabad, India
R.K. Thakur
Thoracic and Cardiovascular Institute, Sparrow Health System,
Michigan State University, 405 West Greenlawn, Suite 400,
Lansing, MI 48910
A. Natale et al. (eds.), Cardiac Electrophysiology, 95
DOI: 10.1007/978-1-84996-390-9_23, Springer-Verlag London Limited 2011
96 Y.Y. Lokhandwala et al.

Fig.23.2 Para-Hisian pacing

time clearly is shortest with His bundle capture (second com- In Fig.23.4, tachycardia has a left bundle branch block
plex). The VA pattern of the first complex is different from (LBBB) morphology. Compared to the previous tracing, the
the other two complexes. The first retrograde A is earliest in VA time is clearly longer in this case. This is consistent with
CS 5-6. The second and third retrograde A are earliest in CS our prior diagnosis of a left-sided AP.
7-8. Thus the second and third retrograde A are via the AV Ablation was performed during right ventricular (RV) pac-
node; the first is likely via a left AP. ing (Fig. 23.5) because this allows for catheter stability. If
The tachycardia displays varying degrees of right bundle ablation is performed during reentrant tachycardia, the catheter
branch block (RBBB) during SVT (Fig.23.3) (normal HV). may move suddenly when RT terminates during RF applica-
The earliest retrograde A is consistently in CS 5-6, with a tion. So, catheter stability can be obtained by entraining the RT
constant VA interval. CS 9-10 was at CS os (ostium). Thus at 1020 ms faster than the tachycardia CL. The VA conduc-
the retrograde A is via a concealed left posterior AP. tion during RV pacing changes from left AP to AV node.
Case 23 97

Fig.23.3 Development of a right bundle branch block (RBBB)


98 Y.Y. Lokhandwala et al.

Fig.23.4 Development of a left bundle branch block (LBBB)


Case 23 99

Fig.23.5 Ablation of AP

Case 24

Marmar Vaseghi, Noel G. Boyle, and Kalyanam Shivkumar

Case Summary graphy demonstrated a normal ejection fraction at the time of


arrhythmia. Surface electrocardiogram (ECG) was consistent
with atypical atrial flutter (AFL).
A 60-year-old male status post orthotopic heart transplanta- The patient was taken to the electrophysiology (EP) labo-
tion (OHT) in June 1999 presented with drug refractory ratory. Intracardiac ECGs along with location of the catheters
palpitations 8 years after his transplantation. Cardiac cathe- are shown in (Fig. 24.1). What is the likely mechanism of
terization and biopsy showed no evidence of transplant this arrhythmia based on the duodecapolar and coronary
vasculopathy or acute rejection, respectively. Echocardio sinus (CS) catheter recordings?

A B I
aVF
200 msec
V6
D1
D2
D3
D4
D5
D6
D7
D8
D9
D10
CS 9,10
CS 7,8
CS 5,6
CS 3,4
CS 1,2
RVa

Fig.24.1 Panel A: Fluoroscopic LAO view: The position of the duode- CS coronary sinus, D1D10 duodcapolar bipoles 110, LAO left ante-
capolar and CS catheters are shown in right atrium. Panel B: Intracardiac rior oblique, RVa RV catheter bipole
tracings obtained from the duodecapolar and CS catheters are shown.

M. Vaseghi (*), N.G. Boyle, and K. Shivkumar


UCLA Cardiac Arrhythmia Center,
David Geffen School of Medicine at UCLA,
10833 Le Conte Ave, A2-237 CHS, Los Angeles, CA 90095-1679
e-mail: mvaseghi@mednet.ucla.edu

A. Natale et al. (eds.), Cardiac Electrophysiology, 101


DOI: 10.1007/978-1-84996-390-9_24, Springer-Verlag London Limited 2011
102 M. Vaseghi et al.

Case Discussion around the tricuspid annulus. Hence this left AFL spreads
in a figure-eight fashion around both the tricuspid and
Although typical clockwise AFL is the most common type mitral annuli. Mapping with ablation catheters showed
of supraventricular tachycardia (SVT) in stable post-OHT earliest activation at CS 1-2 and ostium of the left inferior
patients, this patients intracardiac tracings show earliest pulmonary vein. The activation map obtained via the abla-
activation in the distal CS consistent with a left AFL. tion catheter and the ablation line, which successfully ter-
Further, the flutter pattern spreads from the distal CS to the minated the tachycardia, are shown in (Fig. 24.2). The
proximal CS and then to the right atrium where D10 (or patient has been without recurrence during the 10 months
septal activation is earliest). This is followed by activation of follow-up.

Fig.24.2 Activation map: the


flutter circuit begins in the
inferior wall of the LA (pink)
close to the CS 3-4 bipole, and
then propagates across to the
RA and around the tricuspid
annulus, as well as around and
over the mitral annulus. LA left
atrium, LAA left atrial append-
age, LIPV left inferior pulmo-
nary vein, LSPV left superior
pulmonary vein, RA right
atrium, RSPV right superior
pulmonary vein, SVC superior
vena cava
Case 25

Bradley P. Knight

Case Summary tachycardia was induced by an atrial premature beat that


conducts to the ventricle, with a QRS morphology that is the
same as that during tachycardia, makes VT very unlikely.
A female marathon runner without structural heart disease
SVT with aberrancy can also be excluded because a His-
had a documented regular wide complex tachycardia
bundle potential could not be recorded before the QRS com-
(Fig. 25.1). At the time of electrophysiology (EP) testing,
plex. Therefore, the most likely diagnosis is ART.
she had normal sinus and atrioventricular (AV) node func-
In a patient with no preexcitation during sinus rhythm, the
tion, no ventricular preexcitation, and no inducible ventricu-
most common mechanism of ART is a slowly conducting,
lar tachycardia (VT). A wide complex tachycardia was
right-sided atriofascicular accessory pathway (AP). A study
induced with an atrial premature extrastimulus during sinus
of 384 patients with a single AP found that anterograde dec-
rhythm (Fig. 25.2). The QRS morphology was a right bundle
remental conduction was seen only in the right free wall
branch block (RBBB) pattern with an inferior axis. There
location. However, the QRS morphology in this patient is not
was not His-bundle potential recordable before the QRS dur-
consistent with a right-sided AP. In this case, the earliest ven-
ing the tachycardia. What is the most likely diagnosis?
tricular activation was along the mitral annulus where abla-
tion was successful (Fig. 25.3). This represents a very unusual
location for a slowly conducting anterograde AP.
Case Discussion

The differential of a wide complex tachycardia includes VT,


supraventricular tachycardia (SVT) with aberrancy, and anti-
dromic AV reentrant tachycardia (ART). The fact that this

B.P. Knight
Division of Cardiology, Northwestern Medical Center, 676 N.
St. Clair, Suite 600, Chicago, IL 60611

A. Natale et al. (eds.), Cardiac Electrophysiology, 103


DOI: 10.1007/978-1-84996-390-9_25, Springer-Verlag London Limited 2011
104 B.P. Knight

Fig.25.1 Shown is a two lead rhythm strip showing the onset of a sustained wide-QRS complex tachycardia. The tracing was recorded from an
ambulatory event recorder
Case 25 105

Fig.25.2 This tracing was recorded during an ablation procedure in a tioned at the His bundle location, and the right ventricular apex (RVA).
patient with a wide complex tachycardia. Shown are surface electro- Note the initiation of a wide complex tachycardia with a single atrial
grams from leads I, II, V1, and V3, and the intracardiac electrograms extrastimulus delivered during sinus rhythm
from the high right atrium (HRA), the ablation catheter (ABL) posi-
106 B.P. Knight

Fig.25.3 This tracing was recorded after the ablation catheter was positioned along the mitral annulus at the site of the earliest ventricular local
activation. The format is the same as figure 25.2
Case 26

Richard H. Hongo and Andrea Natale

Case Summary She has been treated with a variety of antiarrhythmic


agents including verapamil, sotalol, and amiodarone with
The patient is a 53-year-old woman with recurrent palpi- variable success.
tations since age 16 with documented episodes of tach- She presented for electrophysiology (EP) study with possi-
yarrhythmia (Fig. 26.1). Discrete P-waves at a rate of ble catheter ablation because of highly symptomatic episodes
approximately 180 bpm have been apparent with atrio- of atrial arrhythmia that continued to recur despite medical
ventricular (AV) node blockade from adenosine (Fig.26.2). therapy. High-dose isoproterenol infusion (20 mcg/min)

Fig.26.1 Continuous 3-lead (II, V1, V5) ECG capturing recurrent episodes of narrow-complex regular tachyarrhythmia at a rate of approximately
180 bpm

R.H. Hongo
Sutter Pacific Medical Foundation, California Pacific Medical Center,
2100 Webster Street, Suite 521, San Francisco, CA 94115, USA
e-mail: hongor@sutterhealth.org
A. Natale ()
Texas Cardiac Arrhythmia Institute, St. Davids Medical Center,
1015 East 32nd Street, Suite 516, Austin, TX 78705
e-mail: dr.natale@gmail.com

A. Natale et al. (eds.), Cardiac Electrophysiology, 107


DOI: 10.1007/978-1-84996-390-9_26, Springer-Verlag London Limited 2011
108 R.H. Hongo and A. Natale

during EP study induced a tachyarrhythmia with a ventricular Case Discussion


rate of 150 bpm (Fig.26.3). Double transseptal puncture was
performed and a 20-mm 10-electrode lasso catheter and a AV block during adenosine infusion (Fig.26.2) uncovers dis-
3.5-mm F-curve saline irrigated ablation catheter were crete P-waves most consistent with atrial tachycardia. Distinct
advanced into the left atrium (LA). isoelectric segments between the P-waves in all twelve leads
Intracardiac activation of the atrial arrhythmia was recor of the ECG during isoproterenol infusion (Fig.26.3) makes
ded (Fig.26.4) with the lasso catheter at the os of the right both typical and atypical flutter less likely. Loss of 1:1 AV
superior pulmonary vein (RSPV) and the ablation catheter association eliminates atrioventricular reciprocating tachy-
approximately 6 mm within the same vein (Fig. 26.5). cardia (ART) as a diagnosis. Intracardiac ECGs (Fig. 26.4)
A 20-electrode deflectable catheter was introduced from the reveal the earliest activation to be at the distal ablation cathe-
right internal jugular vein and placed along the lateral wall of ter positioned within the RSPV (Fig.26.5), clearly before the
the right atrium (RA) with the distal 10 electrodes in the activation around the lasso catheter at the vein os. The most
coronary sinus (CS). appropriate ablation strategy is to isolate the RSPV. Ablation
What is the tachyarrhythmia based on the surface electro- at the earliest activation site within the vein should be avoided
cardiogram (ECG) before and during adenosine, and assum- because of the risk of PV stenosis. If either atrial fibrillation
ing it is the same arrhythmia induced with isoproterenol? (AF) or typical atrial flutter (AFL) were observed clinically,
What is the tachyarrhythmia based on the intracardiac ECGs? isolation of all four pulmonary veins and cavotricuspid isth-
What is the most appropriate next step? mus ablation, respectively, would also be appropriate.

Fig.26.2 Continuous 3-lead (II, V1, V5) ECG during adenosine-induced AV block. Uncovered discrete P-waves are most consistent with atrial
tachycardia
Case 26 109

Fig.26.3 12-lead ECG during isoproterenol infusion (20 mcg/min) demonstrates sustained atrial tachyarrhythmia. Distinct isoelectric PR
segments are seen in all leads

I
200 ms
aVF
V1 S
V6
HRA 19,20
HRA 17,18
HRA 15,16
HRA 13,14
HRA 11,12
ABL p
ABL d
CS 9,10
CS 7,8 C
CS 5,6 C

CS 3,4 C

CS d C
Ls 1,2
Ls 2,3
Ls 3,4
Ls 4,5
Ls 5,6
Ls 6,7
Ls 7,8
Ls 8,9
Ls 9,10

10:06:09 AM 10:06:10 AM
CardioLab v6.5
GE Medical Systems

Fig.26.4 Simultaneous surface and intracardiac ECGs during the atrial tachyarrhythmia reveal the earliest activation to be at the distal electrode
pair of the ablation catheter (ABL d), clearly in front of the activation seen on the Lasso catheter. HRA = high right atrial catheter; CS = coronary
sinus catheter; Ls = Lasso catheter
110 R.H. Hongo and A. Natale

Fig.26.5 Fluoroscopic image in shallow LAO projection, showing the


position of the catheters that correspond with the intracardiac ECGs
from Fig. 26.4. The Lasso catheter is placed at the right superior pulmo-
nary vein (RSPV) os. The ablation catheter is shown positioned within
the RSPV
Case 27

M. Eyman Mortada, Jasbir S. Sra, and Masood Akhtar

Case Summary Seven years prior to this presentation, the patient had
short RP and narrow complex tachycardia. The differential
diagnoses of that rhythm include: typical AVNRT, atrial
A 63-year-old female with a history of tachycardia 7 years
tachycardia (AT) with long PR interval, and AV reentry
ago with the electrocardiogram (ECG) shown in Fig. 27.1
tachycardia via an accessory pathway (AP).
presented complaining of recurrent palpitations with the
The presence of both rhythms in one patient means this is
tachycardia shown in Fig.27.2.
probably AVNRT. The first rhythm was probably a typical
During an electrophysiology (EP) study, the responses from
tachycardia. The patient now presents with atypical AVNRT.
premature ventricular beat and ventricular pacing were recorded
The first intracardiac tracing reveals earliest atrial activa-
during the tachycardia, as shown in Figs.27.3 and 27.4.
tion is in the proximal coronary sinus (CS), suggesting an AT
What is the diagnosis?
originating in the lower septum, retrograde activation over
the AV node, or retrograde activation over a septal AP. A
spontaneous premature ventricular capture (PVC) during the
Case Discussion tachycardia, at the time of the His refractory, was observed.
At that time, the tachycardia cycle length (CL) was not
The presenting tachycardia is characterized by a long RP, nar- changed. The presence of a PVC reduces the likelihood that
row complex tachycardia, with low septal to high atrial activity the mechanism is an AV reentry tachycardia via a septal AP.
(negative P-wave in lead II and positive P-waves in leads aVR The second intracardiac tracing illustrates ventricular
and aVL). Therefore, the differential diagnoses include: low entrainment during the tachycardia. Postventricular pacing
septal atrial tachycardia, atypical AV nodal reentry tachycardia shows a V-A-V response, excluding the diagnosis of a low
(AVNRT), and persistent junctional reciprocating tachycardia. septal AT. Thus, the diagnosis is atypical AVNRT.

M.E. Mortada (*) J.S. Sra, and M. Akhtar


Department of Electrophysiology, Aurora Cardiovascular Services,
Aurora Sinai/Aurora St. Lukes Medical Centers,
University of Wisconsin School of Medicine and Public Health,
2801 W. Kinnickinnic River Parkway, #777,
Milwaukee, WI 53215

A. Natale et al. (eds.), Cardiac Electrophysiology, 111


DOI: 10.1007/978-1-84996-390-9_27, Springer-Verlag London Limited 2011
112 M.E. Mortada et al.

Fig.27.1 Surface ECG of the tachycardia seven years prior to the current presentation

Fig.27.2 Surface ECG of the current tachycardia


Case 27 113

Fig.27.3 Spontaneous PVC during the tachycardia at the His refractory period. From top to bottom: surface ECG with leads I, II and V1, followed
by intracardiac tracing of HRA, proximal to distal CS, and finally His catheters

Fig.27.4 Response from ventricular entrainment of the tachycardia



Case 28

Yash Y. Lokhandwala, Anoop K. Gupta, and Ranjan K. Thakur

Case Summary during tachycardia; this caused advancement of the next


QRS. The QRS advancement by a His-refractory APD
A 21-year-old female with a normal heart comes to the electro- proves the presence of an extranodal (atriofascicular or atrio-
physiology (EP) lab because of palpitations. A diagnosis of ventricular accessory pathway) (Fig.28.4). The HA during
typical atrioventricular reentrant tachycardia (AVNRT) is tachycardia is identical to HA during RV pacing; thus the
made (Fig. 28.1). However, after atrial pacing, long PR and Mahaim-like accessory pathway is not a bystander.
sudden left bundle branch block (LBBB) like morphology is Final diagnosis: (1) Right lateral Mahaim-like AP with
seen (Figs.28.2 and 28.3). What maneuver is being performed? true antidromic AVRT and (2) slowfast, typical AVNRT
What does this say about the tachycardia mechanism? (Figs.28.5 and 28.6).

Case Discussion
An atrial premature depolarization (APD) was delivered
from a radiofrequency (RF) catheter in the right atrium (RA),

Y.Y. Lokhandwala ()
KEM Hospital, Parel, Mumbai, India
A.K. Gupta
Apollo Hospital, Ahmedabad, India
R.K. Thakur
Thoracic and Cardiovascular Institute,
Sparrow Health System, Michigan State University,
405 West Greenlawn, Suite 400,
Lansing, MI 48910

A. Natale et al. (eds.), Cardiac Electrophysiology, 115


DOI: 10.1007/978-1-84996-390-9_28, Springer-Verlag London Limited 2011
116 Y.Y. Lokhandwala et al.

RUBY HALL CLINIC Version WIN2000/XP : EPTRACER V0.77


2008051
I
AVF
V1
V6

CS910

CS78

CS56

CS34

CS12

HISP

RFD

RFP

10 mm/mV 100 mm/s

Fig.28.1 Typical atrioventricular reentrant tachycardia (AVNRT)


Case 28 117

RUBY HALL CLINIC Version WIN2000/XP : EPTRACER V0.77


2008051

II

III

AVR

AVL

AVF

V1

V2

V3

V4

V5

V6

10 mm/mV 25 mm/s

Fig.28.2 Change to left bundle branch block (LBBB) morphology is seen with atrial pacing
118 Y.Y. Lokhandwala et al.

Version WIN2000/XP : EPTRACER V0.77


2008051
I

AVF

V1

V6

CS910

CS78

CS56

CS34

CS12

HISP

RFD

10 mm/mV 100 mm/s

Fig.28.3 A diagnostic maneuver. Does this help with a diagnosis?


Case 28 119

Version WIN2000/XP : EPTRACER V0.77


2008051
I
AVF
V1
V6

CS910

CS78

CS56

CS34

CS12

HISP

RFD

10 mm/mV 200 mm/s

Fig.28.4 APC delivered from the RA free wall


120 Y.Y. Lokhandwala et al.

RUBY HALL CLINIC Version WIN2000/XP : EPTRACER V0.77


2008051
I

AVF

V1

V6

CS78

CS56

CS34

CS12

HISP

HISM

HISD

RBP

RBD

RFD

10 mm/mV 200 mm/s

Fig.28.5 Intracardiac recording during SVT


Case 28 121

Version WIN2000/XP : EPTRACER V0.77


2008051
I

AVF

V1

V6

CS78

CS56

CS34
CS12

HISP

HISM

HISD

RBP

RBD

RFD
10 mm/mV 200 mm/s

Fig.28.6 RV pacing

Case 291

Eric Buch, Shiro Nakahara, Marmar Vaseghi, Noel G. Boyle,


and Kalyanam Shivkumar

Case Summary Case Discussion


A 57-year-old female with recurrent atrial flutter (AFL) was A duodecapolar (DD) catheter was positioned around the tri-
referred for catheter ablation. The surface electrocardiogram cuspid annulus in the anatomical right atrium (RA) with the
(ECG) suggested clockwise AFL with positive F-waves in II, distal tip located at the medial cavotricuspid isthmus (Fig.29.1).
III, and AVF, and negative F-wave in V1. The complete absence Electrophysiological mapping of the AFL demonstrated earli-
of an R-wave in lead I was noted. The chest x-ray demon- est activation in the electrogram recorded from DD poles 19,20
strated dextrocardia, and echocardiography established com- and latest activation at DD poles 1,2. Top left image shows an
plete situs inversus with otherwise normal intracardiac LAO view of the heart (camera in the RAO position relative to
anatomy. Based on the ECG, what is the likely diagnosis? the torso) and direction of flutter (white arrows) as shown by

Fig.29.1 Fluoroscopy of intracardiac catheters. Top left panel shows LAO position relative to the torso). CS coronary sinus catheter, Abl
LAO view of the heart (camera in the RAO position relative to the ablation catheter, HRA high right atrial catheter, His His bundle
torso). Top right panel shows the RAO view of the heart (camera in the catheter

1
This case is adapted from heart rhythm volume 2, issue 6, pages
673-674 (June 2005)

E. Buch (), S. Nakahara, M. Vaseghi,


N.G. Boyle, and K. Shivkumar
UCLA Cardiac Arrhythmia Center,David Geffen School of
Medicine at UCLA, 10833 Le Conte Ave, A2-237 CHS,
Los Angeles, CA 90095-1679
e-mail: ebuch@mednet.ucla.edu

A. Natale et al. (eds.), Cardiac Electrophysiology, 123


DOI: 10.1007/978-1-84996-390-9_29, Springer-Verlag London Limited 2011
124 E. Buch et al.

the intracardiac tracings, demonstrating early activation in tricuspid valve inferior vena cava (IVC) isthmus was a part
DD19, 20 and counterclockwise rotation of flutter in the RA of the circuit and this tachycardia was cured by ablation of the
(Fig. 29.2). Entrainment pacing demonstrated that the isthmus.

I
II 200 ms
III
aVF
V1
HRA
DD19,20
DD17,18
DD15,16
DD13,14
DD11,12
DD9,10
DD7,8
DD5,6
DD3,4
DD1,2
HIS p
HIS m
HIS d
CS 9,10
CS 7,8
CS 5,6
CS 3,4
CS 1,2
RVa
Stim 3

12:32:15 PM 12:32:16 PM 12:32:17 PM

Fig.29.2 Intracardiac electrograms. In this patient with mirror-image is attributed to the fact that in the situs inversus heart, because of the
dextrocardia, typical flutter was counterclockwise (CCW) in the RA, as rotated position of the atria and ventricles (mirror image of a situs solitus
demonstrated by the wave front of activation on duodecapolar (DD) cath- heart), CCW flutter appears CW (its mirror image) on the surface EKG.
eter. However, the inferior leads of the EKG inscribe F-waves that are Hence, the position of the heart in the thorax must be kept in mind when
positive, which is suggestive of clockwise (CW) flutter. This discrepancy evaluating CW versus CCW flutter based on a surface EKG
Case 30

Luis C. Senz and Miguel A. Vacca

Case Summary increasing the doses of the AAD medications, a transitory


and regular rhythm with 1 per 1 atrioventricular conduc-
The patient is a 5-year-old male with a past medical history tion was registered, and is showed in Fig.30.2b.
significant for paroxysmal palpitations which became inces- Considering tachycardiomyopathy and the failure of the
sant 8 months ago despite taking various anti-arrhythmic AAD to prevent arrhythmia recurrences, the patient was
agents (AADs). brought to the electrophysiology lab for ablation. CARTO
The patient had worsening congestive heart failure and was used during the procedure. A decapolar multielectrode
was hospitalized. He had no other significant past medical catheter was introduced into the coronary sinus for map-
history. The tachycardia EKG taken in the emergency room ping. In Fig.30.3, the coronary sinus channels are showed
is showed in Fig.30.1. His echocardiogram showed left ven- from proximal (SC5) to distal (SCD). The third coronary
tricular dilatation and ejection fraction of 38%. sinus channel is not showed due to electrical noise. The
While the patient was in the hospital another tracing was intra-cardiac electrograms from the roving CARTO map-
taken showing spontaneous changes in his electrical rhythm ping and ablation catheter are showed as RFD and RFP for
(Fig. 30.2a and b). Variable atrioventricular conduction the bipolar electrograms that were taken from the distal1,2
during the tachycardia with an isoelectric interval between and proximal3,4 par of electrodes, respectively. The unipolar
the p waves is showed in Fig. 30.2a suggesting an atrial electrogram from the electrodes 1 and 3 of the CARTO rov-
tachycardia as the mechanism of the arrhythmia. After ing catheter are showed as 1 and 3, respectively. A CARTO

Fig.30.1 EKG of tachycardia taken in the emergency room

L.C. Senz (*)


Jefe Servicio de Electrofisiologa, Cardilogo-Electrofisilogo,
Fundacin Cardio Infantil-Instituto de Cardiologa, Calle 163A
No 28-60, Bogot, Colombia
M.A. Vacca
Cardiac Electrophysiologist, Centro Internacional de Arritmias FCI,
Fundacion Cardioinfantil, Instituto de Cardiologia, Calle 163 A No.
13B-60, Bogota, Colombia

A. Natale et al. (eds.), Cardiac Electrophysiology, 125


DOI: 10.1007/978-1-84996-390-9_30, Springer-Verlag London Limited 2011
126 L.C. Senz and M.A. Vacca

Fig.30.2 (a and b) Spontaneous changes in the electrical rhythm

right atrial and superior vena cava activation map was con- Remarkably, the electrograms registered from the coro-
structed during the tachycardia and shown in Fig.30.3. The nary sinus are almost isochronic with the electrograms taken
CARTO map suggested a focal origin of the tachycardia and from the earliest CARTO zone in the posterior aspect of the
showed the site of earliest electrical activation (in red) of the superior vena cava, Fig.30.3. In order to simplify the case,
mapped chambers over the posterior aspect of the superior the CARTO activation and the sequence of the propagation
vena cava where fractionated and almost double electro- maps (from left to right) of the right atrium, superior vena
grams were found and marked as blue points, Fig.30.3. cava, and coronary sinus is shown in Fig.30.4.
Case 30 127

Fig.30.3 Right atrium and


superior vena cava CARTO
activation map and intra-cardiac
electrograms registered during
tachycardia

By analyzing the EKG in Fig.30.1, what is the possible shows a really broad zone of early activation (in red) sug-
origin of the tachycardia? Considering both the electrograms gesting a passive activation from another site.2
and CARTO maps, why does the posterior aspect of the Moreover, the isochronal activation between the distal
superior vena cava and the distal coronary sinus have almost coronary sinus electrode and the earliest electrogram over
the same time of activation? What is the best next step to do the superior vena cava suggests that the cardiac site respon-
in this case? sible for originating the tachycardia is activating these struc-
tures almost at the same time. In normal conditions this place
would be equidistant between the distal coronary sinus and
the superior vena cava. In this way, the origin of the tachy-
Case Discussion cardia would be in the left atrium.3 In fact, the EKG of the
tachycardia (Fig.30.1) shows a positive p wave in the pre-
The accuracy and reliability of a CARTO map are dependent cordial and inferior leads with negative p wave in aVL and
on having an adequate number of contact points taken in iso-biphasic in DI suggesting that the origin of the tachycar-
order to construct the chamber(s) of interest. The CARTO dia is from the left pulmonary veins.4
system constructs a map solely with information taken from A CARTO left atrial activation map integrating the elec-
sites where the tip of the roving catheter contacts cardiac tis- trical information of the right atrium, superior vena cava, and
sue. The system can construct a map that provides informa- coronary sinus was performed and showed in Fig.30.5. This
tion about the site of earliest activation. When using the map showed the actual site of earliest activity to be over the
CARTO system it is important to correlate the 3D color maps Carina between the left pulmonary veins as shown by the
with information provided from the electrograms taken from tachycardia EKG.
different cardiac places and verify and appropriate informa- The corresponding propagation CARTO map is showed
tion provided by the system. in Fig.30.6. The sequence shows a focal origin of the tachy-
In Fig.30.3, the CARTO activation map shows the ear- cardia between the left pulmonary veins (photo 1 of the
liest activation zone over the posterior aspect of the supe- sequence) with spreading of the activation to the posterior
rior vena cava. However, the intra-cardiac electrograms in wall and the roof of the left atrium (photo 2 and 3 of the
Fig.30.3 showed an almost isochronal activation between sequence). The activation reaches the anterior wall of the LA
the distal coronary sinus electrode and the earliest electro- and the posterior aspect of the superior vena cava and almost
gram over the superior vena cava. Moreover, the coronary at the same time the distal coronary sinus is depolarized from
sinus electrodes show a discrete distal to proximal activa- the posterior wall of the LA (photo 4 of the sequence).
tion. The bipolar electrograms registered from the poste- Finally, the RA is passively activated through an iso-
rior aspect of the superior vena cava are double potentials chronic front of waves from the posterior aspect of the supe-
of high amplitude and just 13ms before the onset of the p rior vena cava and the distal coronary sinus (photo 4 of the
wave. The corresponding unipolar electrograms from this sequence).
place showed an r/S pattern. The analysis of the intra- The passive activation of one atrial chamber with the elec-
cardiac bipolar and unipolar electrograms suggests that trical activity originating in the other atrium is a phenomenon
despite the CARTO showing that the earliest activation is that can cause confusion during mapping. Unnecessary radi
over the posterior aspect of the superior vena cava, it would ofrequency applications maybe made targeting the passive
be a passive electrical activation from another cardiac site.1 activation sites without having an effect on the tachycardia.
Furthermore, the CARTO map of the superior vena cava Inter-atrial connections have been described and are
128 L.C. Senz and M.A. Vacca

Fig.30.4 CARTO activation and propagation maps of the right atrium, superior vena cava, and coronary sinus shown in a posterior view

Posterior view Anterior view

Fig.30.5 The CARTO


activation map integrating the
electrical information of the right
atrium, superior vena cava,
coronary sinus, and the left
atrium
Case 30 129

Fig.30.6 CARTO propagation map integrating the electrical information of the right atrium, superior vena cava, coronary sinus, and the left
atrium shown as a sequence of pictures from the superior to inferior views
130 L.C. Senz and M.A. Vacca

responsible for this phenomenon.5 The most recognized inter- tions as through the fossa ovalis and between the right pulmo-
atrial connection between the atrial chambers is through the nary veins and the superior vena cava, Fig.30.8.
Bachmann Bundle, which connects the left atrial appendage Remarkably, in this particular case the earliest activation
and the superior portion of the Crista Terminalis in the ante- of the RA was founded over the posterior aspect of the supe-
rior aspect of the superior vena cava, Fig.30.7. Another con- rior vena cava. So, it seems to correspond to a passive acti-
nection is between the posterior wall of the LA and the vation through a connection between the right superior
coronary sinus and between the coronary sinus and the RA, pulmonary vein and this part of the superior vena cava as
Fig.30.7. There are other less recognized inter-atrial connec- showed in Fig.30.8.

Anterior-superior view

LAO view
Posterior view

Bachmann
CT

Fig.30.7 Anatomical and


CARTO pictures showing the
inter-atrial connections LAO view

Fig.30.8 Drawing and CARTO pictures showing the inter-atrial connections


Case 30 131

The atrial tachycardia was slowed by radiofrequency organized and 1:1 atrioventricular conduction rhythm
applications over the carina between the left pulmonary veins (showed in Fig.30.2b and magnified in Fig.30.10a) would
as showed in Fig.30.9. This site revealed atrial bipolar elec- be the same ablated atrial tachycardia with fixed exit block of
trogram of low amplitude, fractionated, and earlier 52 ms the conduction from the pulmonary vein to the LA that can
than the onset of the p wave. The corresponding unipolar be confused with a normal sinus rhythm.
electrogram showed a QS pattern. See Fig. 30.3 for This case points out that although electroanatomic map-
comparison. ping is a useful tool, the information obtained by the system
The EKG post ablation showed a p wave with a positive/ needs to be validated with electrograms and entrainment.
negative morphology in V1, positive in lead I and aVL, and The inter-atrial connections can muddy the interpretation of
was predominantly negative in the inferior leads suggesting the activation map. Because of that registration of electrical
an escape low CT rhythm, Fig.30.10a. Retrospectively, the activity from only one chamber can be misleading.

Fig.30.9 CARTO and ICE showing the location of successful RF ablation


132 L.C. Senz and M.A. Vacca

a Pre ablation Post ablation

II

III

AVR

AVL

AVF

V1

V2

V3

V4

V5

V6

Fig.30.10 (a and b) Comparison between the pre- and post-ablation p waves morphology

References 4. Kistler PM, Roberts-Thomson KC, Haqqani HM, et al. P-wave


morphology in focal atrial tachycardia. J Am Coll Cardiol.
2006;48:1010-1017.
1. Stevenson WG, Soejima K. Recording techniques for clinical elec- 5. Ho SY, Sanchez-Quintana D, Cabrera JA, etal. Anatomy of the left
trophysiology. J Cardiovasc Electrophysiol. 2005;16:1017-1022. atrium: implications for radiofrequency ablation of atrial fibrilla-
2. Markowitz SM, Lerman BB. How to interpret electroanatomic tion. J Cardiovasc Electrophysiol. 1999;10:1525-1533.
maps. Heart Rhythm. 2006;3:240-246.
3. Lemery R, Soucie L, Martin B, etal. Human study of biatrial elec-
trical coupling. Circulation. 2004;110:2083-2089.
Case 31

Bradley P. Knight

Case Summary procedure. The 12-lead ECG is shown in Fig. 31.1. The
results of pacing maneuvers were consistent with orthodro-
A young woman with a permanent form of reciprocat- mic AV reentry using a slowly conducting accessory path-
ing tachycardia (PJRT) underwent an electrophysiology way (AP). Where is the AP most likely located?

Fig. 31.1 A 12-lead rhythm strip showing a nearly incessant, supraventricular tachycardia with a long RP interval

B.P. Knight
Division of Cardiology, Northwestern Medical Center, 676 N.
St. Clair, Suite 600, Chicago, IL 60611

A. Natale et al. (eds.), Cardiac Electrophysiology, 133


DOI: 10.1007/978-1-84996-390-9_31, Springer-Verlag London Limited 2011
134 B.P. Knight

Case Discussion

The most likely location for an AP with decremental


conduction that participates in PJRT is the posterior sep-
tum. Therefore, efforts to map the atrial insertion during
tachycardia should initially focus in this region. However,
it is not uncommon for these pathways to be located else-
where. At least two studies have found that the site of
earliest retrograde atrial activation occurs at annular
locations other than the posteroseptal region in approxi-
mately one fourth of cases.
In this case, the earliest retrograde activation during
single echo beats was along the lateral mitral annulus
across from the distal coronary sinus electrode (Fig.31.2).
Radiofrequency ablation at this site eliminated AP conduc-
tion. Less than 5% of cases are found in the left lateral Fig. 31.2 This tracing was recorded during catheter ablation of a
supraventricular tachycardia. Shown are surface electrograms from
location.
leads I, II, III, V1, and V5, and the intracardiac electrograms from the
high right atrium (HRA), the ablation catheter (Abl) positioned at the
lateral mitral annulus where the earliest local activation could be
recorded, and the coronary sinus (CS). The vertical line shows that the
earliest local atrial activation precedes the onset of the p-wave on
thesurface tracing

Bibliography

Gaita F, Haissaguerre M, Giustetto C, etal. Catheter ablation of PJRT


with RF current. J Am Coll Cardiol. 1995;25:648-654.
Meiltz A, Weber R, Halimi F, etal. PJRT in adults: Peculiar Features
and Results of RF Ablation. Europace. 2006;8:21-28.
Case 32

Luigi Di Biase, Rodney P. Horton, and Andrea Natale

Case Summary Which should be the diagnostic and therapeutic steps for
this patient?
A 66-year-old male with essential hypertension and a history
a) Perform an Echocardiogram
of bilateral hernia surgery, was self-referred to our institution
b) Perform a Transesophageal Echocardiogram
regarding persistent shortness of breath status post-redo
c) Perform a CT scan e/o MRI scan
pulmonary vein isolation performed at another institution.
d) Perform a CT scan and a Transesophageal Echocardiogram
At admission he denied any chest discomfort, palpitations,
e) Perform a ventilation/perfusion scan
presyncope, or syncope.
f) Perform a chest x ray
The patient began experiencing symptomatic episodes of
g) Perform all the above
atrial fibrillation in 2003.
His echocardiogram and blood screening resulted within
the normal range.
Case Discussion
The arrhythmia was initially treated both with propafenone
and flecainade at the appropriate dosages. These AADs were
ineffective. The patient had a CT scan and an echocardiogram 9 months
Since his AF became more persistent, the patient under- after the redo procedure.
went several cardioversions. The echocardiogram showed normal EF and an enlarged
In 2006 he finally underwent pulmonary vein isolation pulmonary artery with increased pulmonic valve velocities
(PVI). The procedure had no complications and a CT scan consistent with pulmonary hypertension.
performed 3 months after the ablation showed no pulmonary Additionally there was mild right ventricular hypertrophy,
vein stenosis. Five months following the ablation, the patient and mild tricuspid regurgitation.
went into atrial flutter which required cardioversion. In 2008 The TEE was technically difficult and with suboptimal
the patient underwent a repeat procedure. echocardiographic images. No thrombus was appreciable in
One month following the second ablation procedure, he the left atrium and the left atrium appendage. Solely the left
started experiencing progressively worsening dyspnea on superior pulmonary vein was visualized. EF was estimated at
exertion with daily activities such as walking across the 60%. Generally all the structures were poorly visualized.
street. He also reported orthopnea. The Doppler echocardiogram showed increased velocities in
At admission, his blood screening and physical examina- the distal portion of the left superior pulmonary vein, consis-
tion resulted normal. tent with a significant stenosis.
The CT scan showed complete occlusion of the LIPV
and severe stenosis of both left and right superior pulmo-
nary veins. An enlarged pulmonary artery was also noted
L. Di Biase (*)
(Fig. 32.1 and 32.2).
Department of Electrophysiology, St. Davids Medical Center, Based on the CT reports which should be the treatment
1015 32nd Street, Suite 506, Austin, TX 78705 stategy?
email: dibbia@gmail.com
a) Anticoagulation and clinical follow up
R.P. Horton
Department of Electrophysiology, Texas Cardiac Arrhythmia Institute,
b) Dilation with baloon and anticoagulation
1015 East 32nd Street, Austin, TX 78705 c) Dilation with balloon, anticoagulation and follow up with
VQ/scan
A. Natale
Texas Cardiac Arrhythmia Institute, St. Davids Medical Center, d) None of the above
1015 East 32nd Street, Suite 408, Austin, TX 78705 e) All the above

A. Natale et al. (eds.), Cardiac Electrophysiology, 135


DOI: 10.1007/978-1-84996-390-9_32, Springer-Verlag London Limited 2011
136 L. Di Biase et al.

There were no significant gradient across these stenosis


following the dilation.
Based on the good angiographic results of the balloon
angiography the patient should:
a) Stop anticoagulation therapy
b) Continue anticoagulation therapy and repeat the CT scan
at follow up
c) Continue anticoagulation therapy and repeat TEE at
follow up
d) All the above
e) None of the above
The reported incidence of PV stenosis/occlusion defined as
>70% narrowing or 90% affects 3.4% of patients following
catheter ablation of atrial fibrillation.13 The incidence has
decreased in the most recent years due the utilization of dif-
ferent techniques for the PV isolation that limit the burnings
at the antrum of the pulmonary veins which is at a consider-
able distance from the true PV ostia.1
Fig.32.1 CT scan showing complete occlusion of the LIPV and severe The clinical presentation of these patients is variable;
stenosis of both left and right superior pulmonary veins from totally asymptomatic or with mild dyspnea to severe
dyspnea with hemoptysis, fever, or pleuritic chest pain.1,3
The pathogenesis is due to an initial ablative insult that
precipitates a healing reaction culminating in an endovascu-
lar contraction and proliferation of the elastic lamina/intima.
Misdiagnosis is very common in these patients (pulmo-
nary embolism, lung cancer, pneumonia, and new onset of
asthma, are the most common1,3), because symptoms may
occur far from the procedural time. This is why imaging fol-
lowing catheter ablation is crucial also in asymptomatic
patient. In fact, PV stenosis may progress insidiously.
Patients can present with a variety of respiratory symp-
toms, but may also remain asymptomatic especially when
only one vein demonstrates severe stenosis.
Chest x-ray is usually not helpful in diagnosing this
condition.
As shown in this case, TEE does not always provide clear
images of the pulmonary veins.
CT scan and/or magnetic resonance imaging are the best
tools for diagnosis.4,5 Many groups suggest that the assess-
ment of PV diameter using CT scan or MRI 3 months after
ablation provides the best identification of PV stenosis
since the caliber remain relatively stable beyond 3 months
Fig.32.2 CT scan showing an enlarged pulmonary artery with severe after ablation. However, late progression from a mild to a
stenosis of the right and left superior pulmonary veins severe PV stenosis has been described and a repeat imaging
is required for any patient who develops new symptoms
consistent with stenosis.1 In patients with moderate to
Because of the significant dyspnea and pulmonary hyperten- severe stenosis, ventilation/perfusion (V/Q) scan may be
sion related to pulmonary vein stenosis of at least three pul- useful because it provides a good measure of the lung func-
monary veins, the decision was made to undergo balloon tionality. The CSI index (cumulative stenosis index [CSI] =
angioplasty and possible stenting. sum of the percent stenosis of the unilateral veins divided
The left superior, left inferior, and right superior pulmo- by the total number of ipsilateral veins) has been proposed
nary veins were found severely occluded (90%) at the with a cutoff value of 75% to identify patients at greatest
angiograms and were dilated with balloon angioplasty. risk of severe symptoms and lung diseases. In these patients,
Case 32 137

early and, when required, repeated PV intervention should References


be considered for restoration of pulmonary flow and pre-
vention of associated lung disease.1 Late opening of the 1. Di Biase L, Fahmy TS, Wazni OM, et al. Pulmonary vein total
vessel although feasible will not be able to reduce patients occlusion following catheter ablation for atrial fibrillation: clinical
symptoms and restore lung functionality. In the absence of implications after long-term follow-up. J Am Coll Cardiol. 2006;48:
2493-2499.
symptoms, there is no consensus on the best treatment
2. Saad EB, Marrouche NF, Saad CP, Natale A etal. Pulmonary vein
strategy and the CSI index may be useful to identify patients stenosis after catheter ablation of atrial fibrillation: emergence
at higher risk. Balloon angioplasty with or without stenting of a new clinical syndrome. Ann Intern Med. 2003 Apr 15;
has been shown to achieve satisfactory results although 138(8):634-638.
3. Saad EB, Rossillo A, Saad CP, etal. Pulmonary vein stenosis after
restenosis requiring repeat intervention is necessary in
radiofrequency ablation of atrial fibrillation: functional character-
nearly 4550% of patients.1,6 ization, evolution, and influence of the ablation strategy. Circulation.
As shown in this case, after the second ablation, the 2003;108:3102-3107.
patient did not undergo CT scan. 4. Packer DL, Keelan P, Munger TM, et al. Clinical presentation,
investigation, and management of pulmonary vein stenosis compli-
As described earlier in the text, imaging is very important
cating ablation for atrial fibrillation. Circulation. 2005;111:
to detect stenosis even in asymptomatic patients. The dyspnea 546-554.
reported by the patient was underestimated for several months. 5. Neumann T, Sperzel J, Dill T, etal. Percutaneous pulmonary vein
This resulted in severe pulmonary hypertension and pulmo- stenting for the treatment of severe stenosis after pulmonary vein
isolation. J Cardiovasc Electrophysiol. 2005;16:1180-1188.
nary artery dilatation (as demonstrated by the CT scan).
6. Qureshi AM, Prieto LR, Latson LA, etal. Transcatheter angioplasty
In such a case, late dilatation of the occluded pulmonary for acquired pulmonary vein stenosis after radiofrequency ablation.
veins although angiographically successful may not solve Circulation. 2003;108:1336-1342.
the patients symptoms.

Case 33

M. Eyman Mortada, Jasbir S. Sra, and Masood Akhtar

Case Summary What is the diagnosis? Radiofrequency ablation was


performed successfully. Where was the ablation performed?
A 55-year-old male presented with recurrent symptoms of
dizziness and palpitations. He was found to have stable vitals
and abnormal rhythm (Fig. 33.1).
The 12-lead ECG at baseline during sinus rhythm is Case Discussion
shown in Fig.33.2.
The patient underwent a complex electrophysiology study. In defining a wide complex tachycardia, it is important
The intracardiac tracing shown in Fig.33.3 demonstrates the to differentiate between ventricular tachycardia (VT) and
wide complex tachycardia rhythm. supraventricular tachycardia (SVT) (associated with aberrancy,

Fig. 33.1 At presentation, the patient was found to have stable vitals and abnormal rhythm demonstrated in this 12-lead ECG

M.E. Mortada(*), J.S. Sra, and M. Akhtar


Department of Electrophysiology, Aurora Cardiovascular Services,
Aurora Sinai/Aurora St. Lukes Medical Centers, University of
Wisconsin School of Medicine and Public Health, 2801 W.
Kinnickinnic River Parkway, #777, Milwaukee, WI 53215
e-mail: publishing4@aurora.org

A. Natale et al. (eds.), Cardiac Electrophysiology, 139


DOI: 10.1007/978-1-84996-390-9_33, Springer-Verlag London Limited 2011
140 M.E. Mortada et al.

preexcitation, preexistent wide QRS complex, or antidromic beat (Fig.33.7). Intravenous adenosine affects the atrioven-
AV reentry tachycardia). All of these are possible diagnostic tricular node; hence it affects almost all SVTs by terminating
options in this case. the tachycardia or producing atrioventricular dissociation.
The seventh complex on the wide QRS complex tachycar- Few ventricular tachycardias are sensitive to adenosine (e.g.,
dia is premature and results in an earlier initiation of the normal heart VT), thus response to adenosine is no help in
tachycardia after the premature beat. This response fits all of making a diagnosis. The absence of structural heart disease
the above mentioned diagnostic possibilities. If this is a VT, is an important piece of information, increasing the likeli-
the premature beat could be a premature ventricular capture hood that the tachycardia is SVT rather than VT in origin.
(PVC) that entrained the ventricular tachycardia and acceler- The most helpful piece of information for differential diag-
ated the rhythm for that one beat (Fig.33.4). nosis in this case was the baseline 12-lead ECG, which showed
If this is atrial flutter with 2:1 AV conduction or reentry the same QRS morphology. Therefore, VT could be easily
atrial tachycardia associated with left bundle branch block, excluded.
the premature beat could be a premature atrial capture (PAC) Acceleration of the tachycardia by one premature beat, as
that entrained the reentry circuit and accelerated the rhythm seen in the first ECG (the seventh beat), favors the diagnosis
for that one beat (though atrial flutter option is the least of orthodromic AV reentry tachycardia (AVRT) over AV
possible, due to the fast rate of the atrial flutter, 366 bpm nodal reentry tachycardia (AVNRT).
[160ms]) (Fig.33.5). Intracardiac electrocardiograms taken during the tachy-
If this is an orthodromic AV reentry tachycardia with left cardia revealed the activation of the atria to be from the distal
bundle branch block, the premature beat could be a PAC or to the proximal coronary sinus, followed by the high right
PVC that entrained the circuit and accelerated the rhythm for atrium and His (septum). It also revealed constant VA link-
that one beat (Fig.33.6). ing. These findings confirmed the diagnosis of an orthodro-
If this is an AV nodal reentry tachycardia with left bundle mic AVRT using a left lateral free-wall accessory pathway
branch block, the premature beat could be a PAC that and the best location for successful radiofrequency ablation
entrained the circuit and accelerated the rhythm for that one is the lateral side of the mitral valve annulus.

Fig.33.2 The patients 12-lead ECG at baseline during sinus rhythm


Case 33 141

Fig.33.3 Intracardiac tracing


demonstrating the wide complex
tachycardia rhythm (Rhythm
tracing from top to bottom: Lead I,
Lead II, Lead V1, high right
atrium recording HRA,
proximal coronary sinus
recording (CS 9,10) to distal
coronary sinus recording (CS 1,2),
and His bundle recording
proximal HSp, and distal HSd.)
HRAp: proximal high right
atrium; CS: coronary sinus; HSp:
proximal His; HSd: distal His

Fig.33.4 The seventh complex on the wide QRS complex tachycardia


Fig. 33.6 If this patient has an orthodromic AV reentry tachycardia
is premature and results in an earlier initiation of the tachycardia after
with left bundle branch block, the premature beat could be a PAC or
the premature beat. If this patient has a ventricular tachycardia (VT),
PVC that entrained the circuit and accelerated the rhythm for that one
the premature beat could be a premature ventricular capture that
beat
entrained the VT and accelerated the rhythm for that one beat

Fig.33.5 If this patient has atrial flutter with 2:1 AV conduction asso- Fig.33.7 If this patient has an AV nodal reentry tachycardia with left
ciated with left bundle branch block, the premature beat could be a pre- bundle branch block, the premature beat could be a PAC that entrained
mature atrial capture that entrained the atrial flutter and accelerated the the circuit and accelerated the rhythm for that one beat
rhythm for that one beat (though this option is the least possible, due to
the fast rate of the atrial flutter--366 bpm 160 ms)

Case 34

Antonio Rossillo, Sakis Themistoclakis, Aldo Bonso,


Andrea Corrado, and Antonio Raviele

Case Summary Case Discussion

The patient is a 28-year-old female with frequent runs of This tachycardia is a long RP tachycardia, thus the differen-
SVT. No relevant structural changes on echocardiography. tial diagnosis includes atypical AVNRT, atrial tachycardia, or
Based on Figs.34.1 and 34.2, what is the likely mechanism AVRT that utilizes a slowly conducting AP. When a single
of the tachycardia? VPC is delivered the tachycardia terminates without reaching

Fig.34.1 12-lead ECG taken


during SVT (paper speed
25mm/s) showing a negative P
wave in the inferior leads with a
short PQ interval and a heart rate
of 120bpm

A. Rossillo (*), S. Themistoclakis, A. Bonso, and


A. Corrado
Cardiovascular Department, Ospedale dellAngelo,
Mestre, Venice, Italy
A. Raviele
Chief of Cardiology, Chief of Cardiovascular,
Cardiovascular Department, Ospedale dellAngelo,
Mestre, Venice, Italy

A. Natale et al. (eds.), Cardiac Electrophysiology, 143


DOI: 10.1007/978-1-84996-390-9_34, Springer-Verlag London Limited 2011
144 A. Rossillo et al.

the atrium. This makes AT unlikely. Distinguishing AVRT refractory VPC, the likely diagnosis is AVRT or PJRT. This
that utilizes a slowly conducting AP versus AVNRT can be tachycardia is often clinically frequent or incessant. In Fig.
difficult, but as the tachycardia terminated during a HIS 34.3 and 34.4 the ablation site is shown.

Fig.34.2 Intracardiac record-


ings taken at baseline during the
electrophysiology study (paper
speed 100mm/s). Four surface
ECG leads are shown (I, aVF,
V1, V6), one bipolar recording
from the pacing catheter (HRA),
three bipolar recordings from the
distal His bundle region (HIS
D=distal, HIS I=medial, and
HIS P=proximal), two bipolar
recordings from the coronary
sinus (CS1=distal CS and CS4
prox CS), and the distal bipolar
recording of the mapping
catheter (MC D Bi). A single
paced beat from the right
ventricle with a coupling interval
allows for anterograde activation
of the His bundle which
interrupted the tachycardia twice.
(A atrium, V ventricle, and H His
bundle)

Fig.34.3 Intracardiac record-


ings (paper speed 200mm/s).
Same display as shown in
Fig.34.2. Ablation site: earliest
activation on MC D Bi
Case 34 145

Fig.34.4 RAO and LAO


projection of mapping catheter
at the ablation site

RAO LAO

Bibliography p athway in the permanent form of junctional reciprocating tachycar-


dia. Am J Cardiol. 1985;55:1639-1641.
Farr J, Ross D, Wiener I, Br FW, Vanagt EJ, Wellens HJ. Reciprocal
Coumel PH, Attuel P, Leclerque JF. Permanent form of junctional recip- tachycardias using accessory pathways with long conduction times.
rocating tachycardia: mechanism, clinical and therapeutic impli Am J Cardiol. 1979;44:1099-1109.
cation. In: Narula OS, ed. Cardiac Arrhythmias. Electrophysiology, Smith RT, Gillette PC, Massumi A, McVey P, Garson A Jr. Transcatheter
Diagnosis and Management. Baltimore/London: Williams & ablative techniques for treatment of the permanent form of junc-
Wilkins; 1979:347-363. tional reciprocating tachycardia in young patients. J Am Coll
Critelli G, Scherillo M, Monda V, DAscia C, Musumeci S, Antignano A. Cardiol. 1986;8:385-390.
Transvenous catheter ablation of the accessory atrioventricular

Case 35

Yash Y. Lokhandwala, Anoop K. Gupta, and Ranjan K. Thakur

Case Summary Case Discussion

A 47-year-old man presented with recurrent tachycardia Local AP conduction in the RF channel is eliminated
(Fig.35.1) which is responsive to verapamil. The patient has evidenced by prolongation of the local VA interval. Yet, an
a normal heart. His ECG is normal in sinus rhythm. In the EP eccentric atrial activation suggesting a left-sided AP persists
lab he is diagnosed with AVRT. Explain what occurs during in the CS recordings.
RF delivery in Fig.35.2. RF delivery at a different location resulted in VA block
(Fig. 35.3). There were two left-sided APs: the first was
anterolateral and the second was posterolateral; they were
34cm away from each other.

Y.Y. Lokhandwala(*)
KEM Hospital, Parel, Mumbai, India
e-mail: yashlokhandwala@hotmail.com
A.K. Gupta
Apollo Hospital, Ahmedabad, India
e-mail: anoopgupta@msn.com
R.K. Thakur
Thoracic and Cardiovascular Institute, Sparrow Health System,
Michigan State University, 405 West Greenlawn, Suite 400,
Lansing, MI 48910

A. Natale et al. (eds.), Cardiac Electrophysiology, 147


DOI: 10.1007/978-1-84996-390-9_35, Springer-Verlag London Limited 2011
148 Y.Y. Lokhandwala et al.

Your Hospital Name in File HEADING.TXT Version WIN200 : EPTRACER V0.48


2008017

II

III

AVR

AVL

AVF

V1

V2

V3

V4

V5

V6

10 mm/mV 25mm/s

Fig.35.1 The ECG suggests that this is likely AVRT, with a left-sided AP (long RP, ST elevation in aVR and ST depression in lead I)
Case 35 149

Your Hospital Name in File HEADING.TXT Version WIN200 : EPTRACER V0.48


2008017
I
AVF
V1
V6

CS910

CS78

CS56

CS34

CS12

HISD

HISP

RFD

RFP

10 mm/mV 100mm/s

Fig.35.2 Intracardiac EGMs during RF energy application. What happens?


150 Y.Y. Lokhandwala et al.

Your Hospital Name in File HEADING.TXT Version WIN200 : EPTRACER V0.48


2008017

AVF

V1

V6

CS910

CS78

CS56

CS34

CS12

HISD

HISP

RFD

RFP

10 mm/mV 100mm/s
Offline printed on 17-05-2008 at 14:23:17.

Fig.35.3 VA block is achieved


Case 36

Eric Buch, Shiro Nakahara, Marmar Vaseghi, Noel G. Boyle,


and Kalyanam Shivkumar

Case Summary cardioversions, she continued to experience tachycardia and


was referred for electrophysiology study and catheter abla-
A 60-year-old female with a history of left atrial myxoma, tion. Surface electrocardiogram showed wide-complex
resected 9 years ago, subsequently developed palpitations, tachycardia with a ventricular rate of 115 beats per minute
fatigue, and tachycardia. Despite antiarrhythmic medica- (Fig.36.1). What is the differential diagnosis of this arrhyth-
tions, including sotalol and amiodarone, and multiple mia based on the electrocardiogram?

Fig.36.1 Surface electrocardiogram shows a regular wide-complex tachycardia at approximately 115 beats per minute

E.Buch (*), S.Nakahara, M.Vaseghi,


N.G. Boyle, and K. Shivkumar
UCLA Cardiac Arrhythmia Center,
David Geffen School of Medicine at UCLA,
10833 Le Conte Ave, A2-237 CHS,
Los Angeles, CA 90095-1679
e-mail: ebuch@mednet.ucla.edu

A. Natale et al. (eds.), Cardiac Electrophysiology, 151


DOI: 10.1007/978-1-84996-390-9_36, Springer-Verlag London Limited 2011
152 E. Buch et al.

Case Discussion Electroanatomic mapping was used to show the right


atrial activation sequence. The tachycardia circuit was around
Diagnostic electrophysiology catheters were placed in the the lateral right atriotomy scar from previous cardiac sur-
high right atrium, at the His bundle, the right ventricular api- gery. Ablation was performed at the superior margin of the
cal septum, and in the coronary sinus. Intracardiac electro- scar and the tachycardia terminated (Fig.36.3). It could not
grams were recorded during tachycardia (Fig.36.2). be reinduced on multiple attempts.

Fig.36.3 Left anterior oblique fluoroscopic view of intracardiac elec-


trophysiology catheters. The tip of the ablation catheter is in the lateral
Fig.36.2 Intracardiac recordings during tachycardia. Left atrial activa- right atrium at the superior margin of the atriotomy scar, at the site of
tion as seen in coronary sinus catheter proceeds from medial to lateral, tachycardia termination
consistent with a focus or reentrant circuit in the right atrium
Case 37

Bradley P. Knight

Case Summary alternative to radiofrequency ablation. One advantage of


cryoenergy over radiofrequency current is the ability to
cryo-map to minimize the likelihood of inadvertent AV
A 25-year-old woman with WolffParkinsonWhite syn-
block.1 As the ablation electrode temperature is lowered
drome was referred for catheter ablation. The 12-lead EKG
below freezing, the tissue reaches a temperature where
is consistent with an anteroseptal accessory pathway (AP)
temporary conduction block occurs well before it reaches a
(Fig. 37.1). A previous attempt at ablation of the AP was
temperature where permanent tissue destruction occurs.
aborted due to its proximity to the His bundle. Another
This permits the ability to monitor during ablation for unde-
attempt at ablation was made. The baseline rhythm was sinus
sirable results, such as AV block, before a permanent lesion
with ventricular preexcitation. Orthodromic AVRT was
is created.
inducible and the AP had a short refractory period that was
Fig.37.2 shows early activation at the para-Hisian region
less than 250ms. The earliest ventricular activation during
with an activation time of -30 ms. Fig.37.3 shows disap-
sinus rhythm was at a site where there was also a large His
pearance of ventricular preexcitation when the temperature
bundle recording during block in the AP. How would you
is lowered during cryoablation. As an iceball forms on the
proceed?
tip of the electrode, the electrogram is lost. After rewarm-
ing of the electrode occurs, a large His bundle recording
can be seen (Fig. 37.4). After elimination of the AP, AV
Case Discussion conduction was intact and there was no VA conduction.
The AP did not recover during long-term follow-up.
In this case, the patient has symptomatic WPW with a high Cryoablation should be considered when attempting to
risk AP. There is a significant risk of causing irreversible ablate an AP that is located in the para-Hisian or midseptal
AV block with RF current. Cryoenergy ablation is an region, to minimize the risk of AV block.

B.P. Knight
Division of Cardiology, Northwestern Medical Center,
676 N. St. Clair, Suite 600, Chicago, IL 60611
e-mail: bknight@nmff.org

A. Natale et al. (eds.), Cardiac Electrophysiology, 153


DOI: 10.1007/978-1-84996-390-9_37, Springer-Verlag London Limited 2011
154 B.P. Knight

Fig.37.1 This 12-lead EKG shows ventricular preexcitation consistent with an anteroseptal accessory pathway

Successful cryo site

Fig.37.2 This tracing was


recorded during catheter ablation
of an anteroseptal accessory
pathway. Shown are surface
electrograms from leads I, II, III,
V1, and V5, and the intracardiac
electrograms from the high right
atrium (HRA), the radiofre- 30 ms
quency ablation catheter (Abl)
positioned near the His bundle
region, the cryo catheter (CRYO)
with the tip positioned at the
earliest local ventricular
activation, and the right
ventricular apex (RVA). The local
ventricular activation recorded
from the cryo catheter is 30 msec
prior to the onset of the delta
wave University of Chivago
Case 37 155

Fig.37.3 This figure shows


During cryoablation
disappearance of preexcitation
during cryo-ablation. The format
is the same as figure 37.2

University of Chicago

Appearance of his bundle


during rewarming after cryo

Fig.37.4 This figure shows the


reappearance of the His-bundle
recording after rewarming of the
cryo-ablation electrode. The
format is the same as figure 37.2 University of Chicago

Reference

1. Gaita F etal. Safety and efficacy of cryoablation of accessory path-


ways adjacent to the normal conduction system. J Cardiovasc
Electrophysiol. 2003;14:825-829.

Case 38

Luigi Di Biase, Rodney P. Horton, and Andrea Natale

Case Summary c) Repeat catheter ablation targeting non PV site


d) AV node ablation and pacing
e) None of the above
A 65-year-old male with a 15-year history of symptomatic
persistent atrial fibrillation was referred to our institution. The
patient had a normal left ventricular ejection fraction. The
patient denied chest discomfort, presyncope, or syncope.
Atrial fibrillation was initially chemically cardioverted
Case Discussion
with Quinidine and Diltiazem.
A few years later the drugs were ineffective and the patient The patient was taken to the electrophysiology laboratory for
underwent three electrical cardioversions. Quinidine and a repeat ablation. During the third procedure, before ablation
Diltiazem were replaced with Tikosyn. Because Tikosyn was in the left atrium, ablation in the persistent left superior vena
ineffective, the patient decided to undergo pulmonary vein cava (SVC) was performed. Once isolation was achieved, the
isolation. Five months following the procedure, the patient ablation catheter was moved to the left atrium. The atrial flut-
experienced symptoms related to atrial flutter with rapid ven- ter was prolonged and interrupted during ablation along the
tricular response. For these reasons the patient underwent a mitral annulus. During and after the infusion of high-dose
second ablation procedure. During the second procedure, the isoproterenol (20mcg/min), there were no evidence of addi-
patient did not show any recovery of conduction around the tional firing sites. After 9 months of follow-up, the patient
pulmonary veins. The presenting arrhythmia was atrial flut- was asymptomatic and free from arrhythmia recurrences.
ter, which was terminated during ablation in the coronary This case shows that in a number of patients electrical
sinus. Two months after the second procedure the patient disconnection of the pulmonary veins is not sufficient to pre-
developed recurrence of left atrial flutter. The CT scan after vent arrhythmic recurrences.
the second procedure is shown (Figs.38.1 and 38.2). This is possible in patients with left atrial scar preced-
What do you propose next for management of this patients ing the ablation procedure, non-paroxysmal AF patients,
rhythm abnormality, would a repeat ablation be an option, if females,1 and in patients with venous anomaly including
so, what is the ablation target? persistent left SVC.2
What do you propose next? In these patients, isolation of the left SVC is necessary
together with the isolation of the pulmonary veins to prevent
a) Clinical follow-up with AADs for rate control recurrences. Whether isolation of the left SVC without isola-
b) Propose a MAZE or COX procedure tion of the PVs would have been sufficient to treat these
patients is not known.
Persistence of the left SVC is a congenital anomaly
L. Di Biase (*) resulting from an abnormal development of the coronary
Department of Electrophysiology, St. Davids Medical Center, sinus. In these patients, the left cardinal vein does not oblit-
1015 32nd Street, Suite 506, Austin, TX 78705
erate during the fetal life, and on the contrary, persists drain-
e-mail: dibbia@gmail.com
ing into either the left atrium or the right atrium through an
R.P. Horton enlarged CS. The described prevalence of left SVC persis-
Department of Electrophysiology, Texas Cardiac Arrhythmia Institute,
1015 East 32nd Street, Austin, TX 78705 tence varies between 0.3% and 2% in individuals with a
e-mail: rodney.horton@gmail.com normal heart.3,4
A. Natale
A persistence of the left SVC can be diagnosed with
Texas Cardiac Arrhythmia Institute, St. Davids Medical Center, echocardiography based an unusual enlarged CS, but the
1015 East 32nd Street, Suite 408, Austin, TX 78705 gold standards are CT scan and MRI.

A. Natale et al. (eds.), Cardiac Electrophysiology, 157


DOI: 10.1007/978-1-84996-390-9_38, Springer-Verlag London Limited 2011
158 L. Di Biase et al.

Fig.38.1 CT scan showing the In the above-mentioned case, spontaneous ectopies origi-
left atrium, the pulmonary nating at different levels in the left SVC were present. It is
veins, and the persistent LSVC
always important to look for unusual sources of AF/Aflutter
especially in patients presenting for repeat procedures. Unusual
sources of AF should be considered after the failure of one or
more procedures for AF ablation. Isolation of the PVs is not
sufficient to prevent recurrences when a left SVC is present.
Thus, diagnosis and isolation of the left SVC appears
critical to avoid AF recurrence in all patients with AF and
with this venous anomaly.

References

1. Verma A, Wazni OM, Marrouche NF, etal. Pre-existent left atrial


scarring in patients undergoing pulmonary vein antrum isolation: an
independent predictor of procedural failure. J Am Coll Cardiol.
2005;45:285-292.
2. Elayi CS, Fahmy TS, Wazni OM, Patel D, Saliba W, Natale A. Left
superior vena cava isolation in patients undergoing pulmonary vein
Fig. 38.2 CT scan showing a antrum isolation: impact on atrial fibrillation recurrence. Heart
persistent LSVC draining into Rhythm. 2006;3:1019-1023.
the coronary sinus 3. Gonzalez-Juanatey C, Testa A, Vidan J, etal. Persistent left superior
vena cava draining into the coronary sinus: report of 10 cases and
literature review. Clin Cardiol. 2004;27:515-518.
The left SVC when present is always a source of ectopies 4. Fraser RS, Dvorkin J, Rossall RE, Eidem R. Left superior vena
that can initiate AF through the electrical connections with cava: a review of associated congenital heart lesions, catheter-
the LA and CS. Hsu etal.5 suggested that as a consequence ization data and roentgenologic findings. Am J Med. 1961;31:7
11-716.
of the abnormal embryologic development, the persistent of 5. Hsu LF, Jais P, Keane D, et al. Atrial fibrillation originating
the left SVC may be associated with the presence of electri- from persistent left superior vena cava. Circulation. 2004;109:
cal tissue, responsible for arrhythmias. 828-832.
Case 39

M. Eyman Mortada, Jasbir S. Sra, and Masood Akhtar

Case Summary changed to 1:1 AV conduction. This excludes the diagnosis


of accessory pathway-mediated tachycardia, as it would not
occur in the presence of 2:1 AV block.
A 38-year-old female complained of recurrent episodes of
The patient was symptomatic with her tachycardia, so the
lightheadedness and palpitations over several months.
best next step in management is to convert her to normal
She came to the emergency room with the same symp-
rhythm and then perform further work-up to diagnose and
toms, at which time her 12-lead ECG was as follows
treat her condition. She was hemodynamically was stable,
(Figs. 39.139.4).
therefore DC cardioversion is not indicated, but adenosine
Two ventricular extrastimuli (S1) at cycle lengths of
IV bolus is the best first-line approach. b-blockers or calcium
400ms were administered during the tachycardia (Fig.39.5).
channel blockers are acceptable therapies, yet, complex elec-
What is the best initial management, and what is the most
trophysiology study post conversion is more appropriate to
likely diagnosis?
confirm the diagnosis and cure the patients arrhythmia by
ablation therapy.
The first intracardiac tracing (Fig. 39.4) showed narrow
Case Discussion complex tachycardia with two atrial activities recorded for
each incident of ventricular activity. When there ventricular
activity, the VA relationship was fixed. Notice that the His
The P-wave occurring between the two R-waves, seen in the
was present before each atrial activity, which suggests the
first 12-lead ECG of this young female with documented
location of the block is in the HisPurkinje system in a 2:1
normal PR interval during sinus rhythm in the past, suggests
pattern due to refractoriness. The first ventricular extrastim-
a diagnosis of supraventricular tachycardia, specifically AV
uli penetrated the Purkinje system but not the His. Therefore,
nodal reentry tachycardia (AVNRT) with 2:1 AV conduction.
it did not engage the circuit and did not change the cycle
Since we were unable to exclude other possibilities, such as
length of the tachycardia. The second ventricular extrastim-
atrial tachycardia a complex electrophysiology study was
uli was given earlier in the tachycardia cycle length and it
required.
penetrated also to a level below the His. Hence, the cycle
When the patients rhythm spontaneously changed from a
length of activation at this vulnerable location of block has
460- to a 230-ms R-R cycle length (Figs 39.1 and 39.3), it
shortened. The His-Purkinje systems refractoriness is
revealed the presence of 2:1 AV conduction, which then
directly related to the duration of proceeding cycle length1.
Therefore, the site of blocks refractory period is also short-
ened. That leads to the penetration of the next atrial activity
M.E. Mortada (*), J.S. Sra, and M. Akhtar into the His-Purkinje system and setting a 1:1 AV conduc-
Department of Electrophysiology, Aurora Cardiovascular Services, tion. Notice the fixed VA relationship before and after the
Aurora Sinai/Aurora St. Lukes Medical Centers, University of
Wisconsin School of Medicine and Public Health, 2801
ventricular extrastimulation. This linking suggests that the
W. Kinnickinnic River Parkway, #777, Milwaukee, WI 53215 tachycardia diagnosis is most likely AV nodal reentry
e-mail: publishing4@aurora.org tachycardia.

A. Natale et al. (eds.), Cardiac Electrophysiology, 159


DOI: 10.1007/978-1-84996-390-9_39, Springer-Verlag London Limited 2011
160 M.E. Mortada et al.

Fig.39.1 The patients 12-lead ECG upon admission to emergency room with symptoms of lightheadedness and palpitations

Fig.39.2 The patients previous baseline 12-lead ECG


Case 39 161

Fig.39.3 The patient spontaneously switched into the rhythm shown in this 12-lead ECG, accompanied by worsening symptoms, but she remained
hemodynamically stable

Fig.39.4 Intracardiac tracing of the same tachycardia the patient pre- HIS, and right ventricular apex recording RVa. Labels used in this
sented first to the ER. (Rhythm tracing from top to bottom: lead I, Lead figure are similar to the labels used in subsequent figures)
II, Lead V1, high right atrium recording HRA, His bundle recording
162 M.E. Mortada et al.

Fig.39.5 Two ventricular extrastimuli (S1) at cycle lengths of 400 ms were administered during the tachycardia. The tachycardia accelerated
from 2:1 AV conduction to 1:1 AV conduction

Reference

1. Akhtar M, Mahmud R, Tchou P, Denker S, Gilbert CJ. Normal elec-


trophysiologic responses of the human heart. Cardiol Clin 1986;4:
365386.
Case 40

Antonio Rossillo, Sakis Themistoclakis, Aldo Bonso, Andrea Corrado,


and Antonio Raviele

Case Summary Case Discussion

The patient is a 40-year-old female with a prior history of This bypass tract is likely located in the posterior septal space
palpitations and is being medically managed for mild hyper- given the morphology of the maximally preexcited tracing
tension. The episodes became more frequent 2months prior (Figs. 40.1, 40.2) and it was successfully as shown in
to the time of ablation. The ECG recordings were taken while figs. 40.340.14.
the patient was having palpitations. The ECG revealed both
orthodromic and antidromic tachycardia. Where is the likely
position of the AP?

A. Rossillo, S. Themistoclakis, A. Bonso, A. Corrado,


and A. Raviele(*)
Cardiovascular Department, Ospedale dellAngelo, Mestre,
Venice, Italy
e-mail: araviele@venicearrhythmias.org

A. Natale et al. (eds.), Cardiac Electrophysiology, 163


DOI: 10.1007/978-1-84996-390-9_40, Springer-Verlag London Limited 2011
164 A. Rossillo et al.

Fig.40.1 The 12-lead resting ECG (paper speed 25mm/s) showing normal sinus rhythm with a ventricular rate of 109bpm with a short PR inter-
val (118ms) and a QRS width of 81ms with mild delta wave in the initial portion of the QRS
Case 40 165

Fig.40.2 12-lead ECG taken during atrial pacing (paper speed 25mm/s) showing maximum ventricular preexitation with an early transition
of the R wave between leads V1 and V2 and negative wave in the inferior leads
166 A. Rossillo et al.

Fig.40.3 Intracardiac recordings taken at baseline during the electro- (3) five unipolar recordings from the coronary sinus (CS), (4) the uni-
physiology study (paper speed 200mm/s). Four surface ECG leads are polar (MC U-CATH) and the distal bipolar recording of the mapping
shown (I, aVF, V1, V6): (1) bipolar recording from the high right atrium catheter (MC D). (A atrium, V ventricle and H His bundle)
(HRA), (2) bipolar recording from the distal His bundle region (HIS D)
Case 40 167

Fig.40.4 Intracardiac recordings taken during baseline while ventricular pacing (paper speed 200mm/s). Same display as shown in Fig.40.3 (A
atrium, V ventricle)
168 A. Rossillo et al.

Fig. 40.5 Intracardiac recordings taken during programmed atrial preexitation was still present and in Panel B: With a coupling interval
stimulation (paper speed 100 mm/s). Same display as shown in of 280 ms suddenly disappeared (ERP of bypass fiber). (A atrium, V
Fig. 40.3. Panel A: At a coupling interval of 290 ms the ventricular ventricle and H His bundle)
Case 40 169

Fig.40.5 (continued)
170 A. Rossillo et al.

Fig.40.6 Intracardiac recordings taken during programmed ventricu- was 130ms and suddenly increased and in Panel B to 180ms with a
lar stimulation (paper speed 200 mm/s). Same display as shown in coupling interval of 270 ms (retrograde ERP of a bypass fiber).
Fig.40.3. Panel A: At a coupling interval of 280ms, the Stim-A interval (Aatrium, V ventricle)
Case 40 171

Fig.40.6 (continued)
172 A. Rossillo et al.

Fig.40.7 Intracardiac recordings taken during programmed atrial stimulation (paper speed 200mm/s). Same display as shown in Fig.40.3. AVRT
induction (AA 280ms) with a single extrastimulus (drive 500ms and S1S2 230ms). (A atrium, V ventricle)
Case 40 173

Fig.40.8 Intracardiac recordings (paper speed 200mm/s). Same dis- fiber without ventricle activity that is present only in the unipolar
play as shown in Fig.40.3. Ablation site. The bipolar recording of the recording. (A atrium, K Kent bundle, and V ventricle)
ablation catheter shows presence of an atrial signal and of the bypass
174 A. Rossillo et al.

Fig.40.9 Intracardiac recordings (paper speed 100mm/s). Same dis- fiber without ventricle activity that is present only in the unipolar
play as shown in Fig.40.3. Ablation site. The bipolar recording of the recording (A atrium, K Kent bundle, and V ventricle)
ablation catheter shows presence of an atrial signal and of the bypass
Case 40 175

Fig.40.10 Intracardiac recording taken during radiofrequency deliv- ardiac recording showed a clear separation between atrial (A) and ven-
ery (paper speed 200mm/s). Same display as shown in Fig.40.3. The tricular signals (V). H His signal
second beat disappeared during ventricular preexitation and the intrac-
176 A. Rossillo et al.

Fig.40.11 Intracardiac recordings taken during radiofrequency deliv- the ventricle signal on the ablation catheter before and after the elimina-
ery (paper speed 25 mm/s). Same display as in Fig. 40.3. The arrow tion of the bypass fibers
shows the elimination of the ventricular preexcitation. The circle shows
Case 40 177

A
Fig.40.12 Intracardiac recordings taken during programmed ventricu- was 160ms and Panel B: At a coupling interval of 270 a decremental
lar stimulation (paper speed 200 mm/s). Same display as shown in VA conduction was present (A atrium, V ventricle)
Fig.40.3. Panel A: At a coupling interval of 280ms, the Stim-A interval
178 A. Rossillo et al.

Fig.40.12 (continued)
Case 40 179

Fig.40.13 Intracardiac recording taken at the end of the procedure (paper speed 200mm/s). The same display as shown in Fig.40.3 (A atrium,
Vventricle, H His bundle)
180 A. Rossillo et al.

Bibliography

Calkins H, Langberg J, Sousa J, etal. Radiofrequency catheter abla-


tion of accessory atrioventricular connections in 250 patients.
Abbreviated therapeutic approach to Wolff-Parkinson-White syn-
drome. Circulation. 1992;85:1337-1346.
Hassaguerre M, Dartigues JF, Warin JF, Le Metayer P, Montserrat P,
Salamon R. Electrogram patterns predictive of successful catheter
ablation of accessory pathways. Value of unipolar recording mode.
Circulation. 1991;84:188-202.
A RAO LAO Jackman WM, Wang XZ, Friday KJ, etal. Catheter ablation of acces-
sory atrioventricular pathways (Wolff-Parkinson-White syndrome)
by radiofrequency current. N Engl J Med. 1991;324:1605-1611.
Scheinman MM, Wang YS, Van Hare GF, Lesh MD. Electrocardio
graphic and electrophysiologic characteristics of anterior, midseptal
and right anterior free wall accessory pathways. J Am Coll Cardiol.
1992;20:1220-1229.
H MV

RA
TT

CS
MCV

TV
B

Fig.40.14 Panel A: RAO and LAO of the mapping catheter at the abla-
tion site in the proximal coronary sinus. Panel B: LAO picture of the
mapping catheter at the ablation site in the proximal coronary sinus (RA
right atrium, TT Todaros tendon, TV tricuspid valve, CS coronary sinus,
MCV middle cardiac vein, H His bundle, MV mitral valve)
Case 41

Yash Y. Lokhandwala, Anoop K. Gupta, and Ranjan K. Thakur

Case Summary Case Discussion

A 36-year-old woman with a normal heart had this ECG in Tachycardias, pre- and post-APDs, are AVNRT. The APDs
the lab (Fig. 41.1). A narrow QRS tachycardia is seen block in the first slow pathway and conduct by another slow
without visible P waves; sharp J point in V1; this likely pathway (Fig. 41.2). RF energy delivered in slow pathway
represents AVNRT. Multiple APDs change or accelerate region eliminated dual AVN physiology and inducibility.
tachycardia how?

Y.Y. Lokhandwala(*)
KEM Hospital, Parel, Mumbai, India
e-mail: yashlokhandwala@hotmail.com
A.K. Gupta
Apollo Hospital, Ahmedabad, India
e-mail: anoopgupta@msn.com
R.K. Thakur
Thoracic and Cardiovascular Institute, Sparrow Health System,
Michigan State University, 405 West Greenlawn, Suite 400,
Lansing, MI 48910
e-mail: thakur@msu.edu

A. Natale et al. (eds.), Cardiac Electrophysiology, 181


DOI: 10.1007/978-1-84996-390-9_41, Springer-Verlag London Limited 2011
182 Y.Y. Lokhandwala et al.

2008083

II

III

AVR

AVL

AVF

V1

V2

V3

V4

V5

V6

10 mm/mV 25 mm/s

Fig.41.1 SVT with change in cycle length


Case 41 183

2008083
I
AVF
V1
V6

CS78

CS34

CS12

HISD

HISP

rfD

rfP

10 mm/mV 50 mm/s

Fig.41.2 SVT with change in antegrade limb resulting in faster cycle length

Case 42

Eric Buch, Shiro Nakahara, Marmar Vaseghi, Noel G. Boyle,


and Kalyanam Shivkumar

Case Summary sinus, middle electrodes along the lateral right atrial wall,
and proximal electrodes near the roof of the right atrium.
A 59-year-old male was referred for radiofrequency catheter Intracardiac activation map showed atrial flutter with distal
ablation of atrial flutter and atrial fibrillation. He had a 3-year to proximal coronary sinus activation and passive counter-
history of highly symptomatic paroxysmal atrial fibrillation clockwise activation of the right atrium (Fig.42.1).
that failed to respond to multiple medications. He underwent After transseptal catheterization, electroanatomic map-
catheter-based radiofrequency ablation (RFA) with pulmo- ping of the left atrium suggested that the tachycardia circuit
nary vein isolation and subsequently a modified left- and utilized an isthmus between the mitral annulus and the left
right-sided surgical maze procedure after recurrence of atrial inferior pulmonary vein. Entrainment mapping confirmed
fibrillation. Following the maze procedure, patient developed that this left-sided isthmus was critical to the tachycardia
incessant atrial flutter causing palpitations and fatigue. He circuit.
was referred for catheter ablation. During ablation in the region between the left inferior pul-
Diagnostic electrophysiology catheters were placed at monary vein and the mitral annulus (Fig.42.2b), the intrac-
the His bundle, the right ventricular apical septum, and in ardiac activation sequence changed abruptly, yet tachycardia
the coronary sinus. A multipolar halo catheter was placed persisted (Fig.42.3). What is responsible for this change in
inthe right atrium with the distal electrodes in the coronary sequence?

E. Buch(*), S. Nakahara, M. Vaseghi,


N.G. Boyle, and K. Shivkumar
UCLA Cardiac Arrhythmia Center,
David Geffen School of Medicine at UCLA,
10833 Le Conte Ave, A2-237 CHS,
Los Angeles, CA 90095-1679
e-mail: ebuch@mednet.ucla.edu

A. Natale et al. (eds.), Cardiac Electrophysiology, 185


DOI: 10.1007/978-1-84996-390-9_42, Springer-Verlag London Limited 2011
186 E. Buch et al.

Fig.42.1 Intracardiac record-


I
ings during tachycardia.
Coronary sinus activation is from aVF
distal to proximal, consistent V1
with a left atrial tachycardia
V6
circuit. Right atrial activation is
earliest at the high septum ABL d
(HALO 10) due to conduction HIS p
through Bachmans bundle, then
conducts along the roof and HIS m
down the lateral right atrial wall HIS d
(HALO 9 to HALO 5). There is
HALO 10
competing activation through the
coronary sinus (HALO 1 to HALO 9
HALO 4), resulting in fusion of HALO 8
right atrial activation
HALO 7
HALO 6
HALO 5
HALO 4
HALO 3
HALO 2
HALO 1
CS 9, 10
CS 7, 8
CS 5, 6
CS 3, 4
CS 1, 2

Case Discussion atrial flutter with a longer tachycardia cycle length. In


effect, the typical right atrial flutter had been entrained by
Successful ablation of the mitral isthmus, which was criti- the left atrial flutter. The patient then underwent further
cal for perpetuation of the left atrial macroreentrant tachy- ablation at the cavotricuspid isthmus, resulting in termina-
cardia, resulted in termination of that rhythm. However, it tion of the atrial flutter, bidirectional isthmus block, and
unmasked an underlying typical counterclockwise right noninducibility of either tachycardia.
Case 42 187

Fig.42.2 (a) Three-dimensional voltage map of the left atrium, left


posterior oblique view. Large areas of scar are seen (in red), likely
a
from prior catheter and surgical ablation procedures. (b)
Posteroanterior view of same patients left atrium, showing ablation
sites (red points) between left inferior pulmonary vein and mitral
annulus. Pacing at these sites showed entrainment of the atrial flutter
with postpacing interval near tachycardia cycle length

b
188 E. Buch et al.

Fig.42.3 Intracardiac record- I


ings during ablation between left
aVF
inferior pulmonary vein and
mitral annulus. Tachycardia V1
showed abrupt change in V6
activation sequence and cycle
ABL d
length during ablation, with
counterclockwise right atrial HIS p
activation followed by medial to HIS m
lateral coronary sinus activation. HIS d
The last few beats are consistent
with typical counterclockwise HALO 10
TCL 228 ms TCL 280 ms
right atrial flutter HALO 9
HALO 8
HALO 7
HALO 6
HALO 5
HALO 4
HALO 3
HALO 2
HALO 1
CS 9, 10
CS 7, 8
CS 5, 6
CS 3, 4
CS 1, 2
Case 43

Bradley P. Knight

Case Summary tachycardia (AT). Atrial overdrive pacing can be used to help
rule in or rule out atrial tachycardia as the mechanism.1,2
The principle is that during atrial overdrive pacing just
A patient with recurrent PSVT undergoes an electrophysiol-
slightly faster than the tachycardia, a 1:1 AV relationship can
ogy procedure. A regular narrow QRS complex tachycardia
sometimes be maintained. In the case of an AT, the first atrial
is induced with a CL 360ms and a 1:1 AV relationship with
depolarization immediately after atrial pacing is stopped will
simultaneous atrial and ventricular activation. Figure 43.1
occur independent of the last QRS complex that was a result
shows the response immediately after overdrive atrial pacing
of AV conduction during atrial pacing. Therefore the first VA
at a CL of 350ms. Shown are the surface recordings I, II,
interval would most likely differ from the VA interval
V1, and V5 and intracardiac recordings from the high right
during tachycardia. In contrast, in the case of an AV nodal
atrium (HRA), at the His bundle region with the ablation
dependent tachycardia, such as AVNRT or ORT, the VA
catheter (Abl), and right ventricular apex (RV). A stimula-
interval of the first return beat is usually the same as the VA
tion channel is also displayed. What is the diagnosis of the
interval during tachycardia because the atrial depolarization
tachycardia?
is dependent on VA conduction. However, there is always a
small chance that the first VA interval will be the same or
similar as that during tachycardia by chance alone.
Case Discussion In this example, the AH interval is increasing during atrial
pacing, but 1:1 AV conduction remains intact. The first VA
During a regular supraventricular tachycardia with a 1:1 AV interval after pacing is the same as that during tachycardia.
relationship, the differential diagnosis includes AV nodal Because the short VA interval excludes ORT as a mecha-
reentry (AVNRT), orthodromic AV reentry (ORT), and atrial nism,2 the diagnosis is most likely AVNRT.

B.P. Knight
Division of Cardiology, Northwestern Medical Center,
676 N. St. Clair, Suite 600, Chicago, IL 60611
e-mail: bknight@nmff.org

A. Natale et al. (eds.), Cardiac Electrophysiology, 189


DOI: 10.1007/978-1-84996-390-9_43, Springer-Verlag London Limited 2011
190 B.P. Knight

Fig. 43.1 Example of atrial overdrive pacing during tachycardia to determine the mechanism of tachycardia. See text for format and
abbreviations

References 2. Knight BP, Ebinger M, Oral H, Kim MH, Sticherling C, Flemming M,


Pelosi F, Michaud GF, Strickberger SA, Morady F. Diagnostic value
of tachycardia features and pacing maneuvers during paroxysmal
1. Knight BP, Zivin A, Souza J, Flemming M, Pelosi F, Goyal R, Man supraventricular tachycardia. J Am Coll Cardiol 2000;36:574-82.
KC, Strickberger SA, Morady F. A technique for the rapid diagnosis
of atrial tachycardia in the electrophysiology laboratory. J Am Coll
Cardiol 1999;33:775-81.
Case 44

Antonio Rossillo, Sakis Themistoclakis, Aldo Bonso,


Andrea Corrado, and Antonio Raviele

Case Summary anterograde conduction through the fast pathway. According


to this finding, a standard ablative approach targeting the
slow pathway may result in complete AV block. Pace map-
The patient is a 76-year-old male with history of AVNRT
ping Kochs triangle to define anterograde and retrograde
since childhood and very frequent episodes occurring in the
structures of the AV node is useful in preventing this
last 2 months. Ischemic heart disease with mild reduction of
complication.
LVEF was present. Coronary angiography showed a moder-
In this case, pace mapping showed an absence of an
ate stenosis in the right and circumflex coronary and no
anterograde fast pathway and the presence of a retrograde fast
significant stenosis of LAD (Fig.44.1).
pathway in the anteroseptal region of the Kochs triangle.
An EP study was scheduled to define the arrhythmia
Therefore, a single RF lesion was applied in the region of
(Fig.44.2).
the fast pathway, and Fig. 44.3 shows a junctional rhythm
What type of tachycardia is this? What ablative strategy
during the ablation. The red arrow shows when the retro-
would you choose?
grade conduction skips from the fast to the slow pathway
(A=atrium, V=ventricle).
A transient complete AV Block (15s) was present at the
Case Discussion end of RF energy delivery.
The EP study performed at the end of the procedure
The tachycardia is clearly an AVNRT and the presence of a showed a normal Wenckebach point and the tachycardia was
first-degree AV block suggests the absence or pathological no longer inducible.

A. Rossillo (*), S. Themistoclakis, A. Bonso, A. Corrado,


and A. Raviele,
Cardiovascular Department, Ospedale dellAngelo, Mestre,
Venice, Italy
e-mail: araviele@venicearrhythmias.org

A. Natale et al. (eds.), Cardiac Electrophysiology, 191


DOI: 10.1007/978-1-84996-390-9_44, Springer-Verlag London Limited 2011
192 A. Rossillo et al.

Fig.44.1 12-lead resting ECG (paper speed 25mm/s) showing sinus rhythm with first-degree AV block

Fig.44.2 Intracardiac recordings taken during programmed atrial stimulation (paper speed 200mm/s) with the induction of the tachycardia with
a cycle length of 428 ms study (A atrium, V ventricle, and H His bundle)
Case 44 193

Fig.44.3 A junctional rhythm during the ablation. The red arrow shows when the retrograde conduction skips from the fast to the slow pathway
(A atrium, V ventricle)

Bibliography guide application of radiofrequency energy. Circulation. 1992;85:


2162-2175.
Hassaguerre M, Jas P, Shah DC, etal. Analysis of electrophysiological
Delise P, Gianfranchi L, Paparella N, etal. Clinical usefulness of slow activity in Kochs triangle relevant to ablation of the slow AV nodal
pathway ablation in patients with both paroxysmal atrioventricular pathway. Pacing Clin Electrophysiol. 1997;20:2470-2481.
nodal reentrant tachycardia and atrial fibrillation. Am J Cardiol. Jackman WM, Beckman KJ, McClelland JH, et al. Treatment of
1997;79:1421-1423. supraventricular tachycardia due to atrioventricular nodal reentry,
Delise P, Sitta N, Bonso A, et al. Pace mapping of Kochs triangle by radiofrequency catheter ablation of slow-pathway conduction.
reduces risk of atrioventricular block during ablation of atrioven- N Engl J Med. 1992;327:313-318.
tricular nodal reentrant tachycardia. J Cardiovasc Electrophysiol. Natale A, Greenfield RA, Geiger MJ, et al. Safety of slow pathway
2005;16:30-35. ablation in patients with long PR interval: further evidence of fast
Haissaguerre M, Gaita F, Fischer B, etal. Elimination of atrioventricu- and slow pathway interaction. Pacing Clin Electrophysiol. 1997;
lar nodal reentrant tachycardia using discrete slow potentials to 20:1698-1703.

Case 45

Yash Y. Lokhandwala, Anoop K. Gupta, and Ranjan K. Thakur

Case Summary Case Discussion

A 25-year-old man with normal heart presents with SVT. His Tachycardia is instantiated by VPDs. VA interval is long
tachycardia is initiated with ventricular pacing (Figs. 45.1 during tachycardia and the atrial pattern during tachycardia is
and 45.2). The response to ventricular pacing is shown in similar to that of the retrograde A waves (in the limited leads
Fig.45.3. What is the diagnosis? available). V pacing at a faster rate than the tachycardia shows
VA dissociation suggesting that this is atrial tachycardia.

Y.Y. Lokhandwala(*)
KEM Hospital, Parel, Mumbai, India
e-mail: yashlokhandwala@hotmail.com
A.K. Gupta
Apollo Hospital, Ahmedabad, India
e-mail: anoopgupta@msn.com
R.K. Thakur
Thoracic and Cardiovascular Institute, Sparrow Health System,
Michigan State University, 405 West Greenlawn, Suite 400, Lansing,
MI 48910, USA
e-mail: thakur@msu.edu

A. Natale et al. (eds.), Cardiac Electrophysiology, 195


DOI: 10.1007/978-1-84996-390-9_45, Springer-Verlag London Limited 2011
196 Y.Y. Lokhandwala et al.

Ruby hall clinic Version WIN2000/XP : EPTRACER V0.77


2008046

AVF

V1

V6

RAD

RAP

HISD

HISP

10 mm/mV 100 mm/s

Fig.45.1 Initiation of SVT


Case 45 197

2008046

AVF

V1

V6

RAD

RAP

HISD

HISP

10 mm/mV 100 mm/s

Fig.45.2 Activation sequence of the SVT


198 Y.Y. Lokhandwala et al.

2008046
I

AVF

V1

V6

RAD

RAP

HISD

HISP

10 mm/mV 100 mm/s

Fig.45.3 Ventricular pacing during SVT


Case 46

Eric Buch, Shiro Nakahara, Marmar Vaseghi, Noel G. Boyle,


and Kalyanam Shivkumar

Case Summary atrial activity with positive flutter waves in lead V1 and
inferior leads, not suggestive of typical counterclockwise
A 79-year-old male with a history of hypertension, coro- atrial flutter (Fig.46.1).
nary artery bypass grafting, and implantable cardioverter- Diagnostic electrophysiology catheters were placed at the
defibrillator for ventricular fibrillation, presented with His bundle, the right ventricular apical septum, and in the cor-
symptomatic supraventricular tachycardia. He had failed onary sinus. Intracardiac activation map obtained during tachy-
antiarrhythmic medications and cardioversion attempts, and cardia showed counterclockwise activation of the right atrium.
was referred for electrophysiology study and catheter abla- Ablation of the cavotricuspid isthmus was performed without
tion. Twelve-lead electrocardiogram showed organized tachycardia termination. Why did the tachycardia persist?

Fig. 46.1 Surface electrocardiogram of clinical tachycardia. There is an organized atrial rhythm with low-amplitude flutter waves that are
predominantly positive in lead V1. Variable conduction to the ventricles results in an irregular ventricular rhythm

E. Buch(*), S. Nakahara, M. Vaseghi,


N.G. Boyle, and K. Shivkumar
UCLA Cardiac Arrhythmia Center,
David Geffen School of Medicine at UCLA,
10833 Le Conte Ave, A2-237 CHS,
Los Angeles, CA 90095-1679
e-mail: ebuch@mednet.ucla.edu

A. Natale et al. (eds.), Cardiac Electrophysiology, 199


DOI: 10.1007/978-1-84996-390-9_46, Springer-Verlag London Limited 2011
200 E. Buch et al.

Case Discussion map shown in Fig.46.2a demonstrates a left atrial tachycar-


dia with centrifugal activation of the left atrium originating
Entrainment mapping from the septal side of the cavotricus- near a scar on the left atrial roof. Several viable sites were
pid isthmus and the distal coronary sinus were performed. found adjacent to the scar, with early activation (Fig.46.2b).
Although the tachycardia could be entrained from both sites, Pacing from these locations resulted in entrainment with
the post-pacing interval was much shorter from the distal concealed fusion, and post-pacing interval near tachycardia
coronary sinus, suggesting a left atrial flutter circuit with cycle length. During ablation (Fig. 46.3) the tachycardia
passive counterclockwise activation of the right atrium. spontaneously terminated, and could not be reinduced. This
After transseptal catheterization, electroanatomic map- case illustrates that typical counterclockwise activation of
ping of the left atrium was performed. Voltage mapping the right atrium does not prove that it is part of the tachy-
revealed an area of scar on the left atrial roof. Activation cardia circuit.

a b

Fig.46.2 (a) Three-dimensional electroanatomical activation map of the area) near the superior aspect of the left atrial septum. (b) Right posterior
left atrium during tachycardia, anteroposterior view. Earliest activation of oblique view. Radiofrequency energy was applied at viable sites adjacent
the left atrium is at the roof (red area), just lateral to a region of scar (gray to the scar (red points within red area) with low-amplitude potentials

ISI
200 ms
aVF
V1
V5
ABL d
ABL p
HIS p C

HIS m C

HIS d C

CS 9, 10
Fig.46.3 During ablation at one CS 7, 8
of the sites shown in Fig.46.2b,
the tachycardia terminated and CS 5, 6
could not be reinduced. Note the CS 3, 4
resumption of atrioventricular
CS 1, 2
paced rhythm at the right side of 11:34:13 AM 11:34:14 AM
the tracing
Case 47

Bradley P. Knight

Case Summary activation of the atrium and ventricle with a septal VA inter-
val that is close to zero. This excludes AV reentry using an
accessory pathway. When attempting to measure the septal
A patient with recurrent PSVT undergoes an electrophysiol-
VA interval, it can be difficult at times to differentiate the
ogy procedure. Fig.47.1 shows a recording during the proce-
atrial signal from the ventricular signal on the His bundle
dure. Shown are the surface recordings and intracardiac
recording. In this example, the proximal coronary sinus
recordings from the His bundle electrogram (HBE) ablation
recordings (CS9-10) can be used to identify the timing of
catheter (Abl), coronary sinus (CS), and right ventricular
septal atrial activation. The termination of the tachycardia is
apex (RV). What is the diagnosis of the tachycardia?
associated with spontaneous AV block. This observation
excludes an AT which would be expected to conduct to the
ventricle upon termination if there is a 1:1 conduction during
Case Discussion the tachycardia. In this case the tachycardia actually slows
slightly before termination. Therefore, it would be extremely
The differential diagnosis of a regular narrow QRS complex unlikely that an atrial tachycardia that spontaneously slows
tachycardia in the EP lab includes AV nodal reentry, ortho- just before termination would be associated with AV block
dromic AV reentry, and atrial tachycardia. The transition on the last beat. Another observation that excludes an atrial
zones during tachycardia can be very useful when trying to tachycardia is that as the tachycardia cycle length increases
make a diagnosis. In this case the tachycardia terminates. the VA interval remains constant. This fixed relationship
Before termination, one can see that there is simultaneous suggests an AV nodaldependent tachycardia.

B.P. Knight
Division of Cardiology, Northwestern Medical Center,
676 N. St. Clair, Suite 600, Chicago, IL, 60611
e-mail: bknight@nmff.org

A. Natale et al. (eds.), Cardiac Electrophysiology, 201


DOI: 10.1007/978-1-84996-390-9_47, Springer-Verlag London Limited 2011
202 B.P. Knight

I 320 330 340 400 780 430

II

III

V1

V5

HBE d

HBE p

Map d

Map p

CS 1-2

CS 3-4

CS 5-6

CS 7-8

CS 9-10

RV d 680 390 780 450

Stim

Fig.47.1 Tracing recorded during an electrophysiology procedure in a patient with recurrent paroxysmal supraventricular tachycardia. Spontaneous
termination of tachycardia occurs. See text for format and abbreviations
Case 48

Antonio Rossillo, Sakis Themistoclakis, Aldo Bonso,


Andrea Corrado, and Antonio Raviele

Case Summary sustained and external DC shock was necessary on three


occasions to restore sinus rhythm. Pharmacologic treatment
(including amiodarone and propafenone) was unsuccessful
A 47-year-old male patient without any structural heart dis-
and the two last episodes of tachycardia were reported to be
ease presented with a history of palpitations since childhood.
faster than usual and poorly tolerated (Figs.48.148.8).
All ECGs recorded while the patient experienced palpita-
Based on the previous tracings, what is the tachycardia
tions showed a wide QRS complex tachycardia with a left
mechanism?
bundle branch block pattern. The arrhythmia was always

Fig.48.1 12-lead resting ECG


(paper speed 25mm/s) showing
sinus rhythm, a normal PR interval
(170ms), and a QRS width of
100ms with slight slurring of the
initial portion of the QRS

A. Rossillo (*), S. Themistoclakis, A. Bonso, A. Corrado,


and A. Raviele
Cardiovascular Department, Ospedale dellAngelo, Mestre,
Venice, Italy

A. Natale et al. (eds.), Cardiac Electrophysiology, 203


DOI: 10.1007/978-1-84996-390-9_48, Springer-Verlag London Limited 2011
204 A. Rossillo et al.

Fig.48.2 Intracardiac recordings at baseline during the electrophysiol- cardial side of the tricuspid annulus obtained by a Cardima catheter
ogy study (paper speed 100mm/s). Four surface ECG leads (I, aVF, V1, positioned in the right coronary artery (C1C7), and one bipolar record-
V6), two bipolar recordings from the His bundle region (HIS D for ing from the coronary sinus (SC). AH interval is 80ms and HV interval
distal and HIS P for proximal), seven bipolar recordings from the epi- 30ms
Case 48 205

Fig. 48.3 Intracardiac recordings taken during programmed atrial H-V interval decreased from 30 to 10ms, and the QRS complex on the
stimulation (paper speed 200 mm/s). Same display as shown in surface ECG became wider with a left bundle branch block
Fig. 48.2. During programmed atrial stimulation (coupling interval morphology
between S1 and S2: 375ms) A-H interval increased from 105 to 125ms,
206 A. Rossillo et al.

Fig.48.4 Intracardiac recordings taken during programmed atrial stimulation (paper speed 200mm/s). Same display as shown in Fig.48.2. At the
coupling interval of 300ms, atrioventricular node duality was demonstrated with an A-H interval which increased suddenly from 111 to 192ms
Case 48 207

Fig.48.5 Intracardiac recordings taken during atrial stimulation (cycle H-V interval, a His bundle potential identified at the very beginning of
length 410 ms; paper speed 100 mm/s). Same display as shown in the QRS complex, and a left bundle branch block morphology on the
Fig.48.2. Ventricular preexcitation is present with a negative (10ms) surface ECG
208 A. Rossillo et al.

Fig.48.6 Intracardiac recordings during atrial stimulation (paper speed 100mm/s). Same display as that shown in Fig.48.2. When atrial pacing
at a cycle length of 270ms is abruptly interrupted, two atrioventricular node echoes appeared (slowfast type)
Case 48 209

Fig.48.7 Intracardiac recordings taken during the tachycardia induced a left bundle branch block morphology. Intracardiac recordings showed
during programmed atrial stimulation (paper speed 100mm/s). Same that the site of earliest atrial activation occurred in the His bundle region
display as Fig.48.2. The tachycardia had a cycle length of 330ms and with a retrograde His deflection
210 A. Rossillo et al.

Fig.48.8 12-lead ECG taken during the induced tachycardia (rate: 186bpm; paper speed 25mm/s) showing left bundle branch block morphology.
In fact, the induced tachycardia on 12-lead ECG is identical to the one observed during rapid atrial stimulation (see Fig.48.6)

Case Discussion

The patient has an atriofascicular pathway or so-called


Manheim. Based on Fig.48.9 this is likely antidromic tachy-
cardia with retrograde conduction over either the fast or slow
pathway (Figs. 48.1048.13).
Case 48 211

Fig.48.9 Intracardiac recordings taken during the tachycardia (paper cardia cycle length (from 670 to 570 ms). This figure illustrates an
200mm/s). Same display as shown in Fig.48.2. A spontaneous change abrupt switch from the slow to the fast retrograde pathway excluding
in H-A interval was observed (from 120 to 69ms) without any change the slowfast atrioventricular node reentrant tachycardia as the mecha-
in atrial retrograde activation pattern but with shortening of the tachy- nism of the induced tachycardia
212 A. Rossillo et al.

Fig.48.10 Intracardiac recordings taken during mapping of the tricus- ablation catheter (MC BIP) is now displayed. In the lateral region of the
pid annulus during sinus rhythm. Same representation as shown in tricuspid annulus a specific potential was recorded (M potential)
Fig.48.2, except that a bipolar recording obtained from the tip of the
Case 48 213

Fig.48.11 Intracardiac recordings taken during mapping of the tricus- recorded, mechanical block occurred (normalization of the QRS com-
pid annulus during sinus rhythm. Same representation as shown in plex on the surface ECG, normalization of H-V interval) (arrow).
Fig. 48.2. When mapping the region where the M potential was Ablation was performed at this site
214 A. Rossillo et al.

Fig.48.12 12-lead ECG recorded after the ablation procedure (paper speed 25mm/s). Radiofrequency current was applied on the site shown in
Fig.48.12. The 12-lead ECG is now normal without any ventricular preexcitation
Case 48 215

Bibliography

Ao Aliot E, de Chillou C, Revault dAllones G. Mahaim tachycardias. Eur


Heart J. 1998;19(suppl E):E25-E31.
Ellenbogen KA, Ramirez NM, Packer DL, etal. Accessory nodoven-
tricular (Mahaim) fibers: a clinical review. PACE. 1986;9:868-875.
Gallagher JJ, Smith WM, Kasell JH, et al. Role of Mahaim fibers in
Cdx cardiac arrhythmias in man. Circulation. 1981;64:176-184.
Haissaguerre M, Cauchemez B, Marcus F, etal. Characteristics of the
H VM ventricular insertion sites of accessory pathways with anterograde
decremental conduction properties. Circulation. 1995;91:1077-
M 1085.
Josephson ME. Nodoventricular and fasciculoventricular bypass tracts.
In: Josephson ME, ed. Clinical Cardiac Electrophysiology:
VT SC Techniques and Interpretation. 2nd ed. Malvern: Lea & Febiger;
1993:396-416.
Mounsey JP, Griffith MJ, McComb JM, etal. Radiofrequency ablation
of Mahaim fiber following localization of Mahaim pathway poten-
tials. J Cardiovasc Electrophysiol. 1994;5:432-434.
Cdx
Tchou P, Lehmann MH, Jazayeri M, etal. Atriofascicular connections
or a nodoventricular Mahaim fiber? Electrophysiologic elucidation
of the pathway and associated reentrant circuit. Circulation.
Fig.48.13 Picture showing the circuit of the arrhythmia (VT=tricus- 1988;77:837-841.
pid valve, MV=mitral valve, Ao=Aorta, Cdx=right coronary artery,
H=His bundle, and M=mahaim fibers)

Case 49

Eric Buch, Shiro Nakahara, Marmar Vaseghi, Noel G. Boyle,


and Kalyanam Shivkumar

Case Summary Case Discussion

A 30-year-old female with primary pulmonary hypertension By definition, typical atrial flutter is a macroreentrant
had recurrent atrial flutter despite amiodarone treatment and tachycardia bound anteriorly by the tricuspid isthmus and
multiple cardioversions. Because of worsened dyspnea posteriorly by the inferior vena cava, the terminal crest, and
resulting from atrial flutter, she was referred for electrophys- most commonly, the superior vena cava.1 The majority of
iology study and catheter ablation. Preprocedural surface these tachycardias are counterclockwise (CCW), with acti-
electrocardiograms, both recorded during the month prior to vation descending the anterolateral and ascending the septal
ablation, showed organized atrial activity with variable flut- RA. Less commonly, isthmus-dependent atrial flutter is
ter wave morphology (Fig.49.1). Based on the 12-lead ECG, clockwise (CW), with activation descending the septal and
what is the diagnosis? Do these arrhythmias arise from ascending the anterolateral RA. Multiple studies have
multiple flutter circuits? attempted to characterize the surface ECG appearance of

Fig.49.1 Surface electrocardiogram showing flutter waves that are predominantly negative in lead V1 and biphasic in the inferior leads, not
typical of cavotricuspid isthmus dependent counterclockwise atrial flutter

E. Buch(*), S. Nakahara, M. Vaseghi,


N.G. Boyle, and K. Shivkumar
UCLA Cardiac Arrhythmia Center,
David Geffen School of Medicine at UCLA,
10833 Le Conte Ave, A2-237 CHS,
Los Angeles, CA 90095-1679
e-mail: ebuch@mednet.ucla.edu

A. Natale et al. (eds.), Cardiac Electrophysiology, 217


DOI: 10.1007/978-1-84996-390-9_49, Springer-Verlag London Limited 2011
218 E. Buch et al.

CW and CCW flutter.2 It is currently accepted that predomi- case, negative F wave deflection in V1 and positive deflec-
nantly negative F wave deflections in the inferior leads and tion in inferior leads (Fig. 49.1a) predicted CW flutter, which
in lead V6 and positive deflection in V1 indicate CCW flut- was established by intracardiac mapping (Fig. 49.2). The
ter, while CW flutter has positive F waves in the inferior other surface ECG (Fig. 49.1b) was likely CCW flutter.
leads and V6, and negative F waves in lead V1.1,2 In this Neither arrhythmia was inducible after ablation at the

Fig.49.2 (a) Left anterior oblique fluoroscopic view of intracardiac isthmus, with the most distal bipole (D1) in the proximal coronary
electrophysiology catheters. The duopecapolar catheter (D1D10) is sinus. (b) Surface and intracardiac recordings. The activation sequence
curled in the right atrium, with the proximal bipoles (D9, D10) near the proceeds from distal to proximal, consistent with clockwise isthmus-
right atrial roof, the middle bipoles (D4D8) along the lateral right dependent atrial flutter
atrial wall, and the distal bipoles (D2, D3) near the cavotricuspid
Case 49 219

I 200 ms
aVF
V1
V6
aVR
ABL d
ABL p
D10
D9
D8
D7
D6
D5
D4
D3
D2
D1

11:15:07 AM 11:15:08 AM

Fig.49.3 With the ablation catheter positioned as shown in Fig.49.2 above, radiofrequency energy was applied. Intracardiac recordings during
ablation, showing termination of tachycardia

cavotricuspid isthmus (Fig. 49.3). Of note, F wave morphol- from the Working Group of Arrhythmias of the European Society
ogy not adhering to the above criteria has been described in of Cardiology and the North American Society of Pacing
and Electrophysiology. J Cardiovasc Electrophysiol. 2001;12:
a minority of patients when the left atrium is activated by 852-866.
Bachmanns bundle. In this case, CCW flutter can manifest 2. Milliez P, Richardson AW, Obioha-Ngwu O, Zimetbaum PJ,
as positive F waves in the inferior leads, and clockwise flut- Papageorgiou P, Josephson ME. Variable electrocardiographic
ter can manifest as negative F waves in the inferior leads. characteristics of isthmus-dependent atrial flutter. J Am Coll
Cardiol. 2002;40:1125-1132.
(Figs49.2 and 49.3).3 3. Oshikawa N, Watanabe I, Masaki R, et al. Relationship between
polarity of the flutter wave in the surface ECG and endocardial
atrial activation sequence in patients with typical counterclockwise
and clockwise atrial flutter. J Interv Card Electrophysiol. 2002;
References 7:215-223.

1. Saoudi N, Cosio F, Waldo A, etal. Classification of atrial flutter and


regular atrial tachycardia according to electrophysiologic mecha-
nism and anatomic bases: a statement from a joint expert group

Case 50

Bradley P. Knight

Case Summary Left-sided slow pathway ablation

An attempt was made to ablate the slow pathway in a


45-year-old woman with recurrent SVT and reproducibly
inducible typical AV nodal reentry. Using a standard 4-mm-
tip electrode ablation catheter, a total of 55 applications of
RF current were delivered using 50 W at several sites just
outside and slightly within the coronary sinus ostium.
Despite adequate tissue heating and the occurrence of an
accelerated junctional rhythm with most lesions, the tachy-
RF site
cardia remained inducible. What other approach could be
used?

Case Discussion

A very small number of patients who have what appear to be


the slowfast type of AV nodal reentry cannot be success-
fully ablated using a right-sided approach. It is likely that
there are left-sided inputs to the AV node that require abla- Fig. 50.1 A fluoroscopic image in a left anterior oblique projection
during ablation of a slow pathway using a left-sided approach. Three
tion to eliminate slow pathway function. Using a retrograde catheters can be seen in the right heart including a right ventricular,
aortic approach the left posterior septal aspect of the mitral His-bundle, and coronary sinus catheter. There is also a left sided abla-
annulus was explored (Fig.50.1). A site was identified that tion catheter that has been positioned along the posteroseptal mitral
was associated with a complex, fractionated atrial electro- annulus using a retrograde aortic approach
gram (Fig.50.2). A single application of RF current at this
site resulted in an accelerated junctional rhythm and elimina-
tion of slow pathway function (Fig.50.3).

B.P. Knight
Division of Cardiology, Northwestern Medical Center, 676 N. St.
Clair, Suite 600, Chicago, IL 60611, USA
e-mail: bknight@nmff.org

A. Natale et al. (eds.), Cardiac Electrophysiology, 221


DOI: 10.1007/978-1-84996-390-9_50, Springer-Verlag London Limited 2011
222 B.P. Knight

Fig.50.2 Tracing showing the electrogram at the successful ablation Fig. 50.3 Tracing demonstrating an accelerated junctional rhythm
site during sinus rhythm in a patient undergoing a left-sided slow AV during delivery of radiofrequency current at the successful ablation site
nodal pathway ablation. Shown are surface recordings from leads I, II, in a patient undergoing a left-sided slow AV nodal pathway ablation.
V1, and V5, and the intracardiac recordings from high right atrium The format and abbreviations are the same as in Figure 50.2
(HRA), His-bundle electrogram region (HBE), left ventricle (LV), and
right ventricular apex (RVA)

Bibliography

Katritsis DG, Becker AE, Ellenbogen KA, etal. Right and left inferior
extensions of the atrioventricular node may represent the anatomic
substrate of the slow pathway in humans. Heart Rhythm.
2004;1:582-586.
Case 51

Yash Y. Lokhandwala, Anoop K. Gupta, and Ranjan K. Thakur

Case Summary Tachycardia is initiated by a decrementally conducting


PVC. VA time is long; earliest in proximal CS. With PVC
placed when the His was refractory, no atrial preexcitation
A 54-year-old woman with a normal heart presents to the EP
was observed, suggesting that the likely mechanism was
lab with an SVT (Fig. 51.1). The tachycardia could be
atypical AVNRT. There is slowintermediate and slowslow
induced with three APCs or with ventricular premature beats
AVNRT, hence ablation of the slow pathway was performed;
(Figs. 51.2 and 51.3). Explain the mechanism of the
no tachycardia was observed after this. The VA time at initia-
tachycardia?
tion of even typical AVNRT often shortens over the initial
few complexes, more so in older patients; hence an appar-
ently atypical AVNRT may later appear typical (Fig.51.4).
Case Discussion

With application of three atrial extrastimuli, tachycardia is


induced. Induction is preceded by lengthening of the AH
interval; this suggests that the likely mechanism is AVNRT.

Y.Y. Lokhandwala(*)
KEM Hospital, Parel, Mumbai, India
e-mail: yashlokhandwala@hotmail.com
A.K. Gupta
Apollo Hospital, Ahmedabad, India
e-mail: anoopgupta@msn.com
R.K. Thakur
Thoracic and Cardiovascular Institute, Sparrow Health System,
Michigan State University, 405 West Greenlawn, Suite 400,
Lansing, MI 48910, USA
e-mail: thakur@msu.edu

A. Natale et al. (eds.), Cardiac Electrophysiology, 223


DOI: 10.1007/978-1-84996-390-9_51, Springer-Verlag London Limited 2011
224 Y.Y. Lokhandwala et al.

RUBY HALL CLINIC Version WIN2000/XP : EPTRACER V0.77


2008045

II

III

AVR

AVL

AVF

V1

V2

V3

V4

V5

V6

10 mm/mV 25 mm/s

Fig.51.1 SVT
Case 51 225

2008045
I

AVF

V1

V6

CS910

CS78

CS56

CS34

CS12

HISD

HISP

RFD

RFP

10 mm/mV 100 mm/s

Fig.51.2 Three PACs initiate tachycardia


226 Y.Y. Lokhandwala et al.

2008045
I

AVF

V1

V6

CS34

CS12

HISD

HISP

RFD

10 mm/mV 100 mm/s

Fig.51.3 Tachycardia initiation with ventricular premature beat


Case 51 227

2008045

AVF

V1

V6

CS910

CS78

CS56

CS34

CS12

HISD

HISP

RFD

Fig.51.4 The patient later showed this pattern typical AVNRT



Case 52

Eric Buch, Shiro Nakahara, Marmar Vaseghi, Noel G. Boyle,


and Kalyanam Shivkumar

Case Summary approximately 220 ms with negative F waves in leads II, III,
and aVF, positive F waves in V1, suggesting typical
A 79-year-old male with a history of hypertension, diabetes, counterclockwise atrial flutter. Conduction to ventricles was
coronary artery bypass grafting, recently diagnosed with predominantly 4:1.
atrial flutter and referred for electrophysiology study and Electroanatomic mapping suggested macroreentrant
catheter ablation. Initial 12-lead electrocardiogram (Fig.52.1) arrhythmia in the right atrium. As seen in Fig.52.2, the direc-
showed organized atrial activity with a cycle length of tion of activation was counterclockwise, and the circuit

Fig.52.1 Surface electrocardiogram of clinical tachycardia. Atrial flutter is seen, with F waves that are predominantly negative in the inferior
leads and upright in V1, consistent with typical counterclockwise atrial flutter

E. Buch(*), S. Nakahara, M. Vaseghi,


N.G. Boyle, and K. Shivkumar
UCLA Cardiac Arrhythmia Center,
David Geffen School of Medicine at UCLA,
10833 Le Conte Ave, A2-237 CHS,
Los Angeles, CA 90095-1679
e-mail: ebuch@mednet.ucla.edu

A. Natale et al. (eds.), Cardiac Electrophysiology, 229


DOI: 10.1007/978-1-84996-390-9_52, Springer-Verlag London Limited 2011
230 E. Buch et al.

Fig.52.2 Three-dimensional
electroanatomic mapping during
tachycardia. Anteroposterior
view of right atrium shows the
reentrant circuit, with early
meeting late activation in a
circuit around the tricuspid
annulus. Pacing from the tip of
the ablation catheter, which was
positioned at the cavotricuspid
isthmus, showed entrainment
with concealed fusion and a
postpacing interval of 10ms
longer than tachycardia cycle
length

involved the cavotricuspid isthmus. What maneuvers are In this case, pacing from the cavotricuspid isthmus showed
useful in proving that the cavotricuspid isthmus is part of the entrainment with concealed fusion, with identical surface
tachycardia? flutter wave morphology and intracardiac activation pattern
as compared to tachycardia. The postpacing interval was
within 10 ms of the flutter cycle length. This showed that the
Case Discussion cavotricuspid isthmus was likely part of the tachycardia
circuit, and therefore radiofrequency ablation of the isthmus
One maneuver that can show whether a given location is close was performed.
to the arrhythmia circuit is entrainment mapping.1 This is The ablation catheter was advanced into the right ventricle,
accomplished by pacing slightly faster than the tachycardia curved, and pulled inferiorly to seat it on the cavotricuspid
cycle length to repeatedly reset the reentrant circuit. If paced isthmus, with the catheter tip at 6 oclock in the left anterior
P wave morphology is identical to that observed during atrial oblique view. A series of radiofrequency applications were
flutter, this is termed entrainment with concealed fusion. If delivered to create a line of conduction block across the
tachycardia continues after cessation of pacing, the first post- cavotricuspid isthmus. During ablation, the cycle length sud-
pacing interval will be close to the tachycardia cycle length, denly prolonged and the arrhythmia terminated with a long
provided that the pacing site is near the reentrant circuit.2 pause (Fig.52.3). Normal sinus node function and atrioven-
Postpacing interval is defined as the time from last paced tricular conduction resumed after a few seconds, and atrial
stimulus to the next local electrogram during tachycardia, flutter was no longer inducible at the end of the ablation
measured at the site of pacing. procedure.
Case 52 231

Fig.52.3 Three-dimensional
electroanatomic mapping
during radiofrequency ablation,
caudal left anterior oblique
view. As the line of conduction
block across the cavotricuspid
isthmus was completed, the
flutter terminated spontaneously
and could not be reinduced

References 2. Stevenson WG, Khan H, Sager P, etal. Identification of reentry cir-


cuit sites during catheter mapping and radiofrequency ablation of
ventricular tachycardia late after myocardial infarction. Circulation.
1. Cosio FG, Lopez Gil M, Arribas F, Palacios J, Goicolea A, Nunez A. 1993;88:1647-1670.
Mechanisms of entrainment of human common flutter studied with
multiple endocardial recordings. Circulation. 1994;89:2117-2125.

Case 53

Bradley P. Knight

Case Summary inferior vena cava. Although there are some risks associated
with crossing the filter with a catheter, it is clear that an elec-
trophysiology procedure can be performed safely in these
A patient is scheduled to undergo a catheter ablation proce-
patients from the femoral vein.1 It is helpful to be certain that
dure for recurrent PSVT, which appears most likely to be
the vena cava is patent before the procedure with a CT scan
typical AV nodal reentry. You discover that the patient under-
or venography. But assuming that the IVC is patent, long
went insertion of an inferior vena cava filter 2 years ago after
guiding sheaths and electrophysiology catheters can be
a pulmonary embolism. How would you proceed with this
passed safely through the filter. Figure 53.1 shows a long
case?
sheath already through the filter, and a second one being
advanced over a guidewire that has been passed through the
filter.
Case Discussion There are times, however, when the vena cava is com-
pletely obstructed. In this situation, options include a supe-
When performing a slow pathway ablation procedure, it is rior or transhepatic central venous approach. One reasonable
important to monitor VA conduction during the accelerated approach that has been shown to be successful in patients
junctional rhythm to avoid AV block. Therefore, it is usually with AV nodal reentry is to place two central venous sheaths
necessary to place two catheters in the heart during the in the right internal jugular vein.2 This allows placement of a
procedure. Unfortunately, some patients represent vascular diagnostic catheter in the high right atrium and an ablation
access challenges. In this case, the patient has a filter in the catheter along the tricuspid annulus (Fig.53.2).

B.P. Knight
Division of Cardiology, Northwestern Medical Center,
676 N. St. Clair, Suite 600, Chicago, IL 60611
e-mail: bknight@nmff.org

A. Natale et al. (eds.), Cardiac Electrophysiology, 233


DOI: 10.1007/978-1-84996-390-9_53, Springer-Verlag London Limited 2011
234 B.P. Knight

Fig.53.1 Fluoroscopic image during catheter placement from a femo-


ral vein approach through an inferior vena cava filter. Two sheaths were Fig.53.2 Right anterior oblique fluoroscopic view after placement of
advanced over a guidewire to access the heart with two electrophysiol- an ablation catheter and diagnostic quadrapolar catheter via the internal
ogy catheters. See text for details jugular vein, during a catheter ablation procedure for typical atrioven-
tricular nodal reentry in a patient without inferior vena cava access

References 2. Salem Y, Burke MC, Morady F, Knight BP. Slow pathway ablation
for atrioventricular nodal reentry using a right internal jugular
vein approach: a case series. Pacing Card Electrophysiol. 2006;
1. Sinha SK, Harnick D, Gomes JA, Mehta D. Electrophysiologic 29:59-62.
interventions in patients with inferior vena cava filters: safety and
efficacy of the transfemoral approach. Heart Rhythm. 2005;2:15-18.
Case 54

Yash Y. Lokhandwala, Anoop K. Gupta, and Ranjan K. Thakur

Case Summary sequence and (2) a right-sided AP should have caused atrial
preexcitation.
This PVC does not rule out AVNRT because to do so, the
An 18-year-old young man with normal heart presents to the
PVC must be delivered when the His is refractory. Thus, the
EP lab. His initial tracing shows AF followed by an SVT
differential diagnoses at this point include AVNRT and atrial
(Fig.54.1). Figures54.2 and 54.3 show the response to ven-
tachycardia.
tricular premature beats and ventricular pacing. Does this
Ventricular pacing is performed during tachycardia. VA
help elucidate the cause of the tachycardia?
dissociation is seen with continuation of tachycardia, prov-
ing a diagnosis of atrial tachycardia.
Case Discussion The tachycardia itself does not lend itself to analysis of
the P waves. The tachycardia continues in spite of bursts of
Even a very early PVC from the right ventricle does not PVCs, suggesting that this is atrial tachycardia. P wave mor-
advance the A. This makes AVRT extremely unlikely phology is revealed after the bursts; upright in I, II, III and
because (1) left-sided AP is ruled out by the activation aVF, suggesting high right atrial origin (Fig.54.4)

Y.Y. Lokhandwala(*)
KEM Hospital, Parel, Mumbai, India
e-mail: yashlokhandwala@hotmail.com
A.K. Gupta
Apollo Hospital, Ahmedabad, India
e-mail: anoopgupta@msn.com
R.K. Thakur
Thoracic and Cardiovascular Institute, Sparrow Health System,
Michigan State University, 405 West Greenlawn, Suite 400,
Lansing, MI 48910, USA
e-mail: thakur@msu.edu

A. Natale et al. (eds.), Cardiac Electrophysiology, 235


DOI: 10.1007/978-1-84996-390-9_54, Springer-Verlag London Limited 2011
236 Y.Y. Lokhandwala et al.

2008043

AVF

V1

V6

CS910

CS56

CS12

HISD

HISP

Fig.54.1 The left half shows AF which initiates a narrow complex SVT. AF termination is followed by a junctional beat and then, SVT. SVT starts
with an A
Case 54 237

2008043

AVF

V1

V6

CS910

CS56

CS12

HISD

HISP

RFD

RFP

10 mm/mV 300 mm/s

Fig.54.2 A VPC is delivered during the tachycardia


238 Y.Y. Lokhandwala et al.

2008043

AVF

V1

V6

CS910

CS56

CS12

HISD

HISP

RFD

RFP

10 mm/mV 150 mm/s

Fig.54.3 V pacing during the tachycardia


Case 54 239

2008043

AVF

V1

V6

CS910

CS56

CS12

HISD

HISP

RFD

RFP

10 mm/mV 150 mm/s

Fig.54.4 P wave morphology can often be uncovered after ventricular pacing



Case 55

Eric Buch, Shiro Nakahara, Marmar Vaseghi, Noel G. Boyle,


and Kalyanam Shivkumar

Case Summary electrocardiogram showed a regular supraventricular tachycardia


at a rate of approximately 250 per minute, with 2:1 conduction to
A 52-year-old male with history of hypertension, diabetes, and the ventricles (Fig.55.1). He was referred for electrophysiology
coronary artery disease status post coronary artery bypass study and catheter ablation. Based on this ECG tracing, what is
grafting was recently seen for palpitations and dyspnea. Surface the most likely mechanism of this tachycardia?

Fig.55.1 Surface electrocardiogram of the clinical tachycardia. Sawtooth flutter waves are predominantly negative in the inferior leads, upright
in V1, and inverted in V6

E. Buch(*), S. Nakahara, M. Vaseghi,


N.G. Boyle, and K. Shivkumar
UCLA Cardiac Arrhythmia Center,
David Geffen School of Medicine at UCLA,
10833 Le Conte Ave, A2-237 CHS,
Los Angeles, CA 90095-1679
e-mail: ebuch@mednet.ucla.edu

A. Natale et al. (eds.), Cardiac Electrophysiology, 241


DOI: 10.1007/978-1-84996-390-9_55, Springer-Verlag London Limited 2011
242 E. Buch et al.

b
I
100 ms
aVF
V1
V6
HRA T

HIS p C

HIS m C

HIS d C

CS 9,10
CS 7,8
CS 5,6
CS 3,4
CS 1,2
DD 19,20
DD 17,18
DD 15,16
DD 13,14
DD 11,12
DD 9,10
DD 7,8
DD 5,6
DD 3,4
DD 1,2
RVA
3:15:00 PM

Fig.55.2 (a) Fluoroscopic left anterior oblique view showing catheter Right atrial activation is seen to proceed from septal to lateral along the
positions. The duodecapolar (DD) catheter is curled in the right atrium superior right atrium, down the anterolateral wall to the cavotricuspid
such that the proximal, mid, and distal bipoles of the catheter are near isthmus area, then up the atrial septum to complete the circuit. As seen
the right atrial roof, anterolateral wall, and coronary sinus os, respec- in the CS catheter, bystander left atrial activation occurs simultaneously
tively. Arrows show the direction of activation. (b) Intracardiac tracings. with right atrial septal activation
Case 55 243

Case Discussion ablation was created across the isthmus, connecting the tri-
cuspid annulus to the inferior vena cava. In the right anterior
Diagnostic electrophysiology catheters were placed at the oblique projection, the ablation catheter was seen to be well
right atrium, the His bundle, the right ventricular septum, separated from the His bundle catheter. In the left anterior
and in the coronary sinus. A multipolar deflectable catheter oblique projection, the ablation catheter was near the 6
was curled in the right atrium with its distal bipoles near the oclock position. During ablation, the flutter terminated.
mouth of the coronary sinus, and the proximal bipoles near After ablation, bidirectional conduction block across the
the roof of the right atrium (Fig.55.2a). Intracardiac record- isthmus was verified by pacing from either side of the
ings from these catheters show that right atrial activation isthmus.
proceeds in a counterclockwise direction in the left anterior The common form of atrial flutter is a macroreentrant
oblique view, from proximal to distal bipole on the duodeca- right atrial arrhythmia with a counterclockwise pattern of
polar catheter (Fig.55.2b). activation as seen from the left anterior oblique view.1 Critical
An ablation catheter was advanced into the right ventricle, to perpetuation of the arrhythmia is a narrow area of slow
pulled back slowly to seat the tip on the cavotricuspid isth- conduction bounded by the inferior vena cava and tricuspid
mus. Beginning at the edge of the tricuspid annulus, where valve, called the cavotricuspid isthmus.2 Surgical or catheter-
the ventricular electrogram from the distal bipole of the abla- based ablation of the isthmus can result in conduction block
tion catheter was larger than the atrial electrogram, a line of and effective treatment of this arrhythmia.3,4

References anatomy of the isthmus. J Cardiovasc Electrophysiol. 1998;9:


1186-1195.
3. Klein GJ, Guiraudon GM, Sharma AD, Milstein S. Demonstration
1. Disertori M, Inama G, Vergara G, Guarnerio M, Del Favero A, of macroreentry and feasibility of operative therapy in the common
Furlanello F. Evidence of a reentry circuit in the common type of type of atrial flutter. Am J Cardiol. 1986;57:587-591.
atrial flutter in man. Circulation. 1983;67:434-440. 4. Feld GK, Fleck RP, Chen PS, etal. Radiofrequency catheter abla-
2. Cabrera JA, Sanchez-Quintana D, Ho SY, Medina A, Anderson tion for the treatment of human type 1 atrial flutter. Identification of
RH. The architecture of the atrial musculature between the a critical zone in the reentrant circuit by endocardial mapping tech-
orifice of the inferior caval vein and the tricuspid valve: the niques. Circulation. 1992;86:1233-1240.

Case 56

Bradley P. Knight

Case Summary QRS complex. Ventricular overdrive pacing is delivered


from the right ventricular apex during tachycardia at a cycle
A regular supraventricular tachycardia with a 1:1 AV length 30ms less than the tachycardia CL. Figure56.1 shows
relationship and a CL of 420ms is induced in the EP lab. the response immediately upon cessation of ventricular
Activation of the high right atrium is coincident with the pacing. What is the mechanism of tachycardia?

II

V1

V5

Fig.56.1 This tracing was 390 390 480 420 430 420 420
recorded when ventricular HRA
overdrive pacing was stopped
after being delivered during a
HBEd
regular supraventricular
tachycardia with 1:1 atrioven-
tricular conduction. Shown are HBEm
surface recordings from leads I,
II, V1, and V5, and the intracar-
diac recordings from high right HBEp
atrium (HRA), His-bundle
electrogram region (HBE), and 390 390
RVA
right ventricular apex (RVA).
Note the response after pacing
can be described as a VAV Stim
response. See text for further
description University of Chicago

B.P. Knight
Division of Cardiology, Northwestern Medical Center,
676 N. St. Clair, Suite 600, Chicago, IL 60611, USA
e-mail: bknight@nmff.org

A. Natale et al. (eds.), Cardiac Electrophysiology, 245


DOI: 10.1007/978-1-84996-390-9_56, Springer-Verlag London Limited 2011
246 B.P. Knight

Case Discussion A

During a regular supraventricular tachycardia with a 1:1


V
AV relationship, the differential diagnosis includes AV
nodal reentry (AVNRT), orthodromic AV reentry (ORT),
Fig.56.3 Ladder diagram showing the response to overdrive ventricu-
and atrial tachycardia (AT). The ventricular overdrive lar pacing during an atrial tachycardia. The response after pacing is a
pacing can be used in the EP laboratory to help rule in or VAV response. This ladder diagram best describes the response seen in
rule out atrial tachycardia as the mechanism.1,2 The prin- Fig. 56.1. See text for more detail
ciple involved is that when ventricular pacing during
tachycardia often results in 1:1 retrograde conduction, the of activation can be described as atrialventricular
atrial rate accelerates to the ventricular pacing rate until (VAV; Fig.56.3).
ventricular pacing is stopped. In the case of an AT, the last When analyzing the response to ventricular pacing the
atrial depolarization that is a result of retrograde VA con- following steps should be taken:
duction will block antegrade, because the AV node will be
refractory in the antegrade direction after recent retrograde 1. Confirm ventricular capture.
activation. After resumption of the AT, the first atrial depo- 2. Confirm acceleration of atrial rate to pacing rate.
larization will conduct to the ventricle. After ventricular 3. Identify last atrial depolarization arising from last
pacing is stopped, the sequence of activation can be ventricular paced beat.
described as atrialatrialventricular (VAAV; Figs. 56.1 4. Identify the next conducted ventricular beat.
and 56.2). In contrast, in the case of an AV nodal depen- 5. Categorize the sequence as VAV or VAAV.
dent tachycardia, such as AVNRT or ORT, ventricular
pacing with 1:1 VA conduction entrains the tachycardia In this case, the response immediately after ventricular pacing is
which, if it does not terminate, will conduct anterograde VAV, which rules out AT. Because the short VA interval excludes
over the AV node as soon as the tachycardia resumes. After ORT as a mechanism,2 the diagnosis must be AVNRT.
entraining an AV nodal dependent tachycardia, the sequence

A References

1. Knight BP, Zivin A, Souza J, et al. A technique for the rapid


V diagnosis of atrial tachycardia in the electrophysiology laboratory.
J Am Coll Cardiol. 1999;33:775-781.
2. Knight BP, Ebinger M, Oral H, etal. Diagnostic value of tachycar-
Fig.56.2 Ladder diagram showing the response to overdrive ventricu-
dia features and pacing maneuvers during paroxysmal supraven-
lar pacing during an atrial tachycardia. The response after pacing is a
tricular tachycardia. J Am Coll Cardiol. 2000;36:574-582.
VAAV response. See text for more detail
Case 57

Yash Y. Lokhandwala, Anoop K. Gupta, and Ranjan K. Thakur

Case Summary Figure 57.1 shows the tachycardia induced during EP


study. What is the interpretation?
A 23-year-old man with recurrent episodes of paroxysmal
palpitations associated with near-syncope was referred for
evaluation. Cardiac evaluation showed a normal heart.
Case Discussion

Y.Y. Lokhandwala(*) The tachycardia is an orthodromic AVRT mediated by a


KEM Hospital, Parel, Mumbai, India right-sided accessory pathway because the tachycardia
e-mail: yashlokhandwala@hotmail.com
becomes faster with the disappearance of RBBB.
A.K. Gupta This conclusion is based on the fact that if the bundle
Apollo Hospital, Ahmedabad, India branch block is on the same side as the accessory pathway,
e-mail: anoopgupta@msn.com
then the tachycardia path is longer during bundle branch
R.K. Thakur block, leading to a slower tachycardia rate during BBB and
Thoracic and Cardiovascular Institute, Sparrow Health System,
Michigan State University, 405 West Greenlawn, Suite 400, Lansing,
faster rate with its disappearance (Fig. 57.2). Parahisian
MI 48910, USA pacing was also performed before and after successful
e-mail: thakur@msu.edu ablation (Figs.57.3 and 57.4).

A. Natale et al. (eds.), Cardiac Electrophysiology, 247


DOI: 10.1007/978-1-84996-390-9_57, Springer-Verlag London Limited 2011
248 Y.Y. Lokhandwala et al.

RUBY HALL CLINIC Version WIN2000/XP : EPTRACER V0.77


2008037

II

III

AVR

AVL

AVF

V1

V2

V3

V4

V5

V6

10 mm/mV 25 mm/s

Fig.57.1 Induced SVT


Case 57 249

RUBY HALL CLINIC Version WIN2000/XP : EPTRACER V0.77


2008037

AVF

V1

V6

CS34

CS12

HISD

HISP

RFD

10 mm/mV 100 mm/s

Fig.57.2 Intracardiac signals during transition. The A is earliest in the breaking the transseptal linking which was perpetuating the RBBB.
RF catheter, followed by the His, and still later in the CS. The two The VA time shortens with the disappearance of RBBB
programmed PVCs eliminate the RBBB by depolarizing the IVS and
250 Y.Y. Lokhandwala et al.

RUBY HALL CLINIC Version WIN2000/XP : EPTRACER V0.77


2008037

AVF

V1

V6

CS34

CS12

HISD

HISP

RFD

10 mm/mV 300 mm/s

Fig.57.3 Para-Hisian pacing before RF. The second complex shows His bundle capture evidenced by a narrower QRS complex. The VA time does
not shorten with His bundle capture. Thus the VA conduction is via an AP
Case 57 251

RUBY HALL CLINIC Version WIN2000/XP : EPTRACER V0.77


2008037

AVF

V1

V6

CS34

CS12

HISD

HISP

RFD

10 mm/mV 100 mm/s

Fig.57.4 Para-Hisian pacing after ablation. There was no VA conduc- complexes show His capture evidenced by narrower complexes. The
tion after ablation; with isoproterenol, VA conduction was seen and, VA time shortens with His bundle capture suggesting that the VA
therefore, para-Hisian pacing was performed. Second through sixth conduction is via the AV node

Case 58

Roopinder Sandhu, Dimpi Patel, William R. Lewis, and Andrea Natale

Case Presentation Figure 58.11 shows an atrial extrastimulus (pac) during


tachycardia.
What does the preceding pacing maneuver prove?
A 22-year-old male with no significant medical history
presented with recurrent palpitations, dizziness, and chest
discomfort. The patient reports having symptoms of short-
ness of breath and fatigue. He is currently not taking any
medications. His 12-Lead ECG (Fig.58.1) shows ventricular
Case Discussion
preexcitation.
The baseline intracardiac electrograms prior to atrial The differential diagnosis for patients with a preexcited
pacing are shown in Fig. 58.2. The baseline intervals are: tachycardia include: atrial arrhythmias, true antidromic
PR, 143; QRS, 104; HV, 27. AV reciprocating tachycardia, AV nodal reentry with a
Figures58.3 and 58.4 show programmed atrial stimula- bystander pathway, or antidromic tachycardia using
tion. The accessory pathway ERP is 330 ms. Figure 58.5 multiple pathways.
shows that atrial fibrillation was induced with atrial pacing. The atrial extrastimulus at the time of the refractory His
Figure58.6 illustrates that the shortest preexcited RR inter- advances the ventricular activation which demonstrates the
val is 298 ms or 201 BPM. These two electrograms suggest presence of an accessory pathway and its participation in the
that the accessory pathway is not a high-risk pathway. tachycardia circuit. The retrograde activation is concentric
Figure58.7 shows that during ventricular pacing at 700 and a retrograde His prior to atrial activation is shown. The
ms retrograde conduction is occurring over the AV node and AV node is the likely retrograde conducting pathway. This is
not the accessory pathway. not consistent with an atrial tachycardia.
Figure58.8 shows that VA block while pacing at 680 ms. We cannot exclude the presence of another pathway (ex
Figure 58.9 reveals that a wide complex tachycardia was concealed accessory pathway not participating in this reentry
inducible. Figure58.10 shows the 12-lead ECG of the wide circuit or a slow conducting retrograde accessory pathway).
QRS tachycardia. The optimal ablation site is the region of earliest ventricu-
lar activation during maximal preexcitation. Presence of an
accessory pathway potential can be guided by a unipolar
R. Sandhu(*) recording. Figure58.12 shows three sites that may eliminate
Department of Cardiology, University of Alberta, Walter Mackenzie
Center, Suite 2C2, 8440 112 St, Edmonton, AB T6G 2B7, Canada
the preexcitation. Site three is the most likely to eliminate
e-mail: rupi.md@gmail.com preexcitation. Figure 58.13 shows application of RF at the
preexcitation site and Fig. 58.14 shows that the PR, 176;
D. Patel
St. Davids Texas Cardiac Arrhythmia Institute, 1015 E. 32nd St. QRS,100; HV, 50 after ablation. Figure 58.15 shows the
#516, Austin, TX 78705, USA post-ablation ECG.
e-mail: dimpi.patel@stdavids.com In summary, the criteria for diagnosing an antidromic
W.R. Lewis AVRT includes that the QRS configuration during the tachy-
Heart and Vascular Center, MetroHealth Medical Center, Case cardia is identical to that obtained during maximal preexcita-
Western Reserve University, 2500 MetroHealth Drive, Suite H322, tion; demonstrate the ventricles participate during the
Cleveland, OH 44109, USA
e-mail: wlewis@metrohealth.org tachycardia (1:1 conduction, atrial premature depolarization
advances the tachycardia with an identical morphology); the
A. Natale
participation of the atria in the tachycardia; and to exclude
Texas Cardiac Arrhythmia Institute, St. Davids Medical Center,
1015 East 32nd Street, Suite 516, Austin, TX 78705 atrial tachycardia, atrial flutter, or AVNRT using a bystander
e-mail: dr.natale@gmail.com pathway.

A. Natale et al. (eds.), Cardiac Electrophysiology, 253


DOI: 10.1007/978-1-84996-390-9_58, Springer-Verlag London Limited 2011
254 R. Sandhu et al.

Fig.58.1 Baseline ECG Baseline ECG

Fig58.2 Baseline intervals: PR, 143; QRS, 104; HV, 27


Case 58 255

Fig.58.3 Atrial extrastimuli

Fig.58.4 AP ERP/AV ERP: 600/330


256 R. Sandhu et al.

Fig.58.5 Atrial fibrillation

Fig.58.6 Shortest preexcited RR


Case 58 257

Fig.58.7 Ventricular pacing: 700 ms

Fig.58.8 VA block: 680 ms


258 R. Sandhu et al.

Fig.58.9 Wide complex tachycardia

Fig.58.10 Antidromic AVRT


Case 58 259

Fig.58.11 Atrial S2 advances tachycardia. CS distal pacing

Site 1 Site 2 Site 3

Fig.58.12 Which site would you select for ablation?


260 R. Sandhu et al.

Fig.58.13 Application of RF energy

Fig.58.14 Post-ablation intervals: Pr, 176msec; QRS 100 msec; HV 50 msec


Case 58 261

Fig.58.15 Post-ablation ECG

Bibliography

Yee R, Klein GJ, Sharma AD, etal. Tachycardia associated with acces-
sory atrioventricular pathways. In: Zipes DP, Jalife J, eds. Cardiac
Electrophysiology. Philadelphia, PA: WB Saunders; 1990:463.

Case 59

M. Eyman Mortada, Jasbir S. Sra, and Masood Akhtar

Case Summary Case Discussion

During ventricular programmed stimulation, the following To recognize the direction of the atrial activation, it is essential to
response was observed (note CS 1,2 is distal) (Fig. 59.1). be acquainted with the normal atrial activation originating from
What are the mechanisms of retrograde conduction seen the sinus node high right atrium. In this patient, the last atrial
in this tracing? beat comes from the high right atrium sinus node (Fig. 59.2).

Fig. 59.1 ECG demonstrating the response of program stimulation d istal coronary sinus recording, followed by the His recording, and
from the right ventricle with one premature beat after pacing in a finally the recording from the right ventricular apex. HRA high right
steady cycle for six beats. At the top is Lead V1 from surface ECG, atrium; CS coronary sinus; HISp proximal His; HISm mid His; RVa right
followed by the intracardiac tracing. From top to bottom: tracing from ventricular apex
the right atrial appendage HRA, then proximal coronary sinus to

M.E. Mortada(*), J.S. Sra, and M. Akhtar


Department of Electrophysiology, Aurora Cardiovascular Services,
Aurora Sinai/Aurora St. Lukes Medical Centers, University of
Wisconsin School of Medicine and Public Health, 2801
W. Kinnickinnic River Parkway, #777, Milwaukee, WI 53215, USA
e-mail: publishing4@aurora.org

A. Natale et al. (eds.), Cardiac Electrophysiology, 263


DOI: 10.1007/978-1-84996-390-9_59, Springer-Verlag London Limited 2011
264 M.E. Mortada et al.

Fig. 59.2 Response of program stimulation from the right ventricle followed by the His recording, and finally the recording from the right
with one premature beat after pacing in a steady cycle for 6 beats, with ventricular apex. HRA high right atrium; CS coronary sinus; HISp
illustration of the activation sequence (arrows). At the top are three proximal His; HISm mid His; HISd distal His; RVa right ventricular
surface ECGs including lead I, II, and V1, followed by the intracardiac apex; H annotation of the His deflection; A annotation of the atrial
tracing. From top to bottom: tracing from the right atrial appendage deflection; V annotation of the ventricular deflection
HRA, then proximal coronary sinus to distal coronary sinus recording,

The first atrial beat appears after a ventricular pacing that the initial electrical activation for the third and fourth
rhythm, with earliest activity seen in the His, suggestive of ventricular complexes is in the right ventricle apex, which is
retrograde activity over the atrioventricular node (AVN). far from the left free-wall accessory pathway. On the other
However, the atrial activity in the second complex is clearly hand, the initial electrical activation of the fifth ventricular
retrograde over the AVN, with earliest activation in the His, complex is over the normal conduction system to both
followed by the proximal coronary sinus, then the high right ventricles simultaneously; thus the electrical activity reaches
atrium. Therefore, the first atrial beat is a fusion beat between the accessory pathway rapidly and, subsequently, the VA
the sinus node (the activity in the high right atrium appears duration becomes shorter.
prior to the activity in the proximal coronary sinus) and the The origin of the first three ventricular complexes (left
retrograde activation over the AVN (the activity in the His is bundle branch block morphology) is right ventricular pacing.
earliest). To understand the fourth ventricular complex, it is first
The third atrial beat starts after premature ventricular necessary to evaluate the fifth ventricular complex. The fourth
extrastimuli. Conduction is seen from the distal to the proxi- atrial complex, as explained previously, has been activated
mal coronary sinus followed by the high right atrium and retrogradely through the left free-wall accessory pathway.
His, suggestive of retrograde activation over a left free-wall After activating the atrium, it comes down through the AVN
accessory pathway. and His-Purkinje system to both ventricles. The AH duration
The fourth atrial beat follows a left bundle QRS complex is longer than the baseline sinus rhythm AH (compared to the
with the same VA duration and activation sequence as the sixth complex) due to the decremental property of the AVN
third atrial complex; hence conduction is retrograde over the node. The HV duration in this complex (the fifth complex)
left free-wall accessory pathway. is the most accurate duration from the His to the earliest
The fifth atrial beat has the same activation sequence as ventricular activation over the normal conduction system.
the previous two atrial complexes, but the VA duration is The fourth ventricular complex has left bundle branch
shorter. The rationale behind the difference in VA duration is block morphology. It is preceded by a His deflection that
Case 59 265

comes prior to atrial activation. Therefore, this complex did Finally, the sixth ventricular complex comes after a sinus
not result from atrial activation. It is either a premature ven- beat, which conducts over the AV node, down to the His-
tricular capture, or it is related to the previous ventricular Purkinje system, then to both ventricles. The possibility of
activity (e.g. bundle branch reentry or repetitive ventricular a ventricular fusion beat occurring over the AV node and
response). The HV duration is slightly longer than the base- the left free-wall accessory pathway in the sixth ventricular
line HV duration when compared to the fifth complex, which complex is excluded due to the fact that the HV duration of
is considered the most accurate HV duration over the normal this beat is the same as the duration of the most accurate
conduction system, as previously mentioned. Hence, the HV duration over the AVN and His-Purkinje system (the
source of this complex is probably bundle branch reentry. fifth beat).

Case 60

Matthew D. Hutchinson

Case Summary may be recorded to the left of the midseptum. Approximately


20% of outflow tract tachycardias will originate from left
sided structures such as: the ASOV, the LVOT, the coronary
A 41-year-old woman without structural heart disease
arterial or coronary venous musculature, or the epicardial
presents with recurrent palpitations due to ventricular
surface.
bigeminy. She had failed multiple antiarrhythmic medications,
Pacemaps from the ASOV produce very different and
including amiodarone. She was referred to our institution
characteristic morphologies. The non-coronary cusp sits
for consideration of ablation. Her clinical PVC is shown in
adjacent to the inter-atrial septum, and a large atrial signal
Fig.60.1; the morphology is left bundle, inferior axis with
(and occasionally a His bundle defection) is recorded there.
a precordial transition at V3.
The right cusp is situated anterior and rightward to the left
Based upon the morphology of the PVCs in Fig. 60.1,
cusp, producing morphologies with later precordial transi-
what is the probable site of origin of the patients
tions (usually at V2 or V3) and a larger R wave in lead I.
arrhythmia?
Pacing more leftward within the right cusp as well as within
the left cusp produces a progressively more rightward frontal
plane axis, a more prominent rS complex in lead I, and an
earlier precordial transition (at V1 or V2). Tachycardias
Case Discussion
originating from between the right and left cusps have a
characteristic notching in the S wave in V1.
The patient presents with frequent PVCs of a left bundle, The most characteristic feature of outflow-type tachycar-
inferior axis morphology. When differentiating sites of ori- dias originating from epicardial structures is delayed initial
gin for outflow tract tachycardias, it is most important to activation in the precordial leads. Daniels and colleagues
consider the precordial transition in light of the anatomical describe the maximum deflection index as the ratio of the
relationships between the RV and LV outflow tracts (RVOT, longest time from QRS onset to maximal QRS deflection
LVOT). The RVOT is positioned anterior to the LVOT rela- (either positive or negative) in any precordial lead to the total
tive to the chest wall; thus the precordial transition for RVOT QRS duration.2 Ratios of 0.55 were predictive of epicardial
sites tends to be later than LVOT sites (typically V4 or later).1 origin with high sensitivity and specificity. Bazan and col-
Early transitions at V1 or V2 suggest an origin more poste- leagues further described the presence of Q wave in lead I
rior than the RVOT, most commonly the aortic sinuses of and the absence of Q waves in the inferior leads as highly
Valsalva (ASOV) or the LVOT. Due to the higher prevalence predictive of epicardial origin from the basal superior LV.3
of RV outflow tract tachycardias, the majority of left bundle Thus for our patient, the precordial transition at V3 presents
inferior axis tachycardias with precordial transition at V3 a variety of possibilities for the site of origin.4
will originate from the septal (posterior or anterior) aspect A combination of activation and pacemapping is utilized
of the RV outflow tract. This is because the septal aspect is in these cases. Activation times were later than the onset of
frequently crescent shaped and both extreme posterior and the QRS in the RVOT, and the pacemap was a poor match
anterior aspects of the septal surface of the outflow region (Fig.60.2). Detailed mapping of the ASOV was performed
which revealed an excellent pacemap from the left cusp with
an activation time 31ms pre-QRS; however ablation at this
M.D. Hutchinson site was unsuccessful (Fig.60.2). We then performed map-
Cardiovascular Division, Department of Medicine,
University of Pennsylvania, 3400 Spruce Street, 9 Founders,
ping of the distal coronary sinus near the origin of the ante-
Philadelphia, PA 19104, USA rior interventricular vein (AIV); the activation time was also
e-mail: matthew.hutchinson@uphs.upenn.edu 31ms pre-QRS, and the pacemap was a near perfect match

A. Natale et al. (eds.), Cardiac Electrophysiology, 267


DOI: 10.1007/978-1-84996-390-9_60, Springer-Verlag London Limited 2011
268 M.D. Hutchinson

Fig.60.1 12-lead ECG I


revealing left bundle, inferior
axis PVCs in a bigeminal II
pattern
III

aVR

aVL

aVF

V1

V2

V3

V4

V5

V6

a b
LCC activation Clinical RVOT RCC LCC

I I

II II
III
III
aVR
aVR
aVL
aVL
aVF

V1 aVF

V2 V1
V3
V2
V4
Fig.60.2 (a) Clinical PVC with V3
activation time 31ms pre-QRS V5
as recorded from the left V4
V6
coronary cusp (LCC). (b)
Clinical PVC with pacemaps ABld V5
obtained from the RV outflow
tract (RVOT), the right and the ABlp V6
left coronary cusps (RCC, LCC)

(Fig.60.3). Ablation in this location abolished the patients ectopy. However, due to individual variability in the anatom-
PVCs. ical relationships between these complex structures, detailed
In summary, the 12-lead ECG provides important infor- activation and pacemapping are of critical importance when
mation about the site of origin of outflow tract ventricular ablating outflow tract ectopy.
Case 60 269

Fig.60.3 (a) Clinical PVC a b c


with activation time 31ms CS activation GCV/AIV Clinical
pre-QRS as recorded from the
proximal portion of the anterior I I
interventricular vein (AIV). II II
(b) Clinical PVC with pacemap
obtained from the AIV. (c) III
III RAO
Fluoroscopy obtained in the aVR
right and left anterior oblique aVR
projections demonstrating the aVL
proximity of two catheters aVF aVL
placed in the LCC and
proximal AIV V1 aVF

V2 V1
V3
V2
V4
V3
V5 LAO
V4
V6

Abld V5

Ablp V6

References 3. Bazan V, Gerstenfeld EP, Garcia FC, etal. Site-specific twelve-lead


ECG features to identify an epicardial origin for left ventricular
tachycardia in the absence of myocardial infarction. Heart Rhythm.
1. Dixit S, Gerstenfeld EP, Callans DJ, Marchlinski FE. Electro 2007;4(11):1403-1410.
cardiographic patterns of superior right ventricular outflow tract 4. Tanner H, Hindricks G, Schirdewahn P, etal. Outflow tract tachy-
tachycardias: distinguishing septal and free-wall sites of origin. cardia with R/S transition in lead V3: six different anatomic
J Cardiovasc Electrophysiol. 2003;14(1):1-7. approaches for successful ablation. J Am Coll Cardiol. 2005;45(3):
2. Daniels DV, Lu YY, Morton JB, etal. Idiopathic epicardial left ven- 418-423.
tricular tachycardia originating remote from the sinus of Valsalva:
electrophysiological characteristics, catheter ablation, and identifi-
cation from the 12-lead electrocardiogram. Circulation. 2006;
113(13):1659-1666.

Case 61

Ronald Lo, Henry H. Hsia, and Amin Al-Ahmad

Case Summary Case Discussion

A 55-year-old woman with no significant past medical his- Analysis of the electrocardiogram demonstrates ventricular
tory has been complaining of palpitations for the past 3 years. bigeminy with left bundle left inferior axis morphology
These symptoms are associated with fatigue, activity intoler- PVCs. This suggested a possible right ventricular outflow
ance, chest discomfort, and lightheadness. A Holter monitor tract origin of the PVCs; however, the early transition raises
was ordered which demonstrated greater than 20,000 PVCs the possibility of a left ventricular outflow tract or aortic cusp
per day. These accounted for approximately 25% of her daily site of origin. Initial mapping of the right ventricle and the
heart beats. right ventricular outflow tract did not locate any points of
Her electrocardiogram is shown in Fig.61.1. Her echocar- earliest activation that were earlier than ventricular activa-
diogram demonstrated an ejection fraction of approximately tion. Mapping was then performed in the left ventricle and
45% with normal valves and cardiac chamber sizes. She was the aortic root along with the aortic cusps. Careful mapping
tried on beta blockers without any effect with respect to using CARTO demonstrated a site between the left and right
symptoms. Where is the origin of her premature ventricular aortic cusps that was 22 ms presystolic to the earliest ven-
contractions? tricular electrogram. The unipolar electrogram also demon-
strated a QS signal as shown in Fig.61.2.
Pace mapping was also performed in the areas around the
aortic cusps, with the best pace mapped site being in between
the right and the left aortic cusps. Further analysis of the
electrocardiogram demonstrated a qrS pattern in leads
V1V3, which appear to be due to a site of activation in
R. Lo(*) between the right and left coronary cusps. The activation
Riverside Electrophysiology, 4000 14th Street, Suite 209, pattern is due to direction of the propagating wave from the
Riverside, CA 92501, USA right and left coronary cusps as a q wave in lead V1, and the
e-mail: heartfailure@gmail.com
anterior activation pattern from the RVOT to the aortic root
H.H. Hsia as an r wave in V1. The remainder of the ventricular activa-
Department of Cardiovascular Medicine, Stanford University, tion produced a large S wave resulting in the qrS pattern seen
300 Pasteur Drive, H2146, Stanford, CA 94305, USA
e-mail: hhsia@stanford.edu on the electrocardiogram.
Catheter ablation in this region (Fig. 61.3) produced
A. Al-Ahmad
School of Medicine, Stanford University, 300 Pasteur Drive,
successful termination of the spontaneous PVCs and repeat
H-2146, Stanford, CA 94305, USA Holter monitors afterward did not demonstrate any PVCs
e-mail: aalahmad@stanford.edu consistent with the ablated morphology.

A. Natale et al. (eds.), Cardiac Electrophysiology, 271


DOI: 10.1007/978-1-84996-390-9_61, Springer-Verlag London Limited 2011
272 R. Lo et al.

Fig.61.1 12-lead ECG with ventricular bigeminy with left bundle left inferior PVCs

Unipolar and bipolar activation mapping

LAT 43 ms

2-Map > 134 Points

AoV

114 ms
R
L

0.70 cm
Fig.61.2 Left ventricular
unipolar and bipolar activation
mapping demonstrating earliest
activation in the region between
the left and right coronary cusp
and the earliest activation at that
site 22 ms presystolic
Case 61 273

Aortogram Successful ablation site Bibliography

Yamada T, Yoshida N, Murakami Y, etal. Electrocardiographic charac-


teristics of ventricular arrhythmias originating from the junction
of the left and right coronary sinuses of Valsalva in the aorta: the
activation pattern as a rationale for the electrocardiographic
characteristics. Heart Rhythm. 2008;5:184-192.

LAO

Fig.61.3 Successful ablation site of premature ventricular contraction


in between the left and right coronary cusp

Case 62

Richard H. Hongo and Andrea Natale

Case Summary onset of the P wave. Pacemapping at this site is only an


11-out-of-12 leads match with the VPC morphology. Despite
the less than ideal activation time and pacemapping match,
The patient is a 57-year-old woman with newly diagnosed
attempting ablation with the 4-mm catheter is appropriate
nonischemic dilated cardiomyopathy (LVEF 2530%) and
before proceeding with the higher risk left-sided mapping
>11,000 uniform VPCs during a 24-h Holter monitor. She
and ablation. Once, however, the lack of VPC flurries and the
presented for electrophysiology study for ablation of the VPC
persistence of VPCs are apparent, further ablation with
focus (Fig.62.1) and to assess for inducible sustained ven-
higher powered catheters is unlikely to be successful, and the
tricular tachycardia. 3-D electroanatomic activation mapping
next most appropriate step is to proceed with mapping of the
was performed with a 4-mm Navistar RMT catheter utilizing
LVOT; the LVOT is more approachable than the epicardium,
the CARTO and Stereotaxis remote magnetic navigation sys-
and the VPC ECG morphology is more suggestive of an
tems. Earliest activation (0ms presystolic) of the recurring
LVOT focus. Following transseptal puncture, 3-D activation
VPCs within the right heart was localized to the anteroseptal
mapping of the recurring VPCs was performed with remote
RVOT, just below the pulmonary valve. Pacemapping was
magnetic navigation within the LVOT. A region of early acti-
also performed and an 11/12-lead morphology match was
vation was found just below the aortic valve, along the pos-
achieved at the same site as the earliest activation.
teroseptal aspect of the LVOT, but was no earlier than the
Ablation was performed at this site with the 4-mm RMT
earliest site in the RVOT (0 ms presystolic). Within this
catheter with temperature and power limited to 52C and 50W,
region, however, there was a single site that demonstrated
respectively. Despite multiple ablations (Fig. 62.2) achieving
presystolic (20ms) activation (Fig.62.3). The first ablation
adequate temperature and power, there were no flurries of ven-
at this site resulted in a brief flurry of VPCs that was fol-
tricular beats during ablations and the VPCs continued to recur.
lowed by complete cessation of VPCs (Fig. 62.4). Several
What is the most appropriate next step? Should this patient
ablations were performed adjacent to the first ablation, tar-
receive an ICD?
geting fractioned electrogram signal. There were no recur-
rent VPCs observed overnight on telemetry monitoring, and
the VPC focus has not recurred after 6 months. The LVEF
Case Discussion has remained depressed (<30%), however, and she has under-
gone insertion of an implantable cardioverter-defibrillator
for primary prevention of sudden cardiac arrest. Elimination
Prominent R waves in precordial leads V1 and V2 is most
of the VPC focus by ablation should precede insertion of an
consistent with an LVOT VPC focus. Right heart electroana-
implantable cardioverter-defibrillator in this patient because
tomic mapping reveals earliest activation at the anteroseptal
there is a reasonable possibility that the newly diagnosed
aspect of the RVOT just below the pulmonary valve. The ear-
dilated cardiomyopathy is caused by the excessive burden of
liest site, however, is on time with, but does not precede, the
VPCs. Persistence of systolic dysfunction (LVEF of 35% or
less) for over 3 months, however, prompted her to be recon-
R.H. Hongo sidered for implantable cardioverter-defibrillator therapy.
Sutter Pacific Medical Foundation, California Pacific Medical Center,
2100 Webster Street, Suite 521, San Francisco, CA 94115, USA
e-mail: hongor@sutterhealth.org
A. Natale(*)
Texas Cardiac Arrhythmia Institute, St. Davids Medical Center,
1015 East 32rd Street, Suite 516, Austin, TX 78705, USA
e-mail: dr.natale@gmail.com

A. Natale et al. (eds.), Cardiac Electrophysiology, 275


DOI: 10.1007/978-1-84996-390-9_62, Springer-Verlag London Limited 2011
276 R.H. Hongo and A. Natale

Fig.62.1 12 lead ECG showing morphology of VPC

Fig.62.2 CARTO map showing ablation lesions in the RVOT


Case 62 277

Fig.62.3 Site of earliest


activation in the LVOT

Fig.62.4 CARTO map showing site of successful ablation



Case 63

David J. Callans

Case Summary nonsustained and sustained polymorphic VT (Fig. 63.1).


What is the best strategy for ablation in this patient?
A 46-year-old woman with a history of renal failure status
post failed transplant received an ICD after an episode of
ventricular fibrillation (VF). Evaluation at that time revealed Case Discussion
normal LV function and no coronary artery disease. She was
well for 6months, but then presented with syncope and ICD In the electrophysiology laboratory, she had multiple mor-
shocks. Interrogation of the ICD showed 60 episodes detected phologies of both left and right bundle PVCs, all of which
in the VF zone over the previous 4days, most of them non- were preceded by an apparent Purkinje potential (Fig.63.2).
sustained, but four resulting in shock delivery. Telemetry Catheter ablation was targeted at elimination of Purkinje
monitoring showed frequent episodes of nonsustained poly- potentials in the area around earliest onset of PVCs.
morphic VT. She was transferred to our hospital for further Afterward, no further spontaneous PVCs were noted despite
evaluation. 12-lead ECG monitoring at this hospital showed isoproterenol infusion (Fig.63.3).
frequent PVCs with several different right bundle, left supe- The patient has been well, without recurrent syncope or
rior axis morphologies which frequently started episodes of ICD shocks over 6months follow-up.

D.J. Callans(*)
Department of Cardiology, University of Pennsylvania,
3400 Spruce Street, Philadelphia, PA 19104, USA
e-mail: david.callans@uphs.upenn.edu

A. Natale et al. (eds.), Cardiac Electrophysiology, 279


DOI: 10.1007/978-1-84996-390-9_63, Springer-Verlag London Limited 2011
280 D.J. Callans

Fig.63.1 Frequent PVCs, with slightly different right bundle, left superior axis morphologies, often causing runs of nonsustained and sustained
polymorphic ventricular tachycardia
Case 63 281

Fig.63.2 Left and right bundle


PVCs with intracardiac I
recordings from a mapping
catheter (Carto P, Carto D), and II
RV apical catheter. Note that
PVCs were preceded by a sharp
potential, presumably from the III
Purkinje system
aVR

aVL

aVF

V1

V2

V3

V4

V5

V6

Carto P

Carto D

RVa
9:55:26 AM
10:10:30 AM

Fig.63.3 Left ventricular voltage mapping in the RAO and LAO projec- the electrograms recorded in this region were normal in morphology.
tions. The voltage maps are essentially normal. Although some abnor- Red dots represent individual ablation lesions which were targeted by
malities are suggested at the junction of the inferior wall and the septum, finding Purkinje-like potentials in sinus rhythm and before PVCs

Case 64

J. David Burkhardt, Dimpi Patel, and Andrea Natale

Case Summary What is the diagnosis? Where is the origin of the VPC?
What is the best strategy for management of this patient?
A 48-year-old Hispanic male presents with a history of
dyspnea on exertion that has been present for two months.
He has been evaluated by an internist who discovered on
monitoring that the patient had frequent PVCs and runs of Case Discussion
monomorphic ventricular tachycardia. He had an echocar-
diogram that revealed an EF of 25% without segmental The case is consistent with a PVC-induced cardiomyopathy.
wall motion abnormality. A nuclear stress test revealed no The high frequency of PVCs is the likely culprit behind the
scar or ischemia. symptoms and left ventricular dysfunction. The location
A 24-h Holter monitor revealed frequent runs of ventricu- appears to be LVOT and specifically aortic cusp. The fea-
lar tachycardia at a rate of 150 BPM up to 12 beats and tures include high amplitude R waves in II, III, and aVf, as
45,000 PVCs. All of the PVCs and VT are of the same mor- well as the early transition and notched upstroke of the
phology. An EKG is included below (Fig.64.1). Laboratory R wave in lead V6.
tests including TSH were unremarkable. He denies syncope With a normal stress test and typical location of normal
or recent illness. heart PVCs further evaluation is not necessary. The LV

J.D. Burkhardt (*) and A. Natale


Texas Cardiac Arrhythmia Institute,
1015 E. 32nd Street, Suite 505, Austin, TX 78705
e-mail: jdavidburkhardt@hotmail.com; dr.natale@gmail.com
D. Patel
St. Davids Texas Cardiac Arrhythmia Institute,
1015 E. 32nd St. #516, Austin, TX 78705
e-mail: dimpi.patel@stdavids.com

A. Natale et al. (eds.), Cardiac Electrophysiology, 283


DOI: 10.1007/978-1-84996-390-9_64, Springer-Verlag London Limited 2011
284 J.D. Burkhardt et al.

dysfunction and congestive heart failure symptoms will recovery. ICE may be a useful aide in this endeavor
likely resolve with elimination of the PVC. An ICD is not (Fig.64.2). It is difficult to appreciate the anatomy of the
indicated since this is likely a reversible condition. The aortic valve using flouroscopy or electroanatomic mapping
patient was tried on a medication; however there was no alone. Pacemapping at the aortic cusp revealed a perfect
decrease in the number of VPCs. Ablation is the next rea- pacemap (Fig.64.3). This location was targeted for ablation
sonable step to give the patient the best chance of full and was successful in eliminating the VPC.

25mm/s 10mm/mV 150Hz 005E 12SL 250 CID:19 EID:23 EDT: 13:20 06-FEB-2006 ORDER: 256788740 ACCOUNT: 0999

Fig.64.1 12-lead ECG showing the VPC

Fig.64.2 Intracardiac echo helps with the anatomy of the aortic valve
region
Case 64 285

Successful site Pacemap

I I
100 ms
II II

III III

aVR aVR

aVL aVL

aVF aVF

V1 S V1

V2 V2

V3 V3

V4 V4

V5 V5

V6 V6

ABL 1,2 ABL 1,2

Abl uni 49 msec Abl uni

ABL 3,4 ABL 3,4


5:58:05 PM 5:42:29 PM
CardioLab v6.0 CardioLab v6.0
GF Medical Systems

Fig.64.3 Spontaneous VPC vs. pacemap



Case 65

Matthew D. Hutchinson

Case Presentation and colleagues found that the inciting PVC originated either
in the right ventricular outflow tract (15%) or the Purkinje
network (85%).1 Those beats originating in the LV Purkinje
A 36-year-old woman was referred to our institution for
network were characterized by a relatively narrow QRS
repetitive, polymorphic ventricular tachycardia and ventricu-
duration (11511 ms) compared to those from the RV
lar fibrillation (PMVT, VF). A comprehensive cardiovascu-
(14310ms).
lar evaluation revealed normal left ventricular structure and
This patients PVC morphology is narrow (ms) and has a
function. The baseline QT interval was normal and there was
right bundle superior axis which is consistent with an origin
no family history of sudden cardiac death. She had an ICD
from the left ventricular Purkinje network. In the aforemen-
placed due to recurrent presyncope with these episodes.
tioned Haissaguerre series, the earliest Purkinje deflections
Intracardiac electrograms and telemetric monitoring revealed
for LV ectopy preceded the local ventricular activation by an
recurrent PMVT episodes which were initiated with short-
average of 4629ms (range 10150ms).The most common
coupled PVCs of uniform morphology. She had failed mul-
ablation strategy is to identify the earliest presystolic Purkinje
tiple antiarrhythmic agents, as well as a previous ablation
activity; this is most easily accomplished via electroanatomi-
attempt. The 12-lead morphology of her initiating PVC is
cal mapping of the Purkinje pre-potentials.
given in Fig.65.1.
Figure65.2 demonstrates mapping along the LV septum
What are the common sites of origin for idiopathic PVCs
in our patient. Note the dramatic prematurity seen in the con-
initiating PMVT/VF?
ducted ectopic beats. Also shown in Fig.65.2 is a more pre-
What is the likely site of origin for the patients inciting
mature Purkinje depolarization which is not conducted to the
PVC, and what strategy is most effective in mapping such
local myocardium. Figure65.3 depicts an episode of NSVT
PVCs?
with early Purkinje pre-potentials driving the tachycardia.
Ablation at this site abolished the clinical PVC; however
a second PVC morphology was then seen. This PVC was
Case Discussion mapped more apically and was noted to have extremely early
Purkinje activation in this region (Fig.65.4). Ablation at this
This case highlights the uncommon clinical entity of idio- site abolished the patients PVCs.
pathic PMVT/VF. These patients have normal corrected QT The majority of these patients have an excellent long-term
intervals and lack the characteristic findings of catecholamin- result after ablation, and 89% of patients in the Haissaguerre
ergic polymorphic ventricular tachycardia on exercise test- group were arrhythmia free off of antiarrhythmic drugs at a
ing. The largest experience of 27 patients by Haissaguerre mean follow-up of 2years.

M.D. Hutchinson
Cardiovascular Division, Department of Medicine,
University of Pennsylvania, 3400 Spruce Street, 9 Founders,
Philadelphia, PA 19104, USA
e-mail: matthew.hutchinson@uphs.upenn.edu

A. Natale et al. (eds.), Cardiac Electrophysiology, 287


DOI: 10.1007/978-1-84996-390-9_65, Springer-Verlag London Limited 2011
288 M.D. Hutchinson

Fig.65.1 The patients dominant I


clinical PVC is shown in a
trigeminal pattern in sinus II
rhythm. The ventricular ectopy is
narrow, and has a right bundle III
superior axis
aVR

aVL

aVF

V1

V2

V3

V4

V5

V6

I
II
III
aVR
aVL
aVF
V1
V2
V3
V4
V5
Fig.65.2 Intracardiac recordings
obtained during LV activation V6
mapping of the clinical PVC. LV5
Bipolar recordings are shown LV4
from the His bundle position (His LV3
15) and LV septum (LV 15). LV2
There are sharp early potentials LV1
(asterisks) on the ablation
Abld
catheter poles (Abl) recorded
after the first sinus beat which Ablp
represent Purkinje potentials His5
which do not activate the LV His4
myocardium. After the second His3
sinus beat, there are later-coupled His2
Purkinje potentials which do His1
conduct to the myocardium and
RVa
produce the clinical PVC
Case 65 289

a b
I
I
II
III
II
aVR
aVL
III
aVF
V1
aVR
V2
V3
aVL V4
V5
aVF V6
LV5
V1 LV4
LV3
V2 LV2
LV1
Abld
V3
Ablp
His5
V4
His4
His3
V5
His2
His1
V6 RVa

Fig.65.3 (a) 12-lead ECG demonstrating a typical episode of NSVT initiated by the clinical PVC. (b) Intracardiac recordings from the episode of
NSVT pictured in panel A. Note the premature Purkinje potentials (asterisks) preceding both the clinical PVC and each beat of nonsustained VT

I Reference
II
III
aVR 1. Haissaguerre M, Shoda M, Jais P, et al. Mapping and ablation
aVL of idiopathic ventricular fibrillation. Circulation. 2002;106(8):
aVF 962-967.
V1
V2
V3
V4
V5
V6
LV5
LV4
LV3
LV2
LV1
Abld
Ablp
His5
His4
His3
His2
His1
RVa

Fig. 65.4 Intracardiac recordings of a second PVC which was seen


after ablation of the clinical PVC. Note the dramatically premature
Purkinje potentials noted on the ablation catheter (asterisks). This PVC
was targeted at the anteroapical LV

Case 66

Matthew D. Hutchinson

Case Presentation the lateral mitral annulus (Fig.66.2). Endocardial pacemaps


for the clinical VT were poor and activation times late during
VT; thus no ablation was performed.
A 73-year-old man with a longstanding nonischemic
The patient was taken back to the EP lab on a later date
cardiomyopathy and an implantable defibrillator placed
for an epicardial approach. Percutaneous epicardial access
for secondary prevention of sudden death was referred for
was obtained using the Sosa technique. Epicardial bipolar
increasingly frequent ICD shocks due to rapid VT.
voltage mapping was performed which revealed a large,
Interrogation of his ICD revealed three distinct intracardiac
confluent area of low voltage along the lateral LV extending
morphologies. He was referred for EP study and voltage
from the mitral annulus to the mid-cavity (Fig.66.3). Despite
mapping, at which time he had a single VT morphology
aggressive programmed stimulation with up to triple extra-
reproducibly induced with programmed stimulation
stimuli and isoproterenol infusion, only polymorphic VT
(Fig. 66.1). The intracardiac electrograms of this induced
was inducible.
tachycardia matched one of the patients dominant clinical
What features of the clinical tachycardia suggest an
arrhythmias. Biventricular voltage maps were normal with
epicardial site of origin?
the exception of a small area of attenuated electrograms at
Based upon the information available, describe the most
effective ablation strategy for this patient.
Sinus rhythm Clinical VT
200 ms
a b
I I
II II Case Discussion
III III
aVR aVR There are several ECG criteria for predicting LV epicardial
aVL aVL VT origin in patients with nonischemic dilated cardiomyo-
aVF aVF pathy. Based upon the ECG morphology of the VT in
V1 Fig. 66.1, one would expect a mid-inferolateral exit site.
V1 Bazan and colleagues prospectively examined several crite-
V2 V2
V3
ria to differentiate basal and apical inferior epicardial tachy-
V3
V4
cardias, and found the following to have significant predictive
V4
V5 value: the presence of Q waves in the inferior leads; a
V5
V6 pseudodelta wave 34ms (measured from onset of QRS to
V6
earliest rapid deflection in precordial leads); and the shortest
Fig. 66.1 (a) Sinus rhythm QRS morphology. (b) Clinical VT precordial RS complex 121 ms (measured from onset of
morphology precordial QRS to earliest nadir of S wave). All of the above
criteria are met for our patients tachycardia (Fig.66.4).
The most effective method of localizing the site for VT
ablation is entrainment mapping; however this is not possible
in many patients due either to arrhythmia noninducibility or
M.D. Hutchinson to hemodynamic compromise during VT. In this case in
Cardiovascular Division, Department of Medicine, which no VT is inducible, an alternate strategy utilizing
University of Pennsylvania, 3400 Spruce Street, 9 Founders, substrate-based ablation is necessary. The technique of sub-
Philadelphia, PA 19104, USA
e-mail: matthew.hutchinson@uphs.upenn.edu strate-based ablation was previously described by Marchlinski

A. Natale et al. (eds.), Cardiac Electrophysiology, 291


DOI: 10.1007/978-1-84996-390-9_66, Springer-Verlag London Limited 2011
292 M.D. Hutchinson

Fig.66.2 Bipolar LV and RV >1.5 mV


Limited endocardial
endocardial voltage maps in LAO
voltage abnormality
(left) and PA (right) projections.
There is a limited region of low
voltage present at the basal
anterolateral LV

LAO PA
<0.5 mV

Lateral LV Sinus rhythm Clinical VT


>1.0 mV 200 ms
epicardium PD = 52 ms
a I
b
I
II II
III III
Base aVR aVR
aVL aVL
aVF aVF
V1
Apex R V1
V2
V2
V3 V3
V4 V4
Fractionated/late
potentials <0.5 mV V5 V5
V6
V6
RS = 164 ms
Fig.66.3 Bipolar LV epicardial voltage map with a large region of low
voltage involving the lateral LV extending from the base to the midwall. Fig.66.4 (a) The sinus rhythm QRS complex with arrows indicating
Areas with abnormal electrograms (late or fractionated potentials) are the absence of Q waves in the inferior leads. (b) The patients VT mor-
labeled with a gray tag phology demonstrates prominent QS waves in the inferior leads. The
pseudodelta wave measured in the precordial leads is 52ms; the short-
est RS complex (measured in V4) is 164ms. These features suggest an
epicardial VT exit site. See text for discussion
and colleagues, and requires a detailed knowledge of the
patients substrate.1 High-density voltage mapping to define
the area of bipolar voltage attenuation is performed; bipolar (defined by a bipolar voltage <0.5mV). Whenever possible,
signals <1.5 mV on the endocardium and <1.0 mV on the targeting areas of markedly abnormal (late and split) electro-
epicardium are characterized as abnormal. Sites with abnor- grams is also performed.
mal electrogram characteristics including fractionated, late, In this case, we have detailed information about the
and split potentials are given anatomical tags; these areas patients substrate from electroanatomical mapping. The
represent regions of surviving myocardium within the scar gray dots in Fig.66.3 represent abnormal bipolar electro-
which exhibit abnormal conduction properties and may par- grams within the low voltage region. We also have a
ticipate in tachycardia circuits. When information regarding 12-lead morphology of one of the patients clinical arrhyth-
the patients spontaneous or induced arrhythmias is avail- mias which was induced during the initial EP study.
able, the approximate exit sites for these tachycardias are Pacemapping of this VT morphology was performed, with
localized by pacing at the border zone of the bipolar voltage the best match from the inferior scar border (Fig. 66.5).
abnormality. Linear lesions sets are designed incorporating Linear lesions were applied extending from the region of
the individual VT exit sites identified by pacemapping and the best pacemap for the clinical VT into the dense scar.
extending through the border zone into the dense scar Additional linear lesions were given to transect regions
Case 66 293

Fig.66.5 Comparison of the best VT pacemap Clinical VT


pacemap morphology to the
clinical VT. The pacing site is
>1.0 mV
indicated by the star on the
electroanatomical map

<0.5 mV

Fractionated/late
potentials

VT pacemap site

a b
>1.0 mV

Fig.66.6 (a) Lesion set design


which incorporates an initial
linear lesion (yellow dashed line)
through the VT exit region
localized by pacemapping and
extending into dense scar. The R R
other linear lesions (white dashed
lines) transect regions with
abnormal electrogram character- <0.5 mV
istics. (b) Final electroanatomical
map with ablation lesions Targeting clinical VT
represented as red tags and late potentials

with abnormal electrograms as shown in Fig. 66.6. The Reference


usual procedural endpoint of arrhythmia noninducibility is
not applicable in this case; however when properly exe- 1. Marchlinski FE, Callans DJ, Gottlieb CD, Zado E. Linear ablation
cuted, substrate-based ablation strategies are as effective lesions for control of unmappable ventricular tachycardia in patients
as entrainment-based strategies in achieving long-term with ischemic and nonischemic cardiomyopathy. Circulation.
2000;101(11):1288-1296.
arrhythmia suppression.

Case 67

Matthew D. Hutchinson

Case Presentation be more endocardial scar than seen here with the typical
CARTO color settings used (<1.5mV), as underlying myo-
cardial hypertrophy with larger baseline local potentials can
A 39-year-old man with familial hypercholesterolemia, prior
mask superficial scar. Adjustment of the color range may be
coronary endarterectomy, mechanical aortic valve replace-
necessary at times to highlight endocardial scar in the setting
ment, and mitral valve annuloplasty presented with symp-
of ventricular hypertrophy.
tomatic recurrent monomorphic ventricular tachycardia
Figures 67.1 and 67.2 demonstrate frequent monomor-
(MMVT). His sinus rhythm ECG indicated prior inferior
phic PVCs which appear to originate from the mid inferior
infarction; his left ventricular ejection fraction was 50% with
wall (apical-septal aspect of this area) based on the ECG.
mild concentric hypertrophy. He was noted to have frequent
Interestingly, the clinical VT which is initiated by this PVC
monomorphic PVCs (right bundle, left superior axis) which
has an overall similar morphology, but exits more basally
appeared to initiate the episodes of MMVT (Fig.67.1). He
(later precordial transition) and slightly further from the sep-
was then brought to the EP lab for mapping and ablation.
tum (wider QRS). The close relationship between single
The left ventricle was accessed by transseptal approach
PVCs and reentrant VT arising from the area of scar in isch-
due to the mechanical aortic prosthesis. His bipolar voltage
emic substrate has been described by Bogun and colleagues.1
map revealed patchy endocardial low voltage regions involv-
When the PVC is frequent, it can serve as a target for map-
ing the basal and mid inferior walls. The clinical VT (right
ping and ablating a potentially arrhythmogenic area of the
bundle, left superior axis morphology; CL 280ms) was reli-
ischemic scar, especially if the clinical VT is not sustained or
ably initiated by a distinct unifocal PVC (Fig. 67.2). The
tolerated to allow for more detailed entrainment mapping.
blood pressure was marginal during VT, and the patient was
In this case we performed activation and pacemapping of
quite symptomatic.
the PVC, leading us to focus on the area just basal and septal
Based on the ECG morphologies, where are the sites of
to the posteromedial papillary muscle. While mapping here,
origin/exit for both the PVC and the sustained VT?
the clinical VT was initiated by a catheter-induced PVC,
What would be your ablation strategy for this VT; how
and large mid-diastolic potentials were seen at this site.
can the frequent PVCs be incorporated into this strategy?
Entrainment was quickly performed while the VT was still
tolerated, and the response suggested that this was an isth-
mus site. RF delivered here terminated the VT. After termi-
nation, we performed pacemapping from the distal and
Case Discussion proximal bipoles of the ablation catheter (spacing configura-
tion 2mm5mm2mm). Interestingly, pacing at threshold
This patient likely has an ischemic substrate for ventricular from the more apical bipole produced a pace match for the
tachycardia based on the history of premature coronary artery spontaneous PVC (Fig.67.3), and pacing at threshold from
disease, ECG evidence of prior inferior infarction, and evi- the more basal bipole produced an excellent pacemap match
dence of patchy endocardial scar by voltage mapping in the for the clinical VT (Fig.67.4). Several additional lesions in
typical inferior regional distribution. Importantly, there may this area significantly raised the pacing threshold, eradicated
the PVCs, and rendered the VT noninducible.

M.D. Hutchinson
Cardiovascular Division, Department of Medicine,
University of Pennsylvania, 3400 Spruce Street, 9 Founders,
Philadelphia, PA 19104, USA
e-mail: matthew.hutchinson@uphs.upenn.edu

A. Natale et al. (eds.), Cardiac Electrophysiology, 295


DOI: 10.1007/978-1-84996-390-9_67, Springer-Verlag London Limited 2011
296 M.D. Hutchinson

Fig.67.1 12-lead surface ECG 800 ms


obtained at rest outside of the
electrophysiology lab. Note
baseline right bundle-branch I
block as well as Q waves in the
inferior leads (arrows). There are
frequent spontaneous monomor- II
phic PVCs (stars), the last one
initiating a run of monomorphic III
ventricular tachycardia
aVR

aVL

aVF

V1

V2

V3

V4

V5

V6

200 ms

II

III

aVR
aVL

aVF

V1

V2

V3
Fig.67.2 12-lead surface ECG
obtained in the electrophysiology V4
lab at start of the ablation
procedure. Again the sinus beat V5
is followed by the same PVC
(star) which initiates the patients V6
clinical VT
Case 67 297

Fig.67.3 Bipolar voltage map Pacemap PVC


of the left ventricle (inferior
view; purple color denotes I I
voltage>1.5mV, red color
II
denotes voltage<0.5mV) with >1.5 mV II
site (star) of threshold pacemap III
Septum III
compared with spontaneous PVC
aVR aVR
aVL aVL

aVF aVF

<0.5 mV V1 V1

Base Apex V2 V2

V3 V3

V4 V4
Posteromedial
papillary V5 V5
muscle
V6 V6

Pacemap VT

I I

>1.5 mV II II

Septum III III

aVR aVR
aVL aVL

aVF aVF

<0.5 mV V1 V1

Base Apex V2 V2

V3 V3
Fig.67.4 Bipolar voltage map
of the left ventricle (inferior
V4 V4
view; purple color denotes Posteromedial
voltage>1.5mV, red color papillary V5 V5
denotes voltage<0.5mV) with muscle
site (star) of threshold pacemap V6 V6
compared with clinical VT

Reference

1. Bogun F, Crawford T, Chalfoun N, etal. Relationship of frequent


postinfarction premature ventricular complexes to the reentry cir-
cuit of scar-related ventricular tachycardia. Heart Rhythm. 2008;
5(3):367-374.

Case 68

Matthew D. Hutchinson

Case Presentation Interestingly, the patients PVC recorded in the EP lab


demonstrated a different precordial morphology with an ear-
lier transition at V2 (Fig.68.2). The difference in morphol-
A 54-year-old man with CAD and previous inferior infarc-
ogy is due to erroneous placement of the precordial leads in
tion and prophylactic single chamber ICD implantation was
Fig.68.1. Leads V1 and V2 should be placed at the fourth
referred for recurrent, symptomatic monomorphic ventricu-
intercostal space along the sternal borders. In this case, the
lar tachycardia and ICD shocks despite amiodarone. He was
lead positioning for V1 and V2 in Fig.68.1 was most likely
also noted to have frequent, monomorphic PVCs with an
at the third interspace (note the similarity between leads V3
intracardiac morphology matching his clinical tachycardia.
on the initial ECG and V2 on the adjacent laboratory record-
He subsequently underwent an electrophysiology study dur-
ing). It is critically important to check the lead placement
ing which the clinical VT was induced and characterized
whenever using the ECG to predict tachycardia site of origin.
with activation mapping. Activation times for the clinical
In this case, the large R wave in lead V2 and biphasic com-
PVCs of up to 30ms pre-QRS were recorded from the right
plex in lead I suggest a more posterior site of origin from the
ventricular outflow tract; however ablation in this area was
aortic sinuses of valsalva.
acutely unsuccessful. The patient was subsequently referred
During the procedure, the patient had very infrequent
to our institution for a repeat ablation procedure. The patients
PVCs and was noninducible for VT with programmed stimu-
admission ECG is shown in Fig.68.1.
lation. The administration of sedative medications can have
Based upon the morphology of the PVCs on the present-
important implications for arrhythmia inducibility. Moreover
ing ECG (Fig.68.1), what is the differential diagnosis for the
the administration of local anesthetic agents subcutaneously
site of origin of the patients arrhythmia?
can result in measurable serum concentrations which can
What are important laboratory considerations when map-
further suppress arrhythmias. On review it was noted that the
ping tachycardias of this type?
patient had been administered intravenous benzodiazepines
for sedation prior to sheath insertion. After administration of
isoproterenol 0.5 mcg/min, the patient had ventricular big-
Case Discussion eminy. Activation times of 37ms pre-QRS were recorded in
the RV outflow tract; however the pacemap did not match the
spontaneous PVC (Fig.68.3). The aortic sinuses were then
The patient presents with frequent PVCs of a left bundle,
mapped via a retrograde approach. The earliest activation
inferior axis morphology. This combination of features most
was 45ms pre-QRS at the junction between the right and left
commonly represents a right ventricular outflow tract origin.
coronary sinuses; pacemapping this region replicated per-
As discussed previously, however, when the ectopy transi-
fectly the clinical PVC morphology (Fig.68.4). Ablation at
tion is at V3 it is important to left-sided sites of origin (the
this site abolished the patients ectopy.
aortic sinuses of valsalva, the left ventricular outflow tract,
the coronary arterial or venous musculature, or the epicardial
surface).

M.D. Hutchinson
Cardiovascular Division, Department of Medicine,
University of Pennsylvania, 3400 Spruce Street, 9 Founders,
Philadelphia, PA 19104, USA
e-mail: matthew.hutchinson@uphs.upenn.edu

A. Natale et al. (eds.), Cardiac Electrophysiology, 299


DOI: 10.1007/978-1-84996-390-9_68, Springer-Verlag London Limited 2011
300 M.D. Hutchinson

Fig.68.1 The presenting 12-lead ECG demonstrates frequent left bundle, right inferior axis ventricular ectopy

a b
I

II

III

aVR

aVL

aVF

V1

V2

V3

V4

V5

V6

Fig.68.2 (a) The same ECG from Fig.68.1 is reproduced; the black at V2 (arrow). This most likely represents erroneous placement of leads
arrow indicates a large R wave in lead V3. (b) The 12-lead ECG taken V1 and V2 in the third intercostal space
in the EP lab for the same patient shows an earlier precordial transition
Case 68 301

Fig.68.3 (a) Comparison of the a b


clinical PVC morphology to the Clinical PVC RVOT pacemap RVOT activation
best pacemap obtained from
the RV outflow tract. Although I I
the frontal plane axis matches the
clinical ectopy well, the II
II
precordial transition is much
later from the RVOT. (b) The III
earliest activation time for the III
clinical PVC from the RVOT is aVR
37ms pre-QRS aVR
aVL

aVL aVF

aVF V1

V2
V1
V3
V2
V4

V3 V5

V4 V6

Abld
V5
Ablp
V6
RVa

a b
Clinical PVC R/LCC pacemap R/LCC activation

I I

II II

III
III
aVR
aVR
aVL

aVL aVF

V1
aVF
V2
V1
V3

V2 V4

Fig.68.4 (a) Comparison of the V3 V5


clinical PVC morphology to the
best overall pacemap obtained V6
from the junction of the right and V4
left coronary cusps. (b) The Abld
earliest overall activation time for V5
the clinical PVC obtained from Ablp
the junction of the right and left
coronary cusps with a sharp V6 RVa
pre-potential 45ms pre-QRS

Case 69

Matthew D. Hutchinson

Case Summary precordial transition at V4 suggesting an exit from the mid


inferior wall adjacent to the septum. Given that the VT cir-
cuits in ischemic patients are often large, multiple different
A 57-year-old man with history of isolated septal myocardial
VT morphologies may share components of the tachycardia
infarction had a CRT-D device implanted several years ago
circuit. The similar tachycardia cycle lengths also suggest a
for ventricular tachycardia and congestive heart failure. He
similar topography for the two VT morphologies. Localizing
presented now with repeated ICD shocks despite treatment
these VT exit sites on this patients voltage map indicates
with amiodarone. Interrogation of the ICD demonstrated two
that they likely originate from opposite sides of the septal
predominant VT morphologies with similar cycle lengths
scar.
(520550ms). The patient was brought to the electrophysi-
In this case, detailed mapping of the septal scar during
ology lab for mapping and ablation.
VT 2 revealed several sites with mid-diastolic potentials
Voltage mapping demonstrated a moderately large area of
(Fig.69.3) which were demonstrated by entrainment to be
basal septal scar, and programmed stimulation with single
isthmus sites. RF lesions at these sites terminated VT 2.
extrastimuli induced two VT morphologies (Figs.69.1 and
Upon repeat programmed stimulation, neither VT 2 nor VT 1
69.2) which matched with the ICD electrograms of the
could be re-induced. Thus it appeared that the two VTs
patients spontaneous tachycardias.
shared a common critical isthmus for reentry, with wave-
Based on the ECG morphologies, where are the likely
fronts travelling in opposing directions and exiting on oppos-
sites of origin/exit for each VT?
ing sides of the scar.
How might these VTs be associated with the scar, and
It is important to remember that scar-related VT can take
what would be your ablation strategy?
long and circuitous routes, especially if there are extensive
areas of scarring. As is seen in this case, the VT cycle length
can be extremely long on antiarrhythmic drug therapy, even
Case Discussion when the scar is not extensive. Thus in patients with exten-
sive scar, two or more VTs of potentially widely disparate
Despite the atypical scar location, this patient has an isch- morphologies are the rule. However, as in this case, some of
emic substrate due to known occlusion of a large septal per- the VTs may simply travel in opposing directions through a
forator. The first VT (Fig. 69.1) has a left bundle, inferior common isthmus, and ablation of this common isthmus
axis morphology with a precordial transition at V3 suggest- would abolish the pathway for multiple reentry circuits.
ing exit from the mid superior septum. The second VT Certainly this is good reason not to be discouraged when
(Fig.69.2) has a right bundle, left superior morphology with multiple VTs are induced at the start of the procedure.

M.D. Hutchinson
Cardiovascular Division, Department of Medicine,
University of Pennsylvania, 3400 Spruce Street, 9 Founders,
Philadelphia, PA 19104, USA
e-mail: matthew.hutchinson@uphs.upenn.edu

A. Natale et al. (eds.), Cardiac Electrophysiology, 303


DOI: 10.1007/978-1-84996-390-9_69, Springer-Verlag London Limited 2011
304 M.D. Hutchinson

Fig.69.1 12-lead surface ECG 200 ms


of the first ventricular tachycar- I
dia (VT) morphology induced
during ablation procedure. The
tachycardia cycle length as II
denoted is 550 ms
III

aVR

aVL

aVF
550 ms
V1

V2

V3

V4

V5

V6

200 ms
I

II

III

aVR

aVL

aVF

526 ms
V1

V2

V3

V4

V5
Fig.69.2 12-lead surface ECG
of the second VT morphology
induced. The tachycardia cycle V6
length as denoted is 526 ms
Case 69 305

Fig.69.3 Bipolar voltage map I 200 ms


of the left ventricle illustrating
VT1 II
the site (white star) of mid-
III
diastolic potentials during VT 2
(Abl recording from ablation RAO
catheter, d distal bipole, aVR
p proximal bipole). The large LV base aVL
white arrows represent the aVF
approximate exit sites for the two V1
VT morphologies induced V2
V3
V4
V5
V6
>1.5 mV

VT2 LV Apex
Abld

Ablp

<0.5 mV

Case 70

Matthew D. Hutchinson

Case Presentation pacing with concealed entrainment suggesting that the


paced beats are exiting the scar in the same fashion as the
VT. However, the local return cycle length is significantly
A 71-year-old man with history of distant inferior myocardial
longer than the tachycardia cycle length (Fig. 70.3). This
infarction with subsequent coronary artery bypass graft sur-
entrainment response suggests that the sharp potential on the
gery and St. Jude mechanical mitral valve prosthesis initially
ablation catheter does not directly participate in perpetuating
presented with symptomatic ventricular tachycardia which
the VT; it is being activated passively as the wavefront trav-
was treated with amiodarone and implantation of an ICD. Now,
els through the scar, thus producing mid-diastolic potentials
several years later, he presented with ICD shocks for recurrent
unrelated to the critical circuit. Ablation at this location
sustained monomorphic VT despite continued amiodarone
would be unlikely to disrupt the reentrant wavefront.
therapy. He was referred for mapping and ablation of VT.
Figure 70.2 is obtained at another site of mid-diastolic
His bipolar voltage mapping revealed an extensive scar
potentials. In this case, an early-coupled extrastimulus termi-
involving the basal inferior and lateral LV. The clinical VT
nates the tachycardia without activating the left ventricular
(right bundle, right inferior axis; CL 510 ms) was easily
myocardium. The response to this so-called non-propagated
induced with programmed stimulation and was well-tolerated
extrastimulus suggests that the pacing stimulus is delivered
hemodynamically allowing detailed entrainment mapping.
within the protected isthmus of the reentrant circuit, thereby
Several sites with mid-diastolic potentials during VT were
extinguishing the reentrant wavefront in both an orthodromic
found in the basal inferolateral wall; Figs. 70.1 and 70.2
(encountering tissue still refractory after the previous reen-
demonstrate responses to attempted entrainment from two of
trant beat) and an antidromic (colliding with the subsequent
these sites.
reentry wavefront as it travels into the isthmus) manner.
Describe the response to entrainment shown in Figs.70.1
Interrupting the reentrant circuit with this extrastimulus also
and 70.2.
implies that the critical isthmus traversing the pacing elec-
Should either site be considered a good target for ablation
trode is relatively narrow, thus susceptible to disruption. It is
of the clinical VT?
also noted that the subsequent pacing stimulus reproduces
the clinical VT morphology perfectly, and activates the myo-
cardium with a long stimulus-QRS interval. This suggests
Case Discussion that the pacing stimulus is delivered proximally within the
reentrant circuit and without antidromic capture. In this case,
VT was repeatedly reinduced with programmed stimulation
Figure 70.1 demonstrates the entrainment response at the
and terminated with nonpropagated extrastimuli. VT was
end of pacing at a site of mid-diastolic potentials. There is
subsequently terminated and rendered noninducible after a
relatively long stimulus-QRS (S-QRS) interval during
single RF lesion in this location.
In summary, locating early diastolic activity during VT
identifies regions of viable myocardium within the scar;
M.D. Hutchinson however these potentials may or may not participate in the
Cardiovascular Division, Department of Medicine,
University of Pennsylvania, 3400 Spruce Street, 9 Founders,
tachycardia circuit. The accurate interpretation of the entrain-
Philadelphia, PA 19104, USA ment response can easily differentiate the critical tachycardia
e-mail: matthew.hutchinson@uphs.upenn.edu isthmus from a passively-activated bystander loop.

A. Natale et al. (eds.), Cardiac Electrophysiology, 307


DOI: 10.1007/978-1-84996-390-9_70, Springer-Verlag London Limited 2011
308 M.D. Hutchinson

Fig.70.1 12-lead surface ECG 200 ms


and intracardiac recordings of the
I
response to entrainment at a site
of mid-diastolic potentials during II
ventricular tachycardia. See text
III
for discussion. Abl ablation
catheter electrogram, d distal aVR
bipole, p proximal bipole, aVL
RVa electrogram recording from
the right ventricular apex aVF
V1
V2
V3
V4
V5
V6
Abld
Ablp
RVa

400 ms

II

III

aVR
aVL

aVF

V1

V2

V3

V4
V5

V6

Abld
Fig.70.2 ECG and intracardiac
recordings from another site of Ablp
mid-diastolic potentials during RVa
ventricular tachycardia
Case 70 309

Fig.70.3 Same snapshot as 200 ms


Fig.70.1, with measured time
I
intervals as denoted (ms)
II

III
aVR
aVL

aVF
V1

V2
V3
V4
V5
V6
551 504
Abld
Ablp
RVa

Case 71

Matthew D. Hutchinson

Case Summary In the EP lab, bipolar voltage mapping revealed a large


area of low voltage involving the distal anterior wall and LV
A 69-year-old man with ischemic cardiomyopathy and apex. The patients clinical VT was easily inducible (CL
remote anterior infarction presented to the Emergency 410ms) with single extrastimuli. As the VT was well-tolerated
Department with palpitations. He was noted on ECG to have hemodynamically, detailed activation and entrainment
monomorphic ventricular tachycardia (left bundle, left supe- mapping was performed. Figures 71.2 and 71.3 depict the
rior axis; CL 420ms). He was treated with intravenous amio- response to entrainment at the sites indicated in the electro-
darone and lidocaine; however he had recurrent arrhythmias anatomical map.
which uniformly responded to anti-tachycardia pacing. An What information about the VT circuit is gained by
example of the 12-lead rhythm strip and intracardiac record- examination of the 12-lead and intracardiac recordings in
ings obtained during the pace termination of an episode of Fig.71.1?
VT is included in Fig.71.1. Coronary angiography revealed Describe the response to entrainment in Figs.71.2 and 71.3.
no significant disease progression since his last evaluation. Ablation at which of these sites would be expected to
He was referred for EP study and ablation. abolish the tachycardia? If neither, where would one expect

II

A III B

aVR
Marker
aVL
channel

aVF

V1
VEGM
V2
Fig.71.1 Panel A 12-lead ECG
V3
illustrating overdrive pacing
during VT from the ICD lead. V4
Panel B Intracardiac recording
simultaneous with ECG in panel A. V5
See text for discussion.
VEGM local bipolar ventricular V6
electrogram

M.D. Hutchinson
Cardiovascular Division, Department of Medicine,
University of Pennsylvania, 3400 Spruce Street, 9 Founders,
Philadelphia, PA 19104, USA
e-mail: matthew.hutchinson@uphs.upenn.edu

A. Natale et al. (eds.), Cardiac Electrophysiology, 311


DOI: 10.1007/978-1-84996-390-9_71, Springer-Verlag London Limited 2011
312 M.D. Hutchinson

Fig.71.2 Panel A Electroanato 200 ms


mical map with large area of low A B I
voltage involving LV anterior II
wall and apex. Panel B Overdrive III
AP view
pacing during VT performed at
aVR
the site indicated with star in
panel A aVL
aVF
V1
V2
V3
V4
V5
Septum
V6
>1.5 mV Abld
Apex Ablp

<0.5 mV

200 ms
A B I
II
III
AP view
aVR
aVL
aVF
V1
V2
V3
V4
V5
Septum
V6
>1.5 mV Abld
Apex Ablp

Fig.71.3 Panel A Electroanato


mical map as described in
Fig.71.2. Panel B Overdrivepac
ing during VT performed at the
site indicated with star in panel A <0.5 mV

to find the most appropriate ablation site based upon the the ICD lead (the local bipolar electrogram [B, bottom panel]
information presented? and the marker channel [A, top channel] are shown). Importantly,
examination of the morphology and post-pacing interval (PPI)
on the local bipolar electrogram reveals intracardiac fusion and
a postpacing interval (PPI) <30 ms of the tachycardia cycle
Case Discussion length (TCL). This suggests that the RV lead pacing site repre-
sents an outer loop of the tachycardia circuit. Examination of
This case illustrates a common presentation of stable, MMVT the 12-lead ECG obtained during ATP confirms these findings.
occurring after healed anterior myocardial infarction. These findings can be particularly helpful when the 12-lead
Figure71.4 demonstrates the response to overdrive pacing from morphology of the clinical VT is not available.
Case 71 313

Fig.71.4 Panel A 12-lead ECG


from Fig.71.1 with QRS fusion I
460 ms 440 ms
during pacing and a postpacing
interval within 30ms of the II
tachycardia cycle length (TCL). A B
Thus, pacing from the RV apex is III
an outer loop site for the clinical
VT. Panel B Intracardiac aVR
recording confirming the Marker
aVL channel
observations in panel A. VEGM
local bipolar ventricular
aVF
electrogram
V1
VEGM
V2
460 440
ms ms
V3

V4

V5

V6

200 ms
A B I
II
III
AP view
aVR
aVL
aVF
V1
V2
V3
V4
V5
Septum 300 300
V6
>1.5 mV Abld
Fig.71.5 Panel A Electroanato
Apex Ablp 423 414
mical map as described in
Fig.71.2. Panel B Overdrive
pacing during VT performed at
the site indicated with star in
panel A. This represents an
entrance site for the clinical VT.
See text for discussion <0.5 mV

Pacing within a protected part of the VT circuit produces a represents an entrance site for the VT which is oriented at the
paced morphology identical to the spontaneous VT morphol- lateral border of the low voltage region. Figure71.3 also shows
ogy. The degree of prematurity of the bipolar electrogram concealed entrainment with a PPI-TCL<30 ms. Here the
(EGM) recorded within the protected circuit relative to the stimulus-QRS interval (equal to the EGM-QRS) is 30ms (8%
onset of the surface QRS complex correlates to the degree of TCL; Fig.71.6); this represents an exit site which is located at
proximity within the circuit.13 Figure71.2 demonstrates con- the septal side of the low voltage region.
cealed entrainment with a PPI-TCL<30ms. The stimulus-QRS Focal ablation of VT can be performed by careful map-
(S-QRS) interval equals the electrogram-QRS (EGM-QRS) ping and targeting of the critical tachycardia isthmus. The
interval and is quite long (300ms, 73% TCL; Fig.71.5). This broad nature and potential redundancy of entrance and exit
314 M.D. Hutchinson

Fig.71.6 Panel A Electroanato 200 ms


mical map as described in I
Fig.71.2. Panel B Overdrive A B
II
pacing during VT performed at III
AP view
the site indicated with star in
aVR
panel A. This represents an exit
site for the clinical VT. See text aVL
for discussion aVF
V1
V2
V3
V4
V5
Septum
V6 30 30
>1.5 mV Abld
Apex Ablp 404 414

<0.5 mV

200 ms
A B I
II
AP view III
aVR
aVL
aVF
V1
V2
V3
V4
V5
Septum 173 173
V6
>1.5 mV Abld
Apex Ablp 405 414
Fig.71.7 Panel A Electroanatomical
map as described in Fig.71.2.
Panel B Overdrive pacing during VT
performed at the site indicated with
star in panel A. This represents an
isthmus site for the clinical VT. See
text for discussion <0.5 mV

sites for a given tachycardia limits the efficacy of focal References


ablation at these sites. Detailed mapping and targeting of
the critical isthmus is more effective in eliminating the 1. Josephson ME. Clinical Cardiac Electrophysiology: Techniques
tachycardia with focal ablation. In this case, mapping ear- and Interpretations, vol. 3. 3rd ed. Philadelphia, PA: Lippincott,
Williams, & Wilkins; 2002.
lier diastolic activity within the dense scar more proximal
2. Almendral JM, Gottlieb CD, Rosenthal ME, etal. Entrainment of
to the exit site identified an isthmus site (Stimulus- ventricular tachycardia: explanation for surface electrocardio-
QRS=EGM-QRS=173ms; 42% TCL; Fig.71.7). Ablation graphic phenomena by analysis of electrograms recorded within the
at this isthmus site terminated the tachycardia and rendered tachycardia circuit. Circulation. 1988;77(3):569-580.
3. Stevenson WG, Khan H, Sager P, et al. Identification of reentry
it noninducible. In summary, a combination of detailed
circuit sites during catheter mapping and radiofrequency ablation of
activation and entrainment mapping remains the most ventricular tachycardia late after myocardial infarction. Circulation.
efficient method of ablating tolerated VT. 1993;88(4 Pt 1):1647-1670.
Section
II
Devices

Case 72

Amin Al-Ahmad and Paul J. Wang

Case Summary Case Discussion

A 19-year-old woman with a history of cardiac transplanta- This ECG shows normal sinus rhythm with low voltage and
tion 5 years prior is admitted with acute rejection. She has a poor R wave progression, in addition to a rightward axis.
dual chamber PPM, Medtronic Kappa 401. Telemetry strip There are pacemaker artifacts after each QRS. This indicates
revealed pacemaker artifact after the QRS. A 12-lead ECG is clear undersensing on the ventricular channel. In this tracing,
shown in Fig.72.1. the pacemaker detects the intrinsic atrial activation and
Her pacemaker settings are as follows: because of ventricular undersensing delivers a pacemaker
stimulus after the AV delay interval ends. Could this pacing
Mode DDD artifact be an atrial stimulus? With ventricular undersensing
LRL/URL 50/150bpm the pacemaker may deliver an atrial stimulus, but would
Sensing, ventricular channel 2.8mV follow this with a ventricular stimulus. As there is only
AV delay one stimulus regularly seen in this ECG, the pacemaker is
adequately sensing the intrinsic atrial activation and tracking
Is the pacemaker function appropriate? Should any program- appropriately.
ming changes be made in this case?

A. Al-Ahmad and P.J. Wang(*)


School of Medicine, Stanford University,
300 Pasteur Drive, H-2146, Stanford, CA 94305, USA
e-mail: paul.j.wang@stanford.edu

A. Natale et al. (eds.), Cardiac Electrophysiology, 317


DOI: 10.1007/978-1-84996-390-9_72, Springer-Verlag London Limited 2011
318 A. Al-Ahmad and P.J. Wang

Fig.72.1 12-lead ECG showing pacing artifact


Case 73

Amin Al-Ahmad and Paul J. Wang

Case Summary a coupling interval of 398ms; this likely represents a ven-


tricular premature contraction (VPC) (Asterisk, Fig. 73.2).
Shortly after the VPC an atrial event can be seen on the atrial
A 32-year-old woman with a history of corrected transposi-
electrogram, but this event is not detected by the pacemaker
tion of the great vessels and a pacemaker placed for sinus
(the marker channel does not show any atrial events) as it is
node dysfunction is seen for a routine pacemaker check. She
likely in the ventricular blanking period (Arrow, Fig.73.2).
has a Medtronic Kappa Model KDR401/403. Atrial sensing
This atrial event may be a retrograde atrial beat, or may sim-
was 2.0mV, the threshold was 0.5V @ 0.25ms. Ventricular
ply be a sinus beat that fortuitously falls after the VPC. This
sensing was 4mV and the threshold was 1.5V @ 0.6ms.
atrial beat does not conduct since it is either retrograde or
Lead impedance on both leads was stable.
because it is just after the VPC. Once the atrial escape inter-
Device settings are as follows:
val is over, an atrial paced event is delivered; however, this is
Mode DDDR
unlikely to capture the atrium since it is delivered while the
atrium is most likely refractory. After the programmed AV
LRL/URL 60/125bpm
delay, a ventricular paced event results in a similar retrograde
Max sensor rate 140bpm
atrial event and the cycle continues. With an AR followed
AV delay 260ms shortly by an AP, the device records this as an atrial high rate
PVARP 240ms episode.
Several atrial high rate episodes were noted. Figure 73.1 It is important not to make assumptions that high rate epi-
shows a stored electrogram of an atrial high rate episode. sodes correspond to atrial tachyarrhythmias in all cases as
Does this electrogram reveal an atrial tachycardia? What is this may lead to inappropriate use of medications. Careful
the cause of the atrial high rate episodes? examination of stored electrograms when available may be
useful to better elucidate the cause of these atrial high rate
episodes.

Case Discussion

The stored strip shows atrial pacing followed by ventricular


sensing initially. A ventricular sensed event occurs with

A. Al-Ahmad and P.J. Wang(*)


School of Medicine, Stanford University,
300 Pasteur Drive, H-2146, Stanford, CA 94305
e-mail: paul.j.wang@stanford.edu

A. Natale et al. (eds.), Cardiac Electrophysiology, 319


DOI: 10.1007/978-1-84996-390-9_73, Springer-Verlag London Limited 2011
320 A. Al-Ahmad and P.J. Wang

Fig.73.1 Stored electrogram showing the atrial EGM and the marker intervals

Fig.73.2 Stored electrogram


showing initiation of atrial high
rate episode. Asterisk shows a
ventricular sensed event (VPC).
Arrow marks the atrial beat that
follows the VPC (probably
retrograde)
Case 74

Kenneth A. Ellenbogen

Case Summary undersensing due to pacing spikes evident shortly after sev-
eral of the patients intrinsic beats. In regard to ventricular
capture, the second beat is paced and the remaining pacer
A 72-year-old female with a past medical history of hyper-
spikes fall within the T wave of the preceding beat, when the
tension and an arrhythmia is admitted for an elective
ventricle is likely refractory (physiologic non-capture). At
cholecystectomy. While the patient is on telemetry, her pace-
this point one is likely to suspect abnormal pacer function
maker is incidentally noted to not be working properly and
with at least R wave undersensing and no evidence of prob-
you are called to evaluate her device. Review of the admis-
lems with capture.
sion chest film reveals a right-sided generator with a single
Interrogation of the device provides the programmed
lead positioned in the RV apex.
lower rate limit (equal to the ventricular escape interval in a
A 12-lead EKG with a rhythm strip is requested and is
VVI pacemaker) of 70bpm along with the ventricular refrac-
shown (Fig. 74.1). What is the underlying rhythm and is
tory period (VRP) of 500ms. Knowledge of these two pro-
there evidence of abnormal pacer function?
grammed parameters along with inspection of the surface
The device is interrogated and the programmed parame-
lead, RV EGM, and marker channel helps to shed light on the
ters are shown (Fig.74.2) with a programmed lower rate of
phenomenon seen on the initial EKG.
70 bpm. A real time intracardiac EGM of the RV and
Review of Fig.74.3 shows that the native beats are prop-
marker channel is obtained during one of the episodes
erly sensed, but they are labeled VR on the marker channel
(Fig. 74.3). There appears to be ventricular pacing shortly
as they fall within the 500ms VRP initiated by the prior beat.
after a sensed QRS labeled as VR on the marker channel at
Every paced or sensed beat reinitiates the ventricular escape
less than the lower rate limit (ventricular escape interval). Is
interval which is the programmed lower rate of 70bpm. The
this consistent with normal pacer function and what is the
exception is if the sensed beat falls within the 500ms VRP in
management for this problem?
which case the ventricular escape interval will not be reset.
Due to the variability of the R-R intervals during atrial fibril-
lation and the intervals between various VR beats, subse-
Case Discussion quent pacing spikes exhibit variable ventricular capture
resulting in pseudo-fused, fused, and completely paced beats.
Review of the 12-lead EKG shows atrial fibrillation with evi- Thus, the pacemaker is exhibiting normal function.
dence of pacer artifacts following several of the QRS com- Shortening the VRP will eliminate some of the undersens-
plexes, but with only the second beat on the strip resulting in ing of native beats and will minimize the inappropriate
ventricular capture. Based on the tracing there is evidence of pacing following these beats.

K.A. Ellenbogen
Cardiac Electrophysiology and Pacing, Department of Cardiology,
VCU School of Medicine, 980053, Richmond, VA 23298-0053, USA
e-mail: kellenbogen@pol.net

A. Natale et al. (eds.), Cardiac Electrophysiology, 321


DOI: 10.1007/978-1-84996-390-9_74, Springer-Verlag London Limited 2011
322 K.A. Ellenbogen

Fig.74.1 12 lead
ECG showing pacer
spikes without
capture

Arrows showing pacing spikes

Fig.74.2 Pacemaker
programmed parameters
Case 74 323

Fig.74.3 Surface ECG, EGM


and marker channel while patient
has the abnormality seen on the
12 lead ECG

Arrows pointing to ventricular pacing with varying degrees of capture



Case 75

Nora Goldschlager

Case Summary Case Discussion

A 63-year-old woman with documented bradycardia The figure displays some of the permanently programmed
tachycardia syndrome but no structural heart disease was parameters, the body surface ECG, atrial intracardiac elec-
implanted with a dual chamber pacing system. The low rate trogram, and marker channel information. All QRS com-
limit was programmed at 70ppm, the upper tracking rate to plexes are paced. Atrial pacing stimuli are being delivered at
100ppm. Ventricular sensitivity was programmed to 2mV a rate of about 100/min (with the exception of the third event
and atrial sensitivity to 1mV. Despite her history of parox- (P)); atrial capture is not occurring. The atrial intracardiac
ysmal atrial arrhythmias, automatic mode switch function electrogram reveals regular atrial activity at a rate of about
had not been programmed on. The procedure was accom- 300/min, suggesting atrial flutter. Note the variable ampli-
plished without incident, and antiarrhythmic drug therapy tude of the atrial electrical signals. Note also that the electro-
was begun to manage her tachyarrhythmias, which con- gram is recorded at a gain of 2.5 mV/cm; thus the atrial
sisted primarily of paroxysmal atrial tachycardia and atrial electrical signals are of very low amplitude, and below the
fibrillation. The patient did well over the ensuing several programmed sensitivity. The atrial tachyarrhythmia is there-
months, but then began to have recurrent palpitations asso- fore not sensed, resulting in delivery of atrial stimulus out-
ciated with breathlessness and chest discomfort. Because puts. More importantly, even if the mode switch function
these symptoms were new after pacemaker placement, she had been programmed on, mode switch would not have
was seen ahead of schedule in the cardiology clinic, where occurred due to the atrial undersensing. Note also that event
physical examination revealed no evidence of heart failure; counters will (erroneously) designate these events as AV
subsequently performed echocardiography revealed normal pacing.
left and right ventricular size and systolic function, normal The atrial fibrillation likely explains the patients clinical
pulmonary artery pressure, and only mild biatrial enlarge- symptoms, which could be due to the atrial arrhythmia itself
ment. A pharmacologic stress test showed no evidence of or to intermittent sensing of fibrillatory impulses, resulting in
ischemia. Because of irregular heart rates observed during rapid and/or irregular ventricular paced rates; the latter would
her workup she was referred to the pacemaker clinic, where, not occur if automatic mode switch is programmed on. The
in the course of the evaluation, Fig. 75.1 was obtained. patients symptoms resolved after reprogramming the atrial
What does the figure illustrate? How does it explain the sensitivity to 0.3 mV, and the mode switch function on
patients problem? What steps could be taken to solve the (DDIR), as well as optimizing her antiarrhythmic medication
problem? regimen.

N. Goldschlager
Department of Cardiology, University of California, San Francisco,
Cardiology Division, 5G1, San Francisco General Hospital,
1001 Potrero Avenue, San Francisco, CA 94110, USA
e-mail: ngoldschlater@medsfgh.ucsf.edu

A. Natale et al. (eds.), Cardiac Electrophysiology, 325


DOI: 10.1007/978-1-84996-390-9_75, Springer-Verlag London Limited 2011
326 N. Goldschlager

Fig.75.1 Program perimeters, surface ECG, marker channel and atrial lead recording of the patient
Case 76

Fred M. Kusumoto, Jennifer Crain, and Nora Goldschlager

Case Summary ventricle 84% of the time (Fig 76.2). The rhythm strip
(Fig76.3) shows paced QRS complexes (P) and retrograde
A 66-year-old man with a history of coronary artery disease P waves.
comes to your office complaining of generalized fatigue. He Patients with retrograde atrial activation can develop
denies chest pain or other symptoms of ischemia. He has a pacemaker syndrome. Originally described almost 40 years
history of a myocardial infarction 6 years ago but has normal ago, this complex process can lead to a constellation of
left ventricular function by recent echocardiogram. He under- vague symptoms including syncope or presyncope, weak-
went permanent pacemaker implantation in 2001 for a slow ness, lightheadedness, orthopnea, paroxysmal nocturnal
heart rate. His comorbidities include diabetes, obesity, and dyspnea, dizziness, and, in some cases even pulmonary
hypertension. He currently is being treated with extended- edema. The pathophysiology of pacemaker syndrome is
release metoprolol 50 mg daily and aspirin. On physical multifactorial: ventricular pacing can lead to worsening
examination his heart rate is 60 beats per minute. His lungs mitral regurgitation, elevated atrial natriuretic peptide, and
are clear and he has an II/VI systolic murmur. higher sympathetic activity. Pacemaker syndrome is most
Initial interrogation of his device is shown in Figs.76.176.3. commonly observed in patients with single-chamber ven-
Although the patients nonspecific symptom of fatigue could be tricular pacemakers implanted for sinus node dysfunction,
due to a variety of problems (diabetes, deconditioning, atypical where ventricular pacing leads to retrograde atrial activa-
presentation of ischemia), what specific rhythm and device- tion and/or AV dyssynchrony. The prevalence of pacemaker
related issues should be considered? syndrome varies depending on definition, but in the Mode
Selection Trial, pacemaker syndrome developed in 18%
of patients with pacemakers implanted for sinus node
dysfunction.
Case Discussion Ventricular pacing can also lead to heart failure and devel-
opment of atrial fibrillation. In an analysis of the MOST data,
On initial evaluation (Fig.76.1) it is seen that the patient has likelihood of heart failure hospitalization increased 2.53
a single-chamber ventricular pacemaker. Evaluation of the fold with ventricular pacing > 80% for the VVI pacing mode,
event counters shows that the patient is paced from the and risk of atrial fibrillation increased by approximately 20%
for each additional 25% increase in cumulative ventricular
pacing.
The device was reprogrammed to VVI with a lower rate
F.M. Kusumoto(*)
Department of Cardiovascular Diseases, Mayo Clinic, of 40 ppm and the beta blocker was temporarily discontin-
4500 San Pablo Road, Jacksonville, FL 32224, USA ued. A rhythm strip recorded 1 week later is shown in
e-mail: kusumoto.fred@mayo.edu Fig.76.4. His symptoms had improved significantly. Beta
J. Crain blockers are a critical component for the treatment of
Electrophysiology and Pacing Service, Division of Cardiovascular patients after myocardial infarction. However, in this case,
Diseases, Department of Medicine, Mayo Clinic, Jacksonville, reinstitution of beta blockade therapy could lead to a higher
FL 32224, USA
percentage of ventricular pacing. An atrial lead could be
N. Goldschlager added with a small but finite risk, and, with current battery
Department of Cardiology, University of California, San Francisco,
Cardiology Division, 5G1, San Francisco General Hospital,
technology, the patient might well require device replace-
1001 Potrero Avenue, San Francisco, CA 94110, USA ments over his lifetime. The Pacemaker and Beta-Blocker
e-mail: ngoldschlater@medsfgh.ucsf.edu Therapy after Myocardial Infarction (PACE-MI) Trial is

A. Natale et al. (eds.), Cardiac Electrophysiology, 327


DOI: 10.1007/978-1-84996-390-9_76, Springer-Verlag London Limited 2011
328 F.M. Kusumoto et al.

Fig.76.1 Interrogation of
baseline pacing parameters.
PPM, pulses per minute; V, volt;
mV, millivolt; ADL, activity of
daily living

Initial interrogation report currently underway and is randomizing patients after myo-
Ventricular long term histogram cardial infarction with contraindications to beta blockade
Sensed Paced due to bradycardia to standard therapy or combined therapy
Percentage of beats with pacing and beta-blockade with a composite pri
50 mary endpoint of total mortality and nonfatal myocardial
40 infarction.
30
20
10
0
< 40 60 80 100 120 140 160 180 >
Rate (BPM)
Ventricular high rate episodes: 0
Pacing (percentage of total):

Sensed 16.2%
Paced 83.8%

Fig.76.2 Initial interrogation of long-term rate counters. BPM, beats


per minute
Case 76 329

Fig.76.3 Baseline rhythm strip.


Lead II and surface ventricular
electrograms are shown. P,
ventricular paced event

Fig.76.4 Baseline rhythm strip


(lead II) and ventricular
electrograms on follow-up.
S,ventricular sensed event

Bibliography dysfunction treated with ventricular-based pacing in the Mode Selection


Trial (MOST). J Am Coll Cardiol. June 2004;43(11):2066-2071.
Sweeney MO, Hellkamp AS, Ellenbogen KA, et al. MOde Selection
Ellenbogen KA, Gilligan DM, Wood MA, etal. The Pacemaker syn- Trial Investigators. Adverse effect of ventricular pacing on heart
drome: a matter of definition. Am J Cardiol. 1997;79:1226-1229. failure and atrial fibrillation among patients with normal baseline
Link MS, Hellkamp AS, Estes NA 3rd, etal. MOST study investigators. QRS duration in a clinical trial of pacemaker therapy for sinus node
High incidence of pacemaker syndrome in patients with sinus node dysfunction. Circulation. 2003;107:2932-2937.

Case 77

Gregory M. Marcus and Nora Goldschlager

Case Summary and that neither urgent device interrogation nor urgent
reprogramming is necessary. How does the telemetry tracing
reassure the implanting electrophysiologist?
A 63-year-old woman with paroxysmal atrial fibrillation
and rapid ventricular response and intermittent presyncope
is found to have symptomatic sinus pauses>3 s on Holter
monitoring as well as an intermittent accelerated junctional
rhythm. In sinus rhythm, her PR interval is 180ms, and she
Case Discussion
has normal QRS complex morphology. She undergoes
placement of a St. Jude Identity ADx dual chamber pace- This case demonstrates the importance of first assessing the
maker without any apparent complications. Notably, the patients underlying rhythm when interpreting paced electro-
majority of the right atrial sites tested for capture and sens- cardiograms. The crucial finding (known from the history
ing thresholds exhibited low intracardiac voltage, and an and evident in Fig.77.1) is that the patient has an intermittent
acceptable atrial sensitivity could be achieved only in the accelerated junctional rhythm.
low right atrium. The first pacing output stimulus is associated with an
At implant, atrial and ventricular sensitivities are 2.7 and inverted P wave in lead II, consistent with atrial capture from
>12mV, respectively. The atrial capture threshold is 1.1V at a low atrial position. The QRS complex immediately follow-
0.5 ms, and the ventricular capture threshold is 0.9 V at ing that P wave occurs too early to represent a conducted com-
0.5 ms. The device is programmed to DDD mode, with a plex and therefore likely originates in the AV junction. The
base rate of 60 ppm, a paced AV delay of 275 ms, and a following pacing output stimulus occurs nearly simultane-
sensed AV delay (PV delay) of 250ms. ously with the onset of the next QRS complex (a second junc-
The evening after the device is placed, the on call hospi- tional complex). Importantly, no intervening P wave between
talist is contacted regarding the telemetry findings displayed the first atrial stimulus and this pacing output is observed, and
in Fig. 77.1. A chest radiograph (Fig. 77.2) confirms that the timing of these two stimulus outputs is exactly 1000ms
both leads are in appropriate positions, with the right atrial (60 ppm, the programmed lower rate limit). Therefore, this
lead placed in the low right atrium for reasons mentioned second pacing output represents a pseudopseudo-fusion
above. Concerned that the pacemaker is not functioning complex (an atrial output stimulus occurring around the time
correctly, the on call physician contacts the implanting elec- of inscription of the QRS complex). Although atrial capture
trophysiologist. After viewing the telemetry strips faxed to cannot be confirmed, since the P wave (if present) would be
his home, the electrophysiologist reassures the on call phy- buried in the QRS complex, there is no evidence that atrial
sician that the device appears to be functioning normally capture does not occur. Indeed, given documentation of atrial
capture of the previous (paced) P wave, the likelihood is that
this stimulus does result in atrial capture.
G.M. Marcus(*) Immediately following the first pseudopseudo-fusion
Division of Cardiology, Electrophysiology Section, complex, ventricular safety pacing is observed. Safety
University of California, 500 Parnassus Avenue, pacing occurs when an event is sensed in the ventricular
MUE434, San Francisco, CA 94143-1354, USA channel during the safety pace interval, an interval in the
e-mail: marcusg@medicine.ucsf.edu
ventricular sensing channel immediately following the
N. Goldschlager ventricular blanking period and preceding the ventricular
Department of Cardiology, University of California,
Cardiology Division, 5G1, General Hospital,
alert period.1 The purpose of safety pacing is to avoid ven-
1001 Potrero Avenue, San Francisco, CA 94110, USA tricular asystole in the event that an electrical event (such
e-mail: ngoldschlater@medsfgh.ucsf.edu as an atrial pacing output stimulus) is oversensed in

A. Natale et al. (eds.), Cardiac Electrophysiology, 331


DOI: 10.1007/978-1-84996-390-9_77, Springer-Verlag London Limited 2011
332 G.M. Marcus and N. Goldschlager

Fig.77.1 Telemetry strip demonstrating a junctional rhythm associated with intermittent pseudopseudo-fusion and safety pacing (see text for
discussion)

Fig.77.2 Posteroanterior and


lateral chest radiographs
demonstrating a dual chamber
pacing system, with one lead in
the low right atrium and a second
lead in the right ventricular apex

the ventricular channel, potentially inhibiting ventricular minute equals a cycle length of 1,000ms; therefore, the VA
pacing stimulus output. In this case, the sensed event that interval must be 1,000 ms275 ms=725 ms, the interval
triggers safety pacing may be the atrial stimulus (particu- observed. It is therefore not surprising that the fourth QRS
larly as the atrial lead is in the low right atrium) or the complex, which presumably has a P wave buried in it and
sensed portion of the QRS complex. Safety pacing can be occurs earlier than 1,000ms after the last atrial paced event,
recognized by the shortened AV interval (usually between is not associated with a pacing output (in fact, this provides
110 and 120 ms) between atrial and ventricular stimulus evidence that atrial sensing is intact).
outputs: In this case, the safety-pace interval is 120 ms, The following two complexes are spontaneous, the lat-
clearly shorter than the programmed AV interval of 275ms ter likely a conducted sinus beat, followed by four pseudo
described above. pseudo-fusion complexes. When the QRS complex occurs
The fourth pacing output stimulus results in atrial capture early enough to fall in the safety pace interval (as in the
and is associated with a spontaneous junctional complex. 7th, 8th, 9th, and 12th complexes), safety pacing occurs.
Importantly, the interval between the ventricular safety pac- When the QRS is sensed after the safety pace interval (and
ing stimulus and this stimulus is shorter than that between therefore in the ventricular alert period, as in the 10th and
the two previous atrial stimuli. This provides further proof 11th complexes), the ventricular output is appropria
that the safety paced ventricular stimulus is indeed ventricu- tely inhibited (demonstrating that ventricular sensing is
lar: The lower rate limit (base rate) is determined by the sum intact).
of the AV interval and the VA interval. We know that the AV Although the pacing system is functioning appropriately,
interval is 275ms and that the lower rate limit of 60 beats per it can be reprogrammed in the hopes of improving patient
Case 77 333

comfort. A simple option in this case would be to increase Reference


the lower rate limit, allowing for more consistent atrial cap-
ture. As the patient has no known AV conduction disease and 1. Wang PJ, MacFie J, Homoud MK, Link MS, Foote CB, Estes M.
as the programmed AV interval is quite long, this program Modes of pacemaker function. In: Kusomoto FM, Goldschlager NF,
change should result in conduction along her normal AV eds. Cardiac Pacing for the Clinician. Philadelphia, PA: Lippincott
Williams & Wilkins; 2001:63-90.
nodeHisPurkinje system and a narrow QRS for each paced
atrial event.

Case 78

Amin Al-Ahmad and Paul J. Wang

Case Summary thresholds make this extremely unlikely. Ventricular over-


sensing causing inhibition from electromagnetic interfer-
ence, diaphragm contraction, or intracardiac signals should
A 65-year-old woman has a permanent pacemaker implanted
be considered, in particular in pacing systems that utilize
on the previous day for episodes of AV block resulting in
unipolar leads. However, in this case inhibition is similarly
syncope. The device is a dual chamber Medtronic EnRhythm
unlikely.
DR. The atrial sensing at implant was 3 mV, the threshold
The pacing mode AAI/DDD offers the best explanation
was 0.8 V at a pulse width of 0.5ms, and the impedance was
of the tracing. This pacing mode provides AAI pacing as
556 W. The ventricular sensing at implant was 15 mV, the
long as AV conduction remains intact. If there is an AA
threshold was 0.4 V at a pulse width of 0.5 ms, and the
interval without a ventricular sense, the pacemaker delivers
impedance was 840 W.
a back-up ventricular pace 80ms after the AA escape inter-
Telemetry shows an episode of AV block; however, there
val. Also, if two out of four AA intervals lack a ventricular
is no ventricular pacing for two P-waves (Fig.78.1, arrow).
sense event consistent with AV block, the device changes to
The device is tested and the lead parameters are unchanged
a DDD mode. The device will then periodically check for
from implant. Device settings are as follows:
AV conduction, and if it is present switches back to an AAI
Mode AAI/DDD
mode.
In this case, programming the device to a DDD mode
LRL/URL 50/150bpm
with a long AV delay would eliminate this ECG finding and
AV delay paced/sensed 180/150ms
provide ventricular back-up for all episodes of AV block.
Programmed sensitivity (RV) 0.9 mV Since AAI/DDD does provide back-up ventricular pacing,
PVARP 310ms making no pacemaker programming changes would also be
What is the cause of the lack of atrial tracking in the reasonable if the patient does not have symptoms due to
tracing? lack of ventricular pacing that may occur on occasion of AV
block.

Case Discussion

In this tracing two P-waves are not followed by a paced ven-


tricular beat. While it is possible that failure to capture can
explain this, the lack of pacing artifact and the outstanding

A. Al-Ahmad and P.J. Wang(*)


School of Medicine, Stanford University, 300 Pasteur Drive, H-2146,
Stanford, CA 94305, USA
e-mail: paul.j.wang@stanford.edu

A. Natale et al. (eds.), Cardiac Electrophysiology, 335


DOI: 10.1007/978-1-84996-390-9_78, Springer-Verlag London Limited 2011
336 A. Al-Ahmad and P.J. Wang

Fig.78.1 This figure shows telemetry strip with intermittent AV block, non-conducted P waves with no pacing stimulus following them
Case 79

Kenneth A. Ellenbogen and Rod Bolanos

Case Summary is that the patient has regained AV nodal conduction after she
underwent revascularization in the setting of an acute coro-
nary syndrome. This explanation is excluded with inspection
A 55-year-old woman with a single chamber pacemaker
of (Fig.79.3a, b) the surface lead and ventricular EGM where
presents to clinic for her 1-month post-implant visit. She has
there is not only a V sensed event on the marker channel for
a history of coronary artery disease and presented to an out-
every QRS but also for every P wave seen on lead II. The
side facility with complete heart block 5 weeks ago in the
rhythm here is sinus rhythm at 75bpm with an accelerated
setting of an acute coronary syndrome. Her escape rhythm at
junctional rhythm at 65 bpm. The ventricular histogram
the time consisted of an accelerated junctional rhythm at
should have shown predominantly V sensed events in the
65bpm. She underwent a four vessel coronary artery bypass
range of 6080bpm based on her rhythm and programmed
grafting with implantation of a 4951 RV epicardial pacing
VVI setting at a lower rate of 50bpm. The oversensing of
lead and Medtronic SigmaTM single-chamber pacemaker
farfield P waves explains the ventricular sensed events up to
generator. She reports persistent exercise intolerance despite
130bpm.
an uneventful post-op course and compliance with cardiac
This patient was fortunate that she had an accelerated
rehabilitation.
junctional rhythm with p wave oversensing would otherwise
Interrogation of the device reveals the following pro-
have likely resulted in underpacing and subsequent
grammed parameters (Fig.79.1) along with the accompany-
syncope.
ing rate histogram (Fig. 79.2), which reveals almost no
Oversensing of P waves is uncommon with apical posi-
ventricular paced events and shows primarily ventricular
tioning of RV pacing leads. It can be seen though when leads
sensed events in a wide spectrum of rates up to 130 bpm.
are placed in a more basal location near the tricuspid valve,
What are some possible explanations for these findings with
coronary sinus, mitral annulus or proximal RVOT, or with
the patients past medical history?
integrated RV defibrillator leads not positioned far enough
Inspection of surface lead II along with the ventricular
out in the apex. In this case, the epicardial RV lead was posi-
EGM and marker channel provides insight into the rate his-
tioned over the basal anterior aspect of the right ventricle,
togram findings (Fig.79.3a). There are clearly more ventric-
which is in relatively close proximity to right atrial append-
ular sensed events than there are QRSs on the surface lead.
age predisposing to farfield sensing of P waves.
What else is being sensed on the ventricular channel? What
In this case the R waves measured 3.5mV and thus decre
are the potential management options in this particular
asing the ventricular sensitivity to avoid the P wave oversensing
patient and in general with this problem?
was not a feasible management option. A noninvasive manage-
ment option for dual chamber pacers with this problem is to
force atrial pacing to take advantage of the cross-chamber
Case Discussion blanking period in the ventricular channel after an atrial event
and the shorter ventricular intervals which will prevent ven-
tricular sensitivity from reaching its lowest values. Unfortu
While the predominance of V sensed events on the histogram
nately, with consistent P wave oversensing the RV lead usually
is surprising, it is not necessarily abnormal. One explanation
requires repositioning. Management in this case consisted of
insertion of new bipolar endocardial RV and atrial leads with
K.A. Ellenbogen () upgrade to a dual chamber pacemaker programmed in the
Cardiac Electrophysiology and Pacing, Department of Cardiology, DDD mode.
VCU School of Medicine, Richmond, VA 23298-0053, USA
e-mail: kellenbogen@pol.net

A. Natale et al. (eds.), Cardiac Electrophysiology, 337


DOI: 10.1007/978-1-84996-390-9_79, Springer-Verlag London Limited 2011
338 K.A. Ellenbogen and R. Bolanos

Fig.79.1 Programmed parameters


for pacemaker Modes Additional Features
Mode VVI Sleep Off
Single chamber hysteresis Off
Rates Transtelephonic monitor Off
Lower rate 50 ppm Extended telemetry Off
Activate diagnostics? Completed
Refractory/Blanking
Ventricular High Rate Episodes
Ventricular refractory 330 ms
V. Min. detection rate 220 ppm
Ventricular Lead V. events to detect Three beats
V. events to terminate Two beats
Amplitude 3.50 V High rate collection method Rolling
Pulse width 0.45 ms
Sensitivity 2.00 mV Selectable Diagnostic
Pace polarity Unipolar
Sense polarity Unipolar Custom rate trend
Lead monitor Monitor only Rate trend collect Rolling
Minimum impedance 200 Include refractory senses? Exclude
Maximum impedance 3,000 Rate trend duration 24 h

Clinical status: 03/06/02 to 07/19/02


Ventricular histogram
Sensed Paced
% of Beats
50
40
30
20
10
0
< 40 60 80 100 120 140 160 180>
Rate (bpm)
Ventricular high rate episodes: 0
Pacing (percentage of total):
Sensed 99.7%
Paced 0.3%

Fig.79.2 Stored pacemaker ventricular histogram


Case 79 339

Fig.79.3 (a, b) Surface ECG Surface lead showing hr of 65 bpm


along with marker channel and
ventricular EGM a

LEAD III 0.1 mV/mm

Marker channel

EGM 1 mV/mm

Oversensing of the atrium on ventricular lead


V Rate 65 bpm

LEAD III 0.1 mV/mm

Marker channel

EGM 1 mV/mm

Oversensing the atrium



Case 80

Nora Goldschlager

Case Summary Explain the events illustrated in Fig.80.2. Do these events


account for the interrogated event counter information? What
steps should be taken to eliminate the problem?
A 73-year-old man underwent dual chamber pacing system
implantation for intermittent advanced and complete AV
block. His ejection fraction was normal, and he was not
known to have any ventricular ectopy. Over the ensuing Case Discussion
6 months, pacemaker interrogation revealed normal dual
chamber operation and no ventricular ectopy (designated Figure80.2 displays some programmed parameters, the body
PVE in this pacing system). One month after his last pace- surface ECG, marker channel information, and the atrial
maker clinic visit the patient began to note increasing effort intracardiac electrogram. (This manufacturer designates P
fatigue and breathlessness. A thorough history and physical as a native atrial event, R as a native ventricular event, A
examination revealed no evidence of heart failure and was as an atrial pace event, and V as a ventricular pace event.)
normal. A 12-lead ECG demonstrated sinus rhythm with first The initial portion of the ECG shows sinus rhythm; regu-
degree AV block and no evidence of atrial or ventricular pac- lar atrial activity is confirmed by the atrial electrogram. First
ing outputs. Interrogation of the pacemaker indicated over degree AV block is present; the PR interval is about 400ms
700,000 PVEs occurring during the prior month, a totally (note the programmed PV interval is 160ms); the P waves
unexpected finding for this patient (Fig.80.1). are not followed by paced ventricular complexes, indicating
Figure80.2 was then recorded. that they are not sensed.

Event counts

Rate (ppm) PV PR AV AR PVE


30 - 54 20 0 636 0 0
55 - 69 64,875 14 529,998 12,999 9
70 - 89 319,697 2,786 2,730,957 54,422 246,901
90 - 109 132,003 7,958 1,034,390 74,729 376,031
110 - 129 19,259 7,043 148,900 9,597 112,734
130 - 149 4,238 3,410 2,645 425 7,532
150 - 179 1 868 306 4 1,962
180 - 224 0 23 0 0 955
225 - 249 0 0 0 0 1
> 250 0 0 0 0 0
Total: 540,093 22,102 4,447,832 152,176 746,125
Fig.80.1 Event counts upon
pacemaker interrogation Total event count: 5,908,328

N. Goldschlager
Department of Cardiology, University of California, San Francisco,
Cardiology Division, 5G1, San Francisco General Hospital,
1001 Potrero Avenue, San Francisco, CA, USA 94110
e-mail: ngoldschlater@medsfgh.ucsf.edu

A. Natale et al. (eds.), Cardiac Electrophysiology, 341


DOI: 10.1007/978-1-84996-390-9_80, Springer-Verlag London Limited 2011
342 N. Goldschlager

Fig.80.2 Real time electrogram with marker channel

As is suggested by the horizontal lines in this marker out, and leads to an atrial paced event, followed by a ven-
channel, indicating (for this manufacturer) refractory peri- tricular paced event. Normal P wave tracking follows. Note
ods, the native P waves are falling in PVARP (the upper hori- that the upper horizontal line representing PVARP is shorter
zontal line) and therefore will not be tracked. The device is in the PV cycles than in the RR cycles; this is due to PVARP
therefore sensing consecutive R waves. Atrial outputs do not extension after a PVE diagnosis is made, and is a program-
follow the sensed R waves since the RR cycle length is mable feature in this device. Thus, PVE sensing with auto-
shorter than the programmed base rate; thus, the RA interval matic PVARP extension fosters continuation of the PVE
timeout is continually aborted. Consecutive R waves without diagnosis and disallows appropriate tracking. The mode of
an intervening atrial sensed event define a PVE, account- onset of this rhythm is not known from the rhythm strip
ing for the erroneously overcounted PVEs, and explaining shown. To eliminate the problem, however, the PVARP
the event counter information. extension on PVE sensing could be programmed OFF; the
A premature ventricular complex in the middle of the P waves would then fall in the alert period of the AV interval
strip is followed by a pause, allowing the RA interval to time and would be appropriately tracked.
Case 81

Fred M. Kusumoto, Jennifer Crain, and Nora Goldschlager

Case Summary lead/connector problems. In this case a clean electrogram


with no baseline artifact (initial signals on the left side of the
strip) is suddenly replaced by both high and low amplitude
An 82-year-old man with permanent atrial fibrillation has a
high frequency signals. This type of signal is commonly
permanent pacemaker implanted after AV nodal ablation.
referred to as noise and usually is due to electromagnetic
During a routine pacing system evaluation, interrogation of
interference or lead/connector problems. Baseline evaluation
his device demonstrates a cluster of ventricular arrhythmias
of the device demonstrated no evidence of lead/connector
on one day. A stored electrogram from one of the events is
problems (stable impedances, thresholds, normal electro-
shown in Fig. 81.1. Evaluation of the device demonstrates
grams). A lead problem that is intermittent can sometimes be
stable thresholds and lead impedances. What should be done
identified by having the patient perform specific maneuvers
next?
such as reaching for the ceiling or placing the arm as far as
possible behind his or her back. In this case, on further ques-
tioning, the patient recalled that on the day on which this
Case Discussion figure was recorded he had undergone hernia surgery.
Radiofrequency current from electrocautery use during
surgery can interact with the sensing circuitry of a pacing
Evaluation of the electrograms from the episode reveals high
system. The likelihood and consequences of this interaction
frequency signals. The differential diagnosis includes appro-
vary from patient to patient. In general, electrocautery can be
priate sensing of a rapid ventricular rate due to ventricular or
safely used if several simple recommendations are followed
supraventricular arrhythmias or to sensing of other, extracar-
(Table81.1). In this case the oversensing (Vs) led to transient
diac electrical signals (oversensing). Oversensing can be due
inhibition of pacing stimulus output, which can lead to
to intracardiac electrical activity (ventricular repolarization
asystole in patients who are pacemaker dependent.
(T waves) or atrial depolarization (P waves)), extracardiac
signals (myopotentials or electromagnetic interference), or

F.M. Kusumoto(*)
Department of Cardiovascular Diseases, Mayo Clinic,
4500 San Pablo Road, Jacksonville, FL 32224, USA
e-mail: kusumoto.fred@mayo.edu
J. Crain
Electrophysiology and Pacing Service, Division of Cardiovascular
Diseases, Department of Medicine, Mayo Clinic, Jacksonville,
FL 32224, USA
N. Goldschlager
Department of Cardiology, University of California, San Francisco,
Cardiology Division, 5G1, San Francisco General Hospital,
1001 Potrero Avenue, San Francisco, CA 94110, USA
e-mail: ngoldschlater@medsfgh.ucsf.edu

A. Natale et al. (eds.), Cardiac Electrophysiology, 343


DOI: 10.1007/978-1-84996-390-9_81, Springer-Verlag London Limited 2011
344 F.M. Kusumoto et al.

Reference

1. Cohan L, Kusumoto FM, Goldschlager NF. Environmental effects


on cardiac pacing systems. In: Kusumoto FM, Goldschlager N, eds.
Cardiac Pacing for the Clinician. 2nd ed. New York, NY: Springer;
2008.

Fig.81.1 Ventricular electrograms from the pacemaker during the high ventricular rate episode (V-Tachy). VS, ventricular sensed event; VP,
ventricular paced event

Table81.1 Recommendations for the use of electrocautery during surgery


Rate response and any special algorithms should be programmed off prior to the surgical procedure. It is also important to ascertain ahead
of time specific characteristics of that manufacturers magnet response since magnet application may be required during the surgical
procedure.
The pacemaker can be programmed to an asynchronous pacing mode (VOO, AOO, or DOO) just prior to surgery. Programming the pace-
maker to an asynchronous mode may be preferable to placing a magnet over the device particularly if there is little or no intrinsic ventricular
activity since the manufacturers magnet rates may be higher than desired (e.g., 100/min St. Jude devices). However, problems can arise
during asynchronous pacing if the pacemaker competes with the patients intrinsic rhythm. This can result in the induction of tachyarrythmias.
Use bipolar cautery if possible. This type of system has a short current path, which greatly reduces the area of significant electrical signal
generation to roughly a 6-in. circle centered on the site of electrocautery application. In addition, an ultrasonic scalpel may reduce EMI.
If a unipolar cautery system must be used, the indifferent electrode (grounding pad) should be placed such that the current flow between it and
the cautery tip will not intersect the pacing system. For example, the thigh ipsilateral to the surgical site can be used in abdominal procedures.
Good contact between the indifferent electrode and the skin must be maintained to reduce the chance of loss of contact, resulting in the pacing
lead becoming a current sink for the electrocautery.
Do not use electrocautery within 6 in. of the pulse generator.
Use the minimum power settings required for adequate electrocautery.
Use short bursts (preferably less than 1 s in duration) spaced more than 5 s apart. If electrocautery is causing inhibition of the pacemaker, a
longer time between bursts will minimize hemodynamic effects.
Monitor the patient for signs of pacemaker inhibition or triggering. If the ECG tracing is not clear because of interference from the use of
cautery, the patient should be monitored manually or by some other means, such as ear or finger plethosgraphy or arterial pressure display.
Provisions for alternative pacing and defibrillation should be readily available in the operating suite.
Verify function of the pacemaker after the procedure with a complete pacing system interrogation and threshold determinations.
With permission from Cohan etal.1
Case 82

Amin Al-Ahmad and Paul J. Wang

Case Summary pacing at the upper tracking rate, however, with automatic
mode switch the device would be expected to change to a
non-tracking mode (unless the atrial fibrillation electrograms
An 85-year-old man with a history of coronary artery dis-
are undersensed). With ventricular pacing at the upper track-
ease, aortic valve replacement, and a PPM placed for inter-
ing rate one must consider pacemaker-mediated tachycardia
mittent AV block comes to the emergency department with
(endless-loop tachycardia). This can occur is a ventricular
shortness of breath. The pacemaker is a dual chamber
premature beat, results in retrograde atrial activation that is
St. Jude Medical Victory DR 5810. Telemetry reveals ven-
then tracked resulting in ventricular pacing, again with retro-
tricular pacing with a heart rate of 130 bpm (Fig. 82.1).
grade atrial activation, and the cycle continues. It can also
Pacemaker parameters are as follows:
occur if an atrial stimulus fails to capture the atrium, and the
Mode DDD
ventricular stimulus that follows in the DDD mode results in
retrograde atrial activation, resulting in initiation of the
LRL/URL 60/130 bpm
tachycardia.
AV delay sensed/paced 150/200 ms
The post-ventricular atrial refractory period (PVARP)
PVARP 250 ms decreases the likelihood of tracking of retrograde atrial beats
Automatic mode switch On, detection rate 180 bpm as they often occur during this interval. In patients with a
What are the possible causes of the rapid ventricular pacing? short PVARP who have retrograde conduction, PMT is pos-
Should programming changes be made? sible when the retrograde conduction time is longer than the
PAVRP. Current pacemakers have algorithms to prevent or
terminate PMT. The PVC extension algorithm is one that
results in PVARP extension after a PVC is detected by the
Case Discussion pacemaker. In addition, if the pacemaker detects atrial sens-
ing and ventricular pacing at a rapid rate, a PMT termination
Ventricular pacing at the upper rate limit could be due to algorithm may prolong the PVARP interval for a single beat
tracking of either sinus tachycardia or an atrial arrhythmia at to terminate PMT.
that rate. If the atrial rate exceeds the maximum tracking In this case, PMT was diagnosed and the PVARP was
rate, ventricular pacing will display a Wenkeback phenom- extended to 350 ms. This results in termination of PMT
enon and there will be regular changes in rate when the (Fig. 82.2). In addition, in this patient the upper tracking
intrinsic atrial activation occurs during the pacemaker atrial ratewas decreased to 110 bpm given the patients history of
refractory period (PVARP). Atrial fibrillation may also cause coronary disease.

A. Al-Ahmad(*)
School of Medicine, Stanford University, 300 Pasteur
Drive, H-2146, Stanford, CA 94305, USA
e-mail: aalahmad@stanford.edu
P.J. Wang
School of Medicine, Stanford University, 300 Pasteur
Drive, H-2146, Stanford, CA 94305, USA
e-mail: paul.j.wang@stanford.edu

A. Natale et al. (eds.), Cardiac Electrophysiology, 345


DOI: 10.1007/978-1-84996-390-9_82, Springer-Verlag London Limited 2011
346 A. Al-Ahmad and P.J. Wang

Fig.82.1 Telemetry reveals


ventricular pacing with a rate
of 130 bpm

Fig.82.2 This figure shows the termination of rapid pacing due to PMT once the PVARP is increased
Case 83

Kenneth A. Ellenbogen

Case Summary Her pacemaker is interrogated revealing the following


programmed parameters (Fig. 83.1). The surface lead II,
A 71-year-old female has a dual chamber pacemaker atrial and ventricular marker channels, and atrial EGM are
implanted 3 years ago for symptomatic sinus bradycardia. shown (Fig. 83.2) during the patients presenting tachycar-
She presents to the Emergency Room complaining of short- dia. What appears to be the underlying atrial rhythm based
ness of breath and palpitations. She is noted to be tachy- on the atrial EGM? Based on the marker channel, it appears
cardic with a regular, nonpaced rhythm of approximately that the device is sensing only every other atrial beat. Is this
140 bpm. The patient has no known prior history of atrial normal pacemaker function and what are the potential
arrhythmias. adverse effects from this phenomenon?

Fig.83.1 Surface ECG, marker channel and atrial EGM

K.A. Ellenbogen
Cardiac Electrophysiology and Pacing, Department of Cardiology,
VCU School of Medicine, P.O. Box980053, Richmond, VA
23298-0053, USA
e-mail: kellenbogen@pol.net

A. Natale et al. (eds.), Cardiac Electrophysiology, 347


DOI: 10.1007/978-1-84996-390-9_83, Springer-Verlag London Limited 2011
348 K.A. Ellenbogen

Every other p wave falls in PVAB period

Fig.83.2 Surface ECG, marker channel and atrial EGM

Case Discussion could result in tracking of the atrial flutter at the pacemakers
programmed upper tracking rate if the sensed AV interval
were to be exceeded or ventricular conduction absent.
Inspection of the atrial EGM shows a regular atrial rate of
Additionally, the device underreports the time, duration, and
230ms. This finding combined with the appearance of lead
frequency of atrial arrhythmias.
II is most consistent with atrial flutter as the underlying atrial
Management in this case would first involve improved
rhythm. There is 2:1 conduction resulting in a ventricular
ventricular rate control, followed by adjustment of the PVAB
rate of approximately 140bpm. The patients device is exhib-
if necessary to mitigate the observed functional oversensing.
iting normal pacemaker function it is inhibited, even though
Manufacturer-specific algorithms to prevent underdetection
it is undersensing atrial flutter, and only senses every other
of atrial flutter are also available, but still may lead to under-
atrial electrogram. The patients underlying ventricular rate
detection of atrial flutter.
of 140 bpm inhibits ventricular pacing. Every other atrial
electrogram is falling within the post-ventricular atrial blank-
ing period (PVAB) which is programmed in this device to
130ms, and is the initial component of the post-ventricular
atrial refractory period (PVARP). The PVAB is the portion of Bibliography
the PVARP where atrial sensed signals are ignored. As a
result of this atrial functional undersensing, the device may Israel CW, Barold SS. Failure of atrial flutter detection by a pacemaker
fail to mode switch appropriately in this case from DDDR to with a dedicated atrial flutter detection algorithm. PACE. 2002;
the non-atrial tracking mode of DDI. Failure to mode switch 25:1274-1277.
Case 84

Nora Goldschlager

Case Summary designated by this manufacturer) coincides with a spontane-


ous QRS complex. The V pace event producing a P wave
and the P sense event reflecting QRS complex sensing
A 68-year-old woman underwent dual chamber pacing
indicate lead reversal at the pulse generator.
system implantation for sinus node dysfunction associated
Since atrial pacing output is occurring through the
with 58 s pauses in atrial rhythm and severe presyncope.
ventricular channel, the atrial paced events are designated
AV conduction was normal, and intraventricular conduction
V, accounting for the 100% V pace events on interroga-
delays were not present. Pacing system implantation was
tion. And since QRS sensing is occurring through the atrial
accomplished without difficulty.
channel, P is designated, accounting for the 100% P sense
One week later, during a routine wound check visit, a
events on interrogation.
12-lead ECG was recorded with and without magnet, and the
Had a free-running and DOO mode rhythm strip and/or
pacemaker interrogated. Despite no known AV conduction
ECG been recorded at the time of implant and interpreted
system disease, the event counters indicated 100% ventricu-
correctly, the diagnosis of lead reversal would have been
lar pacing and 100% atrial sensing, which would not be
made and the problem corrected immediately. An ECG
expected in a patient with sinus node dysfunction and intact
recorded in DOO mode would have shown that the first of
AV conduction. Fig. 84.1 shows the body surface ECG, the
the two stimulus outputs (the atrial stimulus) was producing
atrial and ventricular intracardiac electrograms, and the
a QRS complex and the second stimulus output (the ven-
marker channel information.
tricular stimulus) was producing a P wave (if temporal
What is the problem with this pacing system and does it
opportunity existed); the first of these events would have
explain the event counter information? What would the ECG
been diagnostic of the problem.
recorded in DOO mode show? How could the problem have
been avoided?

Case Discussion

Scrutiny of the Fig.84.1 shows that the V (V pace event


as designated by this manufacturer) produces a P wave, and
not a QRS complex, and that the P (atrial sense event as

N. Goldschlager
Department of Cardiology, University of California, San Francisco,
Cardiology Division, 5G1, San Francisco General Hospital,
1001 Potrero Avenue, San Francisco, CA 94110, USA
e-mail: ngoldschlater@medsfgh.ucsf.edu

A. Natale et al. (eds.), Cardiac Electrophysiology, 349


DOI: 10.1007/978-1-84996-390-9_84, Springer-Verlag London Limited 2011
350 N. Goldschlager

Fig.84.1 Program settings, surface ECG, marker channel and both the ventricular and atrial EGM
Case 85

Anurag Gupta and Amin Al-Ahmad

Case Summary but are not limited to: (a) sensor driven rate changes, dis-
cussed below; (b) programmed interventions such as rate
smoothing, rate hysteresis, rate drop response, overdrive
A 29-year-old female with history of transposition of the
algorithms, and atrial fibrillation suppression algorithms;
great arteries status post Mustard procedure and sick sinus
(c) reversion to alternate mode and/or rate, for example, as
syndrome status post dual chamber permanent pacemaker
with magnet application, electromagnetic interference, and
presents to the emergency room with 3-day history of cough
battery depletion; and (d) device malfunction including
and nasal congestion thought consistent with upper respira-
component and/or circuit failures with runaway.
tory tract infection. However, the cardiac electrophysiology
In this patient, interrogation of her device effectively rules
service is contacted by the emergency room providers because
out all categories except inappropriate sensor-driven rate.
an ECG obtained with the patient lying supine in bed demon-
Though multiple sensing methods are employed, these notably
strates atrial pacing at 105 ppm (Fig.85.1).
include sensors of body motion and minute ventilation sensors,
Interrogation of her Medtronic Kappa 400 ppm confirms
both of which are active in this patient as represented by the
current atrial pacing at 97 ppm (Fig.85.2). Inhibition of pacing
integrated designation of her sensor. More specifically, sen-
for assessment of underlying rhythm reveals predominantly
sors of body motion generally utilize a piezoelectric crystal that
sinus rhythm at 64 bpm. Device settings are as follows:
reacts to mechanical stress, body vibrations, and/or accelera-
Mode DDDR
tion. None of these stimuli are likely present in this stationary
patient lying supine in her bed without any objects on her
Mode switch Off
device. On the other hand, minute ventilation sensors measure
Lower rate 60 ppm
thoracic impedance by repetitively emitting high frequency
Upper tracking rate 125 ppm current (e.g., 16 times per second for Medtronic Kappa 400
Upper sensor rate 140 ppm ppm) and then measuring voltage between the PM can and lead
Sensor Integrated electrodes; the changes in impedance amplitude and cycle
Sinus preference On length approximate changes in tidal volume and respiratory
rate, respectively, and thus allow estimation of minute ventila-
No other additional features or algorithms are programmed
tion. In this patient, tachypnea and coughing from her respira-
on. What are the possibilities accounting for her atrial pacing
tory infection led to increase in her minute ventilation sensor.
at an elevated rate that exceeds her lower rate limit?
Integrated dual sensors may mimic sinus node function
response more closely and mitigate inappropriate sensing
with cross-checking algorithms that detect discrepancies
Case Discussion between the two sensors. In her case, with further change of
her sensor from integrated to activity mode only (thus
The observation made by the emergency room providers is eliminating the minute ventilation sensor), she promptly
that she has an elevated atrial pacing rate. Notable reasons reverted to an atrial sensed sinus rhythm at 64 bpm. This case
for atrial pacing at rate exceeding the lower rate limit include illustrates the importance of recognizing reasons for changes
in atrial rate, and demonstrates the role of sensors in pacing.

A. Gupta (*) and A. Al-Ahmad


Cardiac Electrophysiology Service, Division of Cardiology,
Department of Medicine, Stanford University Hospital and Clinics,
300 Pasteur Drive, Room H2146, Stanford, CA 94305-5233, USA
e-mail: agupta@stanfordalumni.org

A. Natale et al. (eds.), Cardiac Electrophysiology, 351


DOI: 10.1007/978-1-84996-390-9_85, Springer-Verlag London Limited 2011
352 A. Gupta and A. Al-Ahmed

Fig.85.1 12-lead ECG demonstrating atrial pacing at rate exceeding lower rate limit

Fig.85.2 Intracardiac electrogram verifying atrial pacing at rate exceeding lower rate limit
Case 86

Fred M. Kusumoto, Jennifer Crain, and Nora Goldschlager

Case Summary Experimental studies have suggested that antitachycardia


pacing can be useful for terminating episodes of atrial fibrilla-
tion. In the Atrial Therapy Efficacy and Safety Trial (ATTEST),
A 64-year-old man with a history of atrial fibrillation has been
antitachycardia pacing was effective as defined by the device
receiving sotalol. A dual chamber pacemaker with antitachy-
in terminating 54% of episodes of atrial fibrillation/atrial
cardia pacing capabilities was implanted because of symp-
tachycardia. However, the overall atrial fibrillation burden
tomatic bradycardia associated with sotalol therapy. Since
was not decreased by the use of specialized algorithms for
being placed on sotalol several months ago he has had no epi-
preventing and terminating atrial fibrillation. Thus, success
sodes of symptomatic atrial fibrillation and wonders whether
defined by the device may or may not represent true efficacy.
he can stop anticoagulation. He has hypertension, but other-
The summary and electrograms from a successfully treated
wise does not have a history of coronary artery disease, heart
episode in this patient is shown in Figs.86.2 and 86.3. Notice
failure, prior stroke or transient ischemic attack, or diabetes.
that atrial fibrillation terminates 1 min after the pacing sequ
Initial interrogation of his device is shown in Fig.86.1.
ence. Despite being classified as successful, it is not clear
Does the device provide any additional clinical information
whether the antitachycardia pacing truly had any effect on the
that would help in patient management?
arrhythmia.
The patient later began developing side effects from the
sotalol therapy and he subsequently underwent radiofre-
quency catheter ablation with almost complete elimination
Case Discussion
of his atrial fibrillation (Fig.86.4). Recently published guide-
lines suggest that anticoagulation should be continued for at
Interrogation of the device demonstrates a significant amount least 2 months after radiofrequency catheter ablation. After
of asymptomatic atrial fibrillation. Episodes of asymptom- this initial period, decisions regarding continued anticoagu-
atic atrial fibrillation are up to 12 times more common than lation should take individual risk factors into account. In this
episodes of symptomatic atrial fibrillation; and prolonged case, because his only risk factor for stroke was hyperten-
episodes (>48 h) are observed in up to 45% of patients, of sion, after discussion with the patient the treating physician
which 38% are completely asymptomatic. Implantable chose to discontinue anticoagulation after 2 months. The
cardiac rhythm devices provide a unique opportunity for patient continues to be monitored for recurrence of atrial
evaluating arrhythmia burden and efficacy of therapy. fibrillation through his implanted device.

F.M. Kusumoto(*)
Department of Cardiovascular Diseases, Mayo Clinic,
4500 San Pablo Road, Jacksonville, FL 32224, USA
e-mail: kusumoto.fred@mayo.edu
J. Crain
Electrophysiology and Pacing Service, Division of Cardiovascular
Diseases, Department of Medicine, Mayo Clinic, Jacksonville,
FL 32224, USA
N. Goldschlager
Department of Cardiology, University of California, San Francisco,
Cardiology Division, 5G1, San Francisco General Hospital,
1001 Potrero Avenue, San Francisco, CA 94110, USA
e-mail: ngoldschlater@medsfgh.ucsf.edu

A. Natale et al. (eds.), Cardiac Electrophysiology, 353


DOI: 10.1007/978-1-84996-390-9_86, Springer-Verlag London Limited 2011
354 F.M. Kusumoto et al.

Initial interrogation: Cardiac compass trends Page 1


P = Program
I = Interrogate
_ = Remote P I P P I I I P PI I

AT/AF total 24
hours/day 20
16
12
8
4
0

V. rate duting AT/AF >200


(bpm)
max/day 150
avg/day

108

<50
Jun 2005 Aug 2005 Oct 2005 Dec 2005 Feb 2006 Apr 2006 Jun 2006

Fig. 86.1 Interrogation of the pacemaker showing amount of atrial arrhythmias: AT/AF atrial tachycardia/atrial fibrillation, bpm beats per
minute

Date of Visit: 14-Jul-2006 13:05:19


9987 Software Version 1.2
Device: EnRhythm P1501DR Copyright Medtronic, Inc. 2002
Treated AT/AF Episode #531
ATP Time Duration Avg bpm
Type Success ID# Date
Seq hh:mm hh:mm:ss A/V
AT/AF 2 Yes 531 10-Jun-2006 16:31 :02:25 221/97
V-V A-A Fast A.= 200 ms AT/AF = 350 ms

Interval (ms) Onset Detection First ATP Term.


1,500 19 s 1.1 min 1.1 min

1,200

900

600

400

200

15 10 5 0 0 5 0 10 5 0
Time (s)

Fig.86.2 Summary of a successfully treated episode of atrial fibril- (ATP)). Notice that the patient had an episode of atrial fibrillation that
lation. Notice that the atrial rate remains elevated (average cycle length terminated spontaneously prior to the treated event
200ms or approximately 300bpm even after the antitachycardia pacing
Case 86 355

Fig.86.3 Electrograms from the episode. Atrial pacing (TP) does not at the end of the strip, the patient has ventricular pacing. TD tachycardia
terminate the atrial fibrillation. Notice the relatively controlled ventric- detect, FD fibrillation detect, FS fibrillation sense
ular rate during the episode of atrial fibrillation. During sinus rhythm,

Fig.86.4 The patient underwent


radiofrequency catheter ablation
for atrial fibrillation (arrow).
After ablation a significant
decrease in episodes of atrial
fibrillation was recorded.
Ideviceinterrogation, P device
programmed
356 F.M. Kusumoto et al.

Bibliography Lee MA, Weachter R, Pollak S, etal. The effect of atrial pacing thera-
pies on atrial tachyarrhythmia burden and frequency: results of a
randomized trial in patients with bradycardia and atrial tachyar-
Calkins H, Brugada J, Packer DL, et al. HRS/EHRA/ECAS expert rhythmias. J Am Coll Cardiol. 2003;41:1926-1932.
consensus statement on catheter and surgical ablation of atrial Page RL, Wilkinson WE, Clair EA, McCarthy EA, Pritchett EL.
fibrillation: recommendations for personnel, policy, procedures and Asymptomatic arrhythmias in patients with symptomatic paroxys-
follow-up. Heart Rhythm. 2007;4:816-861. mal atrial fibrillation and paroxysmal supraventricular tachycardia.
Israel CW, Gronefeld G, Ehrlich JR, Li YG, Hohnloser SH. Long-term Circulation. 1994;89:224-227.
risk of recurrent atrial fibrillation as document by an implantable
monitoring device: Implications for optimal patient care. J Am Coll
Cardiol. 2004;43:47-52.
Case 87

Kenneth A. Ellenbogen

Case Summary Case Discussion

A 54-year-old man presents to pacemaker clinic for routine The episode summary reveals an atrial arrhythmia with a
device follow-up. He has a history of hypertension, typical median cycle length of 230ms falling in the AF zone which
isthmus-dependent atrial flutter, and atrial tachycardia. Six is successfully treated with a single sequence of ATP. Review
months ago he underwent implantation of a dual chamber of the interval plot (Fig.87.1) shows the ATP delivered at the
Medtronic pacemaker with atrial antitachycardia pacing far right hand side with termination of the arrhythmia as
capabilities. On further questioning he reports an episode evidenced by the boxes returning to an approximate atrial
of palpitations that occurred last week lasting several cycle length of approximately 800 ms. The corresponding
minutes. marker channel (Fig.87.3) shows the delivery of ATP consist-
Patient underwent interrogation of his device and the ing of eight pulses beginning at 190ms with each subsequent
episode summary correlated with his reported symptoms. It pulse decrementing by 10ms with successful termination of
showed an atrial tachycardia with an atrial median cycle the atrial arrhythmia.
length of 230ms and a duration of 1.1 min. It was terminated Devices with ATP capability can have ATP therapy pro-
by the first round of ramp pacing and had a ventricular rate grammed either with Ramp where there is a train of decre-
of 120140 bpm. The interval plot for the event is shown mental pulses or with Burst+ where there is a drive train of
(Fig. 87.1) with what key event seen at the far right hand side nondecremental pulses followed by two atrial extrastimuli.
of the interval plot? The accompanying atrial and ventricular In those with atrial tachycardia cycle lengths >240ms, Ramp
marker channels at the onset of the atrial arrhythmia is shown ATP has been shown to be more efficacious than Burst+ with
(Fig. 87.2) followed by the marker channels and EGM likely similar efficacy in atrial arrhythmias with cycle lengths
(Atip-RV ring) during delivery of ATP therapy (Fig. 87.3) <240 ms. In this case, the patients symptomatic atrial
correlating with the far right hand side of the interval plot. arrhythmia was effectively and consistently terminated by
What is the arrhythmia and what does the pacemaker do? atrial antitachycardia pacing. This feature has proven to be
less useful now, as many patients with organized atrial
arrhythmias will undergo a curative ablative procedure if the
arrhythmias are recurrent and symptomatic, while patients
K.A. Ellenbogen who have atrial fibrillation and more disorganized atrial
Cardiac Electrophysiology and Pacing, Department of Cardiology,
VCU School of Medicine, P.O. Box980053, Richmond, VA
arrhythmias are much less likely to have the arrhythmia
23298-0053, USA terminated by pacing. However, there is still clearly a role for
e-mail: kellenbogen@pol.net anti-tachycardia atrial pacing in selected patients.

A. Natale et al. (eds.), Cardiac Electrophysiology, 357


DOI: 10.1007/978-1-84996-390-9_87, Springer-Verlag London Limited 2011
358 K.A. Ellenbogen

Fig.87.1 Stored interval plot of episode


Case 87 359

Fig.87.2 Atrial and ventricular marker channel during episode


360 K.A. Ellenbogen

Fig.87.3 Atrial EGM and marker channel during delivery of ATP

Bibliography

Gulizia M, Mangiameli S, Orazi S, et al. Randomized comparison


between Ramp and Burst+ atrial antitachycardia pacing therapies in
patients suffering from sinus node disease and atrial fibrillation and
implanted with a DDDRP device. Europace. 2006;8(7):465-73.
Case 88

Nora Goldschlager

Case Summary indicating misconnection of the atrial and ventricular leads at


the pulse generator. The atrial stimulus delivery is followed
about 108ms later by a sensed event in the ventricular channel
A 72-year-old man had a dual chamber pacing system
(R). Because of the misconnection, the designated sensed
implanted for bradycardiatachycardia syndrome, associated
R event is in fact an atrial event. This atrial event might be a
with palpitations, chest discomfort, and presyncopal spells.
native P wave buried within the paced QRS complex or a far-
The implant procedure was completed without difficulty.
field signal due to the pacing stimulus delivery itself (cross
The patient remained in sinus rhythm throughout his recov-
talk); the differential diagnosis cannot be made from Fig.88.1,
ery. At his first postimplant clinic visit, pacemaker interroga-
nor would an atrial electrogram solve the problem, as it would
tion revealed 100% atrial pacing and 100% spontaneous
register an electrical event in either case. Cross talk could be
QRS complexes, considered to be appropriate for his previ-
diagnosed if reduction in stimulus voltage output resulted in
ously documented arrhythmia. A 12-lead ECG recorded
disappearance of the atrial sensed event (designated R),
with a magnet placed over the pulse generator revealed ven-
whereas a native P wave would be unaffected.
tricular capture; atrial stimulus outputs were not well seen
Ventricular pacing through the atrial lead in this patient is
and atrial capture could not be confirmed. (Note that for this
not desirable, and could lead to atrial fibrillation, systolic
manufacturer, the DOO rate is 100ppm and the AV interval
dysfunction, AV dyssynchrony or 1:1 retrograde atrial acti-
is 110ms.) Rate programming to a low base rate was per-
vation with its attendant hemodynamic problems of hypoten-
formed in order to assess the patients underlying rhythm and
sion, AV valve regurgitation, and clinical heart failure. Had
rate. Figure 88.1 illustrates the findings with the base rate
the lead misconnection diagnosis been made at the time of
programmed to 45 ppm. What is the problem? How is the
implant the problem would have been corrected immediately,
marker channel information explained?
making pacing system revision unnecessary. It is mandatory
to verify the origin of the depolarization resulting from pac-
ing stimulus delivery: In this case, atrial stimulus delivery
Case Discussion causing ventricular depolarization would have been obvious
had appropriate attention been paid to the monitored rhythms
during the implantation procedure.
Figure88.1 illustrates the body surface ECG, the ventricular
intracardiac electrogram, and marker channel information.
The atrial pacing stimulus (A) is producing a QRS complex,

N. Goldschlager
Department of Cardiology, University of California, San Francisco,
Cardiology Division, 5G1, San Francisco General Hospital,
1001 Potrero Avenue, San Francisco, CA 94110, USA
e-mail: ngoldschlater@medsfgh.ucsf.edu

A. Natale et al. (eds.), Cardiac Electrophysiology, 361


DOI: 10.1007/978-1-84996-390-9_88, Springer-Verlag London Limited 2011
362 N. Goldschlager

ECG Controls Programmed Parameters


Surface ECG On Mode DDDR
Position 1 Base Rate 45 ppm
Gain 1 mV/cm A-V Delay 250 ms
Filter On P-V Delay 175 ms
Markers On Magnet Response Temporary Off
Position 2 Temporary 30 Off
IEGM On
Position 3
Gain 10 mV/cm
Configuration V IEGM Bi
Sweep Speed 25 mm/s

1.0 Second 25 Apr 2002 16:07

Fig.88.1 Programmed parameters, surface ECG, marker channel and ventricular EGM
Case 89

Kenneth A. Ellenbogen, Rod Bolanos, and Mark A. Wood

Case Summary Case Discussion

A 67-year-old female with a past medical history of sick Review of the 12-lead EKG shows a wide complex tachycar-
sinus syndrome underwent implantation of a dual chamber dia with two pacing spikes associated with each QRS consis-
pacemaker 2 years ago. The patient presented to her local tent with dual chamber pacing at a rate of 150bpm. There is
emergency room (ER) after being bitten by a spider. While in no evidence of undersensing or failure to capture (in the
the ER, she was placed on a cardiac and respiratory atrium or ventricle) on this strip. To know if the pacer was
monitor. functioning properly, one would need access to the pro-
While receiving treatment for the spider bite, she was grammed parameters of the device (Fig.89.2), which revealed
noted to have a wide complex tachycardia of 150 bpm on that the device was programmed DDDR with a dual sensor
telemetry. The patient was given a 150-mg bolus of amio- rate response at a maximum sensor rate of 150bpm match-
darone and transferred to the nearest regional medical center ing the clinical tachycardia. Therefore, the pacer exhibited
for further management of her tachycardia. The 12-lead EKG normal function based on its programming, but its sensor
of the tachycardia is shown (Fig. 89.1). Is there evidence of response was inappropriately triggered by a nonphysiologic
normal pacemaker function on the strip? The patients cause, the respiratory monitor in the emergency room.
tachycardia suddenly stopped while en route to the regional Review of Fig. 89.3 shows that the onset and offset of the
medical center. Upon arrival there, her pacemaker was inter- sensor-driven tachycardia along with a brief respite shown
rogated. Her device was a Guidant Pulsar MaxTM II dual as a blip on the Trending Plot correlated to the patients
chamber pacemaker with a 4469 FinelineTM 2 active fixation exposure to the respiratory monitor and other electrical
atrial lead and 4470 FinelineTM 2 active fixation ventricular equipment in the emergency room.
lead. The device programmed parameters are shown This patients sensor-driven tachycardia was initiated by
(Fig. 89.2). What programmed parameter is particularly the MV sensor which responds to changes in transthoracic
significant? impedance and not the accelerometer which is triggered by
Further interrogation of the device reveals the Trending movement/motion affecting a piezocrystal located in the
Plot/Sensor Replay (Fig. 89.3) summarizing the Minute device generator. Review of the MV impedance measure-
Ventilation (MV) data which in this instance was sampled by ments (Fig. 89.4) shows that the impedance first went up
the device every 60 s over 102 h. What can one garner from around the time of the patients presentation to the ER likely
the plot in terms of the onset, offset, and duration of the due to hyperventilation. The impedance rose again and
tachycardia, and thus the possible etiology of the patients remained elevated after the patient was placed on the moni-
tachycardia? What steps can be taken to prevent this problem tor. Devices such as this patients may pace at the upper
in other cases? sensor rate in response to electrocautery, hyperventilation
(physiologically appropriate), and to other medical electronic
equipment such as respiratory monitors in clinical care areas
and those associated with echocardiographic machines.
Therefore, it is recommended that rate sensor modulated
K.A. Ellenbogen ()
pacing features be disabled in any patient that is undergoing
Cardiac Electrophysiology and Pacing, Department of Cardiology,
VCU School of Medicine, Richmond, VA 23298-0053, USA a surgical procedure or who is to be exposed to monitoring in
e-mail: kellenbogen@pol.net a critical care unit.

A. Natale et al. (eds.), Cardiac Electrophysiology, 363


DOI: 10.1007/978-1-84996-390-9_89, Springer-Verlag London Limited 2011
364 K.A. Ellenbogen et al.

Fig.89.1 12 lead ECG of presenting tachycardia


Case 89 365

Fig.89.2 Device programmed


Brady Parameters
parameters

Initial Present
Value Value
Mode DDDR DDDR

Lower Rate Limit 60 60 ppm


Max Tracking Rate 130 130 ppm
Max Sensor Rate 150 150 ppm
AV Delay (paced) DYN -- DYN -- ms

Atrial
Pulse Width 0.40 0.40 ms
Amplitude 2.6 2.6 V
Sensitivity 0.25 0.25 mV
Refractory (PVARP) DYN -- DYN -- ms

Ventricular
Pulse Width 0.40 0.40 ms
Amplitude 2.6 2.6 V
Sensitivity AUTO 3.08 AUTO 3.08 mV
Refractory 250 250 ms

AV Delay

Initial Present
Value Value
Dynamic AV Delay On On
Maximum Delay 270 270 ms
Minimum Delay 120 120 ms
Sensed AV Offset 30 30 ms

AV Search Hysteresis
Search Interval 32 32 cycles
AV Increase 30 30 %

Sensor (s)

Initial Present
Value Value
Accelerometer On On
Activity Threshold Medium Medium
Reaction Time 30 30 sec
Response Factor AUTO 11 AUTO 11
Recovery Time 2 2 min
Minute Ventilation On On
MV Lead Ventricle Ventricle
Response Factor AUTO 3 AUTO 3
High Rate Response Factor 85 85 %
HIgh Rate Break Point 115 115 ppm
Age 66 66
Gender Female Female
Auto Response On On
ACC Initial Response Factor 8 8
MV Initial Response Factor 3 3

Sensor Rate Target 110 110 ppm


Time Dependent Blend On On
366 K.A. Ellenbogen et al.

Fig.89.3 Trending replay parameters and plot data


Case 89 367

Fig.89.4 Minute ventilation


impedance data

Bibliography

Seeger W, Kleinert M. An unexpected rate response of a minute ventila-


tion dependent pacemaker. PACE. 1989;12:1707.

Case 90

Fred M. Kusumoto, Jennifer Crain, and Nora Goldschlager

Case Summary

A 75-year-old man with an 8-year history of permanent atrial


fibrillation comes to your office for a second opinion. He had
a single chamber ventricular pacemaker placed 4 years ago
for bradycardia. He began developing symptoms of increas-
ing shortness of breath and had his pacing rate increased
from 60 to 70ppm. Progressive shortness of breath developed;
an echocardiogram demonstrated severe mitral regurgitation
(Fig.90.1). He has now been referred for mitral valve replace-
ment. What pacing system programming changes should be
considered at this point?

Fig.90.1 Echocardiogram obtained during ventricular pacing


Case Discussion

The patient has severe mitral regurgitation associated with


ventricular pacing. The pacemaker was reprogrammed to a
lower rate limit of 40 bpm to allow intrinsic ventricular
activation; the mitral regurgitation improved significantly
(Fig.90.2), as did the patients symptoms.
Several large randomized trials have reported an increased
incidence in heart failure associated with right ventricular
pacing. Case reports have described patients in which right

F.M. Kusumoto(*)
Department of Cardiovascular Diseases, Mayo Clinic,
4500 San Pablo Road, Jacksonville, FL 32224, USA
e-mail: kusumoto.fred@mayo.edu Fig.90.2 Echocardiogram obtained during spontaneous cardiac rhythm
J. Crain
Electrophysiology and Pacing Service, Division of Cardiovascular
Diseases, Department of Medicine, Mayo Clinic, Jacksonville, FL
32224, USA
ventricular pacing significantly worsened mitral regurgita-
N. Goldschlager tion. In one study of 256 patients that underwent AV nodal
Department of Cardiology, University of California, San Francisco,
Cardiology Division, 5G1, San Francisco General Hospital,
ablation and permanent pacing, approximately 5% of pati
1001 Potrero Avenue, San Francisco, CA 94110, USA ents developed severe mitral regurgitation and four patients
e-mail: ngoldschlater@medsfgh.ucsf.edu underwent mitral valve replacement; at surgery, no structural

A. Natale et al. (eds.), Cardiac Electrophysiology, 369


DOI: 10.1007/978-1-84996-390-9_90, Springer-Verlag London Limited 2011
370 F.M. Kusumoto et al.

abnormalities of the mitral valve were identified. Right ven- Bibliography


tricular pacing causes abnormal cardiac activation that in
susceptible patients can worsen mitral regurgitation by Berglund H, Nishioka T, Hackner E, et al. Ventricular pacing: a cause of
changing the depolarization contraction timing sequence reversible severe mitral regurgitation. Am Heart J. 1996;131:1035-1037.
of the mitral valve apparatus and altering papillary muscle Sweeney MO, Hellkamp AS, Ellenbogen KA, etal. Adverse effect of
ventricular pacing on heart failure and atrial fibrillation among
alignment.
patients with normal baseline QRS duration in a clinical trial of
In patients that require pacing for rate support, biventricu- pacemaker therapy for sinus node dysfunction. Circulation. 2003;
lar pacing can improve right ventricular pacingassociated 107:2932-2937.
mitral regurgitation. In this case, ensuring minimal right Wilkoff BL, Cook JR, Epstein AE, etal. Dual-chamber pacing or ven-
tricular backup pacing in patients with an implantable defibrillator:
ventricular pacing by simple programming changes improved
the Dual Chamber and VVI Implantable Defibrillator (DAVID)
the patients symptoms and averted the planned surgery. trial. JAMA. 2002;288:3115-3123.
Case 91

M. Eyman Mortada, Jasbir S. Sra, and Masood Akhtar

Case Summary Case Discussion

An 83-year-old male with sick sinus syndrome received a Pacemaker interrogation revealed sinus rhythm in the first
permanent pacemaker. He has a history of coronary artery four beats. Then, a premature atrial contraction occurred
disease with preserved left ventricular function. He com- in the post ventricular atrial refractory period (PVARP),
plained of multiple episodes of palpitations. Interrogation of followed by a prolonged PR interval. This premature
his pacemaker revealed multiple episodes of tachycardia, beat initiated the tachycardia with a very short, fixed RP
where one of them was stored as shown in Fig. 91.1. interval (70ms) and a cycle length of 400ms. The con-
His baseline 12-lead ECG demonstrated sinus rhythm stant fixed RP interval suggests ventricular-atrial link-
with first-degree AV block. The tachycardia was induced in ing due to retrograde activation over a pathway (AV nodal
the electrophysiology lab with atrial extrastimuli. The results or accessory). The short RP duration presumably
of ventricular entrainment of this tachycardia are shown in excludes atypical AV reentry tachycardia and orthodro-
Fig.91.2. mic AV reentry tachycardia via an accessory pathway.
What is the diagnosis and what would be the best man- Therefore, the most likely diagnosis is typical AV nodal
agement in this case? reentry tachycardia.

Fig.91.1 Interrogation of the patients pacemaker revealed multiple episodes of tachycardia, one of which is shown here. (Simultaneous recording
[from top to bottom] of: atrial bipolar recording, ventricular bipolar recording, and Markers channel.) AS atrial sensing; VS ventricular sensing

M.E. Mortada (*), J.S. Sra, and M. Akhtar


Department of Electrophysiology,
Aurora Cardiovascular Services, Aurora Sinai/Aurora
St. Lukes Medical Centers, University of Wisconsin School of
Medicine and Public Health, 2801 W. Kinnickinnic River
Parkway, #777, Milwaukee, WI 53215, USA
e-mail: publishing4@aurora.org

A. Natale et al. (eds.), Cardiac Electrophysiology, 371


DOI: 10.1007/978-1-84996-390-9_91, Springer-Verlag London Limited 2011
372 M.E. Mortada et al.

Fig.91.2 Tachycardia was induced in the electrophysiology lab with sinus (CS 9,10) to distal coronary sinus (CS 1,2) recording, His record-
atrial extrastimuli. Ventricular entrainment of this tachycardia is shown ing, and right ventricular recording.) HRA high right atrium; CS coro-
here. (Simultaneous recording [from top to bottom] of: surface ECG nary sinus; HISp proximal His; HISd distal His; RVa apical right
and leads I, II and V1, high right atrial recording, proximal coronary ventricle

During the electrophysiology study, the post ventricular earliest atrial activity duration (200ms) excludes AV reentry
entrainment demonstrated V-A-V response excluding atrial tachycardia via a septal accessory pathway and atypical AV
tachycardia. The short VA duration seen during the tachy- nodal reentry tachycardia. Hence, the diagnosis is confirmed
cardia (46 ms), concentric atrial activation, the long post- as typical AV nodal reentry tachycardia, and the best man-
pacing interval (780ms), and the long ventricular spike to agement in this case is AV nodal modification.
Case 92

Amin Al-Ahmad

Case Summary Case Discussion

A 65-year-old man with a history of congestive heart failure Defibrillation threshold testing is an important step during
due to ischemic cardiomyopathy and an implantable cardio- the implantation of ICDs. Both sensing of ventricular fibril-
verter defibrillator is admitted to the intensive care unit after lation and determination of the threshold energy needed to
a witnessed collapse. First responders found him to be in terminate ventricular fibrillation can be determined during
ventricular fibrillation and were able to successfully shock defibrillation threshold testing. High defibrillation thresholds
him to normal sinus rhythm using an external defibrillator. can be seen in younger patients, those taking amiodarone,
Interrogation of his Guidant Contak Renewal 3 HE ICD patients with non-ischemic cardiomyopathy, and those with
revealed an episode of ventricular fibrillation that was not a lower ejection fraction.1 Patients with a defibrillation
successfully converted despite multiple maximum energy threshold that has a safety margin of less than 10 J should be
shocks (Fig.92.1). Device therapy was exhausted. considered for device modification.
Records obtained from his implanting institution demon- High defibrillation thresholds can occasionally be decrea
strate failure to convert ventricular fibrillation during defi- sed by using sotalol or dofetolide. Removal of the proximal
brillation threshold testing at 31 J, and failure once at 36 J. coil from the circuit or changing the polarity of shock may
Maximum energy (41 J) was successful in terminating also be of value. Placement of a subcutaneous shock coil or a
ventricular fibrillation. shock coil in the azygous vein may also decrease the defibril-
Device settings are as follows: lation thresholds. Some devices allow for a change in the
programmed defibrillation waveform pulse width or the tilt.
Mode DDD It this case the best option would have been to advance the
Lower rate limit/Upper rate limit 60/120ppm lead further into the right ventricle. Figure92.2 shows the tip
VT Zone (rate>180bpm) ATP3 (burst), 41 J5 of the shock lead is placed in the right ventricular inflow area
VF Zone (rate>220bpm) 41 J5 just beyond the tricuspid valve annulus. Advancement of the
lead such that the tip reaches the right ventricular apex will
A chest X-ray taken in the intensive care unit is shown likely improve the defibrillation threshold.
(Fig. 92.2). What steps could have been taken during the
implant to prevent this from occurring?

A. Al-Ahmad
School of Medicine, Stanford University, 300 Pasteur Drive, H-2146,
Stanford, CA 94305, USA
e-mail: aalahmad@stanford.edu

A. Natale et al. (eds.), Cardiac Electrophysiology, 373


DOI: 10.1007/978-1-84996-390-9_92, Springer-Verlag London Limited 2011
374 A. Al-Ahmad

Fig.92.1 Episode of ventricular fibrillation with failed 41 J shock

Reference

1. Russo AM, Sauer W, Gerstenfeld EP, etal. Defibrillation threshold


testing: is it really necessary at the time of implantable cardio-
verter-defibrillator insertion? Heart Rhythm. 2005; 2:456-461.

Fig.92.2 Chest X-rays showing biventricular system; note lack of left


ventricular lead due to failure to place lead during implant
Case 93

M. Eyman Mortada, Jasbir S. Sra, and Masood Akhtar

Case Summary Case Discussion

A 62-year-old male had a history of nonischemic cardiomyo- Current models of pacemakers have advanced programming
pathy with a left ventricular ejection fraction of 30%, conges- capabilities. There are multiple features that can be adapted
tive heart failure (New York Heart Association class II-III), to the individual patients needs. However, these sophisti-
and atrial fibrillation. His rhythm was converted to sinus cated programming options can create complications of their
rhythm with DC cardioversion and maintained with Tikosyn own. This case is one example.
therapy. The cardiomyopathy persisted after optimizing his Atrial fibrillation (AF) suppression, by pacing the
medical therapy for 34 months. Therefore, he received a atrium faster than the sinus rate, is a great tool to reduce
dual-chamber implantable cardioverter defibrillator, and the the frequency of AF. This patient had no episodes of AF
device was programmed to suppress the atrial fibrillation by since the device was implanted. Yet, he had multiple symp-
pacing the atrium 10 beats faster than his sinus rate to a maxi- tomatic episodes of other types of tachycardia. The tachy-
mum of 110bpm with an AF suppression cycle of 30 beats. cardias started with a programmed pacing in the atrium at
He has a first-degree AV block with AV delay of 205ms, so faster rates (10bpm faster) than the sinus rate to suppress
the sensed AV delay was programmed at 250 ms and the AF, and, at the same time, he had two premature ventricu-
paced AV delay was programmed at 300ms. lar captures (PVC). The second PVC had a retrograde
On an office visit, he complained of multiple episodes of atrial activity, which landed in the PVARP. It was then fol-
palpitations. During the interrogation of the device, an episode lowed by an atrial pacing at the programmed cycle length
of tachycardia had occurred, similar to multiple episodes that (10 bpm faster than the sensed sinus rate) which did not
were disclosed on the stored EGMs (Fig. 93.1). capture the atrium due to its refractory period status (func-
What is the mechanism of the tachycardia? What is the tional non-capture). After 300 ms (paced AV delay), the
best course of action? ventricle was paced and subsequently gave a retrograde
Fig. 93.2 shows a recording after shortening the Post- atrial activity that landed in the PVARP again, leading to
Ventricular Atrial Refractory Period (PVARP), shortening continuous-circuit symptomatic tachycardia. This mani-
the AV delay, and turning ON the VIP (Ventricular Intrinsic festation had been described by Barold and Levine as
Preference) mode. pacemaker-repetitive nonreentrant ventriculoatrial syn-
What is the most likely mechanism of the tachycardia? chronous rhythm1. Currently, there is no programmable
What is the best course of action? algorithm available to detect and terminate this condition.
There are a few algorithms which may prevent this situa-
tion from happening:
1. Prolongation of the atrial escape interval to allow atrial
myocardium time to recover: This algorithm can be pro-
grammed in St. Jude and Medtronic devices by extending
the atrial escape interval to 300350ms from the sensed
M.E. Mortada (*), J.S. Sra, and M. Akhtar atrial activity when it lands in PVARP (e.g. PVC with
Department of Electrophysiology, retrograde atrial conduction). This would prevent the
Aurora Cardiovascular Services, Aurora Sinai/Aurora St. Lukes functional non-capture condition.
Medical Centers, University of Wisconsin School of Medicine and
2. A decrease in the lower base rate (increase lower rate inter-
Public Health, 2801 W. Kinnickinnic River Parkway, #777,
Milwaukee, WI 53215, USA val): In this scenario, it would be feasible to lower the base
e-mail: publishing4@aurora.org rate only if one turned off the atrial fibrillation suppression

A. Natale et al. (eds.), Cardiac Electrophysiology, 375


DOI: 10.1007/978-1-84996-390-9_93, Springer-Verlag London Limited 2011
376 M.E. Mortada et al.

Fig. 93.1 During the interrogation of the device, an episode of Markers channel, atrial bipolar recording, right ventricular bipolar
tachycardia had occurred. It was similar to multiple episodes that were recording, and key parameters. Labels used in this figure are defined in
disclosed on the stored EGMs. From top to bottom: Leadless ECG, the text

Fig.93.2 After shortening the PVARP, shortening the AV delay, and turning ON the VIP (Ventricular Intrinsic Preference) mode; this recording
was observed
Case 93 377

mode. But that would increase the risk of atrial fibrillation. the rhythm recorded over the ventricle during the tachycardia
Hence, using atrial pacing for AF suppression and termi- is all paced beats. Therefore, it cannot be spontaneous _ven-
nating this condition if it occurs reduces the number of tricular tachycardia.
pacing cycles. That, in turn, increases the frequency of The best course of action is to increase the PVARP to
detections of intrinsic atrial activity, leading to interruption prevent the PMT from happening. However, this increases
of the pacemaker-repetitive nonreentrant ventriculoatrial the risk of pacemaker-repetitive nonreentrant ventriculoatrial
synchronous rhythm, if it is present. synchronous rhythm. The following modifications of the
3. Shortening of the AV delay: This feature is helpful in algorithm prevented the two observed tachycardias:
patients who need ventricular pacing due to atrioventricu-
1. PVARP duration same as it was in the beginning.
lar block. It allows for longer VA duration in a fixed cycle
2. Detection of PMT algorithm turned ON to terminate the
length, as in this case. When there is longer VA duration,
tachycardia when it happens.
the next atrial pacing comes further out, allowing more
3. Shortened AV delay with longer ventricular intrinsic
time for the atrial tissue to recover during the PVARP if
preference.
there is atrial activity, leading to capture of the atrium and
4. Extended atrial escape interval when atrial activity is
preventing this condition from happening.
sensed in the PVARP.
After shortening the PVARP, shortening the AV delay, and 5. Reduced number of pacing cycles during atrial pacing in
switching the Ventricular Intrinsic Preference (VIP) mode to the AF suppression algorithm.
ON, the patient again had two PVCs, which induced another
Each patient is unique in regards to the function of his
type of tachycardia. After the second PVC (fourth sensed
conduction system and its response to the different pacing
ventricular activity), the patient had a retrograde atrial activ-
maneuvers. Thus, it is essential to try the above algorithms
ity during the PVARP, leading to pacemaker-repetitive non-
at multiple intervals, to fit the requirements of each patient,
reentrant ventriculoatrial synchronous rhythm for two beats.
until the issue is solved. As a last resort, it may be neces-
In these two beats, after the PVC, the paced AV delay was
sary to turn OFF the AF suppression program to prevent
long, with no escaped intrinsic ventricular activity. Due to
pacemaker-repetitive nonreentrant ventriculoatrial syn-
the programmed VIP, the paced AV delay was shortened,
chronous rhythm. If that is the case, it is important to
leading to termination of the above condition, but causing
remember to increase the PVARP Neither decreasing
the retrograde atrial activity to be out of the PVARP due to
PVARP nor changing atrial sensitivity solves the problem
the decremental VA conduction, and, therefore, tracking it
of PMT2.
with ventricular pacing and creating pacemaker-mediated
tachycardia (PMT). It is less likely to be an atrial arrhythmia
(atrial tachycardia) due to the fixed VA conduction time and
the fixed rate at max track rate (120bpm 500ms). There References
was no far-field ventricular sensing, since the atrial pacing
was not inhibited, and there was atrial activity present at the 1. Barold SS, Levine PA. Pacemaker repetitive nonreentrant ventricu-
same cycle length of the sinus rate (the sinus cycle length loatrial synchronous rhythm. A review. J Interv Card Electrophysiol
2001;5:45-58.
was 630640 ms as seen in the first interrogation). 2. Ellenbogen KA, Kay GN, Lau CP, Wilkoff BL, Lau CP. Clinical
Additionally, there were extra markers on the atrial lead at Cardiac Pacing, Defibrillation and Resynchronization Therapy. 3rd
the same time as the markers on the ventricular lead. Finally, ed. Edinburgh, UK: Saunders/Elsevier Health Science, 2006, pp 101.

Case 94

Kenneth A. Ellenbogen and Rod Bolanos

Case Summary The ICD lead is positioned more distally out in the apex
and a new true bipolar rate sense lead is inserted and posi-
tioned in the right ventricular septum. No further evidence of
A 68-year-old man undergoes implantation of a dual cham-
far-field P wave oversensing is seen. The patient underwent
ber ICD for sustained monomorphic ventricular tachycardia
repeat DFT testing successfully at 14 J. While the pocket
(VT) induced during electrophysiology study. His past medi-
was being closed, the patient received two successive shocks
cal history is significant for an ischemic cardiomyopathy
while the cardiac monitor revealed sinus bradycardia in the
with an LVEF of 40% and a prior history of syncope.
1940. Device interrogation shows surface lead I, Atrial egm,
The patient receives a Guidant VitalityTM 2 EL T 167 ICD
ventricular rate/sense egm, and marker channel (Fig. 94.3)
with a 4087 FlextendTM active fixation atrial lead and a dual
just prior to delivery of the shocks. Does the rapid sensed
coil active fixation RelianceTM G 0185 Gore ICD lead. The R
ventricular event on the ventricular rate/sense EGM have a
waves at implant were 14mV and the P waves were 2.5mV.
likely physiologic explanation?
The following stored EGM was recorded just prior to DFT
testing (Fig. 94.1). The first and sixth QRS complexes are
preceded by a V sensed event and the third QRS complex has
two closely coupled V sensed events. Do these sensed events
on the ventricular channel correlate with any physiologic
Case Discussion
electrical activity? The patient undergoes successful DFT
testing at 14J twice, and the events seen on the ventricular The atrial, ventricular, and shock electrograms of one of the
channel are no longer seen. The pocket is closed and the two episodes are shown (Fig.94.4). The rapid ventricular
patient is returned to his room uneventfully. event is seen only on the rate/sense lead. What should be the
The next morning during the post implant device check, next step in diagnosing the cause of this problem? The pocket
the following EGM is elicited during pacing (Fig.94.2). The was opened and the set screws on the rate sense port exam-
atrial output was 3.5V at 0.5ms. There is intermittent sens- ined. The screws were appropriately tightened but the arti-
ing on the ventricular channel of an event that appears to fact remained. A second generator was connected with
reproducibly follow the paced P wave. What are possible resolution of the rapid, rhythmic activity previously seen
explanations for why the P wave is being sensed on the only on the ventricular rate/sense lead. The original genera-
ventricular channel (far-field P wave oversensing)? What tor underwent evaluation by the manufacturer where a large
diagnostic study may aid in explaining this finding? Can the tear in the rate/sense header sealing ring was seen.
choice of ventricular lead affect the prevalence of this In Fig. 94.1, the VS event preceding the first and sixth
problem? QRS complexes have a sharp EGM and do not appear to cor-
Review of the patients morning chest x-ray shows that relate with any atrial activity or with any other part of the
the ventricular lead has come back and has lost most of its ventricular electrogram such as the T wave and should be
heel in the right atrium. The decision is made to bring the considered nonphysiologic. The VS event preceding the
patient back to the EP lab to reposition the ventricular lead. third QRS complex could possibly be explained by far-field
P wave oversensing. At that time no further evidence of
inappropriate sensing on the ventricular channel was seen
and DFTs were performed successfully.
K.A. Ellenbogen () The next morning there is clear evidence of intermittent
Cardiac Electrophysiology and Pacing, Department of Cardiology,
VCU School of Medicine, P.O. Box980053, Richmond, VA
sensing of the P wave on the ventricular channel. In this case,
23298-0053, USA lead position must be assessed, and this can be done with a post
e-mail: kellenbogen@pol.net implant day chest x-ray. If the RV lead is dislodged, implanted

A. Natale et al. (eds.), Cardiac Electrophysiology, 379


DOI: 10.1007/978-1-84996-390-9_94, Springer-Verlag London Limited 2011
380 K.A. Ellenbogen and R. Bolanos

Fig.94.1 Surface ECG, atrial and ventricular EGM prior to DFT testing

too proximally in the RV septum, or in the RV outflow tract on all EGMs. Diaphragmatic myopotential oversensing is
then P wave oversensing can be seen. The P wave pacing out- typically seen on the rate sense channel with integrated leads
put is not particularly high in this patient, but a high atrial pac- like the one used in this patient; but the sharp, well-demarcated
ing output would be more likely to result in far-field P wave mostly regular appearing EGMs are not typical of diaphrag-
oversensing. The ICD lead utilized in this case is an integrated matic myopotentials. The appearance of such nonphysiologic
bipolar lead and is more susceptible to this phenomenon. The artifacts on only one lead particularly after a recent implant
chest film revealed that the ICD lead had pulled back some and demands close inspection of that lead and the connection to
this likely was the fundamental event that resulted in oversens- the device header.
ing of the P wave on the ventricular rate/sense lead. The rate sense lead was inspected and was unremarkable.
The patient was appropriately returned to the lab and not With insertion of the header torque wrench, the artifact could
only was the ICD lead positioned more distally, but a sepa- be reproduced and the generator was exchanged with elimina-
rate true bipolar rate sense lead was inserted to minimize the tion of the artifact. The original device underwent inspection
likelihood of future far-field P wave oversensing. Review of by the manufacturer that revealed a large tear in the sealing
the EGMs associated with the two inappropriate shocks ring of the rate/sense lead port. Compromise of the set-screw
during pocket closure shows rapid, sharp, and rhythmic VS seal plug can lead to oversensing due to air escaping from the
events that fell into the VF zone and resulted in two shocks. header connectivity cavity. This oversensing is typically seen
The VS EGMs do not correlate with any ventricular activity on the ventricular rate sense lead and usually lasts only 12
and there is no clear atrial activity that is being oversensed in days. This phenomenon can lead to inappropriate detection of
this instance. When both the rate/sense EGM and Shock coil ventricular tachycardia/ventricular fibrillation in approxi-
EGMs are compared, it is clear that only the rate/sense lead mately 20% of cases. To prevent damage to the seal plug dur-
is being affected. Potential nonphysiologic causes of EMI ing insertion of the torque wrench, the manufacturer
typically effect both leads and EMI typically has a pulsed, recommended approaching the sealing ring at a 45 angle with
high frequency appearance that is of varying duration seen the torque wrench prior to fully engaging the header screw.
Case 94 381

Fig.94.2 Surface ECG, atrial and ventricular EGM on post-op day one

Fig.94.3 Surface ECG, atrial and ventricular EGM after lead revision and DFT
382 K.A. Ellenbogen and R. Bolanos

Fig.94.4 Atrial, ventricular and shock electrograms during episode that resulted in shock

Bibliography Lee BP, Wood MA, Ellenbogen KA. Oversensing in a newly implanted
dual-chamber implantable cardioverter-defibrillator: what is the
mechanism? Heart Rhythm. 2005;2:782-783.
Cheung JW, Iwai S, Lerman BB, Mittal S. Shock-induced ventricular Weretka S, Michaelson J, Becker R, et al. Ventricular oversensing:
oversensing due to seal plug damage: a potential mechanism of inap- A study of 101 patients implanted with dual chamber defibrillators
propriate device therapies in implantable cardioverter-defibrillators. and two different lead systems. PACE. 2003;26(Part 1):65-70.
Heart Rhythm. 2005;2:1371-1375.
Case 95

Byron K. Lee

Case Summary arrhythmia is likely a supraventricular tachycardia but it does


not rule out ventricular tachycardia. AV association can also
occur with ventricular tachycardia when there is 1:1 retro-
A 46-year-old man with a history of ventricular tachycardia
grade conduction.
and ICD implantation presented urgently to the Device Clinic
Further examination of the intracardiac electrogram shows
1 day after experiencing several shocks while having sex.
that during ATP there is clear capture of the ventricle. During
After four painful and startling shocks, they suddenly stopped
ventricular capture, the atrial rate is unchanged and contin-
occurring. Several minutes later he was back to his baseline
ues on at exactly the same rate as the clinical tachycardia.
status and chose to come into the clinic the next day, rather
This indicates that the clinical tachycardia is driven by the
than to go to the emergency department immediately.
atria, clinching the diagnosis of a supraventricular tachycar-
Interrogation of his Medtronic Marquis ICD showed that
dia rather than ventricular tachycardia. In this case, exertion
there were several episodes of tachycardia that crossed the
preceded the shocks, strongly suggesting that the supraven-
lower rate cutoff of the VT zone and triggered therapies
tricular tachycardia is simply sinus tachycardia. Therefore,
(Fig.95.1). These episodes corresponded to when the patient
the shocks were inappropriate.
felt his shocks the night before. The device settings are
The shocks experienced by this patient could probably
shown in Fig.95.2.
have been avoided if his therapy zones were programmed
The intracardiac electrogram from one of the episodes of
more appropriately for someone his age. 220 age is a rea-
antitachycardic pacing (ATP) is shown in Fig.95.3. Can we
sonable estimate of the maximum sinus rate for a patient.
determine if the clinical arrhythmia is a supraventricular
Therefore, you could expect this patient would reach sinus
tachycardia or a ventricular tachycardia?
rates of around 174 bpm with heavy exertion. This rate is
higher than the programmed lower rate cutoff for his VT
zone which was set at 167bpm. Therefore, it was not surpris-
Case Discussion ing that he had inappropriate shocks. The patient had the
lower rate cutoff for his VT zone increased to 182bpm and
he has had no further inappropriate shocks since.
The beginning of the recording shows the clinical tachy
cardia. There is AV association with one A electrogram
for every V electrogram. This suggests that the clinical

B.K. Lee
Division of Cardiology, University of California, San Francisco,
500 Parnassus Avenue, San Francisco, CA 94143, USA
e-mail: leeb@medicine.ucsf.edu

A. Natale et al. (eds.), Cardiac Electrophysiology, 383


DOI: 10.1007/978-1-84996-390-9_95, Springer-Verlag London Limited 2011
384 B.K. Lee

Fig.95.1 Episode list Jul 14, 2005 15:21:26


ICD Model: Marquis DR 7274 9966 Software Version 4.0
Serial Number: PKC 138209H Copyright Medtronic, Inc. 2001
Episode Lists Report
Last Interrogation: Jul 14, 2005 13:18:17
Episodes Last Cleared: Jun 27, 2005 12:30:27

VT/VF Episodes
ID# Date/Time Type V. Cycle Last Rx Success Duration
9 Jun 30 15:36:04 VT 350 ms VT Rx 6 No 2.8 min
8 Jun 30 15:35:07 VT 350 ms VT Rx 2 Yes 47 s
7 Jun 30 15:23:02 VT 350 ms VT Rx 2 Yes 39 s
Last Session (Jun 27, 2005)
(Data prior to last session has not been interrogated.)

SVT/NST Episodes
ID# Date/Time A. Cycle V. Cycle Duration Reason
(No data since last session.)
Last Session (Jun 27, 2005)
(Data prior to last session has not been interrogated.)

Jul 14, 2005 15:20:20


ICD Model: Marquis DR 7274 9966 Software Version 4.0
Serial Number: PKC 138209H Copyright Medtronic, Inc. 2001
Parameter Summary Report
Therapy VT VF
167-188 bpm 188-500 bpm

1 Burst Pacing Defib 26 J


2 Ramp Pacing Defib 30 J
3 CV 15 J Defib 30 J
4 CV 30 J Defib 30 J
5 CV 30 J Defib 30 J
6 CV 30 J Defib 30 J

Brady Pacing
Mode DDD
Lower Rate 50 ppm
Upper Tracking Rate 130 ppm
Fig.95.2 Device parameters Upper Sensor Rate 95 ppm
Case 95 385

Fig.95.3 Intracardiac electrogram during ATP



Case 96

Amin Al-Ahmad and Paul J. Wang

Case Summary Case Discussion

A 20-year-old man with a history of cardiomyopathy, long Examination of the episode electrogram reveals episodic
QT syndrome, ventricular tachycardia, and an implantable bursts of high ventricular rates on the ventricular electro-
cardioverter defibrillator is admitted with multiple ICD gram. The cycle length during these bursts is less than 200ms
shocks. The patient has a Medtronic Maximo DR 7278 that and does not appear to be consistent with a physiologic sig-
was implanted 2years ago. Interrogation of the device reveals nal. This electrogram is most consistent with noise related to
five episodes classified as nonsustained VF and three epi- lead fracture. Lead fracture is not uncommon in younger
sodes classified as VF that are treated with shocks. A repre- patients who are very active, and can be the cause of painful
sentative episode is seen in Fig.96.1. Device parameters are inappropriate shocks. An impedance rise is commonly seen
as follows: with lead fracture, although it is worth noting that at times
the lead impedance can be normal. Asking the patient to per-
Mode AAI form isometric contractions of the upper extremities while
LRL/URL 70bpm checking the lead impedance can unmask a high lead imped-
VF zone 300ms (200bpm) ance when it is not immediately seen. Figure96.2 illustrates
Ventricular sensitivity 0.3mV the lead performance trends report; this shows the lead
impedance fluctuating from a normal reading to a very high
What is the cause of the shock? Should any programming reading.
changes be made?

Fig.96.1 Episode electrogram prior to shock delivery

A. Al-Ahmad and P.J. Wang(*)


School of Medicine, Stanford University, 300 Pasteur Drive, H-2146,
Stanford, CA 94305, USA
e-mail: aalahmad@stanford.edu; paul.j.wang@stanford.edu

A. Natale et al. (eds.), Cardiac Electrophysiology, 387


DOI: 10.1007/978-1-84996-390-9_96, Springer-Verlag London Limited 2011
388 A. Al-Ahmad and P.J. Wang

Ventricular pacing impendance


At implant 584 Highest >3,000
Last 584 Lowest 400


>3,000

2,000
1,500

1,000
800
600

400
300

<200
08/11/06 10/20/06 12/29/06 03/09/07 05/18/07 07/27/07 10/05/07 12/13/07 12/26/07
Last 80 weeks (min/max per week) Last 14 days

Fig.96.2 Lead performance report obtained during interrogation of the device

There are no programming solutions that will reliably performed safely by experienced operators and would be
solve this problem. The fractured lead was extracted without appropriate to consider in a young individual who may
difficulty and a new lead placed. Lead extraction can be require multiple leads over a lifetime.
Case 97

Kenneth A. Ellenbogen and Rod Bolanos

Case Summary the shock. What is the larger amplitude EGMs seen on all
three channels due to?
The rapid, sharp electrograms associated with the shock
A 67-year-old man with a history of syncope, an ischemic
episode are remarkable because they are seen on all three
cardiomyopathy, and an ejection fraction of 2530% is
channels. The finding of rapid, high frequency activity of
referred for electrophysiologic evaluation. Nonsustained VT
constant amplitude that is evident on all the channels occu-
is seen on telemetry after admission. The patient underwent
pying the entire cardiac cycle is highly characteristic of
electrophysiology studies and received a dual chamber
electromagnetic interference (EMI). When the noise is first
Guidant Ventak PrizmTM DR HE ICD for inducible, sustained
evident on the top right of Fig.97.1, not only is a tachyar-
monomorphic ventricular tachycardia.
rhythmia detected, but atrial and ventricular pacing is
The patient then presented to the emergency room after
inhibited. Prior to the noise the patient was being paced in
receiving a shock from his device while performing mainte-
both the atria and ventricle at approximately 80 beats per
nance on his car engine. He denied any prior sensation of
minute. EMI thus can result in both oversensing leading to
palpitations or lightheadedness before receiving the shock.
suppression of pacing and inappropriate shocks as seen in
The device was interrogated and the atrial, ventricular,
this case. Syncope may be due to inhibition of pacing in a
and shock electrograms associated with the episode are
pacing-dependent patient.
shown (Fig.97.1). What is the cause of the patients shock?
After the shock is delivered (bottom of Fig. 97.1), the
What should be done about it?
noise persists, and is interspersed with the larger EGMs seen
on all the channels correlating with an underlying sinus
rhythm at a lower rate of approximately 60 beats per minute.
Case Discussion Another EGM characteristic seen here that is consistent with
EMI is the larger amplitude of the signal on the farfield
On the top tracing at the far right hand side, rapid activity is (Shock) electrogram compared to the nearfield (Ventricular)
seen on all three channels with both an atrial (AF) and a ven- electrogram. This is in contrast to the reverse pattern seen
tricular arrhythmia (VF) detected per the marker channel. typically with diaphragmatic myopotentials.
What else occurs as a result of this rapid, sharp activity as In this case the diagnosis of inappropriate shocks due to
seen on the ventricular channel? At the far left of the bottom EMI was made with the EMI source traced to the patients
tracing, a VF episode is declared. Therapy is then delivered automobile alternator. As a result, the patient was instructed
with the remainder of the tracing at the bottom of the page to get no closer than 2 ft from his cars alternator as recom-
showing the post-shock activity on all three channels. There mended by the manufacturer. Clearly, all patients receiving
appears to be persistence or recurrence of the rapid activity ICDs should be counseled about activities and equipment in
(180ms) on both the atrial and ventricular channels despite the environment that may result in EMI.

K.A. Ellenbogen ()
Cardiac Electrophysiology and Pacing, Department of Cardiology,
VCU School of Medicine, Richmond, VA 23298-0053, USA
e-mail: kellenbogen@pol.net

A. Natale et al. (eds.), Cardiac Electrophysiology, 389


DOI: 10.1007/978-1-84996-390-9_97, Springer-Verlag London Limited 2011
390 K.A. Ellenbogen and R. Bolanos

Fig.97.1 Atrial, ventricular and shock electrograms during stored episode

Bibliography Yerra L, Reddy PC. Effects of electromagnetic interference on implanted


cardiac devices and their management. Cardiol Rev. 2007;15(6):
304-309.
Sweesy MW, Holland JL, Smith KW. Electromagnetic interference in
cardiac rhythm management devices. AACN Clin Issues. 2004;
15(3):391-403.
Case 98

Kenneth A. Ellenbogen

Case Summary Case Discussion

A 70-year-old man with a history of an ischemic cardiomyo- Review of the telemetry strip (Fig.98.1) reveals sinus rhythm
pathy and ventricular tachycardia was found to have pacer at approximately 80bpm with all QRS complex morpholo-
spikes on the QRS while on telemetry after elective hip gies appearing narrow and of normal duration. Beginning
replacement. The patient has a Medtronic GemDRTM dual with the eighth QRS complex, there are two closely spaced
chamber ICD. The telemetry strip is shown (Fig.98.1). What spikes evident concurrent with the initial upslope of the QRS
is the underlying rhythm? There are two successive pacer which do not result in a visible depolarization on the surface
spikes just after the initiation of the QRS on beats 8, 10, 12, leads. The device settings reveal DDD programming with
and 14. Is this normal pacemaker function? respective sensed and paced AV delays of 160 and 190ms.
Interrogation of the device revealed the following bradycar- The delay between the first and second spikes on the telem-
dia settings (Fig.98.2). Note the sensed and paced AV delays etry strip is 120ms. This is normal pacemaker function as a
are 160 and 190ms, respectively. Measurement of the interval result of undersensing of P waves on the atrial channel. The
separating the two spikes on the telemetry strip shows an inter- appearance of a second closely spaced pacer spike usually
val of 120ms. Next the surface lead II, atrial and ventricular within 80120 ms after an atrial paced depolarization is
marker channels, and ventricular EGMs are shown (Fig.98.3). called Safety PacingTM and is designed to eliminate cross talk
Here one sees that the two closely spaced spikes seen on telem- between the ventricle and the atrium with resultant oversens-
etry correlate with an A paced event followed by a V ing or inhibition of ventricular pacing and asystole in a
sensed EGM followed by a second spike right after the QRS patient who has complete heart block.
with an associated pacing artifact. Why does the second spike Cross talk can result from oversensing of atrial pacing
only appear after an A paced event? What is this called? stimuli on the ventricular lead due to the atrial pacing stim-
What management options can avoid this phenomenon? uli, a VPC, or other noise. Cross talk is more likely to occur

Fig.98.1 Telemetry strip


showing pacing spikes

K.A. Ellenbogen
Cardiac Electrophysiology and Pacing, Department of Cardiology,
VCU School of Medicine, Richmond, VA 23298-0053, USA
e-mail: kellenbogen@pol.net

A. Natale et al. (eds.), Cardiac Electrophysiology, 391


DOI: 10.1007/978-1-84996-390-9_98, Springer-Verlag London Limited 2011
392 K.A. Ellenbogen

Fig.98.2 Pacemaker Modes/rates Atrial lead


programmed parameters
Mode DDD Amplitude 4V
Mode switch On Pulse width 0.6 ms
A. Detect rate 176 bpm Sensitivity 0.45 mV
Lower rate 70 ppm Pace blanking 240 ms
Upper tracking rate 120 ppm
Upper sensor rate 105 ppm Ventricular lead
Amplitude 3V
A-V Intervals Pulse width 0.4 ms
Paced AV 190 ms Sensitivity 0.3 mV
Sensed AV 160 ms Pace blanking 200 ms
Rate adaptive AV On
Refractory
Start rate 60 bpm
Stop rate 150 bpm PVARP 310 ms
Minimum paced AV 140 ms PVAB 150 ms
Minimum sensed AV 30 ms

Fig.98.3 Surface ECG, marker


channel and Ventricular EGM
Case 98 393

Fig.98.4 Surface ECG, marker channel and Ventricular EGM after programming changes made

with high programmed atrial outputs, unipolar atrial pacing patients rate is faster than the lower pacing rate by about
(e.g., not possible in defibrillators), and with high pro- 10bpm. As a result, if the VV interval is short enough, the
grammed ventricular sensitivity (e.g., low values). Any ven- device will not undersense the P wave initiating the series of
tricular sensed event after atrial pacing falling in the AV events resulting in Safety PacingTM.
interval starting from the end of the ventricular blanking To correct this problem, the atrial sensitivity can be
period out to 110 ms (cross talk window) will result in increased so that P waves are sensed appropriately eliminat-
ventricular Safety PacingTM as this AV interval is deemed to ing the inappropriate atrial pacing. In this case, the lower rate
be nonphysiologic by the device. In this case, the ventricu- was increased to force atrial pacing and capture (Fig.98.4)
lar sensed event falling in the cross talk window is a con- with the AV delays programmed to minimize the likelihood
ducted QRS that is preceded by an inappropriate atrial pacing of the intrinsic QRS falling in the cross talk window and thus
spike as a result of undersensing of the P wave by the atrial abolishing the Safety PacingTM.
lead. The phenomenon does not occur on every beat as the

Case 99

Amin Al-Ahmad and Paul J. Wang

Case Summary Case Discussion

A 19-year-old man with a history of familial dilated cardio- Evaluation of the interval plot (Fig. 99.1) reveals that the
myopathy, congestive heart failure, and a biventricular ventricular rate has been approximately 330ms and is dis-
implantable cardioverter defibrillator has an episode of syn- sociated from the slower atrial rate indicating that ventricular
cope followed by a shock. tachycardia has been ongoing for at least 16s prior to device
The device is a Medtronic InSync ICD 7272. The pacing detection. This ventricular tachycardia is the likely cause of
lower rate limit is set to 60 beats/min and the upper rate limit the syncope in this patient with heart failure. The interval
is set to 140 beats/min. Pacing thresholds, sensing, and plot also shows an acceleration of the ventricular rate which
impedance were all unchanged from prior and within accept- results in detection and ultimate therapy with a 33-J shock.
able limits. The device is set to a fast VT zone via VF with a Evaluation of the stored electrograms (Fig.99.2) of this
rate cutoff of 240 ms, the VF zone is set to 280 ms. The episode show that the patient remains in ventricular tachycar-
device is programmed to deliver a single sequence of ATP dia at 330ms throughout the duration of the episode; however
prior to shock in the FVT zone. The episode interval plot that double counting of each ventricular tachycardia beat resulted
resulted in syncope and shock is shown in Fig.99.1. in detection in the VF zone and VF therapy delivery.
What is the cause of the syncope? And why does the While it is unclear what causes the double counting, it is
patient receive a shock? possible that a widening of the QRS during VT may result

VT/VF Episode #11 Report


ICD Model: InSync ICD 7272 Serial Number: PJP 234201S Date of Visit: Aug 16, 2007

ID# Date/Time Type V. Cycle Last Rx Success Duration


11 Jun 01 00:20:40 VF 160 ms VF Rx 1 Yes 22 s

V-V A-A VF = 280 ms FVT = 240 ms


Interval (ms)
33.0 J
1,800
1,500
1,200
900
600
400
200

Fig.99.1 Device interval plot


showing the episode that resulted 20 15 10 5 0 5 10 15 20 25
in syncope Time (s) [0 = Detection]

A. Al-Ahmad(*) and P.J. Wang


School of Medicine, Stanford University, 300 Pasteur Drive, H-2146,
Stanford, CA 94305, USA
e-mail: aalahmad@stanford.edu; paul.j.wang@stanford.edu

A. Natale et al. (eds.), Cardiac Electrophysiology, 395


DOI: 10.1007/978-1-84996-390-9_99, Springer-Verlag London Limited 2011
396 A. Al-Ahmad and P.J. Wang

Fig.99.2 Device electrograms showing VT with a cycle length of 330ms that is double counted
Case 99 397

in this phenomenon. In either case, it is worth noting Reprogramming the device to a lower rate cutoff or add-
that the VT was slower than the device rate cutoff and ing a VT zone would be appropriate. In addition, the addition
would not have resulted in therapy had it not been double of ATP may also be useful as it may reduce the likelihood of
counted. shock.

Case 100

Kenneth A. Ellenbogen

Case Summary Case Discussion

A 56-year-old man with a history of ischemic cardiomyopa- The nonsustained episodes seen on the arrhythmia logbook
thy presents to clinic for a routine 3-month evaluation of his are all due to the same cause, device oversensing of noise on
ICD. He underwent implantation of a Guidant PrizmTM 2 DR the ventricular channel. The noise is nearly constant across
Defibrillator after sustained monomorphic VT was elicited systole and diastole and is of regular amplitude. EMI can
during electrophysiology study. On questioning, the patient have this pattern, but its presence on only one channel makes
denies receiving any shocks but does report a few episodes of EMI less likely the culprit. Diaphragmatic myopotentials are
dizziness lasting several seconds. classically seen predominantly on the RV channel particu-
His device is programmed DDD 60120 beats per minute larly if the lead is positioned in the inferior apex of the right
(bpm) with a VT zone >180 and VF >200 bpm. Interrogation ventricle. The EGMs due to oversensing of myopotentials
of the device reveals several nonsustained episodes of rapid tend to be more varied in amplitude and frequency with often
ventricular rates on the device arrhythmia logbook respiratory variability. Further interrogation of the device
(Fig.100.1) with one event categorized as VF with an asso- revealed high impedance on the RV lead pointing to a lead
ciated diverted shock. The atrial, RV (nearfield), and shock fracture as the underlying problem.
(farfield) electrograms at the initiation of this event are The patients symptoms were likely due to the profound
shown (Fig.100.2). At the top of the figure, one sees marked bradycardia caused by inhibition of ventricular pacing from
bradycardia with no ventricular pacing despite the pro- the oversensing of the noise on the RV lead. Pacing was
grammed lower rate limit of 60bpm. On the bottom of the intermittently seen as evidence by the VP-Ns seen on the
figure, there are several ventricular sensed events falling marker channel due to a feature of the device where if there
within the VT and VF zones on the RV EGM with no cor- is continuous noise during the noise window of the device
relating EGMs on the shock electrogram. The sharp, nearly (VN markers on the marker channel), the device will pace at
constant activity being sensed on the RV channel is most the lower rate limit to prevent asystole from oversensing
likely due to what? What is the likely cause of the patients noise.
reported presyncope. What feature in this device guards In this instance, the oversensing nearly resulted in deliv-
against absolute inhibition of pacing in this case? ery of a shock after the device declared a VF episode (bottom
As a result of the oversensing of the noise on the RV chan- left, Fig.100.2), but the therapy was diverted during redetec-
nel, an episode is declared by the device. At the top left hand tion (Fig.100.3). This particular device requires 6/10 beats
side of Fig.100.3, the capacitor begins to charge, but ulti- in the tachycardia zone during redetection to proceed with
mately the shock is aborted. Why did this occur? therapy.

K.A. Ellenbogen
Cardiac Electrophysiology and Pacing, Department of Cardiology,
VCU School of Medicine, 980053, Richmond, VA 23298-0053, USA
e-mail: kellenbogen@pol.net

A. Natale et al. (eds.), Cardiac Electrophysiology, 399


DOI: 10.1007/978-1-84996-390-9_100, Springer-Verlag London Limited 2011
400 K.A. Ellenbogen

Fig.100.1 Device arrhythmia


Guidant VENTAK PRIZM 2 DR
logbook

Arrhythmia Logbook Report

Rate A
V
bpm Therapy/ Stab F
Episode Date/Time Type > Ons
Duration ms i
zone A
b
34 05 - DEC - 03 09 : 58 ATR 137 00 : 16 m : s
33 04 - DEC - 03 19 : 45 ATR 253 00 : 07 m : s
32 04 - DEC - 03 19 : 45 ATR 150 00 : 08 m : s
31 04 - DEC - 03 16 : 51 ATR 122 00 : 05 m : s
30 27 - NOV - 03 20 : 11 Spont 138 Nonsustained F N/R O 50%
29 10 - NOV - 03 19 : 32 Spont 132 Nonsustained F N/R O 56%
28 01 - OCT - 03 20 : 19 Spont 174 Nonsustained F N/R O 59%
27 09 - SEP - 03 15 : 03 ATR 100 00 : 05 m : s
26 27 - JUL - 03 19 : 05 Spont 130 Nonsustained F N/R O 56%
25 15 - JUL - 03 21 : 25 Spont 164 Nonsustained F N/R O 59%
24 14 - JUL - 03 20 : 02 Spont 176 Nonsustained F N/R O 34%
23 13 - APR - 03 13 : 39 ATR 104 00 : 14 m : s
22 13 - JAN - 03 20 : 54 Spont 169 Nonsustained F N/R O 59%
21 07 - JAN - 03 19 : 54 Spont 143 Nonsustained F N/R O 66%
20 31 - DEC - 02 19 : 44 Spont VF 185 Diverted T 219 O 59%
19 29 - DEC - 02 20 : 34 Spont 118 Nonsustained F N/R O 47%
18 29 - DEC - 02 08 : 18 ATR 93 00 : 06 m : s
17 23 - DEC - 02 22 : 44 Spont 155 Nonsustained F N/R O 72%
16 25 - NOV - 02 00 : 22 PMT 120
15 24 - NOV - 02 23 : 20 PMT 120
14 24 - NOV - 02 22 : 28 PMT 120
13 24 - NOV - 02 22 : 18 PMT 120
12 24 - NOV - 02 20 : 46 PMT 120
11 24 - NOV - 02 07 : 07 PMT 120
10 24 - NOV - 02 06 : 45 PMT 120
9 24 - NOV - 02 03 : 15 PMT 120
8 22 - NOV - 02 04 : 35 PMT 120
7 22 - NOV - 02 04 : 01 PMT 120
6 15 - OCT - 02 19 : 45 Spont 155 Nonsustained F N/R O 72%
5 14 - MAY - 02 11 : 06 ATR 105 00 : 06 m : s
4 20 - JAN - 02 14 : 58 ATR 89 00 : 07 m : s
3 08 - AUG - 01 10 : 49 Induce VF 255 17 J T 41 O N/R
2 08 - AUG - 01 10 : 41 Induce VF 316 11 J, 17J T 54 O N/R
1 08 - AUG - 01 10 : 35 Induce VF 245 31J T 96 O N/R

End of Report
Case 100 401

Fig.100.2 Atrial, ventricular and shock electrograms during onset of stored episode
402 K.A. Ellenbogen

Fig.100.3 Atrial, ventricular and shock electrograms during stored episode. Note, therapy diverted
Case 101

Amin Al-Ahmad and Paul J. Wang

Case Summary conducted atrial flutter, in which case an atrial flutter abla-
tion would be potentially helpful. Or is this ventricular
tachycardia?
A 68-year-old man with coronary artery disease, congestive
In this case we do not have an intrinsic electrogram in
heart failure, and ICD is admitted with multiple ICD shocks
sinus rhythm (or atrial flutter) to compare the shock mor-
for an ablation procedure. The device, a Boston Scientific
phology with that of the rapid ventricular rate. While con-
Vitality HE, is interrogated. Atrial and ventricular lead
ducted atrial flutter is possible, it is very unlikely as we would
parameters are within acceptable limits and are unchanged
not expect a patient who is in ventricular pacing during atrial
from prior device testing. An example of a representative
flutter to suddenly begin to rapidly conduct. Indeed, further
episode is shown in Fig.101.1. His device is set with a VT
history reveals that the patient is pacemaker dependent. Thus,
zone at 165 bpm and a VF zone at 200 bpm. He is pro-
this episode represents VT.
grammed to receive two ATP trains followed by shock in the
Programming the device to add more ATP or to change
VT zone, and to maximum energy (41 J) shocks in the VF
the ATP to be more aggressive may be helpful, although this
zone. Would an atrial flutter ablation result in a reduction in
may result in a higher risk of inducing VF. In addition, he has
the number of shocks?
not been responding to ATP and had been receiving multiple
shocks despite ATP. Antiarrhythmic medications may also
play a role in the management of this condition. In this
Case Discussion patient, a VT ablation resulted in a significant reduction of
spontaneous VT. Atrial flutter ablation was also performed at
that time.
Examination of the stored electrogram reveals atrial flutter
with ventricular pacing followed by an acceleration of the
ventricular rate. Is this acceleration of the ventricular rate

A. Al-Ahmad(*) and P.J. Wang


School of Medicine, Stanford University, 300 Pasteur Drive, H-2146,
Stanford CA 94305, USA
e-mail: aalahmad@stanford.edu; paul.j.wang@stanford.edu

A. Natale et al. (eds.), Cardiac Electrophysiology, 403


DOI: 10.1007/978-1-84996-390-9_101, Springer-Verlag London Limited 2011
404 A. Al-Ahmad and P.J. Wang

Fig.101.1 The shock electrogram showing a typical episode resulting in shock


Case 102

Kenneth A. Ellenbogen

Case Summary differential diagnosis for the tachyarrhythmia falling in the


VT zone? Why does the device characterize the arrhythmia
as VT?
A 72-year-old man with a history of congestive heart failure
Once the device commits to therapy, ATP is delivered
due to non-ischemic cardiomyopathy and an implantable
with the third ATP resulting in termination of the tachycardia
defibrillator presents to clinic for evaluation of palpitations
(Fig.102.2).
and presyncope. The patient denies having received any
shocks since his device was implanted 9 months ago. Prior
interrogation of his device showed normal function except
for atrial lead sensing issues. The atrial sensitivity was pro- Case Discussion
grammed to 0.9mV.
Interrogation of his Medtronic GemTM DR 7271 reveals
In this case, the third ATP sequence resulted in termination
several VT/VF episodes and no shocks were delivered.
of the tachyarrhythmia with return to normal sinus rhythm
Device settings are as follows:
(NSR) with some PVCs. Review of the sensed atrial rate
Mode DDD
(marker channel) during detection (Fig. 102.1) shows no
clear correlation with the ventricular rate with clearly more
Lower rate limit/upper rate limit 60/120
Vs than As. Yet on the atrial channel the atrial EGM shows
VT zone (rate>180) ATP3 (burst), 21J1, 31J4
an atrial rate concordant with the ventricular rate as sensed
VF zone (rate>210bpm) 31J5 on the ventricular lead. Here there are rapid atrial EGMs that
The following atrial and ventricular EGMs with the cor- are at the same rate as the ventricular EGMs, but are not
responding marker channels are shown (Fig.102.1) during sensed by the device as evidenced by their absence on the
detection of a tachyarrhythmia in the VT zone. What is the marker channel along with slower atrial EGMS falling at the

Fig.102.1 Stored EGM showing the atrial and ventricular EGM, the marker channel and intervals at the onset of tachycardia event

K.A. Ellenbogen
Cardiac Electrophysiology and Pacing, Department of Cardiology,
VCU School of Medicine, Richmond, VA 23298-0053, USA
e-mail: kellenbogen@pol.net

A. Natale et al. (eds.), Cardiac Electrophysiology, 405


DOI: 10.1007/978-1-84996-390-9_102, Springer-Verlag London Limited 2011
406 K.A. Ellenbogen

Fig.102.2 Stored EGM showing ATP terminating the tachycardia

sinus bradycardia rate which are sensed and evident on the and an atrial EGM displaying an atrial signal plus far-field
marker channel. R waves that are appropriately not sensed as evident on the
Based on these observations, the arrhythmia mechanism marker channel (although far-field R waves are confus-
differential diagnoses include SVT with a rapid ventricular ingly visible on the marker channel). The device correctly
response with atrial undersensing, ventricular tachycardia classified the arrhythmia as VT based on the ventricular
with VA dissociation (appropriate A sensing), and less likely EGM and V>A. Additionally, this is an example of far-
ventricular tachycardia with VA conduction and atrial under- field and near-field electrograms. On the atrial channel,
sensing of the retrograde As. VT with 1:1 VA conduction the near-field EGM is the sharp signal and this is sensed
and atrial undersensing is less likely as the sensed atrial rate correctly by the device as atrial activity. The other electro-
appears to march through at a regular rate independent of the gram on the atrial channel is far field, and it represents ven-
ventricular rate. If there was 1:1 VA conduction, the atrial tricular activity sensed in the atrium. It is a far-field EGM
rate would be dictated solely by the ventricular rate. as it has a low frequency, non-sharp appearance consis-
SVT with one to one conduction is less likely as the tent with a sensed signal from a further away source. It is
atrial sensed events march out regularly independent of the appropriately not sensed on the atrial channel, but certainly
ventricular rate. This event is an example of VT with VA is confusing when one first looks at the recorded strip
dissociation, appropriate atrial sensing during sinus rhythm, shown here.
Case 103

Amin Al-Ahmad and Paul J. Wang

Case Summary in the shock? Why does the patient still receive a shock
despite termination of the arrhythmia?
A 58-year-old male with a history of congestive heart failure
and an ICD comes to clinic after receiving an ICD shock
while mowing the lawn. The patient was not symptomatic
during the episode. His left ventricular ejection fraction is Case Discussion
25% and he is being treated with appropriate medications
including an angiotensin converting enzyme inhibitor and a Examination of the interval plot reveals that the patient is
beta-blocker. initially tachycardic with a heart rate of 133 bpm prior to
Evaluation of his Medtronic EnTrust ICD shows lead detection (Fig.103.1). The electrogram (Fig.103.2) is con-
thresholds, sensing, and impedance in an acceptable range sistent with sinus tachycardia. The heart rate then accelerates
and similar to prior device testing. The device is programmed to approximately 300ms (200bpm). The electrogram during
with a VT zone set to 400ms (150bpm) and a VF zone set this 1:1 tachycardia is similar to that of the electrogram prior
to 320 ms (188 bpm). Interrogation of the episode that to rate acceleration. This suggests that this rhythm accelera-
resulted in shock reveals a 1:1 tachycardia with a cycle length tion is more consistent with a supraventricular tachycardia
of 300ms (200bpm) that spontaneously terminates prior to (SVT) such as atrial tachycardia with 1:1 conduction, rather
shock (Figs.103.1 and 103.2). What is the rhythm that results than ventricular tachycardia with retrograde 1:1 conduction.

ATP Time Duration Avg bpm Max bpm Activity at


Type Shocks Success ID# Date
Seq hh:mm hh:mm:ss A/V A/V Onset
VF 0 35J Yes 21 08-Dec-2006 23:53 :15 214/214 ---/--- Rest
VV AA VF = 320 ms VT = 400 ms
Detection Term.
Interval (ms) 34.3 J
1,500
1,200
900
600

400

200

Fig.103.1 Interval plot showing


a 1:1 tachycardia that results in 20 15 10 5 0 5 10 15 20
shock Time (s)

A. Al-Ahmad(*) and P.J. Wang


School of Medicine, Stanford University, 300 Pasteur Drive, H-2146,
Stanford, CA 94305, USA
e-mail: aalahmad@stanford.edu; paul.j.wang@stanford.edu

A. Natale et al. (eds.), Cardiac Electrophysiology, 407


DOI: 10.1007/978-1-84996-390-9_103, Springer-Verlag London Limited 2011
408 A. Al-Ahmad and P.J. Wang

Fig.103.2 Episode electrograms and marker channels


Case 103 409

Despite the fact that this SVT spontaneously terminates this is possible under detection of VT. Another option may
and the heart rate decreases to 133 bpm, the patient still be to increase the number of intervals needed to detect VF
receives a shock. To understand this we need to understand from 12/16 to 18/24; however this may cause delay in ther-
the reconfirmation algorithm in this device. As the device apy. In this case it was determined that the patient was not
capacitor charge ends (CE on the marker channel), the device taking his beta-blockers. Resumption of beta-blocker therapy
attempts to reconfirm prior to discharge. To reconfirm this would potentially decrease the maximum rates during exer-
device requires two intervals to be detected after the charge tion and would decrease the likelihood of this type of inap-
end that are faster than the VT interval+60ms, in this case propriate shock without any device programming changes.
460ms. In this case the intervals are 450ms; thus although This case illustrates the importance of understanding the
these intervals are lower than the VT rate cutoff, they still detection and reconfirmation algorithms of ICDs.
result in therapy delivery.
Reprogramming of the device to increase the VT rate cut-
off may eliminate this problem; however the risk of doing

Case 104

Kenneth A. Ellenbogen

Case Summary fibrillation with a rapid and irregular ventricular rhythm.


This relationship is evident at the beginning of the interval
plot (Fig.104.2).
A 65-year-old woman with a history of syncope and conges-
Subsequently, the ventricular rate abruptly drops into the
tive heart failure due to nonischemic cardiomyopathy pres-
VF zone twice as self-terminating rapid VT/VF episodes fol-
ents to the ER after reportedly receiving several shocks from
lowed by a third VF episode that is sustained long enough to
her ICD. Telemetry in the ER shows normal sinus rhythm
not only be detected, but to result in delivery of a 34.5-J
with some single premature ventricular contractions (PVCs).
shock. An alternative but less likely explanation is that these
Interrogation of her Medtronic MaximoTM DR 7278
are brief episodes of a rapid and regular ventricular response
reveals that the device had delivered three shocks for a tach-
to the atrial arrhythmia. This is less likely because the
yarrhythmia in the VF zone. Device settings are as follows:
patients atrial rate is extremely rapid (although monomor-
phic in appearance), and thus more consistent with atrial
Mode DDD fibrillation, than an organized atrial tachycardia. Additionally,
Lower rate limit/upper rate limit 60/120 there are subtle changes in the near field, or ventricular
VT zone (rate>166bpm) ATP1 (burst), 21J1, 31J4 sensed electrogram suggesting this arrhythmia is more likely
VF zone (rate>188bpm) 31J5 a ventricular tachyarrhythmia (a rapid SVT with aberration
cannot be excluded). This first shock terminates the atrial
arrhythmia (Fig.104.3) and transiently terminates ventricu-
The atrial and ventricular EGMs along with the marker
lar tachycardia, as well. The ventricular tachycardia quickly
channel are shown (Fig.104.1) leading up to the delivery of
reinitiates, with the rate almost identical to the rate of the
the first shock. The device categorizes the ventricular arrhyth-
rapid and regular tachycardia seen prior to the first shock,
mia as VF and the atrial arrhythmia as an SVT (double tachy-
and requiring a second 34.5-J shock that is also unsuccessful.
cardia). Is this correct and why?
In Fig.104.4, we see the successful termination of the ven-
The interval plot is shown (Fig.104.2). What effect does
tricular tachyarrhythmia with the third 34.5 J shock. It is
the first 34.5-J shock have on the double tachycardia?
worth noting here, that the atrial rhythm is now sinus and we
clearly have a ventricular tachycardia with AV dissociation
prior to shock delivery. The ventricular electrogram recorded
from the rate sensing electrodes here is also different from
Case Discussion
what the rate sensing electrogram recorded during the atrial
tachyarrhythmia prior to the first shock during what we
At the initiation of the event, a regular rapid arrhythmia of thought were short bursts of ventricular tachycardia, even
190 ms is evident on the atrial channel with a coexisting though the rates are similar. We have no simple explanation
irregular ventricular rhythm well below the VT zone cutoff for this discrepancy.
of 166 bpm. The most likely initial diagnosis is atrial This case is most likely an example of appropriate classifi-
cation by a device of a double tachycardia with termination of
the atrial arrhythmia by the first shock followed by subsequent
termination of the ventricular tachycardia by the third shock.
K.A. Ellenbogen (*) In patients with VT/VF, dual tachycardias are quite common
Cardiac Electrophysiology and Pacing,
with a prevalence as high as 20%. Additionally, when therapy
Department of Cardiology, VCU School of Medicine,
P.O. Box980053, Richmond, VA, USA, 23298-0053 delivered for the VT/VF fails to convert the atrial arrhythmia,
e-mail: kellenbogen@pol.net the time to the next VT/VF therapy is significantly shorter than

A. Natale et al. (eds.), Cardiac Electrophysiology, 411


DOI: 10.1007/978-1-84996-390-9_104, Springer-Verlag London Limited 2011
412 K.A. Ellenbogen

Fig.104.1 Stored EGM showing the atrial and ventricular EGM, the marker channel and intervals at the onset of the episode
Case 104 413

Fig.104.2 Episode interval plot

AFL
terminates

VT continues

Fig.104.3 Stored EGM showing the first shock


414 K.A. Ellenbogen

Fig.104.4 Stored EGM showing final successful shock

when both arrhythmias are terminated by the initial shock. It dual-chamber implantable cardioverter/defibrillators capable of
has been shown that implanting dual chamber devices with atrial therapy delivery: the REVERT-AF Study. Europace. July
2007;9(7):534-539.
atrial therapy capabilities in those with standard ICD indica- Stein KM, Euler DE, Mehra R, etal. Do atrial tachyarrhythmias beget
tions does not reduce the incidence of VT/VF episodes. ventricular tachyarrhythmias in defibrillator recipients? J Am Coll
Cardiol. July 2002;40(2):335-340.

Bibliography

Gradaus R, Seidl K, Korte T, etal. Reduction of ventricular tachyar-


rhythmia by treatment of atrial fibrillation in ICD patients with
Case 105

Kenneth A. Ellenbogen and Rod Bolanos

Case Summary Interrogation of his device (Fig.105.1) reveals an episode


lasting 2 min during which six episodes of VF(rate<330ms)
A 59-year-old man presented to a local Emergency Room were detected resulting in a total of six shocks. RV lead
after receiving six shocks from his defibrillator. He reported impedance (tip to can) was stable and within normal limits.
feeling palpitations and lightheadedness followed by six con- Review of the short interval counter showed multiple brief
secutive shocks. He denied frank syncope. The patient had a nonsustained episodes with intervals <140ms.
single chamber Medtronic ICD and a MedtronicTM 6936 defi- The rate/sense and marker channels are shown for the first
brillator lead implanted approximately 2 years ago. episode (Fig. 105.2). Is there evidence of normal device

Fig.105.1 Stored EGM showing the atrial and ventricular EGM, the marker channel and intervals at the onset of the episode

K.A. Ellenbogen ()
Department of Cardiology, VCU School of Medicine, 980053,
Richmond, VA 23298-0053, USA
e-mail: kellenbogen@pol.net

A. Natale et al. (eds.), Cardiac Electrophysiology, 415


DOI: 10.1007/978-1-84996-390-9_105, Springer-Verlag London Limited 2011
416 K.A. Ellenbogen and R. Bolanos

Fig.105.2 Episode interval plot

function in this case? Immediately after delivery of the initial initial shock. In this case, the etiology lies in an inherent
shock, the device detects a second episode also in the VF flaw in the design of the ICD lead utilized. The MDTTM
zone (Fig.105.3). What is remarkable about the marker chan- 6936 lead is a coaxial lead utilizing polyurethane polymers
nel intervals in this episode compared to the first episode? for insulation. This ICD leads with polyurethane insulation
The patient goes on to receive four more successive shocks is prone to breakdown of the insulation as a result of metal
for events falling in the VF zone. The RV EGM is shown ion oxidation (MIO). This appears to be especially likely to
(Fig.105.4) along with the marker channel for one of these be noted after a high energy shock. Almost all ICD leads
episodes. What is potentially being over sensed, and what are today are designed with a multilumen approach where
the steps that may be taken to avoid the problem? Why were the conductors and coils are no longer wrapped around
shortly coupled intervals only seen after the first shock? each other, but placed individually inside the lumen of
thelead.
Polyurethane lead failure of the MDTTM 6936 is charac-
terized by oversensing of ventricular events typically follow-
Case Discussion ing a shock possibly due to a noncontact defect between the
pace/sense ring conductor and the right ventricular high volt-
In this case, a patient received multiple shocks over a short age conductor. This oversensing of electrical noise after a
period of time. The first episode (Fig.105.2) shows appropriate shock is a signature presentation for failure of the 6936
device function with detection and termination of an event in lead. Monitoring of the ring to coil impedance and the device
the VF zone. This is an episode of rapid ventricular tachycar- short interval counter may allow early detection of MIO-
dia. Appropriate therapy is delivered for this episode. However, induced damage prior to frank lead failure allowing for elec-
this episode is quickly followed by five subsequent shocks. tive replacement of the ICD lead.
The second episode detected (Fig.105.3) immediately after the Finally, management of a patient who is receiving multi-
delivery of the first shock is characterized by several short cou- ple inappropriate shocks should be mentioned. The etiolo-
pled sensed intervals that are nonphysiologic (<150ms) and is gies for multiple shocks can include SVT, lead fracture, or
representative of the sequence of events associated with the rarely recurrent EMI or even incessant, rapidly recurring VT
remaining shocks. Inspection of the RV EGM (Fig. 105.4) or VF. Immediate deactivation of the device is necessary
shows an intermittently fractionated electrogram correlating to after multiple recurrent (inappropriate or appropriate) shocks,
the short coupled events on the marker channel. and this can be accomplished in most devices by placing a
The presence of very short coupled intervals points to a doughnut shaped magnet over the device. In the vast major-
nonphysiologic cause for the five shocks that followed the ity of devices, this will suspend VT/VF therapy delivery, but
Case 105 417

Fig.105.3 Stored EGM showing the first shock

Fig.105.4 Stored EGM showing the first shock

permit continued bradycardia pacing. The patient must Bibliography


remain on telemetry so appropriate therapy for VT/VF may
be delivered when necessary, or other appropriate therapies Ellenbogen KA, Wood MA, Shepard RK, etal. Detection and manage-
initiated (e.g., treatment for cardiac ischemia, antiarrhythmic ment of an implantable cardioverter defibrillator lead failure: inci-
drug infusion, etc.). dence and clinical implications. J Am Coll Cardiol. 2003;41:73-80.

Case 106

Kenneth A. Ellenbogen

Case Summary Episode 81

Elapsed Time Date Time Type


A 73-year-old man presents to device clinic after receiving a 04-JAN-00 21:18 Spontaneous
shock from his defibrillator. He has a past medical history of Initial Detection VT Zone
an ischemic cardiomyopathy with LVEF of 2025% and Pre-Attempt Avg A Rate 101 bpm
symptomatic bradycardia. He had recently undergone implan- Pre-Attempt Avg V Rate 202 bpm
Measured Onset 38 %. 240 ms
tation of a dual chamber ICD with a CPITM 0154 active fixa- Measured Stability 58 ms
tion defibrillator lead. On questioning, the patient denies any A Fib FALSE
lightheadedness, chest pain, or palpitations around the time V Rate > A Rate TRUE
of the shock. Interrogation of his ICD (Fig. 106.1) reveals 00:04 Attempt 1 VT Shock 1
Therapy Delivered Diverted -Reconfirm
two episodes detected by the device in the VT zone with ther-
Charge Time 2.9 sec
apy delivered for the second episode but not the first. Shocking Impedance
The atrial, right ventricular (RV), and shock electrograms Post-Attempt Avg A Rate 101 bpm
(Fig.106.2) are shown during the initiation of the first epi- Post-Attempt Avg V Rate 101 bpm

sode and during device charging (Fig.106.3). Why are the Redetection VT Zone
Pre-Attempt Avg A Rate 99 bpm
EGMs on the RV lead most consistent with nonphysiologic
Pre-Attempt Avg V Rate 199 bpm
noise, and why is therapy not delivered for the first episode? Measured Stability 115 ms
After therapy is diverted, the device detects a second episode A Fib OFF
for which therapy is delivered (Fig.106.4). The post-shock V Rate > A Rate OFF

electrograms are remarkable for what findings on the atrial 00:20 Attempt 2 VT Shock 1
and ventricular channels? What is one possible cause for the Therapy Delivered 17 J. Biphasic
Charge Time 0.3 sec
nonphysiologic noise? Shocking Impedance 49
Post-Attempt Avg A Rate bpm
Post-Attempt Avg V Rate 127 bpm
00:52 End of Episode
Case Discussion
Fig.106.1 Details of stored device therapy episode
Review of the atrial, RV, and shock electrograms in Fig.106.2
shows sharp, well-demarcated sensed events only on the RV
electrogram channel (near field channel) and not on the typically one sees an EGM characterized by multiple, closely
shock electrogram. The sensed EGMs are in addition cou- coupled signals of varying amplitude. Noise due to diaphrag-
pled to the preceding surface QRS with varying intervals. matic myopotentials typically cannot be reproduced with
Oversensing of diaphragmatic myopotentials can be seen on isometric exercise or with changes in breathing, etc. Further
the RV lead due to its proximity to the diaphragm; but questioning did not reveal any clear exposure to potential
sources of EMI. At this point some other type of nonphysi-
ologic signal is the most likely culprit.
The first shock was diverted during redetection as two out
of three beats failed to fall in the VT zone (Fig. 106.3).
K.A. Ellenbogen (*)
Subsequently, oversensing is seen again only on the RV
Department of Cardiology, VCU School of Medicine,
P.O. Box980053, Richmond, VA 23298-0053, USA channel, but this time the shock is committed after the first
e-mail: kellenbogen@pol.net therapy and delivered (Fig.106.4).

A. Natale et al. (eds.), Cardiac Electrophysiology, 419


DOI: 10.1007/978-1-84996-390-9_106, Springer-Verlag London Limited 2011
420 K.A. Ellenbogen

Fig.106.2 Atrial, ventricular and shock electrograms during initiation of episode

Fig.106.3 Atrial, ventricular and shock electrograms during device charging


Case 106 421

Fig.106.4 Atrial, ventricular and shock electrograms during redetected episode

The fact that the device was recently implanted is an ability of the lead to defibrillate. The lead was ultimately
important clue, particularly since the noise was seen on only redesigned with elimination of the problem. This patient was
one of the channels. In this case, one must consider a possi- brought back to the lab and a new rate sense lead was
ble problem with the lead pin/device header connection. implanted.
External manipulation of the device header did not elicit the
noise. Lead impedance trends were stable and not elevated,
and thus not suggestive of a possible loose set screw.
The source of the problem in this case was an inherent Bibliography
defect in the ICD lead involving the helix stabilizing post. If
the lead was not securely attached to the myocardium, the Doshi RN, Goodman J, Naik AM, Shivkumar K, Chen PS, Peter CT.
Initial experience with an active-fixation defibrillation electrode
binding post could move resulting in the nonphysiologic and the presence of nonphysiological sensing. Pacing Clin
chatter on the rate sense channel (RV). Sometimes this is Electrophysiol. Dec 2001;24(12):1713-1720.
evident at the time of implant and can be corrected by retract- Mela T, Ngarmukos T, Rosenthal L, Mittleman R. Inappropriate ICD
ing the helix and repositioning the lead in another part of the therapy due to lead-related noise in an active fixation ICD lead.
JInvasive Cardiol. May 2001;13(5):406-408.
myocardium. This defect in lead design did not affect the

Case 107

Paul A. Friedman and Charles D. Swerdlow

Case Summary Case Discussion

A 62-year-old woman had previously received an implant- The tracing demonstrating ventricular pacing that results in
able defibrillator due to a history of ventricular tachycardia. atrial capture (note third and fourth complex from the left).
She presented following a shock that was not preceded The VP (ventricular pacing) on the marker channel identi-
by symptoms. A fracture of the right ventricular 6936 fies the delivery of pacing pulses. In a single chamber defi-
Transvene lead was identified as causing noise that led to brillator placed for sudden death prevention, the pace/sense
inappropriate shock. Due to this, the 6936 lead was aban- lead is always placed in the ventricle. The differential diag-
doned, the proximal electrodes capped, and a new Sprint nosis for ventricular pacing leading to atrial capture includes
6945 lead placed in the right ventricular apex, taking care to reversal of leads in the header (in dual chamber devices) or
avoid mechanical contact with the abandoned lead. The dislodgement of the ventricular lead into the atrium. In this
morning after implantation of the new lead, a pacing thresh- case, the ventricular lead has dislodged to the atrium, near
old test is performed (Fig.107.1). On the basis of this trac- the tricuspid annulus. Note that for the third complex, ven-
ing, what is your diagnosis? tricular pacing results in atrial capture (seen on the surface

Fig.107.1 Tracings
recorded the morning
after insertion of a
new lead. See text for
details

P.A. Friedman(*)
Division of Cardiovascular Diseases and Department of Internal
Medicine, Mayo Clinic, 200 First Street SW, Rochester,
MN, USA 55905
e-mail: pfriedman@mayo.edu
C.D. Swerdlow
Division of Cardiology, David Geffen School of Medicine,
Cedars-Sinai Medical Center, Los Angeles, CA, USA
e-mail: swerdlow@ucla.edu

A. Natale et al. (eds.), Cardiac Electrophysiology, 423


DOI: 10.1007/978-1-84996-390-9_107, Springer-Verlag London Limited 2011
424 P.A. Friedman and C.D. Swerdlow

Fig.107.2 Chest X-ray


following new lead
implanation. Details in text

ECG). The captured P-wave conducts to the ventricle, and


the resultant QRS complex is sensed by the ICD (note the
VS marker beneath the third QRS complex).
The chest x-ray (Fig.107.2) confirms dislodgment of the
new lead. The left panel show the anteroposterior view, and
the right panel the lateral view. The arrow in the left and right
panels indicate the new, dislodged lead. Note that in the lat-
eral view it has retracted proximally (farther from the ster-
num), toward the tricuspid annulus. The position of the right
ventricular apex in both views is approximated by the non-
dislodged, nonfunctioning chronic lead. Fig.107.3 True bipolar and integrated lead designs
The middle panel in a zoomed-up view of the distal potion
of the two leads shown in the lateral x-ray. Two important
lead characteristics are observed. First, note that the lead
conductor proximal to the defibrillation coil (short arrow) is electrode. Since in the integrated lead pacing and sensing
a thin longitudinal conductor in the new, dislodged lead occur between the distal tip and the large surface area defi-
Sprint lead. In contrast, the chronic, nonfunctioning lead brillation coil, a large bipole is creating permitting sensing of
is composed of coaxial conductors that appear thicker radio- ventricular complexes by the ICD despite the lead tips posi-
graphically. Coaxial defibrillation leads with circumferential tion in the atrium (as confirmed by atrial capture during pac-
filers (i.e., the Medtronic Transvene lead) have an increased ing). Prior to lead revision, the patient developed a tachycardia
risk of fracture and warrant close follow up. Second, the new inappropriately detected in the VF zone due to inappropriate
lead is an integrated bipolar lead. As shown in Fig. 107.3, sensing of atrial and ventricular signals. Characteristic inter-
sensing and pacing occur between a dedicated tip and ring val plots and electrograms are shown in Fig. 107.4. The
electrode in true bipolar leads. In integrated bipolar leads, interval plot demonstrates a railroad track pattern, caused
sensing and pacing occur between a tip electrode and the by alternating short and long intervals, and leads to overde-
large distal coil, which also functions as a defibrillation tection of a slow tachycardia as VF.
Case 107 425

VT/VF Episode #4 Report Page 1


ID# Date/Time Type V. Cycle Last Rx Success Duration
4 Jun 14 09:14:33 VF 220 ms VF Rx 1 Yes 3.2 min Railroad track
VV VF = 320 ms VT = 400 ms
alternating intervals
VV Interval (ms)
seen in oversensing
1,800 32.2 J of cardiac signals
1,500
1,200
900
600

400

200

20 15 10 5 0 5 10 15 20 25 30 35
Time (s) [0 = Detection]

Far field

Near field

Fig.107.4 Recordings of a tachycardia event following new lead placement. Tpo left panel shows the R-R intervals preceding and following
detection. Bottom right panel shows electrograms recorded during tachycardia

Case 108

Paul A. Friedman and Charles D. Swerdlow

Case Summary The jugular venous distension measures 8cm, lungs are clear,
gallops are absent, the abdomen is unremarkable, and trace
A 73-year-old man with coronary artery disease, hyperten- lower extremity edema is noted. Figure108.1 shows the VT/
sion, and depressed ventricular function presents after receiv- VF episode list. A representative episode is shown in
ing multiple shocks from his single chamber ICD. On physical Fig.108.2. Based on this, what is your diagnosis? Do you rec-
examination, the blood pressure is 148/73, with heart rate 86. ommend device reprogramming or surgical system revision?

P.A. Friedman(*)
Division of Cardiovascular Diseases and Department of Internal
Medicine, Mayo Clinic, 200 First Street SW, Rochester, MN,
USA, 55905
e-mail: pfriedman@mayo.edu
C.D. Swerdlow
Division of Cardiology, David Geffen School of Medicine,
Cedars-Sinai Medical Center, Los Angeles, CA, USA
e-mail: swerdlow@ucla.edu

A. Natale et al. (eds.), Cardiac Electrophysiology, 427


DOI: 10.1007/978-1-84996-390-9_108, Springer-Verlag London Limited 2011
428 P.A. Friedman and C.D. Swerdlow

Fig.108.1 VT/VF Episode List


Case 108 429

Fig.108.2 Sample V-V VF = 320 ms


episode. See text for details VV Interval (ms)
1,800
1,500
1,200
900
600

400

200

30 25 20 15 10 5 0 5 * * * * * *
Time (s) [0 = Detection]

Case Discussion as shown by the corresponding FS (for fib sense) mark-


ers. Sensed events that do not augment a tachycardia counter
The VT/VF episode list (Fig.108.1) confirms the delivery of are labeled VS. Thus, the shorter cycle-length events
multiple ICD shocks, and suggests malfunction by the pres- depicted on the interval plot are due to T-wave oversensing.
ence of untreated VF episodes with duration up to 3.3 min. A sufficient number of such events are present to result in
The top panel in Fig.108.2 illustrates the interval plot lead- repetitive inappropriate detection and shock.
ing to detection. The V-V intervals fall into two groups: one As shown in Fig.108.3, T-wave oversensing can be clas-
at 600ms, and a second at approximately 250300ms. The sified into one of three types. Post-pacing T-wave oversens-
electrogram tracing in the bottom panel shows (from top to ing (left panel) results in pauses in a paced rhythm or ATP
bottom) the far-field electrogram (labeled HVA to HVB), the since the sensed event resets the bradycardia (or ATP) escape
near-field electrogram (Vtip to Vring) and the Marker interval. It does not lead to inappropriate shock unless it
Annotations and V-V intervals. The far-field elctrogram, occurs during reconfimation. When T-wave oversensing
which approximates the surface ECG, shows the presence of occurs during spontaneous rhythm, as in the present case,
normal sinus rhythm. In the near-field recording, ventricular inappropriate therapies may be delivered. If the R-waves are
electrograms (sharp spikes that align with the far-field QRS) small relative to the size of the T-wave (as in the present
highly variable amplitudes and large T-wave electrograms example), reprogramming is not likely to result in acceptable
are present. The T-wave electrograms at times exceed the sensing, and lead revision is required. If the R-waves are
amplitude of the R-wave electrograms. The changing pat- larger than the oversensed T-waves, reprogramming may
terns and amplitudes are suggestive of respiratory variation correct the problem. In this patient, a separate pace-sense
in lead function. The T waves are sufficiently large to be lead (a Novus 4076 bipolar pacing lead) was placed in the
counted as sensed events, resulting in T-wave oversensing. right ventricle since the was no evidence of impaired lead
Due to the close coupling interval between a QRS complex integrity and the pre-existing lead resulted in a satisfactory
in its T-wave, oversensed T-waves augment the VF counter, DFT. Figure108.4 shows how reprogramming can eliminate
430 P.A. Friedman and C.D. Swerdlow

T-wave oversensing with the R-wave is large relative to the sensing is dynamic, and is minimized upon R wave sensing
T-wave. On the left is shown the maximum/minimum sensi- to followed by progressively increasing sensitivity. This
tivity settings. At a setting of 0.3 mV, T waves are over- minimized the risk of T-wave oversensing by decreasing sen-
sensed. Decreasing sensitivity to 0.45 mV eliminates sitivity immediately follow an R-wave, but prevents under-
oversensing. A setting of 0.45mV indicates that any signal sensing the small amplitude electrograms of ventricular
with smaller amplitude will not be sensed. Ventricular fibrillation by progressively increasing sensitivity.

Fig.108.3 Classification of
T wave oversensing

Reference

1. Swerdlow CD, Friedman PA. Advanced ICD troubleshooting: Part I


PACE1. Dec 2005;28(12):13221346.

Fig.108.4 Effect of reprogramming sensitivity on T wave oversens-


ing in Medtronic defibrillators
Case 109

Paul A. Friedman and Charles D. Swerdlow

Case Summary receives a shock 1 month after system revision. He transmits


ICD system information from home via the Carelink
The 73-year-old man of case 109 (who underwent addition remote monitoring network. The stored episode is shown in
of a separate pace-sense lead due to T-wave oversensing) Fig.109.1. Based on your review of this tracing, what is your

Fig.109.1 Electrograms transmitted remotely via careLinkTM network. See text for details

P.A. Friedman(*)
Division of Cardiovascular Diseases and Department of Internal
Medicine, Mayo Clinic, 200 First Street SW, Rochester,
MN 55905, USA
e-mail: pfriedman@mayo.edu
C.D. Swerdlow
Division of Cardiology, David Geffen School of Medicine,
Cedars-Sinai Medical Center, Los Angeles, CA, USA
e-mail: swerdlow@ucla.edu

A. Natale et al. (eds.), Cardiac Electrophysiology, 431


DOI: 10.1007/978-1-84996-390-9_109, Springer-Verlag London Limited 2011
432 P.A. Friedman and C.D. Swerdlow

diagnosis? How do you counsel the patient? Should he pres- fibrillation. The baseline artifact suggests that signal is highly
ent immediately, follow up in 23days, or in 23months? amplified, consistent with a small amplitude electrogram.
Note that after the shock, the far-field electrogram shows a
wide, idioventricular rhythm and the near-field tracing shows
Case Discussion termination of fibrillation, and a rate that is slower than the
far-field ventricular rate. The only explanation for these find-
From top to bottom in Fig. 109.1 are shown the far-field ings is dislodgement of the separate pace-sense lead from the
electrogram (HVA-HVB), the near-field electrogram (Vtip- ventricle into the atrium. The dislodgment may have caused
Vring), the Marker Channel, and the V-V intervals. The the atrial fibrillation, resulting in the inappropriate shock.
far-field electrograms are recorded between the distal defi- Since the far-field signals are recorded from the non-
brillation coil in the right ventricle and the ICD pulse genera- dislodged chronic defibrillation lead, they continue to accu-
tor shell. Due to the large surface area of the electrodes and rately show ventricular activation; the near-field signal from
their wide spacing, this approximates the surface ECG the dislodged bipolar lead records atrial activity. Figure109.2
recording. The far-field tracings demonstrate an irrgular nar- shows a real-time electrogram post-shock, transmitted
row complex tachycardia with cycle length approximately remotely. It is now clear that the pace-sense lead had dis-
400ms (two boxes), and minor variations in the QRS mor- lodged to the atrium and is recording atrial electrograms.
phology. This is consistent with atrial fibrillation, with the The baseline artifact again confirms the small amplitude of
minor QRS morphology changes due to longitudinal disso- the signals (possibly due to poor contact) and the high ampli-
ciation. The near-field electrogram, which is recorded fier gain. The patient was advised to present expeditiously
between the tip and ring of the separate pace-sense lead that for lead revision in order to restore appropriate device
had been placed one month before this event, shows function.

Fig.109.2 Real-time electrogram obtained at the time of CareLinkTM transmission. Details in the text
Case 110

Paul A. Friedman and Charles D. Swerdlow

Case Summary between 400 and 500 to 230 ms, consistent with a change from
variable AV conduction near 2:11:1 AV conduction. This is
seen in the interval plot in Fig.110.3. The ventricular cycle
A 68-year-old man with ischemic cardiomyopathy (NYHA
length is shorter than the programmed SVT limit of 240 ms, so
Class II, left-ventricular ejection fraction 26%) underwent
no SVT-VT discriminators apply. The amplitude of the ven-
implantation of a prophylactic dual-chamber ICD (Medtronic
tricular electrogram is greater during tachycardia than at base-
EnTrust D154ATG) 2 years ago. His medications include
line (note truncated peaks of ventricular electrograms denoted
lisinopril 20 mg daily, carvedilol 12.5 mg daily, simvastatin
by arrow). Allowing for the change in amplitude, the morphol-
40 ms daily, and aspirin 325 mg daily. The ICD was pro-
ogies of the tachycardia and sinus ventricular electrograms are
grammed for single zone detection as shown in the upper
nearly identical. During ventricular antitachycardia pacing
panel of Fig.110.1. Six months ago, he had presyncope fol-
(during charging) the atrial rhythm persists unaltered. These
lowed by a shock. ICD interrogation showed ineffective anti-
observations all indicate that the 1:1 tachycardia is 1:1 AV
tachycardia pacing followed by a successful shock for VT
conduction of atrial flutter rather than 1:1 VA conduction of
with AV dissociation at cycle length 240 ms.
VT. The shock terminates atrial flutter (Fig.110.2). By way of
The patient was in his usual state of good health until he
comparison (Fig.110.4) shows an actual VT episode.
received a shock while shaving with his electric razor on
The Cardiac Compass trend (Fig. 110.3) shows that the
December 9, 2007. At interrogation on December 20, 2007, the
patient developed persistent asymptomatic atrial fibrillation or
baseline rhythm is shown in Fig.110.1, lower panel (HVB=right
flutter in mid October. The maximum ventricular rate exceeded
ventricular coil). Figure110.2 shows the stored electrograms
the VF detection rate once in late October and approached it
from the episode, which was detected as VF. The upper panel
twice in November. Internet-based alerts triggered by persistent
in Fig.110.3 shows the corresponding interval plot. The lower
atrial fibrillation/flutter or rapid ventricular rate would have
panel shows a portion of the Cardiac Compass trend.
alerted the physician both to the presence of atrial fibrillation/
What is the diagnosis? Was this shock preventable? What
flutter and the need for better rate control. The physician would
steps should be taken now?
then have been able to initiate therapy with rate control and/or
rhythm control as well as anticoagulation. This would have
reduced the risk of cardioversion of persistent atrial fibrillation/
flutter without anticoagulation, as happened on December 9.
Case Discussion
At this time, anticoagulation with warfarin should be initi-
ated. Therapeutic options include increasing the dose of beta
The stored electrogram in Fig. 110.2 shows a regular atrial blockers for rate control and consideration of pharmacological,
rhythm at cycle length 230 ms consistent with atrial flutter. or ablative strategies for rhythm control. In some dual-chamber
The ventricular cycle length abruptly decreases from a range ICDs, atrial antitachycardia pacing would also be an option.
Reprogramming the ICD would likely be of minimal
value because of the short cycle length of 230 ms during 1:1
P.A. Friedman(*)
Division of Cardiovascular Diseases and Department of Internal conduction of atrial flutter. Programming ON the 1:1 SVT
Medicine, Mayo Clinic, 200 First Street SW, Rochester, rejection rule, which rejects SVT with near simultaneous
MN 55905, USA atrial and ventricular activation, might be of limited value.
e-mail: pfriedman@mayo.edu But the sinus tachycardia rejection rule would not prevent
C.D. Swerdlow therapy of 1:1 atrial flutter because it requires a gradual
Division of Cardiology, David Geffen School of Medicine, change of ventricular cycle lengths within an expected range
Cedars-Sinai Medical Center, Los Angeles, CA, USA
e-mail: swerdlow@ucla.edu
to diagnose sinus tachycardia.

A. Natale et al. (eds.), Cardiac Electrophysiology, 433


DOI: 10.1007/978-1-84996-390-9_110, Springer-Verlag London Limited 2011
434 P.A. Friedman and C.D. Swerdlow

Fig.110.1 Upper panel:


programmed settings. Lower
panel: baseline rhythm
Case 110 435

Fig.110.2 Stored electrograms from episode of ICD-detected VF


436 P.A. Friedman and C.D. Swerdlow

Fig.110.3 Upper panel: interval plot. Lower panel: Cardiac Compass trend plot for the last 80 weeks showing the total number of hours per
day of atrial fibrillation/flutter in the upper panel
Case 110 437

Fig.110.4 Stored electrograms for an episode of actual VT. Not that the atria (top tracing) are in sinus rhythm during this episode, and that the
ventricular morphology (second tracing) is markedly different than the sinus or atrial flutter morphology

Case 111

Paul A. Friedman and Charles D. Swerdlow

Case Summary SVT-VT discrimination does not apply in the VF zone of


St. Jude ICDs or those made by most other manufacturers.
SVT conducted at shorter cycle length than the shortest
A 62-year-old man with ischemic cardiomyopathy (NYHA
one at which SVT discriminators apply accounted for about
Class II, left-ventricular ejection fraction 29%) underwent
half of all inappropriate therapy of SVT.1
implantation of a prophylactic dual-chamber ICD (Atlas DR
There are several diagnostic and therapeutic consider-
V-242) 2 years ago. During a febrile illness, he received six
ations: Increasing the beta blocker dose will reduce the like-
shocks without premonitory symptoms.
lihood of rapid and rapidly conducted atrial arrhythmia. The
The ICD is programmed for nominal two detection zones,
atrial cycle length of 260 ms is probably too fast for sinus
Tach A (VT) at 400ms and Fib (VF) at 300ms, and nominal
tachycardia, and additional pharmacological or ablative ther-
duration for detection (12 intervals). The ventricular electro-
apy for the atrial arrhythmia may be considered. Atrial anti-
gram morphology discriminator (MD) is programmed
tachycardia pacing therapy might be considered in other
ON. SVT is diagnosed if five of eight ventricular electro-
ICDs. However, since this arrhythmia occurred only once
grams in a sliding window have a match threshold of or
during a febrile illness, increasing beta blockers may be a
higher 60%. The stored electrogram is shown in Fig.111.1.
reasonable first step.
What is the diagnosis? What steps should be taken now?
The ICD should also be reprogrammed: The Fib interval
should be shortened to a short cycle length (e.g., 240ms). A
second VT zone (Tach B) may be programmed. But whether
or not two or three zone programming is used, the morphol-
Case Discussion ogy discriminator should be programmed throughout the VT
zone(s).
The upper panel shows a tachycardia with 1:1 AV relation- The potential risk is that VT with 1:1 VA conduction might
ship and AV interval shorter than the VA interval with cycle occur at a short cycle length and match the sinus morphology.
length 360ms. Almost all morphology match scores exceed The risk in a multicenter study was 10%2 and is probably
60%, and VT therapy is inhibited at the I= markers in left lower in this case because of the primary prevention indica-
side of upper panel and near middle of lower panel. Toward tion. It may be reduced by programming a sustained-duration
the middle of the lower panel, the atrial cycle length shortens override feature (SVT discrimination timeout in St. Jude
gradually from about 360 to 260ms, associated with a cor- ICDs). These features deliver therapy if an arrhythmia satis-
responding decrease in ventricular cycle length, but no fies the ventricular rate criterion for a long programmed dura-
change in morphology match score. VF is detected at the tion even if discriminators indicate SVT. The premise is that
D marker despite an excellent morphology match because VT will continue to satisfy the rate criterion for the pro-
grammed duration while the ventricular rate during transient
sinus tachycardia or atrial fibrillation will decrease below the
VT rate boundary. The limitation is delivery of inappropriate
P.A. Friedman(*)
Division of Cardiovascular Diseases and Department therapy when SVT exceeds the programmed duration, which
of Internal Medicine, Mayo Clinic, 200 First Street SW, Rochester, occurs in ~10% of SVTs at at 3 min.1
MN 55905, USA
e-mail: pfriedman@mayo.edu
C.D. Swerdlow
Division of Cardiology, David Geffen School of Medicine,
Cedars-Sinai Medical Center, Los Angeles, CA, USA
e-mail: swerdlow@ucla.edu

A. Natale et al. (eds.), Cardiac Electrophysiology, 439


DOI: 10.1007/978-1-84996-390-9_111, Springer-Verlag London Limited 2011
440 P.A. Friedman and C.D. Swerdlow

Fig. 111.1 Right atrial (RA) and right ventricular (RA) true bipolar respectively. Check mark above match scores indicates match at or above
electrograms are shown. The top row of numerical values between the the 60% threshold. X marks indicate match scores less than 60%. ICD
electrograms shows the morphology algorithm percent match score. The rhythm diagnoses are highlighted with squares. I= marker denotes inhibi-
middle and lower numbers show atrial and ventricular cycle length, tion of therapy in the rate branch with equal atrial and ventricular rates

References 2. Glikson M, Swerdlow CD, Gurevitz OT, etal. Optimal combination


of discriminators for differentiating ventricular from supraventricu-
lar tachycardia by dual-chamber defibrillators. J Cardiovasc Electr
1. Klein G, Manolis A, Viskin S, etal. Clinical performance of wave- ophysiol. July 2005;16(7):732-739.
let morphology discrimination algorithm in a worldwide single
chamber ICD population (abstract). Circulation. 2003;110:III-345.
Case 112

Paul A. Friedman and Charles D. Swerdlow

Case Summary alignment. An identical tachycardia electrogram may be


acquired at a sufficiently fast rate that Automatic Sensitivity
Control does not reach its most sensitive value at the onset of
A 57-year-old woman with ischemic cardiomyopathy and a
the R-wave. If this occurs, the low-amplitude peak at the
dual-chamber St. Jude Atlas II DR V-265 ICD presented
onset of the ventricular electrogram may not be used for
with inappropriate shocks for sinus tachycardia detected as
alignment. If identical template and tachycardia electrograms
VT (Fig. 112.1, upper panel). The ventricular electrogram
are then compared, their representations in the morphology
morphology discriminator (MD)1,2 was programmed ON.
algorithm may not match.
This discriminator diagnoses SVT and withholds VT therapy
In this case, most electrograms during inappropriately
if five of eight ventricular electrograms in a sliding window
detected VT (e.g., fourth electrogram denoted by arrowhead)
have a match score of or higher 60% with the template elec-
have morphology similar to that recorded in sinus rhythm in
trogram. The stored electrogram is shown in Fig.112.1. The
the lower left panel (arrowhead). The low-amplitude peak at
real-time electrogram in sinus rhythm, shown in the lower
the onset is probably sensed and used for alignment in sinus
left panel shows a morphology match score of 100% on sinus
rhythm. In sinus tachycardia, Automatic Sensitivity Control
beats. The lower right panel shows a real-time electrogram
does not reach minimum. Thus the low-amplitude peak is not
recorded during rapid atrial pacing.
sensed and not used for alignment.
What is the problem? What are the possible solutions?
The lower right panel identifies a second source of mor-
phology mismatch, rate-related bundle branch aberrancy.
The surface ECG shows a wider complex during atrial pac-
ing than in sinus rhythm. The ventricular electrogram mor-
Case Discussion phology is distinctly different from that in sinus rhythm,
but similar to that of the 11th beat in the top panel (asterisk)
This case illustrates two limitations of morphology templates and the final electrograms before the shock (asterisks, sec-
acquired in slow sinus rhythm.3 Accurate alignment of sinus ond row).
template and tachycardia electrogram in the St. Jude mor- In patients who have dual-chamber ICDs and intact AV
phology algorithm is sensitive to the value of the sensing conduction, this problem may be solved by acquiring or veri-
threshold at the onset of the ventricular electrogram, as deter- fying the template during atrial pacing at a cycle length
mined by Automatic Sensitivity Control. If a template shorter than the VT detection interval. Automatic template
electrogram is acquired at the most sensitive setting of updating should be disabled. In single-chamber ICDs, solu-
Automatic Sensitivity Control (either because of a slow sinus tions to the first alignment problems may include altering the
rate or after a ventricular paced beat), a low amplitude peak minimum sensitivity, Threshold Start, or Threshold Delay, or
at the onset of the ventricular electrogram may be used for acquiring a template during exercise testing.

P.A. Friedman (*)


Division of Cardiovascular Diseases and Department
of Internal Medicine, Mayo Clinic, 200 First Street SW, Rochester,
MN 55905, USA
e-mail: pfriedman@mayo.edu
C.D. Swerdlow
Division of Cardiology, David Geffen School of Medicine,
Cedars-Sinai Medical Center, Los Angeles, CA, USA
e-mail: swerdlow@ucla.edu

A. Natale et al. (eds.), Cardiac Electrophysiology, 441


DOI: 10.1007/978-1-84996-390-9_112, Springer-Verlag London Limited 2011
442 P.A. Friedman and C.D. Swerdlow

Fig. 112.1 Top panel shows stored electrogram from an episode of percent match score. The middle and lower numbers show atrial and
ICD-detected VT. Lower left panel shows real-time electrogram of ventricular cycle length, respectively. Check mark above match scores
sinus rhythm indicating template match in of 100% for sinus beats and indicates match at or above the 60% threshold. X marks indicate match
0% for premature ventricular complexes. Lower right panel shows real- scores less than 60%. ICD rhythm diagnoses are highlighted with
time electrogram in atrial pacing. In each panel, the top row of numeri- squares. D= marker denotes detection of VT in the rate branch with
cal values between the electrograms shows the morphology algorithm equal atrial and ventricular rates

References 2. Boriani G, Biffi M, DallAcqua A, etal. Rhythm discrimination by


rate branch and QRS morphology in dual chamber implantable car-
dioverter defibrillators. Pacing Clin Electrophysiol. 2003;26(1 Pt 2):
1. Duru F, Bauersfeld U, Rahn-Schonbeck M, Candinas R. Morphology 466-470.
discriminator feature for enhanced ventricular tachycardia discrimi- 3. Swerdlow CD, Friedman PA. Advanced ICD troubleshooting: part I.
nation in implantable cardioverter defibrillators. Pacing Clin Pacing Clin Electrophysiol. 2005;28(12):1322-1346.
Electrophysiol. 2000;23(9):1365-1374.
Case 113

Paul A. Friedman and Charles D. Swerdlow

Case Summary indicates that in the 5 seconds before detection, the device
was sensing intervals shorter than the detection cutoff of
320ms, while absence of rapid complexes is confirmed by
A 64-year-old man with ischemic cardiomyopathy, refrac-
the surface ECG. A loose set screw, lead dislodgement, elec-
tory congestive heart failure (New York Heart Association
trode contact with an abandoned lead (if present), and lead
Class III), and chronic atrial fibrillation undergoes implan-
fracture could all result in early post-implant inappropriate
tion of an InSync Sentry 7299 CRT-D defibrillator. The
shock. The atrial impedance >2,500 W is not germane to the
morning after the implantation, he receives a shock while
shock; as seen in the VT/VF report, the shock was delivered
shaking his cardiologists hand. The hospital telemetry dur-
in the VF zone based solely on the rapid ventricular rates;
ing the handshake is shown in Fig. 113.1 (top panel), and
detection enhancements did not play a role. In this case, the
episode report is shown in Fig. 113.1 bottom panel.
atrial impedance is high due to absence of an atrial lead; an
Figure.113.2 shows lead impedance values (top panel) taken
atrial lead was not implanted due to chronic atrial fibrillation.
following the shock, and the stored electrograms of the event
The RV lead impedance is high and suggestive of loose set
(bottom panel). In order to identify the cause of the problem,
screw or fracture; typically, any value >2,000 W is abnormal.
detection is turned off, and various electrograms are record-
In the perioperative period, a loose set screw would be far
ing during a provocative maneuver (hand shake), as shown in
more likely than lead fracture to give rise to these findings.
Fig.113.3. Based on this information, what is your diagno-
The episode electrogram (Fig.113.2, bottom panel) shows a
sis? What is your recommended plan of action?
near-field electrogram (Vtip-Vring) that is oversaturated with
the type of noise characteristic of make-break contact artifact
seen in lead fracture or loose set screw. The presence of V-V
Case Discussion intervals in the nonphysiologic range below 150 ms also
favor fracture/loose set-screw. These short intervals lead to
detection of VF (FD, the fourth marker). Following detec-
Hospital telemetry at the time of the shock shows atrial fibril-
tion, only VS (ventricular sensed event) markers are pres-
lation with a ventricular rate around 100bpm, a shock arti-
ent, irrespective of the V-V interval. This is an idiosyncracy
fact, followed by a slightly accelerated rhythm with a wider
of the Medtronic system, which only displays VS markers
QRS complex (Fig.113.1). The delivery of a shock during a
once detection is met. The CE marker denotes charge end,
normal rhythm with heart rate below a tachycardia detec-
and CD charge delivery after a brief confirmation interval.
tion zone indicates a system-related malfunction. The wider
In Fig. 113.3, provocative maneuvers are performed with
rhythm following the shock is likely a brief accelerated idio-
detection turned off to further identify the malfunctioning com-
ventricular rhythm, although post-shock aberrancy cannot be
ponent. In each panel, electrograms using different sources are
excluded. The VT/VF episode report (Fig. 113.1, bottom)
recorded during a hand shake. Note that the only electrogram
that did not have artifact present during the handshake was the
P.A. Friedman(*) can-RV coil (top right panel). The RVtip-RVring (top left),
Division of Cardiovascular Diseases and Department RVtip-RV distal coil (bottom left), and RVtip-LVtip (bottom
of Internal Medicine, Mayo Clinic, 200 First Street SW, Rochester, right panel) all contain artifact (sharp, nonphysiologic deflec-
MN 55905, USA
e-mail: pfriedman@mayo.edu
tions). Thus, any electrogram that incorporates the RV tip has a
noisy signal during handshaking, indicating a loose setscrew or
C.D. Swerdlow
lead fracture involving the RV tip conductor. At surgical revi-
Division of Cardiology, David Geffen School of Medicine,
Cedars-Sinai Medical Center, Los Angeles, CA, USA sion, reinserting the RV distal electrode connector in the header
e-mail: swerdlow@ucla.edu and tightening the screw eliminated the malfunction.

A. Natale et al. (eds.), Cardiac Electrophysiology, 443


DOI: 10.1007/978-1-84996-390-9_113, Springer-Verlag London Limited 2011
444 P.A. Friedman and C.D. Swerdlow

May 02, 2006 09:50:41


ICD Model: InSync Sentry 7299 9998 Software Version 2.0
Serial Number: PRK 124359H Copyright Medtronic, Inc. 2004
VT/VF Episode #12 Report Page 1
ID# Date/Time Type V. Cycle Last Rx Success Duration
12 May 02 10:20:49 VF 140 ms VF Rx 1 Yes 10 s

VV AA VF = 320 ms FVT = 260 ms


Interval (ms)
2,000 34.8 J
1,700
1,400
1,100
800

600

400

200

40 35 30 25 20 15 10 5 0 5 10 15
Time (s) [0 = Detection]

Fig.113.1 Top panel: surface telemetry recorded at the time of shock delivered during a handshake. Bottom panel: Episode report from the same
episode

Lead Performance Atrial RV LV

EGM amplitude Not Taken 6.8 mV


pacing impedance >2,500 1,936 324
Defibrillation impedance 32
SVC impedance 40

Note 1: 035 J charge time not available.

Atip Aring

Fig.113.2 Interrogation Vtip Vring


following the shock. Top
panel: lead impedance values.
Bottom panel: stored
electrograms from the event
that triggered the shock
Case 113 445

Fig.113.3 Electrograms recorded during provactive maneuver (handshake) while detection is turned off

Case 114

Paul A. Friedman and Charles D. Swerdlow

Case Summary interfence is characterized by normal lead impedance val-


ues, noise throughout the cardiac cycle, and a greater
amplitude signal on the far-field than the near-field elec-
A 67-year-old man received a Guidant 1850 single chamber
trogram. The latter occurs since the far-field electrogram
defibrillator on an Endotak right ventricular lead for sud-
is recorded from larger electrodes with a greater interelec-
den death prevention in the setting of an ischemic cardio-
torde spacing than the near-field electrogram, resulting in
myopathy. He was using an auger when he received a shock
a larger antenna that is more susceptible to external
without antecedent symptoms. The ICD is interrogated and
signals. Integrated bipolar leads (which includes the
the episode electrograms shown in Fig.114.1. Lead imped-
Endotak) are more likely to record EMI than true bipo-
ance values were normal. What is your diagnosis, and
lar leads. See Case 108 for depiction of integrated and
management plan?
bipolar leads. Figure114.2 shows examples of oversens-
ing of extra-cardiac noise. The far-left panel shows a lead
fracture, characterized by saturated high frequency elec-
Case Discussion trograms that account for less than 10% of the cardiac
cycle. Artifact is seen on the near-field recording to a
The tracing shows (from top to bottom) the near-field ven- greater extent than the far-field recording. With a lead
tricular electrogram, far-field ventricular electrogram, and fracture, the lead impedance may be elevated; an imped-
markers and V-V intervals. Halfway through the top panel, ance>2,000 W is highly suggestive. The middle panel
high frequency noise becomes apparent on both the near- shows myopotential oversensing. The electrogram affected
field and far-field electrograms. The presence of multiple depends on the source; diaphragmatic oversensing affects
high frequency events/cardiac cycle that are unrelated to the near-field channel and may lead to inappropriate
the R-wave is characteristic of oversensing of extra- shocks. Pectoralis oversensing distorts morphology
cardiac signals. Extra-cardiac signals may be due to lead templates in some systems, leading to SVT-VT misclas-
fracture/loose set screw, myopotential oversensing, or sification with use of morphology-based detection
electromagnetic interference (EMI). Electormagnetic enhancements. The far right panel depicts EMI.

P.A. Friedman(*)
Division of Cardiovascular Diseases and Department
of Internal Medicine, Mayo Clinic, 200 First Street SW, Rochester,
MN 55905, USA
e-mail: pfriedman@mayo.edu
C.D. Swerdlow
Division of Cardiology, David Geffen School of Medicine,
Cedars-Sinai Medical Center, Los Angeles, CA, USA
e-mail: swerdlow@ucla.edu

A. Natale et al. (eds.), Cardiac Electrophysiology, 447


DOI: 10.1007/978-1-84996-390-9_114, Springer-Verlag London Limited 2011
448 P.A. Friedman and C.D. Swerdlow

Fig.114.1 Device interrogation following shock delivery


Case 114 449

Fig.114.2 Characteristic
electrograms recorded in the
setting of lead fracture
(left panel), myopotential
oversensing (middle panel),
and electromagnetic interference
(EMI, right panel)

Case 115

Paul A. Friedman and Charles D. Swerdlow

Case Summary What changes in bradycardia pacing parameters should be


made?
A 78-year-old man with ischemic cardiomyopathy (NYHA
Class IV, left-ventricular ejection fraction 14%) and renal fail-
ure underwent implantation of a prophylactic, cardiac-resyn- Case Discussion
chronization ICD in 2005. He had sinus rhythm with first
degree AV block and left bundle branch block. His heart failure
Figure115.1 shows an episode of monomorphic VT at cycle
improved to Class III and ejection fraction improved to 21%.
length 340 ms terminated by antitachycardia pacing. The
In May 2007, he saw his cardiologist for a routine follow-
post-therapy rhythm is a SVT with 1:1 AV association. The
up visit. ICD interrogation showed 96% ventricular pacing
pattern of atrial and ventricular timing at the end of the anti-
and no episodes of VT. ECG monitoring showed frequent
tachycardia pacing sequence indicates that the likely diagno-
ventricular tracking of atrial premature complexes, resulting
sis is atrial tachycardia with first degree AV block.
in asymptomatic rhythm irregularities. To reduce this track-
The rhythm preceding VT is sinus tachycardia at cycle
ing, bradycardia programming was changed to the settings
length 560 ms without cardiac-resynchronization pacing.
shown in left panel of Fig.115.1. In August 2007, he returned
The patients heart failure has worsened because of reduction
for a routine visit, reporting an increase in exertional dysp-
in cardiac resynchronization pacing, which generally needs
nea and reduction in exercise capacity. ICD interrogation
to be applied to more than 90% of QRS complexes for
showed 41% ventricular pacing and asymptomatic, device-
therapeutic effect. The root cause of insufficient cardiac-
detected VT, all terminated by antitachycardia pacing. The
resynchronization pacing is the long post-ventricular atrial
plasma natriuretic peptide concentration has increased from
refractory period (PVARP) of 500 ms, which was increased
165 pg/mL in May to 536 pg/mL. Figure115.1 (right panel)
in May to reduce tracking of atrial premature complexes.
shows a stored EGM of device-detected VT with onset iden-
This prevents tracking of the sinus P waves that fall within
tical to the seven other episodes. ICD programming for
the PVARP (VSAR interval 200270 ms) as the sinus cycle
detection and therapy of VT/VF is shown below:
length decreases.
Intervals Detection Therapy This episode and the other seven episodes of VT were
to detect interval (ms) preceded by an atrial-paced event, setting up safety pac-
VF 18/24 320 35 J Shock6 ing, which initiated VT. Assuming the activity sensor at 0,
Fast VT 18/24 240 ATP X1, 35 J Shock6 the device will pace at the lower rate (70 bpm), and thus
the VSAP interval=lower rate interval PAV~850130
VT 16 400 ATP X3, 35 J Shock6
ms=720 ms. After the VSVS interval shortens to 510 ms,
probably because of a premature ventricular complex, the
measured VSAP interval is 710 ms, indicating that the sen-
P.A. Friedman(*) sor was driving the pacing rate slightly faster than the
Division of Cardiovascular Diseases and Department lower rate of 70 bpm. Note also that ARAP interval is 300
of Internal Medicine, Mayo Clinic, 200 First Street SW, Rochester, ms, which means that the noncompetitive atrial pacing
MN 55905, USA
e-mail: pfriedman@mayo.edu feature (NCAP), designed to prevent pacing-induced atrial
fibrillation, withheld the Ap event until the NCAP timer
C.D. Swerdlow
Division of Cardiology, David Geffen School of Medicine,
expired. The conducted VS beat from the preceding AR P
Cedars-Sinai Medical Center, Los Angeles, CA, USA wave times in the cross-talk window after the AP event.
e-mail: swerdlow@ucla.edu This initiates Safety Pacing with an APVP interval of

A. Natale et al. (eds.), Cardiac Electrophysiology, 451


DOI: 10.1007/978-1-84996-390-9_115, Springer-Verlag London Limited 2011
452 P.A. Friedman and C.D. Swerdlow

Fig.115.1 Bradycardia pacing parameters and stored electrogram of VT terminated by antitachycardia pacing. See text for details

110 ms, and the resulting fusion beat. The fusion beat is the incidence of VT by preventing pacing-related
followed by a VS beat with V-V interval of 430 ms. This proarrhythmia.
completes a short-long-short sequence that initiates VT.
Without electrograms, we determine from the intervals
alone if this last beat is a premature ventricular beat or if
Reference
it is conducted from the NCAP Ap beat with a Ap-VS inter-
val of 480 ms. The latter explanation suggests proarrhyth-
1. Sweeney MO, Ruetz LL, Belk P, Mullen TJ, Johnson JW, Sheldon
mia related to specific features of bradycardia pacing (long
T. Bradycardia pacing-induced short-long-short sequences at the
PVARP and NCAP) and first degree AV block.1 The cor- onset of ventricular tachyarrhythmias: a possible mechanism of
rect action to increase percent of cardiac resynchroniza- proarrhythmia? J Am Coll Cardiol. 2007;50(7):614-622.
tion pacing is to shorten the PVARP. This may also reduce
Case 116

Paul A. Friedman and Charles D. Swerdlow

Case Summary undersensing during ventricular fibrillation.1 The interval


plot at lower left shows two unsuccessful attempts at anti-
tachycardia pacing denoted by arrowheads. The stored EGM
The 78-year-old man described in previous case 116 is fol-
at right shows, relatively slow, late-stage VF, probably with
lowed. No programming was performed at an August 2007
substantial undersensing. It is difficult to assess the magni-
visit. Instead, amiodarone therapy was initiated to prevent
tude of undersensing because the true-bipolar sensing EGM
asymptomatic VT requiring termination by antitachycardia
was not recorded.
pacing.
The most likely explanation is that amiodarone slowed
Six weeks later, on September 28, 2007, the patient devel-
the rate of VT so that it remained undetected. The patient
oped rapid palpitations after receiving bad news. Within a
became ischemic, and the rhythm degenerated into VF.
few minutes, he experienced dyspnea and his typical angina.
Antiarrhythmic drugs that slow the rate of VT below the pro-
He used nitroglycerin spray twice without relief. Twenty
grammed VT rate cut-off can prevent initial detection of VT2
minutes later he collapsed and died. Figure116.1 shows the
or divert appropriate therapy during reconfirmation.
data retrieved postmortem from an episode of ICD-detected
In this case, VT could likely have been prevented or
VT that was recorded about 20 min after he collapsed.
reduced in frequency by reprogramming the PVARP. Even in
What is the diagnosis? What steps could have been taken
the absence of proarrhythmia, infrequent asymptomatic VT
to prevent the patients death?
terminated by antitachycardia pacing may not require phar-
macological prophylaxis. Amiodarone often slows the VT
cycle length by 100 ms or more, and patients with advanced
Case Discussion heart failure may not tolerate even relatively slow VT. 2 When
therapy with amiodarone is initiated either for therapy of
The Flashback intervals in the top left panel show RR atrial or ventricular arrhythmias in a patient with known
intervals for approximately 11 min preceding ICD detection monomorphic VT, the VT detection interval should usually
of VT. The extreme RR variability is characteristic of be increased by 50 ms.

P.A. Friedman(*)
Division of Cardiovascular Diseases and Department
of Internal Medicine, Mayo Clinic, 200 First Street SW, Rochester,
MN 55905, USA
e-mail: pfriedman@mayo.edu
C.D. Swerdlow
Division of Cardiology, David Geffen School of Medicine,
Cedars-Sinai Medical Center, Los Angeles, CA, USA
e-mail: swerdlow@ucla.edu

A. Natale et al. (eds.), Cardiac Electrophysiology, 453


DOI: 10.1007/978-1-84996-390-9_116, Springer-Verlag London Limited 2011
454 P.A. Friedman and C.D. Swerdlow

Stored electrogram

RR Interval (ms) VT 400 ms


Flashback intervals 1,000
FVT 320 ms
VF 240 ms
800
600
400
200
0
600 400 200 0
Time (s) [0 = Detection]

Interval plot
VV AA VF = 320 ms FVT = 240 ms VT = 400 ms
Interval (ms)
2,000 Burst
1,700
1,400
1,100
800
600
400
200

40 30 20 10 0 10 20 30 * * *
Time (s) [0 = Detection]
Septermber 28, 2007

Fig.116.1 Flashback intervals, interval plot, and stored electrogram from ICD-detected VT. See text for details

References 2. Bansch D, Castrucci M, Bocker D, Breithardt G, Block M.


Ventricular tachycardias above the initially programmed tachycar-
dia detection interval in patients with implantable cardioverter-
1. Swerdlow CD, Friedman PA. Advanced ICD troubleshooting: part II. defibrillators: incidence, prediction and significance. J Am Coll
Pacing Clin Electrophysiol. 2006;29(1):70-96. Cardiol. 2000;36(2):557-565.
Case 117

Paul A. Friedman and Charles D. Swerdlow

Case Summary a Novus 5076 right atrial lead. While in hospital on teleme-
try, she develops recurrent episodes of pacing failure, ven-
tricular arrhythmias, and shocks (Fig. 117.1). The device
A 24-year-old woman with complex congenital heart dis-
status report is shown in Fig.117.2. In order to further trou-
ease, depressed systemic ventricular function, and conges-
bleshoot the device malfunction, a real-time electrogram and
tive heart failure previously received an InSync II Marquis
surface ECG are simultaneously recorded (Fig.117.3). What
7289 CRT-D utilizing a Medtronic Sprint Fidelis 6949 right
is the diagnosis? How is the problem corrected?
ventricular defibrillation lead, 4193 left ventricular lead, and

Fig.117.1 Telemetry
recordings obtained during
hospitalization. See text for
details

P.A. Friedman (*)


Division of Cardiovascular Diseases and Department
of Internal Medicine, Mayo Clinic, 200 First Street SW, Rochester,
MN 55905, USA
e-mail: pfriedman@mayo.edu
C.D. Swerdlow
Division of Cardiology, David Geffen School of Medicine,
Cedars-Sinai Medical Center, Los Angeles, CA, USA
e-mail: swerdlow@ucla.edu

A. Natale et al. (eds.), Cardiac Electrophysiology, 455


DOI: 10.1007/978-1-84996-390-9_117, Springer-Verlag London Limited 2011
456 P.A. Friedman and C.D. Swerdlow

Aug 08, 2007 09:43:11


Fig.117.2 ICD status report. ICD Model: InSync II Marquis DR 7289 9989 Software Version 2.0
Note impedance values. Serial Number: PRJ 620851S Copyright Medtronic, Inc. 2002
Discussion in text Status report Page 1
Last Interrogation: Aug 08, 2007 07:13:28

Battery voltage Lead impedance


(ERI = 2.62 V) Aug 08, 2007 03:00:03
Aug 08, 2007 07:13:26 A. Pacing 432
Voltage 2.88 V RV Pacing 560
Last capacitor formation LV Pacing 368
Mar 30, 2007 19:54:26 V. Defib 49
Charge time 7.96 s SVC Defib None
Energy 0.0 - 30.0 J EGM amplitude
Last charge Aug 08, 2007 05:26:38
Aug 08, 2007 05:27:42 P wave amplitude 1.4 mV
Charge time 4.51 s RVR-wave amplitude 5.0 mV
Energy 0.0 24.0 J Last high voltage rherapy
Sensing integrity counter Aug 08, 2007 05:08:31
(if>300 counts, check for sensing issues) Measure impedance 42
Since Aug 08, 2007 03:25:21 Delivered energy 24.1 J
120130 ms VV intervals 17 Vaveform Biphasic
Pathway B>AX

Case Discussion ventricular event is sensed during the cross-talk window, the
ventricular pacing stimulus follows shortly after its onset (due
The patient has striking episodes of failure of ventricular out- to the abbreviated AVI), avoiding pacing during repolarization
put (seen in Fig.117.1 and 117.3) leading to shortlongshort and proarrhythmia. In the present case, by its design, the 7289
intervals, which are proarrythmic and induce polymorphic CRT-ICD delivers safety pacing pulses only to the right ven-
ventricular arrhythmias (Fig.117.1). The tracing in Fig.117.3 tricular lead. Due to failure of the Fidelis lead in this patient,
is duplicated in Fig.117.4 with additional labels added. The output failure occurs, awnd the ventricle is not paced. Note
arrows indicate the paced atrial beats not followed by a QRS, that the other (captured) pacing pulses in Fig.117.4 are associ-
which lead to alternating V-V intervals. Note that the marker ated with the BV marker for biventricular pacing. It is likely
channel for these beats displays VS with two bars of differ- that only the left ventricular lead is capturing during pacing, so
ing heights (circled in Fig. 117.4). This double-bar marker that during safety pacing, which is delivered only to the non-
indicates safety pacing. Safety pacing prevents cross talk, function right ventricular lead, ironically, no ventricular cap-
which occurs when an atrial pacing event is sensed on the ven- ture occurs. Figure117.6 further corroborates this. Note that
tricular channel, inhibiting ventricular output. As shown in after an external shock is delivered to terminate ventricular
Fig.117.5 an atrial pacing event is followed by a ventricular tachycardia, biventricular pacing occurs that captures only the
blanking period, then a cross-talk window. Any event sensed LV (first arrow). Due to the width of the LV-only paced QRS,
during the cross talk window will result in delivery of a ven- the same ventricular complex is sensed on the RV lead, giving
tricular pacing stimulus with an abbreviated atrioventricular rise to an FS marker (second arrow), for fib sense event.
interval (AVI). Thus, if an atrial event is sensed on the ven- Thus, while the Fidelis fails to pace, it remains capable of
tricular channel during the cross-talk window, a pacing stimu- sensing. Also note that the impedance of the failed lead is not
lus is delivered after an abbreviated AVI that may be suboptimal elevated (Fig.117.2). Fidelis lead malfuncntion without sig-
hemodynamically, but preferable to asystole. If an intrinsic nificant increases in impedance have been described.
Case 117 457

Fig.117.3 Simultaneous
surface telemetry (top panel)
and device markers and
electrograms (bottom panel)

Fig.117.4 Simultaneous
surface and device recordings,
as in 118.3. Large arrows
indicated paced p waves without
subsequent QRS. Each such
event has a characteristic marker
(The first one is circled). Details
in text
458 P.A. Friedman and C.D. Swerdlow

Fig.117.5 Venricular sensing


following a paced atrial event.
Sensing during the cross talk
window will trigger a paced
ventricular event with an
abbreviated AV interval. Figure
courtesy of Dr. David Hayes

Fig.117.6 Episode from same


patient treated with shock. Note
the last complex is biventricu-
larly paced, but sensed on the RV
channel. See text for discussion
Case 118

Paul A. Friedman and Charles D. Swerdlow

Case Summary the right ventricular coil is measured between that coil and
the ICD can. The impedance for the superior vena cava elec-
trode is measured between that electrode and the right ven-
A 59-year-old man with ischemic cardiomyopathy had a
tricular coil, the high-voltage electrode of opposite polarity.
single-chamber Medtronic Entrust ICD and Sprint Fide
Simultaneous fluctuations in both measurements indicate
lis Model 6949 dual-coil true bipolar lead implanted over
intermittent conduction failure in the common portion of the
2 years ago after out-of-hospital VF. His NYHA Class is III
circuit that includes right ventricular coil. The differential
and left ventricular ejection fraction is 34%. After an episode
diagnosis includes isolated high-voltage conductor failure in
of monomorphic VT storm a year ago, he has been treated
the lead or failure of conductive contact in the header (e.g.,
with amiodarone 200mg per day to decrease the frequency
loose set screw).
of VT.
Recalled Fidelis leads have been implanted in more
He now presents for a routine follow-up visit. The pacing
patients than any other lead family.1 About 95% of Fidelis
threshold is 0.5V at 0.4ms, and the pacing lead impedance
lead failure involves one of the two pace-sense electrodes.
is stable at 560 W. One episode of device-detected VT is
Failures of pace-sense electrodes present with intermittent
stored in the ICD. The patient had no awareness of this epi-
high pacing impedance measurements and evidence of over-
sode, which occurred 6weeks ago. Figure118.1 shows the
sensing, including inappropriate shocks.2 See Fig. 118.3.
corresponding stored electrogram. Figure 118.2 shows
A modification of previous lead failure algorithm may assist
80-week trend plots for R wave amplitude and impedance
in identifying lead failures before patients receive inappro-
measurements corresponding to each-high voltage lead
priate shocks.3
which are displayed with each interrogation.
Isolated high-voltage failures represent only about 5% of
What is the diagnosis? What steps should be taken now?
reported Fidelis failures. But the time course in this case is
unusual for header-connector problems, which usually occur
in the first year after implant. So the differential diagnosis
includes two unlikely events, both of which require operative
Case Discussion intervention despite the fact that the patient is asymptomatic
and therapy of VT was successful; successful antitachycar-
Figure118.1 an episode of monomorphic VT with AV dis- dia pacing provides no information about the present or
sociation that was detected immediately and terminated by future efficacy of defibrillation, and the likelihood of need-
antitachycardia pacing. ing defibrillation is significant in a patient with a history of
The high-voltage impedance trends show intermittent high out-of-hospital VF. If no problem is identified within the
values over the last 5weeks. The defibrillation impedance for header at surgery, the lead should be replaced regardless of
intraoperative impedance measurements, which may be nor-
mal in cases of intermittent conductor failure. No other diag-
nostic testing is helpful. Chest radiography does not identify
P.A. Friedman(*)
Division of Cardiovascular Diseases and Department of Internal Fidelis lead failures, and noninvasive defibrillation testing
Medicine, Mayo Clinic, 200 First Street SW, Rochester, may not identify intermittent high-voltage failures.
MN 55905, USA
e-mail: pfriedman@mayo.edu
C.D. Swerdlow
Division of Cardiology, David Geffen School of Medicine,
Cedars-Sinai Medical Center, Los Angeles, CA, USA
e-mail: swerdlow@ucla.edu

A. Natale et al. (eds.), Cardiac Electrophysiology, 459


DOI: 10.1007/978-1-84996-390-9_118, Springer-Verlag London Limited 2011
460 P.A. Friedman and C.D. Swerdlow

Fig.118.1 Stored electrogram of device-detected VT


Case 118 461

R wave amplitude
At implant 15.8 mV Highest 16.6 mV
Last 15.2 mV Lowest 11.2 mV

mV
>20.0

10.0

5.0
3.0
2.0
1.0
0.0
<0.3
09/08/06 11/17/06 01/26/07 04/06/07 06/15/07 08/24/07 11/02/07 01/05/08 01/18/08
Last 80 weeks (min/max per week) Last 14 days

V. Defibrillation impedance
At implant 46 Highest 144
Last 50 Lowest 38


>200
150

100
80
60

40
30

<20
09/08/06 11/17/06 01/26/07 04/06/07 06/15/07 08/24/07 11/02/07 01/05/08 01/18/08
Last 80 weeks (min/max per week) Last 14 days

SVC (HVX) Deffib impendance


At implant 62 Highest 168
Last 63 Lowest 49


>200
150

100
80
60

40
30

<20
09/08/06 11/17/06 01/26/07 04/06/07 06/15/07 08/24/07 11/02/07 01/05/08 01/18/08
Last 80 weeks (min/max per week) Last 14 days

Fig.118.2 R wave and high-voltage impedance trends. Data points indicate daily measurements in last 2 weeks and the range of weekly values
before then
462 P.A. Friedman and C.D. Swerdlow

Atrial EGM

V Tip-Ring

CareAlert
RV pacing impendance
At implant 728 Highest >3,000
Last >3,000 Lowest 552


2,498 short VV intervals (120 130 ms) in last week >3,000
2,000
1,500
1,000
800
600
400
300
<200
05/27/05 08/05/05 10/14/05 12/23/05 03/03/06 05/12/06 07/20/06 08/02/06
Last 80 weeks (min/max per week) Last 14 days

Fig.118.3 Inappropriate shock caused by failure of the ring conductor provides a cumulative count of nonphysiologic short intervals caused by
in a Fidelis lead. The upper panel shows characteristic oversensing oversensing normal values short intervals are less than 300 (over about a
caused by lead or connector (header, adapter, or set-screw) problems.3,4 3-month period),5,6 but values for true-bipolar leads rarely exceed 50.
Oversensing is intermittent and usually occurs only during a small frac- Intermittent high measurements of the pacing-lead began 12days prior
tion (<10%) of the cardiac cycle. Nonphysiologically short inter- to the interrogation, indicating complete or partial interruption of the
vals130 ms are recorded, and make-break potentials saturate the pace-sense circuit. These triggered an Internet-based alert in the
sensing amplifier. The lower panel shows that 2,498 short intervals have Medtronic CareLink follow-up network. The patient was advised to have
been recorded by the Sensing Integrity Counter in the last week. It the lead replaced at that time, but refused because he was feeling well
Case 118 463

References 4. Swerdlow CD, Friedman PA. Advanced ICD troubleshooting: part


II. Pacing Clin Electrophysiol. January 2006;29(1):70-96.
5. Gunderson BD, Patel AS, Bounds CA, Ellenbogen KA. Automatic
1. Hauser RG, Kallinen LM, Almquist AK, Gornick CC, Katsiyiannis WT. identification of clinical lead dysfunctions. Pacing Clin Electrophysiol.
Early failure of a small-diameter high-voltage implantable cardioverter- January 2005;28(Suppl 1):S63-67.
defibrillator lead. Heart Rhythm. July 2007;4(7):892-896. 6. Vollmann D, Erdogan A, Himmrich E, etal. Patient Alert to detect
2. Swerdlow CD. Small-diameter defibrillation electrodes: can they ICD lead failure: efficacy, limitations, and implications for future
take a licking and keep hearts ticking? Heart Rhythm. July algorithms. Europace. May 2006;8(5):371-376.
2007;4(7):900-903.
3. Gunderson B, Patel A, Bounds C. Automatic identification of
implantable cardioverterdefibrillator lead problems using intracar-
diac electrograms. Comput Cardiol. 2002;29:121-124.

Case 119

M. Eyman Mortada, Jasbir S. Sra, and Masood Akhtar

Case Summary When comparing the current EGM to the one recorded at
the time of ICD shock, the T-wave is smaller and the over-
sensing episodes are fewer, but still present in a few beats.
A 62-year-old Caucasian male with history of cardiomyopa-
Thus, tachycardia in this patient increased the amplitude of
thy and implantable cardioverter defibrillator (ICD) implanted
the T-wave and the risk for oversensing and ICD shock.
8 months ago came to the emergency room after experienc-
Therefore, treatment must be aimed at slowing the heart rate
ing two ICD shocks. Both occurred without prodromal
by reducing physical activity and starting b-blocker therapy.
symptoms and after he did heavy work in his backyard.
Amiodarone therapy has no role in resolving T-wave
The stored EGM from the time when the patient received
oversensing.
the shock is shown in Fig. 119.1a and b (Panel B is a continu-
The device could be reprogrammed to reduce these events
ous recording from panel A).
by lowering the sensitivity. After lowering the sensitivity
The current EGM recording is shown in Fig.119.2.
from 0.3 to 0.45 mV, T-wave oversensing disappeared, as
What is the most appropriate next step for managing this
shown in the following real-time EGM (Fig.119.3).
condition?
Unfortunately, in this case, the R-waves are small to start
with. When the sensitivity was reduced slightly further,
Case Discussion down to 0.5 mV, some of the R-waves were not detected.
Therefore, there is no safety margin to detect real ventricular
Interrogation of the device indicates a single-lead device. fibrillation, and the patient would be at high risk for sudden
The EGM recording (V-tip to V-ring) prior to the shock cardiac death. If large R-waves are present and a good safety
showed fast rhythm with a rate of 460480ms. This fast margin exists within the sensitivity range, one could reduce
rhythm (most likely sinus tachycardia) is due to the the sensitivity and perform a defibrillation test to confirm the
patients physical activity (heavy labor in his backyard). efficacy of the device. If the device is a St. Jude ICD,
Yet, the device detected a faster rhythm with a rate of oversensing of spontaneous T-waves can be reduced by pro-
220270ms, almost double the rate of the actual rhythm grammable threshold start and Decay Delay. The best
detected on the EGM. The extraventricular detection solution would be lead revision. Rarely, SVT-VT morphol-
recorded by the device matches the location of the tall ogy discriminator and stability algorithm may help in reduc-
T-wave, leading to the diagnosis of T-wave oversensing. ing the inappropriate shock from T-wave oversensing.
Interestingly, after the ICD shock, the T-wave becomes Finally, forcing ventricular pacing might eliminate the
smaller and the oversensing resolves for several beats. T-wave oversensing due to alteration of the repolarization
This could be related to the effect of the electrical jolt on sequence, but this may result in adverse, desynchronizing
ventricular repolarization. hemodynamic effects.

M.E. Mortada(*), J.S. Sra, and M. Akhtar


Department of Electrophysiology, Aurora Cardiovascular Services,
Aurora Sinai/Aurora St. Lukes Medical Centers, University of
Wisconsin School of Medicine and Public Health, 2801
W. Kinnickinnic River Parkway, #777, Milwaukee, WI 53215, USA
e-mail: publishing4@aurora.org

A. Natale et al. (eds.), Cardiac Electrophysiology, 465


DOI: 10.1007/978-1-84996-390-9_119, Springer-Verlag London Limited 2011
466 M.E. Mortada et al.

Fig.119.1 Panel A illustrates the ICD reading from top to bottom: V-tip to V-ring electrocardiograms (EGM), leadless ECG, channel markers,
and event markers. Panel B is a continuous recording from panel A, which demonstrated the shock with 34.4 Joules

Fig.119.2 ICD reading at the time of the visit. From top to bottom: V-tip to V-ring EGM, channel markers, and then the leadless ECG. It shows
one T-wave oversensing after the fifth ventricular beat
Case 119 467

Fig.119.3 ICD reading after lowering the sensitivity from 0.3 to 0.45 mV causing the resolution of T-wave oversensing. From top to bottom: V-tip
to V-ring EGM, channel markers, and then the leadless ECG

Case 120

Paul A. Friedman and Charles D. Swerdlow

Case Summary which begins during atrial-sensed, ventricular-paced rhythm.


VT begins with a non-physiologically short PR interval.
The next ventricular beat is not preceded by an atrial electro-
A 76 year-old man with prior inferior wall myocardial
gram, but it and subsequent ventricular beats are followed by
infarction (NYHA Class II, left-ventricular ejection fraction
an atrial electrogram with a fixed VA interval. This pattern
41%, first degree AV block, and narrow QRS) underwent
indicates the onset of VT with 1:1 VA conduction. The ST
implantation of a dual-chamber ICD (Medtronic EnTrust
marker at the 16th tachycardia beat (lower right) indicates
D154ATG) for sustained monomorphic VT requiring cardio-
withholding of VT therapy because of the ICDs diagnosis of
version. He was treated with sotalol 120mg q 12 h to reduce
sinus tachycardia.
the frequency of VT. Figure120.1 shows an episode retrieved
The Sinus Tachycardia Rule applies because this tachy-
from the ICDs SVT log with programmed detection and
cardia has a 1:1 AV relationship and cycle length between
bradycardia pacing parameters. Why is VT misdiagnosed as
the VT detection interval and SVT Limit. Since the tachycar-
sinus tachycardia?
dia begins during ventricular-paced rhythm, the expected
PR window does not apply. The range of expected RR
intervals is wide because of frequent premature ventricular
complexes indicated by short intervals followed by compen-
Case Discussion satory pauses on the interval plot (e.g. third ventricular
electrogram in middle panel). The change in RR interval
Present Medtronic ICDs discriminate 1:1 AV conduction of between sinus tachycardia with cycle length of 500ms and
sinus tachycardia from 1:1 VA conduction of VT using a dual VT with cycle length 430 ms is insufficient to exceed the
chamber onset algorithm (Sinus Tachycardia Rule) that expected range of RR intervals. Thus the 1:1 AV relation-
evaluates both PR and RR interval onset.1 If the predominant ship and expected RR interval are classified as consistent
AV pattern at the onset of tachycardia is 1:1, the algorithm with sinus tachycardia, and the expected PR interval
determines if the RR and PR intervals are within the criterion is not applied.
expected range based on the range of variability for recent This error in classification can be prevented by program-
PR and RR intervals. Sinus tachycardia is diagnosed and ming Managed Ventricular Pacing,2,3 which permits AV
VT therapy is withheld if both the RR and PR intervals are conduction during sinus rhythm with first degree AV block.
within the expected range. Premature ventricular complexes During Managed Ventricular Pacing, the expected PR inter-
increase the expected range of RR intervals. val is required to withhold VT therapy and diagnose sinus
Figure 120.1 shows ICD programming, stored electro- tachycardia, so VT would probably have been diagnosed
gram, and interval plot corresponding to the episode of VT, correctly.

P.A. Friedman(*)
Division of Cardiovascular Diseases and Department of Internal
Medicine, Mayo Clinic, 200 First Street SW, Rochester, MN 55905
e-mail: pfriedman@mayo.edu
C.D. Swerdlow
Division of Cardiology, David Geffen School of Medicine,
Cedars-Sinai Medical Center
e-mail: swerdlow@ucla.edu

A. Natale et al. (eds.), Cardiac Electrophysiology, 469


DOI: 10.1007/978-1-84996-390-9_120, Springer-Verlag London Limited 2011
470 P.A. Friedman and C.D. Swerdlow

Fig.120.1 Top panel shows programmed values for detection of VT/VF, rejection of SVT, and bradycardia pacing. Middle panel shows stored
electrogram. Lower panel shows interval plot. Open squares denote atrial events. Closed circles denote ventricular events. See text for discussion

References 3. Sweeney MO, Ellenbogen KA, Casavant D, etal. Multicenter, pro-


spective, randomized safety and efficacy study of a new atrial-based
managed ventricular pacing mode (MVP) in dual chamber ICDs.
1. Stadler RW, Gunderson B, Pearson A, Gillberg J. Adaptive algorithm JCardiovasc Electrophysiol. Aug 2005;16(8):811-817.
to withhold ICD therapy during sinus tachycardia (abstract). Pacing
Clin Electrophysiol. 2000;23(4 Part II):677.
2. Sweeney MO, Bank AJ, Nsah E, etal. Minimizing ventricular pac-
ing to reduce atrial fibrillation in sinus-node disease. N Engl J Med.
Sep 6 2007;357(10):1000-1008.
Case 121

Kenneth A. Ellenbogen

Case Summary Case Discussion

A 44year old woman with NYHA Class III congestive heart This is a patient with loss of response to CRT (cardiac resyn-
failure, non-ischemic cardiomyopathy and a left bundle chronization pacing). The etiologies are numerous, but can
block (QRS duration: 150ms) underwent implantation of a be divided into SYSTEM RELATED and PATIENT RELA
biventricular ICD 6months ago. The patient returned 1month TED. System related causes include atrial undersensing,
after the initial implant and noted a significant improvement atrial oversensing, ventricular oversensing and loss of left
in heart failure symptoms. She is seen now at her 6month ventricular capture. Patient related causes include atrial
clinic visit and her heart failure symptoms have worsened. fibrillation, poor lead placement, lack of dyssynchrony, sub-
Her device is interrogated and she is pacing her ventricle less optimal AV and VV coupling.
than 50% of the time. A rhythm strip is printed from the pro- In this patient, the problem is clearly ventricular over-
grammer (Fig. 121.1) and the programmed parameters are sensing. The cause appears to be due to T wave oversensing.
shown below: The morphologies of the 2 VS events are very different
and most likely not both PVCs. The second VS event occurs
Atrial Lead only 520ms after the first VS (Fig.121.2). The second VS is
Atrial lead amplitude 3.5V narrow and looks like an intrinsic depolarization. This brings
Pulse width 0.4ms into question whether the first VS is a true PVC. The first VS
Sensitivity 0.3mV following the paced ventricular event is likely due to T wave
Pace blanking 200ms oversensing.
RV Lead
T wave oversensing is causing a loss of BiV pacing. The
patients intrinsic R wave falls shortly after the T wave and
Amplitude 3V
resets the timer. By decreasing the ventricular sensitivity
Pulse width 0.4ms
(increasing its value), the T wave is no longer sensed and
Sensitivity 0.3mV normal biventricular pacing resumes (Fig. 121.3). In this
Pace blanking 200ms case, the ventricular sensitivity was programmed from 0.3 to
LV lead 0.45 mV with a restoration of BiV pacing, and the device
Amplitude 6V was programmed to 0.6mV as the R wave was >12mV dur-
Pulse width 1.1. ms ing sinus rhythm. Defibrillation thresholds (DFTs) should be
Pace polarity LV tip to RV coil
done to confirm adequate sensing during ventricular tachyar-
rhythmias at the programmed sensitivity of 0.6mV.
Why is the ventricle paced only 50% of the time and what
programming changes are recommended?

K.A. Ellenbogen
Department of Cardiology, VCU School of Medicine, 980053,
Richmond, VA 23298-0053
e-mail: kellenbogen@pol.net

A. Natale et al. (eds.), Cardiac Electrophysiology, 471


DOI: 10.1007/978-1-84996-390-9_121, Springer-Verlag London Limited 2011
472 K.A. Ellenbogen

Fig.121.1 Rhythm strip printed


from programmer showing atrial
and ventricular EGMs and the
marker channel

Fig.121.2 Rhythm strip printed


from programmer showing atrial and
ventricular EGMs and the marker
channel. Note TW oversensing

Fig.121.3 No further TW
oversensing after programing
changes
Case 122

Anurag Gupta

Case Summary Case Discussion

A 75 year-old male with permanent atrial fibrillation, a his- Notable reasons for ventricular pacing at rate exceeding the
tory of congestive of heart failure due to ischemic cardio- lower rate limit include but are not limited to: (a) sensor
myopathy, and a biventricular implantable cardioverter driven rate changes, unlikely in this patient as his accelerom-
defibrillator is admitted with medically refractory recurrent eter would not be predicted to lead to sustained elevations in
ventricular tachycardia resulting in disabling shock therapy. heart rate while stationary and/or during sleep; (b) P-wave
He undergoes successful endocardial catheter ablation, with tracking modes, not possible in this patient whose pacing
no subsequent inducible ventricular tachycardia nor sponta- mode is VVIR; (c) programmed interventions such as rate
neous sustained events for the remainder of his hospitaliza- smoothing, rate hysteresis, rate drop response, overdrive
tion. However, in the context of atrial fibrillation, he is noted algorithms, conducted atrial fibrillation response, ventricular
on telemetry to have a persistently elevated paced ventricular sense response, and ventricular rate regularization (discussed
rate of approximately 90 ppm, including overnight while below); (d) reversion to alternate mode and/or rate, for exam-
sleeping comfortably (Fig.122.1). ple as with magnet application, electromagnetic interference,
Interrogation of his Guidant Contak Renewal 3 HE biven- and battery depletion; and (e) device malfunction including
tricular ICD shows adequate battery longevity and stable component and/or circuit failures with runaway.
lead parameters. Device settings are as follows: Several features deserve special mention in individuals
such as the one discussed in this case who have biventricular
Mode VVIR devices and experience frequent PVCs and/or conducted
Lower rate 70ppm atrial tachyarrhythmias. This may lead to unwanted (a) com-
Maximum sensor rate 120ppm petition with pacing and thus decrease in cardiac resynchro-
Pacing chamber Biventricular (LV offset 0ms) nization therapy; and/or (b) irregularity in ventricular cycle
Sensor (accelerometer) Activity threshold: Medium
length, which may lead to hemodynamic and symptomatic
compromise. One intervention, ventricular sense response,
Rate hysteresis Off
allows triggering of ventricular pacing in response to sensed
Rate smoothing Off
ventricular events, mitigating the potential loss of efficacy
Ventricular rate regularization Maximum (maximum pacing rate with conducted tachyarrhythmias or frequent PVCs. An
120ppm)
additional potential therapy, and the one that accounts for
Ventricular tachyarrhythmia Begin at 125bpm
therapies this patients elevated ventricular rates, is ventricular rate
regularization, or similar algorithms that represent modifica-
What are the possibilities accounting for his ventricular tions of rate smoothing. Ventricular rate regularization may
pacing at an elevated rate that exceeds his lower rate limit? reduce symptoms by minimizing VV cycle length variation
during atrial arrhythmias by adjusting the ventricular pacing
rate (up to a programmable maximum rate) in accordance to
a weighted sum of current ventricular cycle lengths.1 A
telemetry strip in this patient demonstrates how ventricular
A. Gupta rate regularization leads to greater maintenance of VV reg-
Cardiac Electrophysiology Service, Division of Cardiology,
Department of Medicine, Stanford University Hospital and Clinics,
ularity despite frequent PVCs and to elevated ventricular
300 Pasteur Drive, Room H2146, Stanford, CA 94305-5233 pacing rate above the lower rate limit (Fig.122.1). Note that
e-mail: agupta@stanfordalumni.org frequent PVCs, as opposed to rapid conduction of his

A. Natale et al. (eds.), Cardiac Electrophysiology, 473


DOI: 10.1007/978-1-84996-390-9_122, Springer-Verlag London Limited 2011
474 A. Gupta

Fig.122.1 Telemetry recording


demonstrating ventricular pacing
above the lower rate limit

Fig.122.2 12 h overnight heart


rate trend. Note the abrupt
transition in ventricular rate after
the ventricular rate regularization
feature was turned off (arrow)

underlying atrial fibrillation, are present in this patient and shows that following a PVC, the next ventricular beat occurs
are responsible for utilization of the ventricular rate regular- at the completion of the lower rate interval, keeping the over-
ization feature. all ventricular rate close to 70ppm though with greater VV
The device was reprogrammed to turn off the ventricular irregularity (Fig.122.3). This case illustrates the importance
rate regularization function. Subsequently, the ventricular of recognizing reasons for changes in ventricular rate, includ-
rate promptly decreased from approximately 9070ppm, as ing programmed interventions used to mitigate irregularity
illustrated during overnight 12 h telemetry trend (Fig.. 122.2). in ventricular pacing such as ventricular rate regularization
More specifically, a telemetry strip with the new settings and/or to minimize loss of biventricular pacing.
Case 122 475

Fig.122.3 Telemetry recording


demonstrating ventricular pacing
with ventricular rate regulariza-
tion feature turned off. Note the
irregularity in VV cycle due to
PVC and the overall slower
paced ventricular rate

Reference

1. Tse HF, Newman D, Ellenbogen KA, Buhr T, Markowitz T, Lau C.


Effects of ventricular rate regularization pacing on quality of life
and symptoms in patients with atrial fibrillation (Atrial fibrillation
symptoms mediated by pacing to mean rates [AF Symptoms
Study]). Am J Cardiol. 2004;94:938-941.

Case 123

Ronald Lo, Amin Al-Ahmad, and Paul J. Wang

Case Summary ventricular rhythm. The ventricular rhythm is secondary to


the variable conduction down the AV node from the rapid
atrial fibrillation. It is unlikely to be a ventricular tachycardia
A 48 year old male with a history of coronary artery disease,
given the irregularity of the rhythm. Careful examination of
hypertension, ischemic cardiomyopathy with an ejection
the episode reveals that the episode was detected and then
fraction of 10% with an biventricular ICD presented with an
aborted, followed by detection in the VF zone with an ICD
shock from his device. He has had increased social stressors
discharge which fortuitously converts him to sinus rhythm.
at home which have resulted in him running out of his heart
Treatment of atrial fibrillation and prevention of rapid con-
failure medications.
duction down the AV node with beta blockers or calcium
Interrogation of his Medtronic InSync Sentry biventricu-
channel blockers is the primary therapeutic treatment for this
lar ICD revealed an episode of atrial fibrillation with a rapid
occurrence.
ventricular response followed by an aborted discharge and
Inappropriate ICD discharges remain frequent, with up
subsequent 34.9 J shock with restoration of sinus rhythm
to 2030% of ICD discharges being inappropriate, with
(Fig.123.1).
supraventricular arrhythmias being a frequent culprit.1
What is the ventricular rhythm, and why does he receive a
Programming the device to a higher VT or VF zone may
shock?
alleviate the problem, but may risk the undertection of true
VT or VF. SVT discriminators may have a role in the preven-
tion of inappropriate discharges; however it may be very
Case Discussion difficult distinguishing true VT from an SVT.
This case illustrates that ICD discharges may occur in the
setting of non-ventricular tachyarrhythmias.
Examination of the interrogation report reveals an atrial and
ventricular electrogram with rapid atrial fibrillation. The
atrial activity is clearly disorganized with an irregular

R. Lo (*)
Riverside Electrophysiology, 4000 14th Street, Suite 209, Riverside,
CA 92501
e-mail: heartfailure@gmail.com
A. Al-Ahmad and P.J. Wang
School of Medicine, Stanford University, 300 Pasteur Drive, H-2146,
Stanford, CA 94305
e-mail: aalahmad@stanford.edu; paul.j.wang@stanford.edu

A. Natale et al. (eds.), Cardiac Electrophysiology, 477


DOI: 10.1007/978-1-84996-390-9_123, Springer-Verlag London Limited 2011
478 R. Lo et al.

Fig.123.1 Device interrogation showing an atrial fibrillation with an ICD discharge


Case 123 479

Reference

1. Korte T, Kditz H, Niehaus M, Thomas PT, Tebbenjohanns J. High


incidence of appropriate and inappropriate ICD therapies in children
and adolescents with implantable cardioverter defibrillator. Pacing
Clin Electrophysiol. 2004;27(7):924-932.

Case 124

Ronald Lo, Amin Al-Ahmad, and Paul J. Wang

Case Summary be conducted to the ventricle and sensed appropriately in the


ventricle. The differential diagnosis here remains ventricular
tachycardia or a supraventricular tachycardia.
A 55 year old female with idiopathic nonischemic dilated
Upon closer inspection of the intervals, one notes that
cardiomyopathy and complete left bundle branch was seen in
there is not a constant VA interval that might suggest ven-
the clinic for significant dyspnea on exertion. She underwent
tricular tachycardia with 1:1 retrograde conduction. Rather,
implantation of a biventricular implantable cardioverter-
the AV interval remains quite constant increasing the likeli-
defibrillator with significant improvement in her symptoms
hood that this is an atrial tachycardia with 1:1 ventricular
of heart failure.
conduction. The device detects this episode in the VT zone
More recently she has been having episodes of palpita-
and delivers anti-tachycardia pacing (ATP) which is seen to
tions due to medication noncompliance, and usually associ-
terminate the tachycardia. The absence of VA conduction
ated with significant weight gain.
during the ATP pacing train also argues against the likeli-
Interrogation of her Medtronic InSync II Marquis device
hood that this is VT with 1:1 retrograde conduction.
revealed an episode of tachycardia as shown in (Fig.124.1).
While ATP is fairly successful in terminating some ven-
The device settings are as follows:
tricular tachycardias, termination of a supraventricular tachy-
Mode: DDD with lower rate limit of 50 and upper rate limit of 120
cardia such as an atrial tachycardia can also occur. ATP may
Mode Switch: On at 155bpm offer the potential for painless termination of ventricular
VF Zone (rate>207bpm) : 30 Joule x 6 tachycardia or other tachyarrhythmias. Reduction in painful
VT Zone (rate>167bpm): ATP x 3 (Burst), 15 Joule x 1 and 30 shocks may improve patient quality-of-life and extend the
Joule x 4 longevity of the implanted device. In this case, ATP success-
What is the mechanism of the tachycardia and how did it fully terminated an atrial tachyarrhythmia and prevented an
terminate? inappropriate shock.

Case Discussion

Examination of the intracardiac electrograms demonstrates a


1:1 tachycardia that starts with an atrial beat that is in the
post ventricular atrial refractory period. This beat appears to

R. Lo(*)
Riverside Electrophysiology, 4000 14th Street,
Suite 209, Riverside, CA 92501
e-mail: heartfailure@gmail.com
A. Al-Ahmad and P.J. Wang
School of Medicine, Stanford University,
300 Pasteur Drive, H-2146, Stanford, CA 94305
e-mail: aalahmad@stanford.edu; paul.j.wang@stanford.edu

A. Natale et al. (eds.), Cardiac Electrophysiology, 481


DOI: 10.1007/978-1-84996-390-9_124, Springer-Verlag London Limited 2011
482 R. Lo et al.

Fig.124.1 Intracardiac electrogram of tachycardia with termination via ATP


Case 125

Ronald Lo, Amin Al-Ahmad, and Paul J. Wang

Case Summary the start of an atrial arrhythmia, likely an atrial tachycardia


with predominately 1:1 conduction to the ventricle. It is
interesting to note that the PR interval during the atrial tachy-
A 72 year old man with a history of nonischemic dilated
cardia is shorter than the PR interval during a paced rhythm
cardiomyopathy with an ejection fraction of 20%, complete
of the first two beats. This may be due to the location of the
left bundle branch block and mechanical aortic and mitral
atrial tachycardia being located closer to the AV node.
valves. He has been having symptomatic Class IIIIV heart
Upon detection of an atrial arrhythmia, or atrial high rate
failure symptoms and subsequently underwent implantation
episodes, the device mode switches from a tracking mode to
of a St Jude Medical Atlas+HF Model V-340 biventricular
a demand backup pacing mode, in this case DDI. In DDI
defibrillator. His symptoms of heart failure have improved
mode, there is sensing in both the atrium and the ventricle,
significantly after implantation of his cardiac resynchroniza-
with the only action the device is taking is inhibition of
tion device. However, he does note occasionally the sensa-
pacing with a sensed complex. The benefit of DDI pacing is
tion of palpitations with subjective worsening of dyspnea on
prevention of rapid atrial tracking, however with less optimal
exertion. His device interrogation revealed the event dis-
atrial and ventricular timings, especially in a cardiac resyn-
played in Fig. 125.1. What is the rhythm displayed? Why
chronization device.
would this rhythm cause him to experience worsening heart
In this patient, there is loss of biventricular pacing as the
failure symptoms?
ventricular complexes are due to the intrinsic conduction
from the atrial tachycardia. The loss of beneficial biventricu-
lar pacing and atrial-ventricular synchrony contributes to
Case Discussion worsening hemodynamic function and mechanical dyssyn-
chrony. Treatment of the atrial tachycardia with beta block-
Interrogation of his device revealed recording of two electro- ers or antiarrhythmic medications may decrease the likelihood
gram channels, the atrial and ventricular channels. Prior to of this from recurring.
the event, he is noted to be tracking his atrium with biven- This case illustrates the importance of maintaining biven-
tricular pacing. The fourth beat of the electrogram reveals tricular pacing in patients who have atrial tachyarrhythmias.

R. Lo(*)
Riverside Electrophysiology,
4000 14th Street, Suite 209, Riverside, CA 92501
e-mail: heartfailure@gmail.com
A. Al-Ahmad and P.J. Wang
School of Medicine, Stanford University,
300 Pasteur Drive, H-2146, Stanford, CA 94305
e-mail: aalahmad@stanford.edu; paul.j.wang@stanford.edu

A. Natale et al. (eds.), Cardiac Electrophysiology, 483


DOI: 10.1007/978-1-84996-390-9_125, Springer-Verlag London Limited 2011
484 R. Lo et al.

Fig.125.1 Device interrogation


Section
III
Clinical Cases

Case 126

Mehmet K. Aktas, Abrar H. Shah, and James P. Daubert

Case Summary Case Discussion

A 51-year-old man on chronic methadone therapy for a his- The QT interval, the electrocardiographic gauge of ventricular
tory of heroin abuse presented to the emergency room with repolarization, is often overlooked or misinterpreted. Alter
cough and dyspnea. On exam he was found to be tachypneic ations to the timing and mechanism of ventricular repolari
and pulse oximetry showed a saturation of 88% on room air zation can lead to ventricular tachyarrhythmias particularly
and a portable chest X-ray revealed a right lower lobe infil- short-long-short sequences, which are often a trigger of
trate. His serum potassium was 3.6 meq/L and his serum torsade de pointes. The factors influencing the QT interval
magnesium was 2.0meq/L. An electrocardiogram performed are complex and may include a variety of channelopathies,
upon arrival showed sinus arrhythmia with evident U waves changes in autonomic innervation, and acquired factors such
and a prolonged QTc interval (Fig.126.1) although this was as drugs or electrolyte disturbances.1,2 The patient described
not recognized at the time. He was given intravenous moxi- was on methadone which is a known QT prolonging drug and
floxacin while in the emergency room and was hospitalized hence prudence would require that the QT interval be closely
for in-patient antibiotic therapy. Twelve hours later the monitored when other QT prolonging drugs are begun.35
patient reported feeling anxious and was found to be diapho- Even with a prolonged baseline QTc (which was overlooked)
retic. Telemetry monitoring was initiated and showed sinus he was started on a fluoroquinolone type antibiotic, moxiflox-
rhythm with frequent ventricular ectopy. An electrocardio- acin, which has been shown to consistently prolong the QT
gram showed ventricular bigeminy with significant QT inter- interval and has rarely been associated with torsade de
val prolongation (Fig. 126.2). Minutes later the patient pointes.6,7 The combination of methadone, and moxifloxacin
became pulseless and apneic and was found to be in ven- in this patient led to significant QT prolongation and torsade
tricular fibrillation (Fig. 126.3). Chest compressions were de pointes.
started and within a minute spontaneous return of sinus Once drug induced torsade de pointes (or even significant
rhythm was noted. Serial electrocardiograms demonstrated QT prolongation without torsade) is identified, immediate
progressive QT prolongation with rate corrected QT inter- discontinuation of the offending drug or drugs is required.
vals (QTc) as high as 630ms. What was the likely cause for Temporary pacing may be considered to prevent pause
this patients cardiac arrest? related ventricular tachyarrhythmias and short-long-short
sequences. Chronotropic agents, such as isoproterenol or
atropine, may also be considered to increase the heart rate in
attempt to shorten the QT interval and eliminate short-long-
short sequences. Intravenous magnesium sulfate, a safe and
M.K. Aktas(*)
Department of Cardiology/Electrophysiology, University of Rochester effective adjunctive therapy, may also be given for the acute
Medical Center, 601 Elmwood Ave, 679C, Rochester, NY 14642 termination of torsade de pointes.8 Potassium should be
e-mail: mehmet_aktas@urmc.rochester.edu maintained in the high normal range. The patient above con-
A.H. Shah tinued to have salvos of ventricular fibrillation despite these
Department of Cardiology, University Cardiovascular Associates, measures and so low-dose intravenous dopamine for chrono-
2365 South Clinton Ave., Suite 100, Rochester, NY 14618 tropic support was started and the episodes of ventricular
e-mail: abrar_shah@urmc.rochester.edu
fibrillation stopped. Moxifloxacin was discontinued and over
J.P. Daubert the ensuing days his QT interval returned to baseline. His
Cardiac Electrophysiology, Cardiology Division,
methadone was gradually tapered with further normalization
Duke University Health System, DUMC Box3174 Duke Hospital
7451H, Durham, NC 27710 of his QTc. Since discharge has remained in sinus rhythm
e-mail: james.daubert@duke.edu with no further arrhythmias or syncope.

A. Natale et al. (eds.), Cardiac Electrophysiology, 487


DOI: 10.1007/978-1-84996-390-9_126, Springer-Verlag London Limited 2011
488 M.K. Aktas et al.

Fig.126.1 Electrocardiogram at presentation showing sinus arrhythmia. The QTc interval is prolonged at 495ms

Fig.126.2 Electrocardiogram shows a ventricular bigeminal rhythm which confounds calculation of the QTc. Nevertheless, the QT interval is
severely prolonged extending out to the succeeding QRS complex. The QTc is estimated at about 630 ms
Case 126 489

Fig.126.3 Telemetry strip


showing ventricular bigeminy
with a short-long-short
sequence followed by torsade de
pointes

References 5. Chugh SS, Socoteanu C, Reinier K, Waltz J, Jui J, Gunson K.


A community-based evaluation of sudden death associated with
therapeutic levels of methadone. Am J Med. 2008;121:66-71.
1. Roden DM. Drug-induced prolongation of the QT interval. N Engl 6. Dale KM, Lertsburapa K, Kluger J, White CM. Moxifloxacin and
J Med. 2004;350:1013-1022. torsade de pointes. Ann Pharmacother. 2007;41:336-340.
2. Moss AJ. Drug-induced QT prolongation: an update. Ann 7. Sherazi S, DiSalle M, et al. (2008). Moxifloxacin-induced torsades
Noninvasive Electrocardiol. 2006;11:1-2. de pointes. Cardiol J 15(1): 71-73.
3. Wedam EF, Bigelow GE, Johnson RE, Nuzzo PA, Haigney MC. 8. Tzivoni D, Banai S, Schuger C, et al. Treatment of torsade de
QT-interval effects of methadone, levomethadyl, and buprenorphine pointes with magnesium sulfate. Circulation. 1988;77:392-397.
in a randomized trial. Arch Intern Med. 2007;167:2469-2475.
4. Krantz MJ, Lewkowiez L, Hays H, Woodroffe MA, Robertson AD,
Mehler PS. Torsade de pointes associated with very-high-dose
methadone. Ann Intern Med. 2002;137:501-504.

Case 127

Loren P. Budge and John P. DiMarco

Case Summary known history of arrhythmias, although he has had several


prior episodes of palpitations in the past which have resolved
A 55-year-old man was brought to the emergency depart- spontaneously for which he had not sought evaluation. The
ment after an episode of palpitations and syncope. He had no rescue squad was called. During transport several short runs
prior cardiac history, and his medical history was significant of a wide complex tachycardia were noted on monitor but no
only for hypertension, which he has been trying to manage strips were saved. Upon arrival at the emergency room, his
with a salt-restricted diet. He has been quite active and denied heart rate was 80, with a blood pressure of 108/72. His elec-
previous symptoms of angina or heart failure. He has not trocardiogram is shown in Fig. 127.1. An echocardiogram
been taking any medications. His symptoms started abruptly was obtained later that day and is shown in Fig.127.2. This
while sitting at his desk at work, and consisted of rapid pal- led to a cardiac magnetic resonance study as shown in
pitations with chest pain, shortness of breath, lightheaded- Fig.127.3.
ness, and diaphoresis. A nurse was present and reported a What is this patients diagnosis? What arrhythmias is this
heart rate near 200bpm. After a few minutes, he briefly lost patient likely to have?
consciousness, then quickly awoke and felt well. He has no What would you do next?

Fig.127.1 ECG on arrival to the emergency department

L.P. Budge(*)
Cardiovascular Division, University of Virginia Health System,
1215 Lee Street, Box800662, Charlottesville, VA, 22908 USA
e-mail: lbudge@virginia.edu
J.P. DiMarco
Cardiovascular Division, University of Virginia Health System,
1215 Lee Street, Charlottesville, VA, 22908 USA
e-mail: jdimarco@virginia.edu

A. Natale et al. (eds.), Cardiac Electrophysiology, 491


DOI: 10.1007/978-1-84996-390-9_127, Springer-Verlag London Limited 2011
492 L.P. Budge and J.P. DiMarco

Fig.127.2 Transthoracic echocardiogram apical four chamber view at


end-systole. This echocardiogram is diagnostic for left ventricular non-
compaction. Pertinent findings include prominent left ventricular trabe- Fig. 127.3 Cardiac magnetic resonance (CMR) apical short axis
culae with a ratio of non-compacted to compacted myocardium >2:1 viewin diastole. This CMR demonstrates the hallmark features of left
(double-sided arrows demarcate compacted and non-compacted seg- ventricular noncompaction. There is a greater than 2.3:1 ratio of
ments). Intertrabecular recesses are also seen (one-sided arrow). Color non-compacted to compacted myocardium (demarcated by the double-
Doppler (not shown) demonstrates flow within these recesses with sided arrows shown) with prominent inferolateral trabeculae. Cine (not
communication to the left ventricular cavity shown) reveals hypokinetic contraction of the noncompacted segments
with evidence of blood flow within the trabecular space with communi-
cation to the left ventricular cavity

Case Discussion
inheritance pattern is usually autosomal dominant. There is
This gentleman presented with an episode of syncope with some phenotypic and genotypic overlap with other cardio-
non-sustained wide complex tachycardia noted by the rescue myopathies, especially hypertrophic cardiomyopathy. LVNC
squad prior to his admission. His admission ECG is nonspe- may also occur with several neuromuscular disorders such
cific since it shows only normal sinus rhythm with lateral T as Barth Syndrome, Charcot-Marie-Tooth 1a, Melnick-
wave inversions. However, his echocardiogram shows prom- Needles Syndrome and Nail-patella Syndrome. It may rarely
inent left ventricular trabeculae and is diagnostic for isolated be seen in conjunction with other forms of congenital heart
left ventricular non-compaction (LVNC). disease, including Ebsteins anomaly, bicuspid aortic valve,
LVNC is an uncommon congenital cardiomyopathy. L-TGA, left atrial appendage isomerism, and ventricular
Patients with LVNC will manifest on echocardiography septal defects. The prevalence of LVNC was originally
prominent left ventricular (LV) trabeculae with deep inter- thought to be extremely rare, but as imaging modalities and
trabecular recesses. This pattern is caused by intrauterine awareness foster recognition of milder forms of the disorder,
arrest of compaction, resulting in two layers of myocar- more cases with less prominent clinical and imaging find-
dium: compacted and non-compacted. There is continuity ings are being diagnosed.
between the LV cavity and the intertrabecular recesses, The diagnostic criteria of isolated LVNC differ depending
without any communication to the epicardial vessels. LVNC on the imaging modality used. Echocardiographic criteria
most commonly affects the inferolateral portion of the LV include an absence of coexisting cardiac abnormalities,
apex, although other areas, including the RV can also be a2:1 ratio of non-compacted to compacted myocardium
affected. There is usually a corresponding decrease in LV (NC/C ratio) at end-systole with thickening of the myocar-
ejection fraction. dial wall (as opposed to apical thrombus, which would not
The most common clinical presentations of symptomatic thicken) and documentation of flow within the intertrabecu-
LVNC are heart failure, arrhythmia (atrial or ventricular), lar recesses. Cardiac magnetic resonance imaging criteria for
chest pain or systemic embolism. The sinus rhythm ECG LVNC require a NC/C ratio of >2.3:1 at end-diastole which
usually shows only non-specific ST or T wave abnormalities. has a sensitivity of 86% and a specificity of 99%.
Associated conduction abnormalities such as bundle branch Prognosis in LVNC is controversial. Most studies report
or fascicular blocks, or Wolf-Parkinson-White syndrome no more than 34year follow-up durations and are biased
may be seen. towards patients with the more severe forms of the disease.
When LVNC occurs in families, it has been linked to Those who present in childhood have a reported 14% 3year
mutations in a number of cytoskeletal proteins and the mortality. LV function often recovers transiently if heart
Case 127 493

failure therapy is initiated, only to again worsen in early Bibliography


adulthood. Of adults who present with symptoms, 41% of
patients had documented ventricular tachycardia (VT), Engberding R, Yelbuz TM, Breithardt G. Isolated noncompaction of the
53% were hospitalized for heart failure, and 24% had a left ventricular myocardium a review of the literature two decades
thromboembolic event over a mean 44 month follow-up after the initial case description. Clin Res Cardiol. 2007;96:481-488.
Epstein AE, DiMarco JP, Ellenbogen KA, etal. ACC/HRS 2008 guide-
period. Transplant-free survival in symptomatic patients
lines for device-based therapy of cardiac rhythm abnormalities: a
was 58% at 5 years, with a 35% mortality rate, half of report of the American College of Cardiology/American Heart
whom died suddenly. However among asymptomatic Association Task Force on Practice Guidelines (Writing Committee to
patients with an incidental finding of LVNC, 5year trans- Revise the ACC/AHA/NASPE 2002 Guideline Update for Implantation
of Cardiac Pacemakers and Antiarrhythmia Devices): developed in
plant-free survival was 97%, with less than a 10% throm-
collaboration with the American Association for Thoracic Surgery and
boembolic rate. Further studies on the long-term prognosis Society of Thoracic Surgeons. Circulation. 2008;117:e350-e408.
of asymptomatic patients with findings of LVNC during Frischknecht BS, Attenhoffer Jost CH, etal. Validation of noncompac-
cardiac imaging are clearly needed. tion criteria in dilated cardiomyopathy, and valvular and hyperten-
sive heart disease. J Am Soc Echo. 2005;18:865-872.
Treatment recommendations include standard medical
Jenni R, Oechslin E, Schneider J, Attenhofer Jost C, Kaufmann PA.
therapy according to ejection fraction, heart failure symp- Echocardiographic and pathoanatomical characteristics of isolated
toms, and atrial arrhythmias. Due to the propensity for left ventricular non-compaction: a step towards classification as a
thrombus formation, warfarin is recommended for patients distinct cardiomyopathy. Heart. 2001;86:666-671.
Kobza R, Jenni R, Erne P, Oechslin E, Duru F. Implantable cardiover-
with an EF40% or with atrial fibrillation even if other risk
ster-defibrillators in patients with left ventricular noncompaction.
factors are absent. Given that sudden cardiac death is com- PACE. 2008;31:461-467.
mon in this population, and there is currently no reliable way Murphy RT, Thaman R, Blanes JG, etal. Natural history and familial
to determine who is at risk for life-threatening arrhythmia, characteristics of isolated left ventricular non-compaction. Eur
Heart J. 2005;26:187-192.
recent guidelines support consideration of ICD therapy as a
Oechslin EN, Attenhofer Jost CH, Rojas JR, Kaufmann PA, Jenni R.
Class 2B indication. Long-term follow-up of 34 adults with isolated left ventricular non-
This patient presented with heart failure and what was compaction: a distinct cardiomyopathy with poor prognosis. J Am
most likely ventricular tachycardia. He was started on ther- Coll Cardiol. 2000;36:493-500.
Petersen SE, Selvanayagam JB, Wiesmann F, et al. Left ventricular
apy with an ACE inhibitor and a beta blocker. A dual cham-
non-compaction: insights from cardiovascular magnetic resonance
ber ICD was inserted. Three months after insertion, he had imaging. J Am Coll Cardiol. 2005;46:101-105.
an episode of lightheadedness and an electrogram stored by
the ICD showed a burst of monomorphic ventricular tachy-
cardia that was broken with antitachycardia pacing. No atrial
arrhythmias have been detected.

Case 128

David J. Callans

Case Summary What is the likely mechanism responsible for these


findings?
A 62-year-old man without previous cardiac history pre-
sented to the hospital with complaints of near syncope and
exercise intolerance. The presenting electrocardiogram Case Discussion
(Fig. 128.1) demonstrates sinus rhythm with a normal PR
interval and narrow QRS on half of the conducted beats, An electrophysiologic study was performed to investigate
intermittent AV block, and alternating left and right bundle the pathophysiology of his AV block (Fig.128.2). A split His
aberrancy on the other half of the conducted beats. potential was recorded and intermittent AV block was

500 ms
I

II

III

aVR

aVL

aVF

V1

V2

V3

V4

V5

V6

4:54:12 PM 4:54:13 PM 4:54:14 PM 4:54:15 PM 4:54:16 PM

Fig.128.1 Presenting electrocardiogram with intermittent AV block and alternating left and right bundle aberration

D.J. Callans
Department of Cardiology, University of Pennsylvania,
3400 Spruce Street, Philadelphia, PA 19104, USA
e-mail: david.callans@uphs.upenn.edu

A. Natale et al. (eds.), Cardiac Electrophysiology, 495


DOI: 10.1007/978-1-84996-390-9_128, Springer-Verlag London Limited 2011
496 D.J. Callans

200 ms
I

aVF

V1 S

V6

hRA CT

HIS d C

HIS p C

4:16:04 PM 4:16:05 PM

Fig.128.2 Surface and intracardiac recordings during sinus rhythm and atrial pacing. A split His potential is seen, and atrial pacing results in
intermittent intra-His conduction block

A proposed mechanism for this phenomenon is presented


in Fig.128.3. Despite normal infra-His conduction, alternat-
ing aberrancy is favored during 3:2 conduction because of
alternating retrograde concealment, resulting in long-short
input to the other bundle branch on the following portion of
the sequence.
Alternating bundle branch block is recognized as a high
risk situation and is considered a Class 1 indication for per-
manent pacing. The patient was treated with dual chamber
pacing, indicated because of the finding of intra His block,
which resolved his symptoms in follow up.

Fig.128.3 Concealment into the distal left bundle branch on the first
aberrant beat protects the left bundle from long-short stimulation Bibliography
(bymaking the pause shorter with reference to the left bundle than the
right) following the sinus complex that blocks at the infra His level. Bharati S, Lev M, Wu D, Denes P, Dhingra R, Rosen KM.
Thepattern reverses itself in the next part of the sequence, shielding the Pathophysiologic correlations in two cases of split His bundle
right bundle branch in the same manner potentials. Circulation. 1974;49:615-623.
Lerman BB, Marchlinski FE, Kempf FC, Buxton AE, Waxman HL,
Josephson ME. Prognosis in patients with intra-Hisian conduction
induced with atrial pacing, with an intra-His level of conduc- disturbances. Int J Cardiol. 1984;5:449-460.
tion block. The alternating bundle branch pattern was not McAnulty JH, Murphy E, Rahimtoola SH. Prospective evaluation of
intrahisian conduction delay. Circulation. 1979;59:1035-1039.
reproduced during the EP study, because a steady state pat-
Wu D, Denes P, Dhingra RC, et al. Electrophysiological and clinical
tern of 3:2 block could not be demonstrated, despite pacing observations in patients with alternating bundle branch block.
autonomic manipulations. Circulation. 1976;53:456-464.
Case 129

Andrew E. Darby and John P. DiMarco

Case Summary he was in sinus rhythm with first degree AV block and left
bundle branch block (Fig.129.1). He was later noted to have
type II second degree AV block alternating with periods of
A 51year-old man is referred for evaluation of heart failure
complete heart block (Fig.129.2).
with AV block. He had been in his usual state of health until
What disease process could account for this patients pre-
6 months earlier when he began to noted easy fatigue. He
sentation of heart failure with advanced AV block?
then developed gradually progressive dyspnea with exertion.
Two weeks prior to presentation, the patients dyspnea with
exertion progressed to the point of shortness of breath with
walking only fifty feet. The onset of orthopnea and paroxys-
mal nocturnal dyspnea finally prompted him to seek care in
Case Discussion
the emergency department.
His medical history includes hypertension, dyslipidemia, The differential diagnosis of heart failure with advanced AV
and obesity. He had donated his left kidney to a brother with block should include infiltrative processes (sarcoidosis or
polycystic kidney disease 13 years earlier. The patient does not myocarditis), lyme carditis, and certain genetic conditions
smoke or use illicit substances. He lives with a wife and one which may present in the fourth or fifth decades of life (lamin
child, and he works as a long-haul commercial truck driver. A/C deficiency or alpha-myosin heavy chain gene mutation).
The patient was afebrile on presentation. He was rela- This patients chest CT showed mediastinal lymphadenopa-
tively hypotensive with a blood pressure of 85/60, and he had thy which suggested sarcoidosis as the potential etiology. A
a heart rate of 90bpm. He was noted to clinically have heart cardiac MRI revealed patchy areas of delayed hyperenhance-
failure with elevated jugular venous pressure, bibasilar ment in a non-coronary distribution suggestive of an infiltra-
crackles, and lower extremity edema. tive process (Fig. 129.3). He subsequently underwent a
Studies obtained included a chest x-ray demonstrating lymph node biopsy which revealed noncaseating granulomas
cardiomegaly with pulmonary vascular congestion. His pre- diagnostic of sarcoidosis. The patients heart failure and
senting electrocardiogram is shown (Fig.129.1). A chest CT advanced AV block were therefore deemed secondary to
scan revealed pulmonary edema with prominent mediastinal cardiac sarcoidosis.
and paratracheal lymphadenopathy. A transthoracic echocar- Sarcoidosis is a chronic, multisystem disorder of unknown
diogram indicated severe left ventricular dysfunction with an etiology. It is characterized by the accumulation of T lympho-
approximate ejection fraction of 25%. A cardiac catheteriza- cytes and macrophages in tissues leading to the formation of
tion demonstrated no significant coronary disease. noncaseating granulomas which disrupt normal tissue archi-
The patients dyspnea improved with diuresis, but he tecture. The disorder may involve any organ system but most
developed further changes in his electrocardiogram. Initially commonly affects the lungs, skin, eyes, liver, and lymphatics.
Cardiac granulomas are found in nearly 25% of patients
with sarcoidosis examined at autopsy. Importantly, it accounts
for 1325% of sarcoidosis-related deaths. Cardiac involve-
A.E. Darby (*) ment may precede, follow, or occur concurrently with other
Department of Cardiology, University of Virginia, organ involvement. The most common cardiac manifesta-
1215 Lee Street, Charlottesville, VA 22908, USA
e-mail: aed6d@virginia.edu tions of sarcoidosis are conduction abnormalities, heart
failure and ventricular arrhythmias.
J.P. DiMarco
Cardiovascular Division, University of Virginia Health System,
Sarcoid granulomas have an affinity for the conduction
1215 Lee Street, Charlottesville, VA 22908, USA system. First degree AV block is common due to involve-
e-mail: jdimarco@virginia.edu ment of the AV node or bundle of His. Interventricular

A. Natale et al. (eds.), Cardiac Electrophysiology, 497


DOI: 10.1007/978-1-84996-390-9_129, Springer-Verlag London Limited 2011
498 A.E. Darby and J.P. DiMarco

Fig.129.1 Presenting electrocardiogram demonstrating a first-degree AV block and left bundle branch block

Fig.129.2 An electrocardiogram demonstrating high grade AV block. AV block with alternating bundle branch aberration or complete AV
Since there is now a right bundle branch block pattern and the original block with a ventricular escape rate the sinus rate
ECG showed a left bundle branch block, this may represent either 2:1

conduction defects (right or left bundle branch block) may Ventricular dysrhythmias may also occur, and patients
also be apparent on the electrocardiogram. The most com- with cardiac sarcoidosis are at increased risk for sudden death.
mon rhythm abnormality among patients with clinically- Infiltrating granulomas can cause inflammation with subse-
evident cardiac sarcoidosis is complete heart block, occurring quent scar formation. This process may create a substrate for
in up to 30% of patients. reentrant dysrhythmias. Ventricular tachyarrhythmias (VT)
Case 129 499

are the second most common mode of presentation of cardiac


sarcoidosis. Sustained or nonsustained ventricular tachycardia
are seen during Holter monitoring in about 23% of patients.
Sudden death due to ventricular arrhythmias or complete
heart block accounts for 2565% of deaths due to cardiac sar-
coidosis. Atrial arrhythmias are less common, occurring in
approximately 19% of patients. Only limited data about the
electrophysiologic findings in patients with sarcoid are avail-
able. Multiple VT morphologies are common in patients who
present with VT. Low voltage areas of scar can be seen in
either ventricle. Both epicardial and endocardial sites of VT
origin are possible. Heart block and VT may occur separately
or together.
Heart failure is another common mode of presentation
of cardiac sarcoidosis. Granulomatous infiltration and the
subsequent inflammatory response may damage the myocar-
dium resulting in systolic dysfunction. The infiltrative Fig.129.3 Cardiac magnetic resonance scan showing diffuse
process may also cause abnormalities in diastolic function.
Ventricular aneurysm formation has been noted to occur, and
immunosuppression with the addition of antimalarials,
they may be a focus for ventricular dysrhythmias.
methotrexate, or azathioprine. Patients with left ventricular
Diagnostic criteria for cardiac sarcoidosis have been pro-
dysfunction should be treated with standard medications
posed (Table129.1). The guidelines in the table do not incor-
including ACE inhibitors, beta-blockers, and diuretics if
porate updated imaging techniques, but they can serve as a
needed.
reference point. Cardiac MRI has emerged as an extremely
Patients with cardiac sarcoidosis often develop indica-
useful diagnostic tool as illustrated in Fig.129.3. Published
tions for permanent cardiac pacing. Strong consideration
reports indicate an approximate 100% sensitivity for detect-
should be given to ICD placement given the high risk of ven-
ing myocardial infiltration suggestive of sarcoidosis.
tricular arrhythmias and sudden cardiac death. Standard
Therapy for cardiac sarcoidosis primarily consists of
guidelines for primary and secondary prevention apply. ICD
immunosuppression. Corticosteroids, such as prednisone
placement has been suggested for primary prevention,
dosed 1mg/kg/day, are the standard of care. Steroid dosing
regardless of left ventricular function, for patients with fre-
should be gradually tapered based upon the clinical
quent ventricular ectopy or nonsustained ventricular tachy-
response. An inadequate response is managed by escalating
cardia. It is likely prudent to avoid amiodarone secondary to
the potential confounding effects of pulmonary toxicity in
patients with sarcoidosis.
Table129.1 Japanese Ministry of Health and Welfare Guidelines for Our patient was started on 70mg of prednisone daily. He
the Diagnosis of Cardiac Sarcoidosis
underwent dual-chamber pacemaker implantation and was
1. Histologic Diagnosis Group: endomyocardial biopsy demon- started on an ACE inhibitor and beta-blocker. The patient
strates epithelioid granulomata without caseating granulomata
declined an ICD as implantation would preclude continuing
2. Clinical Diagnosis Group: in patients with a histological
his job as a commercial truck driver.
diagnosis of extracardiac sarcoidosis, cardiac sarcoidosis is
suspected when (a) and at least one of criteria (b) to (d) is
present, and other etiologies such as hypertension and coronary
artery disease have been excluded:
a.Complete RBBB, LBBB, left axis deviation, AV block, VT, Bibliography
PVC, or pathological Q or ST-T change on resting or
ambulatory electrocardiogram Banba K, Kusano KF, Nakamura K, etal. Relationship between arrhyth-
b.Abnormal wall motion, regional thinning, or dilatation of the mogenesis and disease activity in cardiac sarcoidosis. Heart
left ventricle Rhythm. 2007;4:1292-1299.
c.Perfusion defect by thallium-201 myocardial scintigraphy or Dubrey SW, Bell A, Mittal T. Sarcoid heart disease. Postgrad Med J.
abnormal accumulation of gallium-67 or technetium-99m 2007;83:618-623.
myocardial scintigraphy Furushima H, Cinushi M, Sugiura H, Kasai H, Washizuka T, Aisawa Y.
d.Abnormal intracardiac pressure, low cardiac output, Ventricular tachyarrhythmia associated with cardiac sarcoidosis:
abnormal wall motion, or depressed ejection fraction of the itsmechanisms and outcome. Clin Cardiol. 2004;27:217-222.
left ventricle Hiraga H, Yuwai K, Hiroe M. et al. Guideline for the diagnosis of
e.Interstitial fibrosis or cellular infiltration over moderate grade cardiac sarcoidosis study report on diffuse pulmonary diseases. The
even if the findings are non-specific Japanese Ministry of Health and Welfare 1993; 2324.
500 A.E. Darby and J.P. DiMarco

Iannuzzi MC, Rybicki BA, Teirstein AS. Medical progress: sarcoidosis. Smedema JP, Snoep G, van Kroonenburgh MP, etal. Evaluation of the
N Engl J Med. 2007;357:2153-2165. accuracy of gadolinium-enhanced cardiovascular magnetic reso-
Koplan BA, Soejima K, Baughman K, Epstein LM, Stevenson WG. nance in the diagnosis of cardiac sarcoidosis. J Am Coll Cardiol.
Refractory ventricular tachycardia secondary to cardiac sarcoid: 2005;45:1683-1690.
electrophysiologic characteristics, mapping and ablation. Heart Syed J, Myers R. Sarcoid heart disease. Can J Cardiol. 2004;20:
Rhythm. 2006;3:924-929. 89-93.
Case 130

Thomas J. Sawyer, Burr W. Hall, and James P. Daubert

Case Summary previously, his brother collapsed and died suddenly while
running in his driveway at home. The patients paternal
grandfather had also died suddenly just days before his
A 13-year-old boy is being evaluated for syncope. At the brother. The 13-year-old boys ECG is shown below
time of the syncopal event, he was a national, junior tennis (Fig. 130.1). Representative echocardiographic images are
champion, but had recently complained of increased fatigue also shown (Figs.130.2 and 130.3). Of note, there was no
and shortness of breath while playing tennis. One month LV outflow obstruction noted at rest or with provocative

Fig.130.1 ECG at presentation

T.J. Sawyer(*)
Cardiac Study Center, 1901 South Cedar St.,
Suite 301, Tacoma, WA 98405, USA
e-mail: tsawyer11@gmail.com
B.W. Hall
Department of Cardiology, University of Rochester Medical Center,
601 Elmwood Avenue, Rochester, NY 14618, USA
e-mail: burr_hall@urmc.rochester.edu
J.P. Daubert
Cardiology Division, Duke University Health System,
DUMC Box3174, Duke Hospital 7451H, Durham, NC 27710, USA
e-mail: james.daubert@duke.edu

A. Natale et al. (eds.), Cardiac Electrophysiology, 501


DOI: 10.1007/978-1-84996-390-9_130, Springer-Verlag London Limited 2011
502 T.J. Sawyer et al.

at high risk for sudden death primarily as a result of ven-


tricular arrhythmias. Treatment of the symptoms associated
with HCM and identification of this high-risk subset of
patients is the primary goal of the initial evaluation of these
patients.

Findings in HCM

Classic physical findings include a systolic murmur that


increases with maneuvers, which decrease either preload or
afterload.2 Obstruction is not present in all patients (even
with Valsalva or exercise) and thus a murmur may not be
noted. Evidence of LVH is the hallmark of HCM. This is
Fig. 130.2 Parasternal long axis image late systolic frames. Note often suspected electrocardiographically and confirmed by
absence of SAM (systolic anterior motion of the mitral valve) 2D echocardiography. The hypertrophy is usually asymmet-
ric with hypertrophy of the septum being greater than that of
the free wall, but can be concentric or more predominant in
other regions.1,2

Work Up for HCM Patients

Once the diagnosis has been made, a detailed family history


should be obtained. Special attention is given to a history of
sudden death or unexplained syncope. All first degree relatives
should undergo echocardiographic screening. Initial evalua-
tion includes a 48h Holter monitor and an exercise test. All
patients should be told to avoid dehydration and strenuous
exercise.

Fig.130.3 Parasternal long axis image early systolic frames. Note


absence of SAM (systolic anterior motion of the mitral valve) Risk Stratification in HCM

aneuvers What is your diagnosis? What is an appropriate


m Obviously, the most feared complication of HCM is sudden
management strategy? cardiac death. Despite intense work in this area, the com-
plexity of the disease continues to make the identification of
high risk markers for sudden death problematic. A prior
cardiac arrest or sustained VT denotes very high risk for
sudden death and clearly warrants an ICD. Stratification of
Case Discussion
risk is usually focused on the younger patient as the attain-
ment of advanced age usually indicates a more benign form
Hypertrophic cardiomyopathy (HCM) affects approxi- of the disease. Most experts agree that the factors listed
mately 1 in 500 people (130.2). Thirty to fifty percent of below suggest particularly high risk for the patient without
patients with HCM have associated dynamic left ventricular a prior arrest (primary prevention of sudden cardiac
outflow tract (LVOT) obstruction with even a higher percent arrest)1:
having obstruction with exercise. Hemodynamically based
symptoms consist primarily of shortness of breath and Family history of sudden death
decreased exercise tolerance. Overall mortality in HCM is Syncope unlikely to be neurocardiogenic in etiology
<1% per year.1,2 However, there is clearly a subset of patients Left ventricular wall thickness >30mm
Case 130 503

Abnormal blood pressure response to exercise (hypoten- scar in the basal septal region potentially puts this patients at
sion or failure to increase BP) high-risk for subsequent tachyarrhythmic events.1,2
Non-sustained ventricular tachycardia

Other less potent or agreed upon risk factors include


LVOTobstruction, specific genotype, atrial fibrillation, and Prevention of Sudden Cardiac Death
ischemia.
For patients at high-risk for sudden cardiac death, an implant-
able defibrillator is the treatment of choice. There are no
Treatment definitive criteria to unequivocally determine who will ben-
efit from defibrillator implantation. A prior cardiac arrest or
sustained VT is a clear indication for an ICD. The presence
In the majority of patients, an implantable defibrillator is not of two or more risk factors is widely agreed upon to be a
necessary and therapy is focused on the relief of symptoms. good indication for an ICD.5 Recent data from Maron, etal.
Agents which block the effects of catecholamines and improve suggest that any patient with one or more of the five major
diastolic function are the mainstays of therapy. Increased dia- risk factors listed above should be considered for an ICD, as
stolic filling time improves LVOT obstruction in patients with the chance of experiencing ICD therapy in follow-up was
this variety of the disease. Beta-blockers are usually the initial approximately as high for patients with one risk factor as for
therapeutic choice. Verapamil has also been used effectively those with two or more.6
and, like beta-blockers, has negative chronotropic and inotro-
pic effects without significant alterations in afterload.1 If
patients remain symptomatic despite optimal pharmacologic
therapy, then several invasive options may be considered. References
Surgical septal myomectomy is the gold standard for the
treatment of symptomatic, obstructive cardiomyopathy 1. Nishimura RA, Holmes DR. Hypertrophic Obstructive Cardio
refractory to medical therapy. Successful operations can myopathy. NEJM. 2004;350:1320-1327.
result in complete resolution of both mitral regurgitation and 2. Maron BJ. Hypertrophic cardiomyopathy. A systematic review.
outflow gradient. Excellent long-term follow-up has been JAMA. 2002;287:1308-1320.
3. Elliott PM, Poloniecki J, Dickie S, etal. Sudden death in hypertro-
achieved with sustained improvement in exercise capacity phic cardiomyopathy: identification of high risk patients. J Am Coll
and symptoms. Major complications typically occur in <3% Cardiol. 2000;36:2212-2218.
and include complete heart block, aortic insufficiency, septal 4. Maron BJ, Spirito P, Shen W, et al. Implantable cardioverter-
defects, and death.1,2 defibrillators and prevention of sudden cardiac death in hypertrophic
cardiomyopathy. JAMA. 2007;298:405-412.
Alcohol induced ablation is performed when 100% alcohol
is selectively perfused into a septal perforator supplying the
proximal septum. This induces a controlled myocardial infarc-
tion ultimately resulting in thinning of the basal septal region. Bibliography
Initial results of this procedure demonstrate improved hemo-
dynamics and exercise tolerance but not to the extent seen
Maron BJ, Roberts WC, McAllister HA, etal. Sudden death in young
with surgery. Heart block remains a major complication seen athletes. Circ. 1980;62:218-229.
in 1520%, although lower in some series. The production of Maron BJ. Sudden death in young athletes. NEJM. 2003;349:1064-1075.

Case 131

John P. DiMarco

Case Summary heart rate is 230bpm and her blood pressure is 90 systolic.
She is alert and oriented but aware of her rapid heart rate.
During placement of an intravenous line she converts to sinus
A 21-year-old woman presents to the emergency room with
rhythm (Fig.131.2).
a sustained tachycardia (Fig.131.1). She has a 3 year history
If she had failed to convert spontaneously, what would
of intermittent palpitations but prior episodes have lasted
have been the most appropriate acute management? What
only a few minutes and she has never consulted a physician
would you recommend for chronic therapy?
for this problem. In the emergency department her initial

Fig.131.1 Supraventricular tachycardia on presentation to the emergency room

J.P. DiMarco
Cardiovascular Division, University of Virginia Health System,
1215 Lee Street, Charlottesville, VA 22908 USA
e-mail: jdimarco@virginia.edu

A. Natale et al. (eds.), Cardiac Electrophysiology, 505


DOI: 10.1007/978-1-84996-390-9_131, Springer-Verlag London Limited 2011
506 J.P. DiMarco

Fig.131.2 ECG showing sinus


rhythm with preexcitation after
conversion. The delta waves
polarity suggests a right anterior
pathway location

Case Discussion pathways, this is not a contraindication to its use in


orthodromic AVRT.
The initial ECG during tachycardia shows a regular narrow After conversion, the ECG shows preexcitation. The delta
complex rhythm with a cycle length of 260ms. The early wave polarity is consistent with an anterior or anteroseptal
ST segment is notched suggesting a short RP tachycardia. pathway. Although the anterograde ERP of the pathway, a
In this clinical situation, AV reentrant tachycardia (AVRT) major determinant of risk for sudden death, is unknown, we
is the most likely diagnosis and termination should be pos- do know the pathways retrograde properties can support a
sible if transient AV nodal block can be achieved. Vagal rapid SVT. Catheter ablation of the accessory pathway there-
maneuvers (e.g., carotid massage, breath holding, Valsalva) fore would represent optimal therapy if it can be safely per-
may be effective, particularly in the first few minutes of an formed. In this patient, the probable location of the accessory
episode. More sustained episodes typically require intrave- pathway raises the possibility of inadvertent damage to the
nous drug therapy. Adenozine, verapamil and diltiazem are normal conduction system during mapping and ablation. In
the drugs of choice and all three agents are highly effective this situation, many electrophysiologists would prefer to use
in AV nodal dependent arrhythmias. Most authors would a cryoablation technique rather than use radiofrequency
prefer adenozine in a patient with moderately severe since the initial response with the former is reversible. In this
hypotension but, in most cases, the hypotension will resolve patient, electrophysiologic study confirmed a site adjacent to
if the SVT is terminated by any agent. Although adenozine the His bundle and a successful cryoablation was carried out
may transiently shorten the refractory period of accessory (Figs.131.3 and 131.4).
Case 131 507

Fig.131.3 Mapping of the 0 1000 2000


accessory pathway location. The ABLATION SITE
shortest AV interval was located
just adjacent to the His bundle I
recording site DELTA

II

aVF

V1

HRA A A A
V V V
ABLATED

HBED

RVA

0 1000 2000

Speed: 100 mm/sec Time: 00: 18: 31 Study #1 Protocol #7 Protocol Name: AAP University of Virqinie Medical Center

0 1000 2000
30 Degrees Centigrade
I

II

aVF

V1

DCS

ABLATED
Fig.131.4 Cryomapping of an A
A H
anteroseptal accessory pathway.
The cryo ablation catheter was HBED
used to map the shortest AV
interval and then loss of
RVA
preexcitation was seen when
30 C was reached. Completion
of the lesion was then perfomed
at 70 C for 4 min with 0 1000 2000
continued monitoring of AV
conduction Speed: 100 mm/sec Time: 00: 13: 37 Study #1 Protocol #12 Protocol Name: Mapping University of Virqinie Medical Center

Case 132

John P. DiMarco

Case Summary Case Discussion

A 17-year-old man presents to the emergency room com- The second tracing shows an irregular rhythm with both nar-
plaining of 2 h of rapid palpitations and chest pressure. His row and wide (preexcited) beats and is most consistentwith
ECG is shown in Fig. 132.1. Hes had several similar epi- atrial fibrillation with intermittent preexcitation. Although
sodes in the past but those episodes had terminted spontane- adenosine is a primary option for terminating PSVT, it has
ously and he had not sought medical attention. His blood several electrophysiologic effects that may lead to proar-
pressure during tachycardia is 100/70. He is given 12mg of rhythmia. Adenosine stimulates an outward potassium cur-
intravenous adenosine. His tachycardia converts to a differ- rent in atrial cells that shortens action potential duration and
ent rhythm as shown in Fig.132.2. this facilitates atrial fibrillation induction by spontaneous
What is the second rhythm and was it related to the ectopy. Adenosine also transiently shortens the effective
adenosine? refractory periods of most accessory pathways and may

Fig.132.1 Narrow complex


tachycardia on presentation to
the emergency room

J.P. DiMarco
Cardiovascular Division, University of Virginia Health System,
1215 Lee Street, Charlottesville, VA 22908 USA
e-mail: jdimarco@virginia.edu

A. Natale et al. (eds.), Cardiac Electrophysiology, 509


DOI: 10.1007/978-1-84996-390-9_132, Springer-Verlag London Limited 2011
510 J.P. DiMarco

Fig.132.2 Irregular tachycardia


recorded after adenosine
administration (simultaneous
12lead ECG reformatted)

0 1000 2000 3000 4000


ADENOSINE INDUCED ATRIAL FIBRILLATION

aVF

V1

HRA

HISD

DCS

MCS2
Fig.132.3 Effect of adenosine
at EP Study. Adenosine had been
PCS
administered during an episode
of AV reentrant tachycardia. The
RVA
tachycardia terminated just
before this recording. After the +140
fourth beat, a spontaneous atrial Arterial
premature beat initiated atrial
fibrillation which is conducted 0
over both the AV node and the 0 1000 2000 3000 4000
accessory pathway

accelerate ventricular rates in preexcited atrial arrhythmias. Bibliography


For these reasons, adenosine should only be used with con-
tinuous ECG monitoring with facilities for resuscitation Exner DV, Muzyka T, Gillis AM. Proarrhythmia in patients with the
available. This patient also received an initial adenosine dose Wolff-Parkinson-White syndrome after standard doses of intrave-
of 12 mg. Since many patients will respond to the lower nous adenosine. Ann Intern Med. 1995;122:351-352.
Gupta AK, Shah CP, Maheshwari A, Thakur RK, Hayes OW,
recommended initial dose (i.e., 6mg), the lower dose is pre-
Lokhandwala YY. Adenosine induced ventricular fibrillation in
ferred even if more patients will require two doses to termi- Wolff-Parkinson-White syndrome. Pacing Clin Electrophysiol.
nate SVT. In this patient, the preexcited atrial fibrillation 2002;25:477-480.
spontaneously converted to sinus rhythm after about 45 s and
no intervention was required. During his electrophysiologic
study, a 30 s episode of atrial fibrillation was again observed
when adenosine was readministered (Fig.132.3).
Case 133

John P. DiMarco

Case Summary twice daily but did not add any other medications because the
patient was relatively bradycardic at rest. Three days later
thepatient became acutely short of breath and lightheaded.
A 57-year-old man presented to his local cardiologist with a
He called the rescue squad and was taken to the emergency
complaint of intermittent palpitations. He had a history of
room. His ECG on presentation is shown in Fig.133.1.
mild hypertension for which he was being treated with lisono-
What is the mechanism of this arrhythmia? How might
pril. He had no symptoms of angina and an exercise test
this have been prevented?
1year ago had been negative for signs of ischemia. He had a
chronic left anterior fascicular block. An event recorder was
scheduled and the patient transmitted several episodes of par-
oxysmal atrial fibrillation. An echocardiogram showed mild Case Discussion
left atrial enlargement but was otherwise normal. Routine
laboratory studies were normal. Anticoagulation was dis- The tracing shows a regular wide complex tachycardia with a
cussed but in view of a CHADS2 score of 1, he was started on rate of 200bpm. The QRS duration is about 140ms. AV
aspirin. The cardiologist started him on flecainide 100 mg dissociation is not identifiable but there is extreme left axis

Fig.133.1 Wide complex tachycardia on presentation to the emergency room

J.P. DiMarco
Cardiovascular Division, University of Virginia Health System,
1215 Lee Street, Charlottesville, VA 22908, USA
e-mail: jdimarco@virginia.edu

A. Natale et al. (eds.), Cardiac Electrophysiology, 511


DOI: 10.1007/978-1-84996-390-9_133, Springer-Verlag London Limited 2011
512 J.P. DiMarco

Fig.133.2 Tachycardia 2 h later after administration of metoprolol

deviation. The physicians in the emergency room took the patients, flutter with 1:1 conduction can be prevented if AV
history that the patient had just recently started flecainide for nodal blocking agents are coadministered with either
paroxysmal atrial fibrillation and suspected that this may be a flecainide or propafenone. In some patients, as in the patient
proarrhythmic effect of the flecainide. Atrial proarrhythmia in this case, this may be difficult because of resting sinus
with flecainide is relatively uncommon but this setting is a bradycardia.
typical situation in which it might be expected. Often patients When patients present with wide complex regular tachy-
who previously had atrial fibrillation will have organization of cardia on flecainide for paroxysmal atrial fibrillation, physi-
their arrhythmia to atrial flutter during therapy with either fle- cians should be aware of the possibility of proarrhythmia. In
cainide or propafenone. The flutter cycle length is often longer many cases, a trial of adenosine or another AV nodal block-
than normal because of the effects of the drug. In addition, ing agent will help make the ECG diagnosis when the patient
with the rapid ventricular rate, bundle branch block is often presents.
observed. If the patient is exercising or is not also on an AV
nodal blocking agent, 1:1 conduction with ventricular rates of
200bpm or more may occur. Since Ic antiarrhythmic agents,
like flecainide and propafenone show use dependence, their Bibliography
sodium channel blocking effects become more pronounced at
high rates also setting the stage for ventricular proarrhythmia. Feld GK, Chen PS, Nicod P, Fleck P, Meyer D. Possible atrial proar-
This patient received intravenous metoprolol and a trac- rhythmic effects of class 1C antiarrhythmic drugs. Am J Cardiol.
1990;66:378-383.
ing obtained later that day, before he underwent elective car- Kawabata M, Hirao K, Higuchi K, etal. Clinical and electrophysiologi-
dioversion, is seen in Fig.133.2. Note that the right bundle cal characteristics of patients having atrial flutter with 1:1 atrioven-
branch block pattern has resolved at the lower rate. In most tricular conduction. Euro Pace. 2008;10:284-288.
Case 134

John P. DiMarco

Case Summary catheter had been placed in the coronary sinus at this time.
The tracings show initiation of a narrow complex tachycar-
dia with two atrial extrastimuli. There is a critical AH delay
A 41-year-old woman presents to the emergency room with
and the VA time during the tachycardia is less than 70ms.
palpitations and dyspnea. She had had intermittent palpita-
This tachycardia was initiated on two more occasions and
tions for many years. In the past, the spells had lasted only a
the VA relationship did not change. Ventricular pacing dur-
few minutes and then had terminated spontaneously. She had
ing tachycardia produced a VAV response (not shown). Later
learned how to break most of the episodes by either bearing
in the study, the patient spontaneously developed the rhythm
down or rubbing on her neck. Today, she had been involved
shown in Fig.134.3. The 12 lead ECG during this rhythm
in an argument at work and then developed sustained palpita-
matched the ECG that had been seen in the emergency room.
tions that have persisted for the past 3 h. She states that at
Block below the His spike is seen. The most likely diagnosis
first the palpitations were more severe but that since she has
for these findings is 2:1 infra-Hisian block during AV node
arrived in the hospital, she feels more comfortable. Her elec-
reentrant tachycardia.
trocardiogram is shown in Fig.134.1.
Spontaneous 2:1 AV block during AV node reentrant
What is the most probable mechanism for the arrhythmia?
tachycardia is relatively uncommon in the emergency room
How frequently is this observed?
but can be seen in up to 10% of patients at the time of elec-
trophysiologic study. As reported by Mann et al., 2:1 AV
block during AVNRT is usually due to functional infranodal
Case Discussion block seen because of the rapid rate of the AV nodal reentry.
One also must question whether catheter trauma to the con-
duction sytem or the common use of isoproterenol for
The initial ECG shows a narrow complex tachycardia at a
induction contributes to the increased frequency during
rate of approximately 110bpm. A P wave is visible in the
electrophysiologic studies compared to spontaneous pre-
middle of the RR interval. The P wave morphology appears
sentations. Clinical clues to this diagnosis are the position
to be biphasic in the inferior leads. The differential diagnosis
and morphology of the P wave and the fact that patients
for this rhythm would include an atrial tachycardia, atypical
often report sudden changes in the rate of the tachycardia as
atrial flutter with 2:1 conduction, AV reentry with a relatively
they switch from 1:1 to 2:1 conduction.
long RP interval, and a variant of AV node reentry.
The patient was referred for electrophysiologic study.
Initiation of tachycardia is shown in Fig.134.2. An ablation

J.P. DiMarco
Clinical Electrophysiology Lab, Cardiovascular Division,
University of Virginia Health System, 1215 Lee Street,
Charlottesville, VA 22908, USA
e-mail: jdimarco@virginia.edu

A. Natale et al. (eds.), Cardiac Electrophysiology, 513


DOI: 10.1007/978-1-84996-390-9_134, Springer-Verlag London Limited 2011
514 J.P. DiMarco

Fig.134.1 Tachycardia on presentation in the emergency room

II

aVF

V1

HRA

DCS
ABLATED

ABLATEP PCS

HBED

Fig.134.2 Initiation of typical


AV nodal reentrant tachycardia RVA
with double atrial extrastimuli.
The ablation catheter at this point
had been placed in the coronary 0 1000 2000
sinus
Case 134 515

Fig.134.3 Spontaneous 2:1 AV


block during an episode of I
tachycardia. Note that there is a
1:1 HA relationship with slow
II
pathway anterograde conduction
and fast pathway retrograde
conduction, during the period of aVF
AV block
V1

HRA

DCS

PCS

HBED

HBEP

RVA

0 1000 2000

Bibliography

Mann KC, Prinkman K, Bogun F, etal. 2:1 atrioventricular block dur-


ing atrioventricular node reentrant tachycardia. J Am Coll Cardiol.
1996;28:1770-1774.

Case 135

John P. DiMarco

Case Summary does well for 3 weeks, then reports several episodes of palpita-
tions that occurred when she was running. Her ECG does not
show recurrence of the prior pattern of preexcitation.
A 23-year-old female, medical student is referred from the
What do you think is the most likely mechanism for the
emergency room after an episode of supraventricular tachycar-
recurrent tachycardia?
dia. After conversion in the emergency room, her ECG had
shown preexcitation (Fig.135.1). She undergoes electrophysi-
ologic study and initiation of tachycardia is shown in Figs.135.2 Case Discussion
and 135.3. A left sided accessory pathway is mapped and suc-
cessfully ablated with a single lesion using a transseptal The patient clearly initially has Wolff-Parkinson-White syn-
approach. There is no evidence of accessory pathway conduc- drome with a delta wave on her surface electrocardiogram
tion at the end of the case and her ECG is now normal. She consistent with a left-sided accessory pathway. However,

I aVR V1 V4

II aVL V2 V5

III aVF V3 V6

III

Fig.135.1 Narrow complex tachycardia on presentation to the emergency room. A retrograde Pwave is easily seen in the early ST segment in
many leads

J.P. DiMarco
Cardiovascular Division, University of Virginia Health System,
1215 Lee Street, Charlottesville, VA 22908 USA
e-mail: jdimarco@virginia.edu

A. Natale et al. (eds.), Cardiac Electrophysiology, 517


DOI: 10.1007/978-1-84996-390-9_135, Springer-Verlag London Limited 2011
518 J.P. DiMarco

Fig.135.2 12 lead ECG after tachycardia termination. The delta wave polarity suggests a left free wall pathway

Fig.135.3 Initiation of tachycardia during the first EP study


Case 135 519

when tachycardia was initiated during her electrophysiologic is likely to be a future clinical problem and she probably
study, the first three beats have a different retrograde atrial should have undergone mapping and ablation of the slow
activation sequence. The VA interval for the first three beats pathway at her initial study. At a follow-up study, there was
is most consistent with AV reentry and then the patient no evidence for recovery of conduction over the accessory
switches to eccentric retrograde activation over the accessory pathway. Sustained AV nodal reentry could be initiated dur-
pathway. This phenomena occurred with only two of many ing low dose isoproterenol. After a slow pathway ablation,
SVT initiations during the electrophysiologic study and was she has remained symptom free.
not noted by the operators.
The coexistence of dual AV nodal pathways and acces-
sory pathways is relatively common in people with Wolff-
Parkinson-White syndrome. In a recent report by Sclapfer Bibliography
and Fromer, recuurent tachycardia after ablation was caused
by recovery of accessory pathway conduction and by AV Calkins H, Yong P, Miller JM, etal. Catheter ablation of accessory path-
node reentry in approximately equal numbers of patients. ways, atrioventricular nodal reentrant tachycardia, and the atrioven-
tricular junction: final results of a prospective, multicenter clinical
Palpitations may also be reported after ablation even when trial. Atakr Multicenter Investig Group Circ. 1999;99:262-270.
monitoring shows only sinus rhythm of atrial or ventricular Schlapfer J, Fromer M. Late clinical outcome after successful radiofre-
premature beats. It is often a difficult clinical decision as to quency catheter ablation of accessory pathways. Euro Heart J.
whether to proceed to slow pathway ablation if this is an 2001;22:605-609.
Schluter M, Cappato R, Ouyang F, Antz M, Schluter CA, Kuck KH.
incidental finding during an electrophysiologic study. Clinical recurrences after successful accessory pathway ablation:
However, in this case, the fact that three AV nodal echo beats the role of dormant accessory pathways. J Cardiovascr Electro
are seen at the beginning of the tachycardia suggests that this physiol. 1997;8:1366-1372.

Case 136

John P. DiMarco

Case Summary At electrophysiologic study, a tachycardia is induced with


atrial pacing and the response to adenosine is assessed
(Fig.136.2).
A 35-year-old man has a several year history of recurrent
What is the likely mechanism of the tachycardia?
palpitations. He has no history of structural heart disease and
a 2-D echocardiogram obtained 2 years ago was normal.
Recently, he presented to his local emergency room and was Case Discussion
found to be in a wide complex tachycardia (Fig.136.1). He
was electrically cardioverted and referred for evaluation. His The initial tracing during tachycardia shows a regular wide
ECG at baseline shows sinus rhythm and was felt to be within complex tachycardia with a left bundle branch block mor-
normal limits. phology. AV dissociation is not evident. The QRS axis does

Fig.136.1 Wide comple tachycardia with a left bundle bracnch block configuration on presentation to the emergency room

J.P. DiMarco
Clinical Electrophysiology Lab, Cardiovascular Division,
University of Virginia Health System, 1215 Lee Street,
Charlottesville, VA 22908, USA
e-mail: jdimarco@virginia.edu

A. Natale et al. (eds.), Cardiac Electrophysiology, 521


DOI: 10.1007/978-1-84996-390-9_136, Springer-Verlag London Limited 2011
522 J.P. DiMarco

Fig.136.2 Findings at EP study.


The first four beats show the I
wide complex, preexcited
tachycardia with conduction over
the atriofascicular pathway. II
Adenosine, 9mg, was infused
and the tachycardia stops when
block during anterograde
conduction over the atriofascicu- aVF
lar pathway is observed. Note
that the next sinus beat is
normally conducted but that V1
effects on the AV node are still
present and the second sinus
beats conducts over the HRA
atriofascicular tract

HIS d

HIS p

RVA

not suggest a right ventricular outflow tract VT. During the morphology, often with a left axis. Unlike most other acces-
EP study, there is a 1:1 VA relationship but the wide QRS sory pathways, conduction over atriofascicular pathways is
complexes are not preceded by a His potential making aber- adenosine sensitive as shown in Fig.136.2. During mapping
rancy unlikely. Adenosine terminates the tachycardia by pro- the appropriate ablation site can be located by recording a
ducing block during antegrade conduction. These findings, discrete potential near the tricuspid annulus.
and the normal ECG in sinus rhythm, should suggest that the
patient has an atriofascicular fiber, also known as a Mahaim
tract. Atriofascicular pathways typically originate in the
Bibliography
right atrium and the fiber traverses the tricuspid annulus and
inserts into the distal right bundle. Atriofascicular fibers have
Ellenbogen KA, Rogers R, Old W. Pharmacological characterization of
relatively slow antegrade conduction and do not manifest ret- conduction over a Mahaim fiber: evidence for adenosine sensitive
rograde conduction. Preexcitation is usually not evident dur- conduction. Pacing Clin Electrophysiol. 1989;12:1396-1404.
ing sinus rhythm but can be brought out by atrial, especially Prystowsky E, Yee R, Klein GJ. Wolff-Parkinson-White syndrome. In:
low right atrial, pacing. Due to the pathways distal insertion, Zipes DP, Jalife J, eds. Cardiac Electrophysiology: From Bench to
Bedside. 4th ed. Philadelphia, PA: Saunders; 2004:868-878.
the local electrogram at the RV apex precedes the ventricu- Sternick EB, Cruz FES, Timmermans C, etal. Electrocardiogram during
lar electrogram on the His catheter during tachycardia. The tachycardia in patients with anterograde conduction over aMahaim
classic Mahaim tract tachycardia has a left bundle branch fiber: old criteria revisited. Heart Rhythm. 2004;4:406-413.
Case 137

John P. DiMarco

Case Summary a nodofascicular pathway, and AV node reentry with proxi-


mal common pathway retrograde block.
The patient was started on carvedilol and his average
A 24-year-old African-American man is referred for evalua-
heart rate over 24 h decreased to about 110bpm. His symp-
tion of persistent tachycardia. The patient states that he was
toms of heart failure improved slightly, but after 4 weeks of
told as a child that his heart rate had always been too high.
therapy, his left ventricular dysfunction was still severely
At age 17, he was evaluated by a pediatric cardiologist
depressed. The patient was taken to the electrophysiology
because of a persistent tachycardia detected during a high
(EP) laboratory. In the EP lab, he was in the same tachycar-
school physical. At that time, an echocardiogram was repo-
dia in an incessant fashion. Initiation of the tachycardia
rted to be normal and he was cleared for a sports partici
could not be tested because of the tachycardias continuous
pation. Recently, he has developed moderate dyspnea on
nature. Adenosine transiently slowed but did not terminate
exertion.
the tachycardia. No VA conduction was present. A His spike
On physical examination, he was a muscular young man
preceded each QRS complex (Fig.137.2). His synchronous
in no acute distress. His jugular venous pressure was elevated
PVCs had no effect on the tachycardia. No evidence for pre-
to 68cm above the clavicle but no murmur or gallop was
excitation was seen with atrial stimulation. These findings
heard. His pulse was irregular with a heart rate of app
are felt to be most consistent with a nonparoxysmal junc-
roximately 115bpm. His ECG is shown in Fig.137.1. Atwo-
tional tachycardia.
dimensional echocardiogram showed four chamber enlar-
Nonparoxysmal junctional tachycardia is a relatively
gement and severe left ventricular dysfunction.
uncommon arrhythmia. It is very rare for it to present in an
What is the probable mechanism for the tachycardia?
incessant fashion and to manifest VA block as it did in this
patient. In one series, only two of eleven patients had VA
block during junctional tachycardia. In this patient, it was
probably the cause of a tachycardia-related cardiomyopathy.
Case Discussion Therapy for junctional tachycardia can include drug therapy
and/or selective catheter ablation. The latter carries some
This case represents an incessant, probably lifelong, tachy- risk of producing AV block.
cardia presenting with congestive heart failure in a young Mapping of the AV junctional area was performed using a
individual. The ECG shows a narrow complex tachycardia cryoablation catheter. Cryoablation lesions were placed in
with dissociated P waves. When the P wave is able to con- the region of the His bundle with careful monitoring of AV
duct, fusion beats are seen. conduction (Fig.137.3). Eventually, the junctional tachycar-
A narrow complex tachycardia with VA block is a very dia was eliminated and the patient remained in sinus rhythm
rare phenomenon. The differential diagnosis includes junc- (Fig. 137.4). Three months later his ejection fraction had
tional tachycardia with VA block, reentrant tachycardia using returned to normal.

J.P. DiMarco (*)


Clinical Electrophysiology Lab, Cardiovascular Division,
University of Virginia Health System, 1215 Lee Street,
Charlottesville, VA 22908, USA
e-mail: jdimarco@virginia.edu

A. Natale et al. (eds.), Cardiac Electrophysiology, 523


DOI: 10.1007/978-1-84996-390-9_137, Springer-Verlag London Limited 2011
524 J.P. DiMarco

Fig.137.1 ECG at the time of referral. The patient was taking carvedilol. 12.5mg bid, at this time

II

aVF

V1

HRA

HIS d

HIS p
Fig.137.2 Intracardiac
recordings made during the
patients EP study. Note the
His spike before QRS and the RVA
AV dissociation except for
fusion beats
Case 137 525

Fig.137.3 Cryoablation of
junctional tachycardia. As the I
temperature on the ablation
catheter falls to 30C, the
junctional tachycardia slows and II
the patient is in sinus rhythm.
The lesion was then continued at
a lower temperature for 4 min
aVF

V1

HRA

HIS d

ABLD

RVA

I aVR V1 V4

II aVL V2 V5

III aVF V3 V6

Fig.137.4 Final ECG at the end


of the procedure

Bibliography Scheinman MM, Gonzalez RP, Cooper MW, Lesh MD, Lee RJ, Epstein
LM. Clinical and electrophysiologic features and role of catheter
ablation techniques in adult patients with automatic atrioventricular
Hamdan MH, Kalman JM, Lesh MD, etal. Narrow complex tachycar- junctional tachycardia. Am J Cardiol. 1994;74:565-572.
dia with VA block: diagnostic and therapeutic implications. Pacing
Clin Electrophysiol. 1998;21:1196-1206.
Hamdan MH, Page RL, Scheinman MM. Diagnostic approach to nar-
row complex tachycardia with VA block. Pacing Clin Electrophysiol.
1997;20:2984-2988.

Case 138

John P. DiMarco

Case Summary appropriate anticoagulation for stroke prevention and whether


he wishes to pursue a rate control a rhythm control strategy.
The patients age and his history of hypertension give him
A 79-year-old man is referred because of recent onset atrial
a CHADS2 score of two indicating that he should be placed
fibrillation. He has a history of hypertension and had two bare
on warfarin therapy unless he has contraindications. No con-
metal stents placed in his right coronary artery after presenting
traindications to long-term anticoagulation were present and
with an acute coronary syndrome 3 years ago. Subsequently,
he was begun on warfarin.
he did well on a medical regimen of enalapril, hyrochlorthi-
The decision on rate control versus rhythm control is some-
azide, metoprolol, atorvastatin and aspirin. He had a brief
what more difficult. Due to his history of ischemic heart disease,
episode of atrial fibrillation 6 months ago that was associated
he would not be a good candidate for a class 1C antiarrhythmic
with a urinary tract infection. He also describes several addi-
drug. Sotalol or dofetilide could be used but he does have mild
tional self-terminating episodes of minor palpitations, which
LVH on his echo and LVH is a potential risk factor for drug-
were not documented electrocardiographically, in the past
induced torsades de pointes. Amiodarone or dronedarone would
several months. Last week he presented to his local physician
therefore be the likely choice for drug therapy and they are
for a routine office visit and the electrocardiogram shown in
likely to be help maintain sinus rhythm in this situation. Catheter
Fig.138.1 was obtained. He was only aware of some minor
ablation would also be an option but the patient did not want to
irregularities in his heart rate and had noted a recent slight
undergo any invasive procedures unless absolutely necessary.
decrease in his exercise tolerance. These symptoms had not
An important point is that the patient had only minimal symp-
been severe enough for him to seek medical attention. A
toms at his initial presentation even though his rate wasnt opti-
two-dimensional echocardiogram showed a preserved left
mally controlled. Therefore, a rate control strategy is also an
ventricular ejection fraction with mild left ventricular
option. It would involve only relatively simple drugs and would
hypertrophy and moderate left atrial enlargement.
have a high probability of keeping him symptom free.
What would you recommend now?
There have been seven published or reported randomized
clinical trials that have compared rate control and rhythm con-
trol strategies in patients with persistent and/or paroxysmal
Case Discussion atrial fibrillation. The largest trial, AFFIRM, showed that there
was no difference in overall survival between the two strategies
This patient gives a typical history for an elderly patient who with a slight trend in favor of rate control. The other studies
presents with atrial fibrillation. He has a history of hyperten- noted that sinus rhythm may be difficult to maintain and that a
sion, one of the most common risk factors for atrial fibrillation. rhythm control strategy was associated with more therapeutic
He has had several episodes of palpitations and atrial fibrillation complications and more hospitalizations. Even in patients with
suggesting that this will be a recurrent problem implying that congestive heart failure, data from the Atrial Fibrillation-
just a cardioversion without drug therapy would not be helpful. Congestive Heart Failure Study (AF-CHF) show that there was
The options for therapy that should be considered include no improvement in survival with a rhythm control strategy.
This patient elected to follow a strategy of rate control and
anticoagulation. He did well on warfarin with no bleeding
complications. His metoprolol dose was increased and he
J.P. DiMarco reported that his exercise tolerance was back to his prior base-
Clinical Electrophysiology Lab, Cardiovascular Division,
line. Several office visits showed resting heart rates of 6080
University of Virginia Health System,
1215 Lee Street, Charlottesville, VA 22908, USA beats per minute. A follow-up electrocardiogram is shown in
e-mail: jdimarco@virginia.edu Fig.138.2. During monitored exercise at his rehab facility, his

A. Natale et al. (eds.), Cardiac Electrophysiology, 527


DOI: 10.1007/978-1-84996-390-9_138, Springer-Verlag London Limited 2011
528 J.P. DiMarco

Fig.138.1 ECG at the time of his initial presentation

Fig.138.2 Follow-up ECG obtained during treatment with metoprolol, 50mg twice daily

peak heart rate during a symptom limited exercise test was Crijns HJ. Rate versus control in patients with atrial fibrillation: what
130bpm. He completed stage 3 of a Bruce protocol. He has the trials really say. Drugs. 2005;65:1651-1667.
Fuster V, Ryden LE, Cannom DS, etal. ACC/AHA/ESC 2006 guide-
continued to be asymptomatic over several years follow-up. lines for the management of patients with atrial fibrillation: a report
of the American College of Cardiology/American Heart Association
Task Force on practice guidelines and the European Society of
Cardiology Committee for Practice Guidelines (Writing Committee
Bibliography to Revise the 2001 guidelines for the management of patients with
atrial fibrillation) developed in collaboration with the European
Heart Rhythm Association and the Heart Rhythm Society. Europace.
Camm AJ, Kirchof P, Lip GYK, et al. Guidelines for the management 2006;8:651-745.
of atrial fibrillation. The Task for the Management of Atrial Wyse DG, Waldo AL, DiMarco JP, etal. A comparison of rate control
Fibrillation of the European Society of Cardiology. Euro Heart and rhythm control in patients with atrial fibrillation. N Engl J Med.
J2010;31:2369-2429. 2002;347:1825-1833.
Case 139

John P. DiMarco

Case Summary

An 85-year-old woman with a history of hypertension had


presented with atrial fibrillation with a rapid ventricular
response 6 months ago. She had been treated with amio-
darone (600mg daily for 7 days, then 200mg daily) after an
elective cardioversion. Her other medications included:
lisinopril 20 mg daily, hydrochlorothiazide 12.5 mg daily,
warfarin 2mg daily, and metoprolol 25mg twice daily. After
her cardioversion, she had no symptomatic recurrences of
atrial fibrillation and had returned to her baseline functional
status. Today, she returns for a routine scheduled office visit.
She reports 3 weeks of a nonproductive cough that was not
associated with fever. She has also noted progressive dysp-
nea on exertion and had difficulty walking from the parking
garage for her office visit. Her primary care physician had
first stopped her lisonopril and then prescribed a 7 day course
of azithromycin for a presumed bronchitis 10days ago. She
completed this without improvement in her symptoms. You
Fig.139.1 Chest x-ray at presentation
order a chest x-ray which is shown in Fig.139.1.
What is the most likely diagnosis? What should you do
now? lower. Based on data from randomized trials, the current esti-
mate for the incidence of pulmonary toxicity on a dose of
200300mg per day is about 2%. Truly long-term data about
incidence of pulmonary toxicity over years of therapy with
Case Discussion amiodarone are not available.
Amiodarone pulmonary toxicity is thought to be multifac-
torial in its etiology. Amiodarone can damage lung tissue
In this patient, amiodarone pulmonary toxicity must be con-
indirectly via immunologic reactions or directly via a cyto-
sidered early in the differential diagnosis. In surveys of
toxic process. Amiodarone induces CD8-positive cytotoxic
patients treated with amiodarone for ventricular arrhythmias,
T cells and can result in the production of oxygen free radi-
often at doses of 400600mg daily, the incidence of amio-
cals. Phospholipid accumulation may occur in lung tissue
darone induced pulmonary toxicity was as high 510%.
and this may have an additional direct cytotoxic effect.
More recently, lower doses of amiodarone have become
A number of different clinical presentations of amio-
standard and the incidence of pulmonary toxicity has been
darone toxicity have been described. Acute or subacute tox-
icity with respiratory failure and an adult respiratory distress
syndrome (ARDS) has mostly been reported in patients with
J.P. DiMarco critical illnesses in intensive care units or after cardiotho-
Clinical Electrophysiology Lab, Cardiovascular Division,
racic surgery. In many of the reported cases, the relationship
University of Virginia Health System, 1215 Lee Street,
Charlottesville, VA 22908, USA to amiodarone has been only circumstantial but there does
e-mail: jdimarco@virginia.edu seem to be real risk of acute toxicity. The more common

A. Natale et al. (eds.), Cardiac Electrophysiology, 529


DOI: 10.1007/978-1-84996-390-9_139, Springer-Verlag London Limited 2011
530 J.P. DiMarco

clinical presentation of amiodarone induced lung toxicity is randomized clinical trials to support this. However, in the
similar to that shown here in this case. Patients will often patient with severe toxicity or an ARDS-like pattern, a trial
present with the insidious onset of dyspnea and a chronic, of steroids should be considered. The usual regimen would
nonproductive cough. The chest x-ray characteristically start with 4060 mg of prednisone daily with a gradual
shows a diffuse interstitial process. Pulmonary function tests decrease over several months. Long-term steroid therapy
will show restrictive disease with a marked decrease in should not be required.
carbon monoxide diffusion capacity (DLCO).
The diagnosis of amiodarone pulmonary toxicity is often
one of exclusion. The chest x-ray picture can be confused
with infection, pulmonary edema, the adult respiratory dis- Bibliography
tress syndrome (ARDS), interstitial or chronic eosinophilic
pneumonia, disseminated malignancy or bronchiolitis oblit- Goldschlager N, Epstein AE, Naccarelli GV, etal. A practical guide for
clinicians who teat patients with amiodarone: 2007. Heart Rhythm.
erans organizing pneumonia unrelated to amiodarone. In 2007;4:1250-1259.
amiodarone toxicity, the onset is usually insidious and the Jessurun GAJ, Boersma WG, Crijns HJGM. Amiodarone-induced
patients symptoms may be much less severe than would be pulmonary toxicity: predisposing factors, clinical symptoms and
expected based on the abnormalities on chest x-ray, pulmo- treatment. Drug Saf. 1998;18:339-344.
Olshansky B, Sami M, Rubin A, et al. Use of amiodarone for atrial
nary function tests and arterial blood gasses. Chest computed fibrillation in patients with preexisting disease in the AFFIRM
tomography will show a diffuse ground glass appearance or study. Am J Cardiol. 2005;95:404-405.
reticular abnormalities. Ott MC, Khoor A, Leventhal JP, Paterick TE, Burger CD. Pulmonary
If amiodarone pulmonary toxicity is suspected, the wisest toxicity in patients receiving low-dose amiodarone. Chest. 2003;
123:646-651.
course of action is to immediately discontinue therapy. If Singh SN, Fisher SG, Deedwania PC, Rohatagi P, Singh BN, Fletcher
amiodarone is continued, irreversible respiratory failure may RD. Pulmonary effect of amiodarone in patients with heart failure.
occur. There is substantial anecdotal experience showing J Am Coll Cardiol. 1997;30:514-517.
favorable responses to corticosteroid therapy but there are no
Case 140

John P. DiMarco

Case Summary ablation lesion and not notice the effects because of the
ventricular pacing.
In this patient, we proceeded with mapping during ven-
A 43-year-old man was referred for electrophysiologic study
tricular pacing using a cryoablation catheter. The ablation
because of a history of recurrent supraventricular tachycar-
site is seen in Fig.140.3. Once a change in retrograde activa-
dia. He had experienced recurrent episodes of SVT since age
tion had been seen, we stopped ventricular pacing and moni-
17. Several episodes had been terminated during emergency
tored AV conduction. First degree AV block was seen but no
room visits with adenosine. His baseline ECG did not show
higher grades of AV block. After completion of the ablation
preexcitation. Two examples of his tachycardia are shown in
lesion, we repeated para-Hisian pacing (Fig.140.4). Before
Fig.140.1, panels A and B.
ablation, the stimulus-to-A interval had stayed the same
During the electrophysiologic study, sustained SVT was
when the His bundle was captured (not shown). After abla-
initiated with a single atrial extrastimulus (Fig.140.2). The
tion, there is a shortening of the VA time when the His bundle
earliest retrograde activation was seen on the proximal His
was captured suggesting that retrograde conduction is now
bundle recording electrodes. Presence of a concealed septal
over the AV node.
accessory pathway was confirmed by advancement of the A
Another method to differentiate between AV nodal ver-
with a His synchronous PVC (not shown).
sus accessory pathway retrograde conduction would phar-
How would you proceed?
macologic administration of verapamil or adenosine. Both
these agents should block or delay retrograde AV nodal con-
duction but should not affect conduction over an accessory
Case Discussion pathway. Veparamil, in particular may be very useful during
mapping is there is a fusion of conduction of the AV node
This patient has a concealed retrograde septal accessory and an accessory pathway during ventricular pacing. Para-
pathway and highly symptomatic SVT. Concealed septal Hisian pacing is however, relatively easy to perform and
pathways are often difficult to treat safely with ablation since straightforward. A catheter is positioned near the His bundle
there is significant risk of damaging the AV conduction recording site and then pacing performed, As the stimulus
system. In particular, if one monitors the ablation during output is increased or as the catheter shifts with respiration
continuous ventricular pacing, one might affect antegrade the His bundle may be captured and retrograde conduction
conduction over the normal AV node His bundle with the assessed.

J.P. DiMarco
Clinical Electrophysiology Lab, Cardiovascular Division,
University of Virginia Health System, 1215 Lee Street,
Charlottesville, VA 22908, USA
e-mail: jdimarco@virginia.edu

A. Natale et al. (eds.), Cardiac Electrophysiology, 531


DOI: 10.1007/978-1-84996-390-9_140, Springer-Verlag London Limited 2011
532 J.P. DiMarco

Fig.140.1 ECGs obtained I aVR V1 V4


during two separate episodes of
tachycardia. Panel A shows a
regular tachycardia with a right
bundle branch block pattern.
Panel B shows another episode of
tachycardia, this time with a left
bundle branch block pattern. II aVL V2 V5
Retrograde atrial activity with a
short RP' interval is seen best in
leads V1 and V2

III aVF V3 V6

I aVR V1 V4

II aVL V2 V5

III aVF V3 V6
Case 140 533

Fig.140.2 Initiation of SVT I


with programmed atrial
stimulation. A single atrial II
extrastimulus initiates tachycar-
dia. The earliest atrial activation aVF
during SVT is recorded on the
His bundle electrodes V1

HRA

CSD

CS 34

CS 56

CS 78

CS 910

HIS d

HIS p

RVA

STIM 600 270

II

aVF

V1

HRA

CSD

CS 34

CS 56

CS 78

CS 910

HIS d

HIS p
Fig.140.3 Mapping during
ventricular pacing. The ABLD
retrograde atrial electrogram on
the ablation catheter is now ABLP
earlier than seen on all other
catheters. The ablation catheter RVA
was now immediately adjacent to
the His bundle catheter on STIM 400 400 400 400
fluoroscopy
534 J.P. DiMarco

Fig.140.4 Para-Hisian pacing I


after successful ablation. The first
and third beats are wider indicatng II
ventricular capture only while the
narrower second, fourth and fifth aVF
beats show His bundle capture.
The Stimulus-to-A interval is V1
longer in the first and third beats
than when the His is captured HRA
(narrower beats 2, 4, and 5)
CSD

CS 34

CS 56

CS 78

CS 910

HIS d

HIS p

ABLD

ABLP

RVA

STIM 500 500 500 500

Bibliograhpy Nakagawa H, Jackman WM. Para-hisian pacing: useful clinical tech-


nique to differentiate retrograde conduction between accessory
atrioventricular pathways and atrioventricular nodal pathways.
Hirao K, Otomo K, Wang X, etal. Para-hisian pacing: a new method for Heart Rhythm. 2005;2:667-672.
differentiating retrograde conduction over an accessory AV path-
way from conduction over the AV node. Circulation. 1996;94:
1027-1035.
Case 141

John P. DiMarco

Case Summary bradycardia and progressive AV block. Skeletal muscle


symptoms, also related to glycogen deposition, may occur.
Many require pacemakers as young adults. Sudden death has
A 26-year-old woman is brought to the emergency room
been reported in some individuals but does not appear to be
complaining of dizzy spells and palpitations. His ECG is
common.
shown in Fig.141.1. Her physical exam was normal except
The inheritance pattern is autosomal dominant with a high
for heart rate. An echocardiogram showed concentric hyper-
degree of penetrance. The genetic locus for this syndrome
trophy without outflow tract obstruction. No valvular abnor-
was first identified on chromosome 7q34q36 in 1995.
malities were seen. Upon further questioning, she admitted
Subsequently, the characteristic mutations have been identi-
that many of her family members had been told they had
fied as missense mutations in the gene (PRKAG2) that
Wolff-Parkinson-White syndrome. Her 21-year-old brothers
encodes the gamma-2 regulatory subunit of AMP activated
ECG is shown in Fig.141.2. He also had a history of palpita-
protein kinase. This results in myocardial hypertrophy without
tions and had been told they were due to paroxysmal atrial
myofibrillar disarray and only minimal fibrosis. Histologically,
fibrillation. Several older members of the family had a his-
the myocytes can be shown to be enlarged by large cytosolic
tory of palpitations as young adults but then required pace-
vacuoles that contain inhomogeneous granular material (gly-
makers later in life.
cogen) and also display contractile elements. These glycogen
What is the most likely diagnosis for the syndrome seen
laden myocytes can both disrupt the annulus fibrosis leading
in this family?
to preexcitation as muscular connections between the atria
and ventricles are created and also infiltrate the conduction
system leading to AV block. A murine model of this syn-
drome has been developed which exhibits a very similar
Case Discussion phenotype.
From a clinical standpoint, it is important to recognize
The syndrome of early preexcitation, left ventricular hyper- this syndrome. Although young patients with a PRKAG2
trophy, and late progressive conduction system disease has defect may present with what looks like preexcitation with
only recently been recognized as a discrete entity. Patients AVRT, the associated findings of significant ventricular
with the syndrome often have a preexcitation pattern noted hypertrophy and the positive family history of both preexci-
on their ECG early in life and may have typical AV reentrant tation and late conduction system disease should make one
tachycardia as children and young adults. Over time, how- cautious before recommending ablation therapy. Patients
ever, they often go on to develop progressive signs of left who undergo ablation are likely to require permanent pacing,
ventricular hypertrophy with diastolic dysfunction, sinus if not immediately, then in the near future.

J.P. DiMarco
Clinical Electrophysiology Lab, Cardiovascular Division,
University of Virginia Health System, 1215 Lee Street,
Charlottesville, VA 22908, USA
e-mail: jdimarco@virginia.edu

A. Natale et al. (eds.), Cardiac Electrophysiology, 535


DOI: 10.1007/978-1-84996-390-9_141, Springer-Verlag London Limited 2011
536 J.P. DiMarco

Fig.141.1 ECG on presentation. Note the short PR interval, the sinus bardycardia and the very unusual preexcited QRS morphology

Fig.141.2 ECG in the patients 21year old brother


Case 141 537

Bibliography Sidhu JS, Rajawat YS, Rami TG, et al. Transgenic mouse model of
ventricular preexcitation and atrioventricular reentrant tachycardia
induced by an AMP-activated protein kinase loss-of-function muta-
Charron P, Genest M, Richard P, Komajda M, Pochmalicki G. A famil- tion responsible for Wolff-Parkinson-White syndrome. Circulation.
ial form of conduction defect related to a mutation in the PRKAG2 2005;111:21-29.
gene. Europace. 2007;9:597-600. Sternick EB, Oliva A, Magalhaes LP, et al. Familial pseudo-Wolff-
Gollob MS, Green MS, Tang AS, etal. Identification of a gene respon- Parkinson-White syndrome. J Cardiovasc Electrophysiol. 2006;
sible for familial Wolff-Parkinson-White syndrome. N Engl J Med. 17:724-732.
2001;344:1823-1831. Wolf CM, Ahmad M, Arad F, etal. Reversibility of PRKAG2 glycogen-
Murphy RT, Morgensen J, McGarry K, et al. Adenosine monophos- storage cardiomyopathy and electrophysiological manifestations.
phate-activated protein kinase disease mimicks hypertrophic cardi- Circulation. 2008;117:144-154.
omyopathy and Wolff-Parkinson-White syndrome: natural history.
J Am Coll Cardiol. 2005;45:922-930.

Case 142

John P. DiMarco

Case Summary flecainide 50 mg bid and had only very rare spells of atrial
fibrillation once her hyperthyroidism had been corrected.
A 59year old woman with severe rheumatoid arthritis devel- Recently, she had undergone back surgery to revise spinal
oped recurrent episodes of a narrow complex tachycardia after hardware in place to correct scoliosis. After the procedure, she
an orthopedic surgical procedure. She had a history of recur- developed episodes of the tachycardia shown in Fig.142.1.
rent supraventricular tachycardia that began at age 17 years. In A trial of intravenous adenosine resulted in transient atrial
1991, at age 42, she had undergone an electrophysiologic fibrillation with a rapid ventricular rate which later converted
study which showed dual AV nodal pathways and easily induc- spontaneously to sinus rhythm. A second episode of SVT
ible typical (slow-fast) AV node reentrant tachycardia. She was treated with intravenous verapamil (10 mg) with termi-
underwent a fast pathway ablation which resulted in a persis- nation of the tachycardia (Fig.142.2). She was then referred
tent first degree AV block (PR interval 240340 ms) that was for electrophysiologic study.
asymptomatic. Seven years after the first electrophysiologic The patient underwent electrophysiologic study using
study, she developed hyperthyroidism and in that setting had standard techniques. Her baseline recording in sinus rhythm
episodes of paroxysmal atrial fibrillation. She was treated with is shown in Fig.142.3.

I aVR V1 V4

II aVL V2 V5

III aVF V3 V6

Fig.142.1 ECG of tachycardia that developed after orthopedic surgery

J.P. DiMarco
Cardiovascular Division, University of Virginia Health System,
1215 Lee Street, Charlottesville, VA 22908, USA
e-mail: jdimarco@virginia.edu

A. Natale et al. (eds.), Cardiac Electrophysiology, 539


DOI: 10.1007/978-1-84996-390-9_142, Springer-Verlag London Limited 2011
540 J.P. DiMarco

Fig.142.2 Rhythm strip shortly after intravenous verapamil

II

aVF

V1

HRA

HIS d

HIS p

RVA

Fig.142.3 Baseline recordings at electrophysiologic study. The PR and AH intervals are prolonged consistent with the patients prior fast pathway
ablation

At electrophysiologic study, the tachycardia could not be The ablation catheter was removed from the coronary
initiated with either rapid atrial pacing or atrial premature sinus and replaced with a 14-pole catheter. Figure 142.5
stimulation. The tachycardia was reliably initiated with a shows the activation sequence during tachycardia and the
single ventricular extrastimulus over a wide range of cou- response to ventricular pacing at a cycle length 40 ms faster
pling intervals (Fig.142.4). than the tachycardia cycle length.
Case 142 541

II

aVF

V1

HRA

CS ABLd

CS ABLp

HIS d

RVA

Fig.142.4 Tachycardia initiation. A quadripolar ablation catheter had atrial electrograms in the coronary sinus recordings precede the right
been positioned in the mid-coronary sinus. A single ventricular extra- atrial signals
stimulus initiates tachycardia with a short RP interval. Note that the

Figure142.6 shows the response to a ventricular prema- available, slow pathway ablation became the preferred
ture stimulus delivered during tachycardia. approach in patients with AVNRT since it carried a much
What do you think is the most likely mechanism of the lower risk for the inadvertent production of AV block. Slow
tachycardia? What structure might you target for ablation? pathway ablation was at least as effective as fast pathway
ablation and the PR interval after ablation remained normal.
Only a few papers have looked at the long-term effects of
fast pathway ablation. During the relatively short term fol-
Case Discussion low-up in those series, late progression to, as opposed to
early development, of high grade AV block was not
This case raises several interesting issues. Fast pathway described.
ablation was an early technique used for catheter ablation in This patient underwent a fast pathway ablation and then
patients with AV node reentry. The technique used required had 17 years without recurrent regular tachycardia, even
that the tip of the ablation catheter be near the His bundle though she did have some intermittent atrial fibrillation after
recording position. The catheter was then withdrawn slightly. a bout of hyperthyroidism. It is interesting that she was able
During RF delivery, effects on the fast pathway were to tolerate flecainide and metoprolol for her paroxysmal
assessed by monitoring the PR interval. A 50% prolongation atrial fibrillation with a very long baseline PR interval with-
of the PR interval and an inability to reinitiate tachycardia out developing higher grade AV block. However, during the
were the usual endpoints for the ablation. After techniques stress of a surgical procedure, she developed recurrent
for localization and ablation of the slow pathway became tachycardias.
542 J.P. DiMarco

II

aVF

V1

HRA

CS 12

CS 34

CS 56

CS 78

CS 910

CS 1112

CS 1314

HIS d

HIS p

RVA

Stim

Fig.142.5 Ventricular pacing during SVT. The coronary sinus catheter is positioned so that poles 1112 are at the coronary sinus os

Working out the probable mechanism for the tachycardia fast-slow forms of AV node reentry or an accessory path-
requires a number of steps. The ECG in Fig.142.1 shows a way mediated tachycardia but would be quite rare for an
relatively short RP interval and is most consistent with either atrial tachycardia. Interestingly, the atrial activation sequence
the slow-slow form of AV node reentry or an accessory demonstrated earliest atrial activity on the coronary sinus
pathway mediated tachycardia. An atrial tachycardia, how- electrogram recordings. The response to ventricular pacing
ever, must also be considered. Verapamil administration in Fig.142.5 shows what has been called a pseudo VAAV
broke the tachycardia and a careful examination of the ECG response. During ventricular pacing, the VA interval pro-
strip at the time of termination shows that the last atrial com- longs and the two right atrial deflections after the last paced
plex conducted to the ventricle (Fig.142.2). Either an atrial beat are both entrained beats so this is really a VAV response.
tachycardia or AVNRT with retrograde block would give this It is also noted that the post-pacing interval on the RV elec-
response. However, this observation makes it unlikely that trogram minus the tachycardia cycle length is very long, a
the tachycardia used an accessory pathway for retrograde finding consistent with retrograde conduction an AV nodal
conduction since most accessory pathways, including those fiber. A premature stimulus delivered when the His is refrac-
with long conduction times, are insensitive to verapamil. The tory in Fig.142.6 does not advance the atrial electrograms.
tachycardia could not be initiated with atrial stimulation but This does not completely exclude an accessory pathway
ventricular premature beats reliably initiated tachycardia. unless the stimulus is delivered near the AV groove and there
This would be common with either the slow-slow or the is no decremental conduction in the pathway.
Case 142 543

II

aVF

V1

HRA

CS 12

CS 34

CS 56

CS 78

CS 910

CS 1112

CS 1314

HIS d

HIS p

RVA

Fig.142.6 A single VPD is introduced during tachycardia

Left sided extensions of AV nodal fibers have been case, however, because of the history of previous AV node
reported. Some authors have said this occurs in up to 68% reentry, the response to verapamil and the long post-pacing
of patients with AV node reentry and the phenomenon interval, it is likely that left sided AV nodal fibers com-
seems to be more common in patients with the slow- prised the retrograde limb of the circuit. Since this patient
slow or fast-slow variants. In some reports, retrograde had a history of a prior fast pathway ablation, we were
atrial activation over what has been called an AV nodal reluctant to try to ablate in the typical slow pathway posi-
fiber has been reported even when the earliest retrograde tion in the triangle of Koch, even though other authors
atrial recording is in the distal coronary sinus. Precise cri- have used this approach safely and successfully. We there-
teria to differentiate between an accessory pathway with fore mapped earliest retrograde activation to a site 1.5 cm
decremental conduction properties and unusually posi- within the coronary sinus os (Fig.142.7). Application of
tioned AV nodal fibers are difficult to define since the RF here terminated the tachycardia and prevented reinitia-
responses to either pharmacologic manipulations or stimu- tion. She has continued to be free of recurrent tachycardia
lation maneuvers may not be entirely conclusive. In this during follow-up.
544 J.P. DiMarco

II

aVF

V1

HRA

CS 1-2

CS 3-4

CS 5-6

ABL d

ABL p

CS 7-8

CS 9-10

CS 11-12

CS 13-14

HIS d

RVA

Stim

Fig.142.7 The ablation catheter tip is about 1.5 cm inside the coronary sinus os. This was the shortest VA time recorded in either atrium or in the
coronary sinus

Bibliography approaches to the AV node for treatment of AV nodal reentrant


tachycardia. J Cardiovasc Electrophysiol. 1997;8:451-468.
Lee MA, Morady F, Kadish A, etal. Catheter modification of the AV
Chen J, Anselme F, Smith TW, etal. Standard right atrial ablation is junction with radiofrequency energy for control of atrioventricular
effective for atrioventricular nodal reentry with earliest activation in nodal reentry tachycardia. Circulation. 1991;83:827-835.
the coronary sinus. J Cardiovasc Electrophysiol. 2004;15:2-7. Mehta D, Gomes JA. Long term results of fast pathway ablation in
Greenberg LBB, Overholt ED M, etal. Differential electrophysiologic atrioventricular nodal reentry tachycardia using a modified tech-
properties of decremental retrograde pathways in long RP tachy- nique. Br Heart J. 1995;74:671-675.
cardia. Circulation. 1987;76:21-31. Nam G-B, Rhee K-S, Kim J, Choi K-J, Kim Y-H. Left atrionodal comn-
Hwang C, Martin DJ, Goodman JS, et al. Atypical atrioventriculart nections in typical and atypical atrioventricular nodal reentrant
nodal reciprocating masquerading as tachycardia using a left-sided tachycardias: activation sequence in the coronary sinus and results
accessory pathway. J Am Coll Cardiol. 1997;30:218-225. of radiofrequency catheter ablation. J Cardiovasc Electrophysiol.
Jais P, Haissaguerre M, Shah DC, etal. Successful radiofrequency abla- 2006;17:171-177.
tion of a slow atrioventricular nodal pathway on the left posterior Vijayaraman P, Kok LC, Rhee B, Ellenbogen KA. Unusual variant of
atrial septum. PACE. 1999;22:525-527. atrioventricular nodal reentrant tachycardia. Heart Rhythm. 2005;2:
Kottkamp H, Hindricks G, Borgreffe M, Breithardt G. Radiofrequency 100-102.
catheter ablation of the anterosuperior and posteroinferior atrial
Case 143

John P. DiMarco

Case Summary her to reach the emergency room. In the emergency room,
she was found to have a narrow complex tachycardia at
A 61 year old woman is referred because of repeated about 155bpm (Fig.143.1).
episodes of narrow complex tachycardia. She had been well The tachycardia was reliably terminated during each of her
with no cardiac history until 2.5 years before referral. At emergency room visits with 6mg of intravenous adenosine. A
that time, she began to notice episodes of palpitations. These trial of oral metoprolol 50mg twice daily was ineffective and
would occur at various intervals from every few days to she was referred after several repeat emergency room visits.
every few weeks. Most terminated on their own within an At electrophysiologic study, the tachycardia could be
hour or two but several episodes had lasted long enough for easily initiated with single atrial extrastimuli (Fig.143.2).

I aVR V1 V4

II aVL V2 V5

III aVF V3 V6

Fig.143.1 ECG of the patients presenting tachycardia in the emergency room

J.P. DiMarco
Cardiovascular Division, University of Virginia Health System,
1215 Lee Street, Charlottesville, VA 22908, USA
e-mail: jdimarco@virginia.edu

A. Natale et al. (eds.), Cardiac Electrophysiology, 545


DOI: 10.1007/978-1-84996-390-9_143, Springer-Verlag London Limited 2011
546 J.P. DiMarco

II

aVF

V1

HRA

CS-D

CS 3-4

CS 5-6

CS 7-8

CS 9-10

ABL-D

HIS d

HIS p

RVA

STIM

Fig.143.2 Initiation of sustained tachycardia with a single atrial extrastimulus (500/320). The tachycardia could be reliably initiated over a wide
range of coupling intervals

The response to ventricular overdrive pacing is seen in These tachycardias, for unknown reasons, occur mostly in
Fig. 143.3. Another tracing of the tachycardia is shown in middle aged or elderly women. As in this case, they may
Fig.143.4. have an ECG pattern that closely resembles AV node reentry,
What is the mechanism of the tachycardia? What therapy particularly when the PR interval is close to the cycle length
would you recommend now? of the tachycardia. They are almost always adenosine sensi-
tive. For these reasons, in many cases, they are incorrectly
thought to be AV nodal reentrant tachycardias. The findings
of a AV dissociation and lack of VA conduction make atrial
Case Discussion tachycardia the likely diagnosis. The atrial mechanism is
uncertain. There tachycardias are easy to start with extrastim-
This patients tachycardia has several interesting characteris- uli. Although they are sensitive to adenosine, they usually do
tics. The baseline electrocardiogram shows a narrow com- not require isoproterenol for initiation. This suggests that
plex tachycardia without obvious P waves visible on the they may be due to reentry involving portions of AV nodal
surface ECG. The initiation of the tachycardia shows tissue rather than being caused by triggered activity or abnor-
prolongation of the AH interval and then what appears to be mal automaticity in the atrium itself. Most can be ablated
a midline retrograde activation sequence. This could easily from the right atrium but there are occasional reports where
be interpreted as AV node reentry. However, in Fig.143.3, ablation from the left side of the atrial septum was required.
during overdrive ventricular pacing, there is no entrainment Mapping and ablation of these tachycardias require care.
of the atrium due to VA block during the tachycardia, and in Although we have previously had success with radiofre-
Fig.143.4 you can see that there is actually AV dissociation quency ablation, the potential risk for producing inadvertent
during running tachycardia. AV block has caused us to shift to cryoablation. With this
These findings are most consistent with an atrial technique, most of these tachycardias can be successfully
tachycardia arising from the region around the AV node. mapped and ablated (Figs.143.5 and 143.6).
Case 143 547

II

aVF

V1

HRA

CS-D

CS 3-4

CS 5-6

CS 7-8

HIS d

HIS p

RVA

STIM

Fig.143.3 Ventricular pacing during an episode of tachycardia


548 J.P. DiMarco

II

aVF

V1

HRA

CS-D

CS 3-4

CS 5-6

CS 7-8

CS 9-10

ABL-D

HIS d

HIS p

RVA

Fig.143.4 Additional recordings during tachycardia made later during the case
Case 143 549

Fig.143.5 Ablation site. A cryoablation catheter was a 4mm tip was used for mapping and ablation. The local electrogram, on the ablation cath-
eter is seen to precede the atrial signal on the His electrogram by about 25ms

Bibliography

Chen CC, Tai CT, Chiang CE, et al. Atrial tachycardias originating
from the atrial septum: Electrophysiologic characteristics and
radiofrequency ablation. J Cardiovasc Electrophysiol. 2000;11:
744-749.
Iesaka Y, Takahashi A, Goya M, etal. Adenosine-sensitive atrial reen-
trant tachycardia originating from the atrioventricular nodal transi-
tional area. J Cardiovasc Electrophysiol. 1997;8:854-864.
Lai LP, Lin JL, Chen TF, Ko WC, Lien WP. Clinical, electrophysiologi-
cal characteristics, and radiofrequency catheter ablation of atrial
tachycardia near the apex of Kochs triangle. PACE. 1998;21:
367-374.
Marrouche NF, SippensGroenewegen A, Yang Y, Dibs S, Scheinman
MM. Clinical and electrophysiologic characteristics of left septal
atrial tachycardia. J Am Coll Cardiol. 2002;40:1133-1139.

Fig.143.6 Ablation site for the septal atrial tachycardia. The 4 mm


tipped croablation catheter is positioned immediately next to the His
bundle catheter

Case 144

John P. DiMarco

Case Summary rates between 130 and 160 beats per min. Her ECG is shown
in Fig.144.1.
A 20-year-old woman is referred for management of refrac- She was afebrile. No murmurs were audible. She had
tory heart failure and consideration of heart transplantation. bilateral pulmonary crackles. Her jugular venous pressure
She was first seen at another hospital with a complaint of was elevated. Her chest x-ray showed cardiomegaly and pul-
dyspnea that began about 3weeks prior to her seeking medi- monary congestion. Her laboratory studies were not remark-
cal attention. Prior to that she had been completely well and able. Her rhythm was initially thought to be sinus tachycardia
had participated in high school sports. She denied any recent in the setting of decompensated heart failure of unknown
substance abuse. Several relatives do have a history of myo- cause. She failed to improve after initiation of an ACE inhib-
cardial infarctions in late middle age. Physical examination itor, a diuretic, and a beta blocker over 10days at the outside
at her initial presentation revealed a persistent tachycardia at hospital. She gradually became more short of breath and

Fig.144.1 ECG at the time of initial presentation

J.P. DiMarco
Clinical Electrophysiology Lab, Cardiovascular Division,
University of Virginia Health System, 1215 Lee Street,
Charlottesville, VA, 22908 USA
e-mail: jdimarco@virginia.edu

A. Natale et al. (eds.), Cardiac Electrophysiology, 551


DOI: 10.1007/978-1-84996-390-9_144, Springer-Verlag London Limited 2011
552 J.P. DiMarco

Fig.144.2 Change in tachycardia during sleep

developed hypotension. These developments led to her trans- usually with catheter ablation for atrial tachycardias and
fer to a referral center. During the next 24h of monitoring, PJRT, may be the only therapy required. At the outside hos-
she was noted to stay in this tachycardia almost continuously. pital, an initial diagnosis of sinus tachycardia had been made
However, on rare occasions during sleep, monitor strips in this patient since the P waves were strongly positive in II,
showed that the rhythm could change spontaneously as III and aVF. Although she did not report any recent febrile
shown in Fig.144.2. illnesses and was afebrile, the referring physicians believed
A 2-dimensional echocardiogram was obtained showing that she had myocarditis. After transfer, the ECG was looked
an ejection fraction that was estimated to be 1520% with at more closely since it was felt that the tachycardia could be
four chamber dilatation. A cardiac magnetic resonance scan the cause of her heart failure. It was now noted that the P
confirmed the depressed ventricular function noted on her waves were negative in lead I and aVL (Fig.144.1), making
echocardiogram but failed to show any evidence for acute sinus tachycardia unlikely. The site of origin most consistent
myocarditis. with this ECG pattern would be the left atrial appendage.
What do you think the mechanism of the tachycardia is? She later underwent electrophysiologic study. Recordings
Do you think this is related to the patients heart failure? made with an ablation catheter positioned at the base of the
left atrial appendage are shown in Fig. 144.3. Ablation at
this site terminated the tachycardia and sinus rhythm was
restored.
Case Discussion After the ablation, the patients symptoms gradually
improved and she was eventually discharged still on an ACE
In young individuals, incessant or nearly incessant tachycar- inhibitor, a beta blocker and a diuretic. Four weeks later, her
dias are an important cause of congestive heart failure. In symptoms had completely resolved and these medications
individuals without associated heart disease, atrial tachycar- were discontinued. A 3 month follow-up echocardiogram
dias and the permanent form of junctional reciprocating showed an ejection fraction of 60% and she was totally
tachycardia (PJRT) are the most common arrhythmias iden- symptom-free.
tified. In older adults, atrial fibrillation with uncontrolled Resolution of a tachycardia-induced cardiomyopathy usu-
ventricular rates would be the most common cause for ally takes at least several weeks. Once the tachycardia has
tachycardia-induced heart failure. It is important to recog- been eliminated or the ventricular rate controlled, the patient
nize this syndrome in young patients who present with heart should be carefully reevaluated to make sure the tachycardia
failure and tachycardia since treatment of the arrhythmia, was the primary cause of the problem.
Case 144 553

II

ABL d

ABLp

CS d

CS 34

CS 56

CS 78

CS p

RVA

Fig.144.3 Activation map during SVT. The ablation catheter is now positioned at the base of the left atrial appendage. This was the earliest site
of activation identified with the local electrogram preceding the P wave onset by 3540ms

Bibliography Wang YL, Li XB, Quan X, etal. Focal atrial tachycardia originating
from the left atrial appendage: electrocardiographic and electro-
physiologic characterization and long-term outcomes of radiofre-
Calo L, De Ruvo E, Sette A, et al. Tachycardia-induced cardiomyo quency ablation. J Cardiovasc Electrophysiol. 2007;18:459-464.
pathy: mechanisms of heart failure and clinical implications. Yamada T, Murakami Y, Yoshida Y, et al. Electrophysiologic and
JCardiovasc Med. 2007;8:138-143. electrocardiographic characteristics and radiofrequency catheter
Kato M, Adachi M, Yano A, et al. Radiofrequency catheter ablation ablation of focal atrial tachycardia originating from the left atrial
for atrial tachycardia originating from the left atrial appendage. appendage. Heart Rhythm. 2007;4:1284-1291.
JInterv Card Electrophysiol. 2007;19:45-48.
Umana E, Solares CA, Alpert MA. Tachycardia-induced cardiomyopa-
thy. Am J Med. 2003;114:51-55.

Case 145

John P. DiMarco

Case Summary be definitely identified. The mechanism of the arrhythmia


cannot be definitely diagnosed on the electrocardiogram.
However, given the patients hypotension and significant
An 81-year-old man is brought to the emergency room
chest discomfort, the most appropriate initial therapy would
complaining of palpitations and dyspnea. His cardiac history
be to terminate the arrhythmia with cardioversion. He was
includes coronary artery bypass surgery performed at age 76
therefore anesthetized and cardioverted electrically. His
because of exertional angina. He had no prior history of any
chest pain immediately resolved.
arrhythmia. Since bypass surgery, he has done extremely
It is important to recall that the diagnosis of the arrhyth-
well and has been fully active. Last year he had a stress test
mia is still uncertain and at this point in time an electrophysi-
which showed no inducible ischemia and no fixed perfusion
ologic study would certainly be appropriate. Given the
defects. His left ventricular ejection fraction was estimated
patients history of coronary disease and his age, one might
to be 60%. He is currently taking: aspirin, metoprolol, lisino-
guess that this was either ventricular tachycardia or an atrial
pril for hypertension, and atorvastatin. Today, while working
tachycardia with left bundle branch block but only an elec-
in his yard, he noted the sudden onset of palpitations and
trophysiologic study in which the clinical tachycardia was
shortness of breath. The rescue squad was called and he was
reproduced would enable one to make the diagnosis with
brought to the emergency room. On arrival to the emergency
certainty.
room, his heart rate was 180bpm. His blood pressure was
He therefore underwent electrophysiologic study. During
80/50. He was complaining of moderately severe chest
the study, AV node reentry was induced (Figs. 145.2 and
discomfort. His electrocardiogram is shown in Fig.145.1.
145.3) on multiple occasions. Most of the episodes had a
What is your diagnosis for his arrhythmia? What would
narrow QRS complex but several showed the left bundle
you do next?
branch block pattern observed clinically. He underwent a
slow pathway ablation and has done well since.
Although the clinical history of coronary artery disease
in this patient could make one suspect either ventricular
Case Discussion tachycardia or an atrial tachycardia, AV node reentry can
appear for the first time in an elderly patient. In some series,
This elderly gentleman presents with a wide complex tachy- up to 1015% of patients with AV node reentrant tachycar-
cardia. Several aspects of the clinical history are important. dia will first present with their arrhythmia after age 65. As in
He has a history of coronary artery disease but does not younger patients, slow pathway ablation in elderly patients
report ever having a myocardial infarction. Recent testing has been safe and highly effective. In this case, the clinical
showed normal ventricular function. This is his first episode history and the ECG were not sufficient to allow a diagnosis
of symptomatic arrhythmia. The ECG shows a regular wide to be made with certainty when the patient presented. The
complex tachycardia at 180bpm. The QRS duration is about clinical situation determined the need for cardioversion and
140ms. There is a fairly typical left bundle branch block pat- the diagnosis was finally made during an elective electro-
tern observed. The QRS axis is normal. Atrial activity cannot physiologic study.

J.P. DiMarco
Clinical Electrophysiology Lab, Cardiovascular Division,
University of Virginia Health System,
1215 Lee Street, Charlottesville, VA, 22908 USA
e-mail: jdimarco@virginia.edu

A. Natale et al. (eds.), Cardiac Electrophysiology, 555


DOI: 10.1007/978-1-84996-390-9_145, Springer-Verlag London Limited 2011
556 J.P. DiMarco

I aVR V1 V4

II aVL V2 V5

III aVF V3 V6

V1

Fig.145.1 ECG and rhythm strip on arrival in the emergency room


Case 145 557

II

V1

HRA

HIS D

RVA

Fig.145.2 Tracings ashowing AV node reentrant tachycardia with a left bundle branch block. Bundle branch block was noted intermittently
d uring episodes of tachycardia
558 J.P. DiMarco

WIANT, WILLIAM 1251911 04/09/08 16 : 11 : 00 Baseline Intervals 150mm/sec 0.400 mv

II

V1

HRA

HIS D

RVA

Fig.145.3 AV node reentrant tachycardia with a normal QRS

Bibliography f eatures, electrophysiologic characteristics and results of radio


frequency ablation. J Am Coll Cardiol. 1994;23:702-708.
Epstein LM, Chiesa N, Wong MN, Lee RJ, Griffin JC, Scheinman MM.
Akhtar M, Shenasa M, Jazayeri M, Caceres J, Tchou PJ. Wide QRS Radiofrequency catheter ablation in the treatment of supraventricu-
complex tachycardia. Reappraisal of a common clinical problem. lar tachycardia in the elderly. J Am Coll Cardiol. 1994;23:1356-
Ann Intern Med. 1988;109:905-912. 1362.
Channamsetty V, Aronow WS, Sorberra C, Butt A, Cohen M. Efficacy Kalusche D, Ott P, Arentz T, etal. AV nodal reentrant tachycardia in
of radiofrequency catheter ablation in treatment of elderly patients elderly patients: clinical presentation and results of radiofrequency
with supraventricular tachyarrhythmias and ventricular tachycardia. catheter ablation therapy. Coron Artery Dis. 1998;9:359-363.
Am J Therapeutics. 2006;13:513-515. Tchou P, Young P, Mahmud R, Denker S, Jazayeri M, Akhtar M. Useful
Chen SA, Chiang CE, Yang CJ, etal. Accessory pathway and atrioven- clinical criteria for the diagnosis of ventricular tachycardia. Am J
tricular node reentrant tachycardia in elderly patients: clinical Med. 1988;84:53-56.
Case 146

Brett A. Faulknier, David T. Huang, and James P. Daubert

Case Summary such as RV outflow tract VT, (4) polymorphic VT due to


long QT syndrome (LQTS), (5) catecholaminergic polymor-
phic VT (CPVT), (6) hypertrophic obstructive cardiomyopa-
A 19-year-old college swimmer suffered syncope during an
thy (HCM) with ventricular tachycardia (VT), (7) an
intercollegiate race. She described losing consciousness for
anomalous coronary artery (or coronary stenosis, unlikely in
two seconds, and coming to the surface gasping for air and
this demographic) resulting in ischemically mediated VT, or
choking. Over the prior month she had also had 23 episodes
(8) myocardial scar-related reentrant VT due to myocardial
of palpitations, one associated with lightheadedness all while
infarction, dilated cardiomyopathy, an infiltrative process,
training or competing. During a stress test she exercised
non-compaction, arrhythmogenic RV cardiomyopathy/dys-
14 min, but during recovery developed a wide complex
plasia (ARVC/D) or sarcoidosis. Aberrant SVT is essentially
tachycardia at 300 beats/min lasting 1520 s without syn-
excluded since negative complexes in leads I and avL do not
cope (Fig.146.1). The past medical history, family history
fit with LBBB aberrancy. Similarly, the morphology of the
and physical examination were unremarkable. A urine drug
QRS complexes rule out an accessory pathway since nega-
screen was negative. An ECG taken in the EP clinic is shown
tivity in leads I and avL points to a left-lateral pathway in
(Fig.146.2). What is the differential diagnosis and what fur-
which case leads V1V3 should show positive delta waves; a
ther evaluation and management should be considered for
Mahaim pathway would show a LBBB appearance but with
this patient?
a normal to leftward axis. Against idiopathic RVOT VT is
the abnormal resting ECG, the irregularity (and also extreme
rapidity) of the VT; palpitations are common but syncope is
not a hallmark of this condition. Normal RV function and
Case Discussion structure should be present for this condition to be confi-
dently diagnosed. Regarding LQTS, repolarization is abnor-
The differential diagnosis of this left bundle type wide QRS mal with inverted T waves in leads V1V4, but the QT is
tachycardia (Fig.146.1) includes: (1) supraventricular tachy- only borderline prolonged, and VT due to LQTS is polymor-
cardia (SVT) with aberrant conduction, (2) a preexcited phic with a torsade de pointes (twisting of the points)
tachycardia, especially atrial fibrillation with a bypass tract appearance. In CPVT, another ion channelopathy, frequent
or a Mahaim pathway, (3) idiopathic normal heart VT, multiform PVCs are usually seen during exercise (or stress),
but were not seen in her stress test, and VT if it occurs is
polymorphic, and often bidirectional. The VT observed
(Fig.146.1) was relatively monomorphic although somewhat
B.A. Faulknier(*)
Department of Cardiology and Electrophysiology, irregular. A coronary angiogram showed normal coronary
West Virginia University Physicians of Charleston, 3100 MacCorkle origin and anatomy, excluding differential number 6. An LV
Avenue SE, Suite 700, Charleston, WV 25304, USA angiogram showed an LVEF of 50% with mild anterobasal
e-mail: bfaulknier@hsc.wvu.edu
and anterolateral hypokinesis. An RV angiogram was per-
D.T. Huang formed (Figs. 146.3 and 146.4) and showed a dilated RV
Department of Cardiology, University of Rochester, with global hypokinesis and more severe hypo-to-akineis of
601 Elmwood Avenue, Box679, Rochester, NY 14642, USA
e-mail: david_huang@urmc.rochester.edu the RV apex and RVOT. A very focal portion of the RV free
wall was mildly dyskinetic. This leaves us in the last cate-
J.P. Daubert
gory, scar related reentry, and points to ARVC/D or possibly
Cardiology Division, Duke University Health System,
DUMC Box3174, Duke Hospital 7451H, Durham, NC 27710, USA sarcoidosis. An RV endomyocardial biopsy showed moder-
e-mail: james.daubert@duke.edu ate to marked interstitial fibrosis (70%), fatty replacement

A. Natale et al. (eds.), Cardiac Electrophysiology, 559


DOI: 10.1007/978-1-84996-390-9_146, Springer-Verlag London Limited 2011
560 B.A. Faulknier et al.

Fig.146.1 Ventricular tachycardia during exercise stress test

Fig.146.2 Baseline ECG showing possible epsilon wave (V2V3), prolongation of the QRS in lead V1 consistent with delayed RV activation,
and inversion of T-waves in the right precordial leads
Case 146 561

(510%), moderate myocyte hypertrophy, and minimal burst pacing. Her LV and RV function remain unchanged.
chronic inflammation. At electrophysiology study the patient She is treated with Sotalol 160mg twice daily.
had inducible left bundle morphology monomorphic VT Arrhyhthmogenic right ventricular cardiomyopathy is
with double extrastimuli at a cycle length of 250 ms characterized macroscopically by a fatty appearance of the
(Fig.146.5). The data was felt to be consistent with ARVC/D. RV free wall. The fibrofatty replacement of the RV myocar-
Ultimately, the patient had a successful abdominal, submus- dium initially produces typical regional wall motion abnor-
cular implantation (rectus sheath pocket) of a dual-chamber malities that later become global, producing RV dilation.
ICD. Six months following implant the patient received an The tissue replacement can also involve areas of the left ven-
appropriate shock for ventricular tachycardia. She had also tricle with relative sparing of the septum.1 Pathologically,
received two treatments with burst pacing within the prior two distinct types of ARVD with differing histologic features
month. Over the following 3years the patient has received have been described. The fibrolipomatous (fibrofatty) pattern
four appropriate ICD shocks for VT and required occasional is characterized by myocardial atrophy and thinning with
replacement of myocardium by both fibrous and fatty tissue,
as well as patchy inflammatory infiltrates. RV aneurysms
and LV involvement are each found in about three-quarters
of patients with fibrofatty disease.2 The lipomatous (fatty)
pattern is characterized by normal or increased myocardial
thickness with exclusively fatty replacement and infrequent
inflammation.2 The fatty type is controversial as to whether
this is the same as ARVD.
Approximately 3050% of cases (or more) are familial.
There are two patterns of inheritance: an autosomal dominant
form, which is most common, and an autosomal recessive
form called Naxos disease, in which ARVD is part of a syn-
drome including hyperkeratosis of the palms and soles and
woolly hair.3 There are at least five established disease-causing
genes that encode desmosomal proteins in the autosomal
Fig.146.3 2-D echocardiogram at end-systole dominant form of the disease: plakoglobin, desmoplakin,

Fig.146.4 RV angiogram in LAO projection at end-diastole (left) and end-systole (right)


562 B.A. Faulknier et al.

Fig.146.5 Monomorphic VT at electrophysiology study, with left bundle branch block type morphology, and cycle length 250ms

plakophilin-2, desmoglein, and desmocollin. Plakoglobin the critearia have also been proposed.13 Beneficial informa-
mutations have been found in Naxos disease. Sen-Chowdhry tion may be obtained from several different diagnostic
etal. have proposed that impaired desmosome function when modalities including ECG, echocardiography, radionuclide
subjected to mechanical stress causes myocyte detachment ventriculography, magnetic resonance imaging, right ven-
and cell death. The myocardial injury may be accompanied tricular angiography, electroanatomic mapping, myocardial
by inflammation as the initial phase of the repair process, biopsy, and other tests.
which ultimately results in fibrofatty replacement of damaged Current guidelines recommend ICD implantation for sec-
myocytes.4 ondary prevention of sudden cardiac death in patients with
The prevalence of ARVD in the general population is esti- sustained VT or VF and for primary prevention in selected
mated to be as high as 1:1,000 and is an important cause of high risk patients.14 Appropriate device interventions are
sudden cardiac death.5,6 Presentation is most common estimated to occur in approximately 4862% of patients.15,16
between the ages of 10 and 50, with a mean age of diagnosis In one study which followed 60 patients with ARVD who
of approximately 30 years.7 The most common symptoms received an ICD for a mean of 80 months, the survival at
are palpitations, syncope, atypical chest pain, and dyspnea.7 1,5, and 10years was 100%, 94%, and 76%, respectively.17
Approximately 50% of patients present with symptomatic Although thin areas of the RV myocardium are rarely perfo-
ventricular arrhythmias, ranging from PVCs to sustained rated during placement of the RV leads, the fibrofatty changes
tachycardia.8 The rate of sudden cardiac death reported var- in the RV often interfere with sensing of arrhythmias.
ies from approximately 410%.9,10 There is an associated risk Moreover, many ARVD patients are young and may require
of VT and sudden cardiac death with exercise and patients multiple ICD and lead replacements over their lifetime.18
should not participate in competitive sports or any activity Antiarrhythmic drugs have not been shown to reduce the
that causes symptoms of palpitations, presyncope, or risk of sudden cardiac death in ARVD. Guidelines suggest
syncope.11 that sotalol or amiodarone may be effective therapies for the
Diagnostic criteria are proposed based upon the presence treatment of sustained VT or VF in patients with ARVD in
of two major criteria, one major plus two minor, or four whom ICD implantation is not feasible.14 Amiodarone or
minor criteria from six categories: (1) family history, (2) sotalol may also be used as an adjunctive therapy for ARVD
ECG depolarization/counduction abnormalities, (3) repolar- patients with an ICD who have frequent ventricular arrhyth-
ization abnormalties, (4) global and/or regional dysfunction mias or shocks, although data from the Northa American
and structural alterations, (5) arryhtymias, and (6) fatty or ARVD Registry suggested that amiodarone is much more
fibrofatty replacement of the RV free wall.12 Modifications to effective than beta-blockers or sotalol.19
Case 146 563

References 12. McKenna WJ, Thiene G, Nava A, Task Force of the Working Group
Myocardial and Pericardial Disease of the European Society of
Cardiology and of the Scientific Council on Cardiomyopathies
1. Richardson P, McKenna WJ, Bristow M, etal. Repert of the 1995 of the International Society and Federation of Cardiology, et al.
WHO/ISFC Task Force on the definition and classification of the Diagnosis of arrhythmogenic right ventricular dysplasia/cardiomy-
cardiomyopathies. Circulation. 1996;93:841. opathy. Br Heart J. 1994;71:215.
2. Basso C, Thiene G, Corrado D, et al. Arrhythmogenic right ven- 13. Marcus FI, McKenna WJ, et al. (2010). Diagnosis of arrhyth-
tricular cardiomyopathy. Dysplasia, dystrophy, or myocarditis? mogenic right ventricular cardiomyopathy/dysplasia: proposed
Circulation. 1996;94:983. modification of the task force criteria. Circulation 121(13):
3. Protonotarios N, Tsatsopoulou A, Patsourakos P, et al. Cardiac 1533-1541.
abnormalities in familial palmoplantar keratosis. Br Heart J. 1986; 14. Zipes DP, Camm AJ, Borggrefe M, et al. ACC/AHA/ESC 2006
56:321. guidelines for management of patients with ventricular arrhythmias
4. Sen-Chowdhry S, Syrris P, McKenna WJ. Genetics of right ventricu- and the prevention of sudden cardiac death: a report of the American
lar cardiomyopathy. J Cardiovasc Electrophysiol. 2005;16:927. College of Cardiology/American Heart Association Task Force and
5. Corrado D, Fontaine G, Marcus FI, Study Group on Arrhythmogenic the European Society of Cardiology committee for Practice Guidelines
Right Ventricular Dysplasia/Cardiomyopathy of the Working (Writing Committee to Develop Guidelines for Management of
Groups on Myocardial and Pericardial Disease and Arrhythmias of Patients With Ventricular Arrhythmias and the Prevention of Sudden
the European Society of Cardiology and of the Scientific Council Cardiac Death). J Am Coll Cardiol. 2006;48:e247.
on Cardiomyopathies of the World Heart Federation, et al. 15. Corrado D, Leoni L, Link MS, et al. Implantable cardioverter-
Arrhythmogenic right ventricular dysplasia/cardiomyopathy: need defibrillator therapy for prevention of sudden death in patients
for an international registry. Circulation. 2000;101:E101. with arrhythmogenic right ventricular cardiomyopathy/dysplasia.
6. Corrado D, Basso C, Schiavon M, etal. Screening for hypertrophic Circulation. 2003;108:3084.
cardiomyopathy in young athletes. N Engl J Med. 1998;339:364. 16. Dalal D, Nasir K, Bomma C, etal. Arrhythmogenic right ventricular
7. Hulot JS, Jouven X, Empana JP, etal. Natural history and risk strati- dysplasia: a United States experience. Circulation. 2005;112:
fication of arrhythmogenic right ventricular dysplasia/cardiomyo- 3823.
pathy. Circulation. 2004;110:1879. 17. Wichter T, Paul M, Wollmann C, et al. Implantable cardioverter-
8. Dalal D, Nasir K, Bomma C, etal. Clinical profile and long-term defibrillator therapy in arrhythmogenic right ventricular
follow-up of 37 families with arrhythmogenic right ventricular cardiomyopathy: single-center experience of long-term follow-up
cardiomyopathy. J Am Coll Cardiol. 2000;36:2226. and complication in 60 patients. Circulation. 2004;109:1503.
9. Tabib a, Loire R, Chalabreysse L, etal. Circumstances of death and 18. Corrado D, Calkins H, et al. (2010). Prophylactic Implantable
gross and microscopic obsevations in a series of 200 cases of sudden Defibrillator in Patients With Arrhythmogenic Right Ventricular
death associated with arrhythmogenic right ventricular cardiomyo- Cardiomyopathy/Dysplasia and No Prior Ventricular Fibrill-
pathy and/or dysplasia. Circulation. 2003;108:3000. ation or Sustained Ventricular Tachycardia. Circulation 122(12):
10. Maron BJ, Carney KP, Lever HM, et al. Relationship of race to 1144-1152.
sudden cardiac deathin competitive athletes with hypertrophic 19. Marcus GM, Glidden DV, et al. (2009). Efficacy of Antiarrhythmic
cardiomyopathy. J Am Coll Cardiol. 2003;41:974. Drugs in Arrhythmogenic Right Ventricular Cardiomyopathy: A
11. Maron BJ, Ackerman MJ, Nishimura RA, etal. Task Force 4: HCM Report From the North American ARVC Registry. J Am Coll
and other cardiomyopathies, mitral valve prolapse, myocarditis, Cardiol 54(7): 609-615.
and Marfan syndrome. J Am Coll Cardiol. 2005;45:1340.

Case 147

Stefan H. Hohnloser and Joachim R. Ehrlich

Case Summary Case Discussion

A 60-year-old business man presented with complaints of a The patient was instructed to use a single oral dose of fle-
rapid irregular heart beat for the first time in 2002. He was cainide, 200300 mg, in case of new-onset AF. During a
found to have atrial fibrillation (AF) with a ventricular rate follow-up period of 5years, the patient experienced on aver-
between 170 and 180bpm. His prior history was completely age 34 episodes of AF per year. All but one episode were
unremarkable with no previous evidence for cardiovascular stopped by orally administered flecainide. The episode that
abnormalities. Shortly before scheduled DC cardioversion, did not respond to oral medication required a DC cardiover-
spontaneous restoration of sinus rhythm occurred. He under- sion, which was performed in the outpatient clinic without
went a thorough clinical examination including a resting complications. At present, the patient continues to do well on
ECG, exercise stress testing, and 2-D transthoracic echocar- the described therapeutic regimen.
diography. No evidence for structural heart disease was In many patients afflicted with paroxysmal AF, no evi-
found but the patient suffered from mild arterial hyperten- dence for structural heart disease can be detected on clinical
sion for which he was started on ramipril 5mg once daily. examination. Many of these patients may suffer from some
Approximately 3months later, the patient was admitted via degree of arterial hypertension. The frequency of AF attacks
the emergency room for a new episode of AF which had varies widely from only a few per year up to almost daily
lasted for approximately 4h. While in hospital, the patient episodes. Patients in whom only few but symptomatic epi-
was continuously monitored and pharmacological cardiover- sodes occur are candidates for intermittent antiarrhythmic
sion by means of intravenous administration of 1 mg/kg drug therapy, the so-called pill-in-the-pocket approach
flecainide was attempted. Approximately 50 min after the (Table147.1). The main rationale of this form of treatment
injection, sinus rhythm was restored. There were no abnor- is to avoid side effects associated with long-term exposure to
mal findings on subsequent standard ECG recordings and on antiarrhythmic drugs and to apply the medication only to
24h Holter monitoring, particularly no widening of the QRS stop an ongoing AF episode rather than preventing recurrent
complex or prolongation of the QT interval as a result of AF attacks. Over the last decade, several clinical trials have
antiarrhythmic drug administration. What long-term therapy evaluated this type of intermittent therapy (Table 147.1).14
should be considered in this patient? As a common feature of these and other similar trials, patients
with significant structural heart disease, abnormalities in
impulse formation or propagation (i.e., bardycardia <60bpm,
evidence for preexcitation), acute coronary syndromes or
congestive heart failure were excluded. Generally, AF was
present for no more than 7days. These studies evaluated the
efficacy and safety of administration of a single dose of class
S.H. Hohnloser(*) IC antiarrhythmic drugs such as propafenone or flecainide.
Department of Cardiology, Section of Clnical Electrophysiology, When compared to placebo,1,2 both drugs restored sinus
J.W. Goethe University, Theodor-Stern-Kai 7, rhythm in a significantly shorter period of time than placebo.
Frankfurt 60590, Germany
e-mail: hohnloser@em.uni-frankfurt.de Importantly, patients were treated in-hospital with continu-
ous ECG monitoring in most studies. In the study by Alboni
J.R. Ehrlich
Division of Clinical Electrophysiology, J.W. Goethe University,
et al., patients were exposed to the drugs for the first time
Theodor-Stern-Kai 7, 60590 Frankfurt, Germany in-hospital; responders (i.e., patients with successful
e-mail: j.ehrlich@em.uni-frankfurt.de pharmacological cardioversion) were then instructed how to

A. Natale et al. (eds.), Cardiac Electrophysiology, 565


DOI: 10.1007/978-1-84996-390-9_147, Springer-Verlag London Limited 2011
566 S.H. Hohnloser and J.R. Ehrlich

Table147.1 Summary of clinical trial examining the use of anti-arrhythmic medications intermittently for acute episodes
Study (reference) Patients Setting Therapy Main outcome
Capucci etal.1 181 In-hospital Propafenone 600mg Propafenone and flecainide superior to
Flecainide 300mg placebo in restoring SR with 72%,
78%, and 39% at 8h
Placebo
Boriani etal.2 240 In-hospital Propafenone 600mg Propafenone superior to placebo in
Placebo restoring SR with 76% and 37% at 8h

Blanc etal.3 86 In-hospital Propafenone 600mg Mean time to restoration of SR shorter for
Amiodarone 30mg/kg propafenone, at 24h no difference

Alboni etal.4 268 In-hospital followed by Propafenone 600mg Therapy successful in 94% of patients
self-administration over Flecainide 300mg over 155month follow-up
15months as needed

self-administer the previously successful antiarrhythmic References


drug.4 This approach worked in almost all patients resulting
in significantly less emergency room visits and hospitaliza- 1. Capucci A, Boriani G, Botto GL, etal. Conversion of recent-onset
tions during follow-up compared to the year before the target atrial fibrillation by a single oral loading dose of propafenone or
AF episode.4 flecainide. Am J Cardiol. 1994;74:503-506.
2. Boriani G, Biffi A, Capucci A, etal. Oral propafenone to convert
The so-called pill-in-the-pocket approach for treatment
recent-onset atrial fibrillation in patients with and without underly-
of paroxysmal AF should be considered in patients with rare ing heart disease. Ann Inter Med. 1997;126:621-625.
to infrequent episodes of AF which are, however, promptly 3. Blanc JJ, Voinov C, Maarek M, on behalf of the PARSIFAL Study
recognized by the patient upon their onset. Before recom- group. Comparison of oral loading dose of propafenone and amio-
darone for converting recent-onset atrial fibrillation. Am J Cardiol.
mending the self-administration of a specific antiarrhythmic
1999;84:1029-1032.
drug, this medication should be administered in a monitored 4. Alboni P, Botto GL, Baldi N, et al. Outpatient treatment of
environment to ensure that no unwanted effects of the antiar- recent-onset atrial fibrillation with the pill-in-the-pocket approach.
rhythmic drug occur. Since clinical studies on this form of N Engl J Med. 2004;351:2384-2391.
intermittent therapy have predominantly used class IC anti-
arrhythmic drugs such as flecainide or propafenone, only
patients with no or at the most minimally structural heart
disease should be considered for this approach.
Case 148

Joachim Ehrlich and Stefan H. Hohnloser

Case Summary recovered fully without neurological sequelae. Prior to hospital


discharge, ECG had returned to normal with a QTc interval of
416ms (Fig.148.2a). Microvolt T-wave alternans testing by
A 62-year-old male patient with a history unremarkable for
means of submaximal exercise testing revealed negative find-
cardiac arrhythmias was admitted to the hospital for a first
ings (Fig.148.2b). Subsequent genetic testing revealed a pre-
episode of symptomatic atrial fibrillation with rapid ventric-
viously unpublished mutation at the KCNE2-gene (exon 1,
ular rate. There was no evidence for structural heart disease
position A269G) leading to a glycine substitution for glu-
by means of history, physical exam and ECG and the patient
tamine within the c-terminus of this potassium channel b-sub-
was started on intravenous therapy with amiodarone for
unit corresponding to the long QT syndrome type VI entity.
pharmacological conversion. There were no signs of abnor-
The present case nicely illustrates several important clini-
mal repolarization on the resting ECG - the QTc (averaged
cal, electrocardiographic and molecular findings in patients
over 3 cycles) measured 430ms. Sinus rhythm was restored
with drug-induced long QT syndrome and torsades de
after 48h of drug infusion with a total dose of 3g of amio-
pointes(Table 148.1). The term reduced repolarization
darone. At this point, the patient developed cardiac arrest
reserve has been coined as a unifying concept to explain the
with ventricular fibrillation and had to be resuscitated. What
variable risk for this syndrome.1,2 This framework suggests
is the likely cause of the cardiac arrest and what further
that the physiological mechanisms that maintain normal car-
testing should be considered?
diac repolarization vary among patients but are not apparent
in the basal state. However, exposure to a substance that pro-
longs the QT interval, or the development of a risk factor
such as bradycardia after restoration of sinus rhythm in a
Case Discussion patient presenting with atrial fibrillation (see Table148.2), is
more likely to cause exaggerated QT prolongation in a
The surface ECG revealed pronounced QTc prolongation susceptible patient than in a nonsusceptible one.
(592ms) along with a prominent macroscopic T-wave altern- Perhaps the most intriguing finding in this patient was
ans (Figs.148.1a, b). Development of this T-wave alternans that of a novel mutation corresponding to the long QT syn-
Coincided with the development of polymorphic ventricular drome type VI entity. Before exposure to amiodarone which
tachycardia of the torsades de pointes type degenerating into was administered for pharmacological conversion of recent
ventricular fibrillation (Fig.148.1c). Amiodarone plasma con- onset atrial fibrillation, the patient had no cardiovascular his-
centration at that time was in the low therapeutic range tory. Similarly, his family history was unremarkable. These
(0.87 mg/L; therapeutic range 0.592.5 mg/L). The patient findings emphasize the fact that the congenital long QT syn-
was successfully resuscitated, taken off amiodarone and drome has been recognized as a syndrome with incomplete

Table148.1 Key Points


J. Ehrlich(*) Although it is a rare phenomenon, amiodarone (particularly at low
Division of Clinical Electrophysiology, J.W. Goethe University, plasma concentrations) can induce proarrhythmia (e.g., torsades
Theodor-Stern-Kai 7, 60590, Frankfurt, Germany de pointes)
e-mail: j.ehrlich@em.uni-frankfurt.de The concept of reduced repolarization reserve has been used to
explain the variable risk of patients to develop drug-induced
S.H. Hohnloser
proarrhythmia
Department of Cardiology, Section of Clnical Electrophysiology,
J.W. Goethe University, Theodor-Stern-Kai 7, There are several clinical risk indicators for drug-associated
Frankfurt 60590, Germany proarrhythmia which need to be checked in all patients prior to
e-mail: hohnloser@em.uni-frankfurt.de initiation of antiarrhythmic drug therapy

A. Natale et al. (eds.), Cardiac Electrophysiology, 567


DOI: 10.1007/978-1-84996-390-9_148, Springer-Verlag London Limited 2011
568 J. Ehrlich and S.H. Hohnloser

Fig.148.1 (a) Normal baseline QT interval during rapidly conducted therapy. (c) Continuous monitoring showing the emergence of poly-
atrial fibrillation. (b) Prominent QT prolongation and macroscopic morphic tachycardia. Horizontal arrows indicate the typical short-
TWA (arrows) after restoration of sinus rhythm during amiodarone long-short initiating sequence of torsades de pointes

penetrance; that is, family members with near-normal QT grossly abnormal repolarization process gave then rise to the
intervals may carry nevertheless the same mutations in genes occurrence of torsades de pointes. The arrhythmia started
associated with the congenital long QT syndrome.3,4 with the typical initiation sequence shown in (Fig.148.1). Of
Current evidence suggests that 510% of patients in whom note, after stopping the offending drug amiodarone and
torsades de pointes develops upon exposure to QT-prolonging subsequent normalization of the QT interval, there was no
drugs harbor mutations associated with the long QT syn- evidence for microscopic T-wave alternans in our patient
drome and can thus be viewed as having a subclinical form of (Fig. 148.2). Whether this observation indicates complete
the congenital syndrome.2,5 This clinical observation nicely reversibility of the proarrhythmic changes induced by amio-
fits in the concept of reduced repolarization reserve arising darone, remains speculative at the present time.
from a mutation in an ion-channel gene which predisposes It appears important to note that amiodarone treatment is
the carrier to drug-induced proarrhythmia. associated with an exceptionally low rate of proarrhythmic
In the present case, several electrocardiographic hall- events compared to other antiarrhythmic drugs.8 If, however,
marks of drug-induced proarrhythmia became evident. amiodarone-induced proarrhythmia occurs, it does typically
Besides marked prolongation of the QT interval, macro- so at lower plasma concentration.9,10 This is in line with
scopic T-wave alternans became evident. This very rare phe- recent experimental studies providing evidence that acute
nomenon has been first described shortly after the invention amiodarone application on the background of reduced repo-
of electrocardiography. In 1948, Kalter and Schwartz exam- larization reserve (induced by augmentation of late INa)
ined the ECGs from 6,059 patients and described anassocia- exhibits a biphasic effect with proarrhythmic actions occur-
tion between macroscopic TWA (observed in 5patients) and ring at lower and anti-arrhythmic effects at higher concentra-
an increased mortality of the affected patients.6 Subsequent tions.11 Low nanomolar concentrations of amiodarone that
case reports described the occurrence of visible TWA in vari- alone caused no significant action potential duration prolon-
ous clinical situations such as myocardial ischemia, coronary gation caused significant prolongation and torsades de
artery spasm, electrolyte disturbances, and particularly in the pointes tachycardia when administered in combination with
setting of the congenital long QT syndrome.7 Obviously, this enhanced late INa. Acutely administered amiodarone inhibits
Case 148 569

Fig.148.2 (a) Resting ECG prior a


to discharge. (b) Negative
microvolt T-wave alternans test
performed after discontinuation
of miodarone treatment

b
120
HR Max. 113
(bpm)
100

80

% bad 20

0
noise (V) 2
1
0

VM

Z
2 uV

Min.
0 2 4 6 8 10 12
Phase 1

HERG currents at low concentrations (proarrhythmic) while Table148.2 Risk factors for drug-induced torsades de pointes
its effect on sodium current (antiarrhythmic) occurs at several Female sex
orders of magnitude higher concentrations.11 Hypokalemia, hypomagnesemia
Finally, our patient is an exceptional example of how it Bradycardia
may be impossible to detect patients at risk for proarrhythmic Recent conversion from atrial fibrillation, particularly with a
QT-prolonging drug
events with classical clinical tools (Table148.2). The diagno- Congestive heart failure
sis of congenital long QT syndrome may in some cases only Digitalis therapy high drug concentration (exceptions: quinidine,
be made on the basis of genetic testing. Whether broader amiodarone)
availability of reliable and fast methodology to detect genetic Rapid rate of infusion with a QT-prolonging drug
alterations may improve safety of antiarrhythmic drug therapy Subclinical long QT syndrome
remains to be seen. Ion-channel polymorphisms
570 J. Ehrlich and S.H. Hohnloser

References 7. Schwartz PJ, Malliani A. Electrical alternation of the T wave:


clinical and experimental evidence of its relationship with the
sympathetic nervous system and with the long-QT syndrome. Am
1. Roden DM. Drug-induced prolongation of the QT interval. N Engl Heart J. 1975;89:45-50.
J Med. 2004;350:1013-1022. 8. Hohnloser SH, Singh BN. Proarrhythmia with class III antiarrhyth-
2. Roden DM. Taking the idio out of idiosyncratic: predicting mic drugs: definition, electrophysiologic mechanisms, incidence,
torsades de pointes. Pacing Clin Electrophysiol. 1998;21:1029- predisposing factors, and clinical implications. J Cardiovasc
1034. Electrophysiol. 1995;6:920-936.
3. Keating MT, Sanguinetti MC. Molecular and cellular mechanisms 9. Tomcsanyi J, Merkely B, Tenczer J, etal. Early proarrhythmia dur-
of cardiac arrhythmias. Cell. 2001;104:569-580. ing intravenous amiodarone treatment. Pacing Clin Electrophysiol.
4. Mohler PJ, Schott JJ, Gramolini AO, et al. Ankyrin-B mutation 1999;22:968-970.
causes type 4 long-QT cardiac arrhythmia and sudden cardiac death. 10. Bertholet M, Dubois C, Materne P, et al. Sudden marked QT
Nature. 2003;421:634-639. prolongation after intravenous administration of amiodarone. Am J
5. Yang P, Kanki H, Drolet B, etal. Allelic variants in long-QT disease Cardiol. 1983;52:1361.
genes in patients with drug-associated torsades de pointes. 11. Wu L, Rajamani S, Shyrode JC, Li H, Ruskin J, Antzelevitch
Circulation. 2002;105:1943-1948. C, Belardinelli L. Augmentation of late sodium current unmasks
6. Kalter HH, Schwartz ML. Electrical alternans. NY State J Med. the proarrhythmic effects of amiodarone. Cardiovasc Res. 2008;
1948;1:1164-1166. 77(3):481-488.
Case 149

Bradley P. Knight

Case Summary She recovered clinically and was referred to a heart fail-
ure clinic a few weeks later, where she was found to have a
A 46-year-old woman, who was previously healthy, presented heart rate was just over 100 bpm. Her electrocardiogram
with cardiogenic shock and a heart rate of 180bpm. An elec- showed a P-wave morphology that was similar to that when
trocardiogram is shown in Fig.149.1. The rhythm is a long she presented initially, and there was mild first degree AV
RP tachycardia with P-waves that have an inferior axis and block (Fig.149.2). Rhythm strips from when the patient was
are deeply inverted in lead V1. An echocardiogram revealed on telemetry during her initial hospitalization were obtained
severe, global left ventricular dysfunction with no significant (Fig. 149.3). Does this patient have sinus tachycardia as
valvular abnormalities. She was treated with inotropic sup- a result of an idiopathic dilated cardiomyopathy, or a
port initially and then with a beta blocker (carvedilol) and an tachycardia-mediated cardiomyopathy?
angiotensin converting enzyme inhibitor (enalapril).

Fig.149.1 12-lead electrocardiogram obtained at the time of initial presentation

B.P. Knight
Division of Cardiology, Northwestern Medical Center,
676 N. St. Clair, Suite 600, Chicago, IL 60611, USA
e-mail: bknight@nmff.org

A. Natale et al. (eds.), Cardiac Electrophysiology, 571


DOI: 10.1007/978-1-84996-390-9_149, Springer-Verlag London Limited 2011
572 B.P. Knight

Fig.149.2 12-lead electrocardiogram obtained a few weeks after the initial presentation

Fig.149.3 Rhythm strip obtained during initial hospitalization


Case 149 573

Case Discussion greater than 150% of that predicted for age, an inverted or
notched P-wave in lead V1, and 2 AV block. This patient
satisfied had all three criteria. Successful catheter ablation
Rapid, sustained ventricular rates can lead to a tachycardia-
of an ectopic atrial tachycardia arising from the right atrium
induced cardiomyopathy. This has been well documented
was performed three months after her initial presentation. An
in patients as well as in pacing-induced heart failure animal
echocardiogram three months later showed normal ventricu-
models.1 However, in a patient with ventricular dysfunc-
lar function.
tion, it can be occasionally difficult to determine whether or
not a supraventricular tachycardia is the primary problem.
This is particularly true when the P-wave morphology is
similar to a sinus P-wave morphology. A useful clue in this References
case can be seen in the monitor strips in Fig.149.3, which
show Wenckebach type, 2 AV block. This clue is useful, 1. Shinbane JS, Wood MA, Jensen DN, Ellenbogen KA, Fitzpatrick
because it would be unlikely for a patient with heart fail- AP, Scheinman MM. Tachycardia-induced cardiomyopathy: a
ure to demonstrate Wenckebach type, 2 AV block during review of animal models and clinical studies. J Am Coll Cardiol.
1997;29:709-715.
sinus tachycardia. A study by Gelb and Garson in 19902 2. Gelb BD, Garson A Jr. Noninvasive discrimination of right atrial
found three observations that differentiated a right atrial ectopic tachycardia from sinus tachycardia in dilated cardiomyo-
ectopic tachycardia from sinus tachycardia: an atrial rate pathy. Am Heart J. 1990;120:886-891.

Case 150

Bradley P. Knight

Case Summary Given her young age and the possibility of a tachycardia-
mediated cardiomyopathy, an ablation procedure should be
considered as first-line therapy for this patient.
A 21-year-old woman presented to her obstetrician with an
The observation that the tachycardia persisted despite
incessant supraventricular tachycardia at 160 bpm during her
transient AV block excludes AV re-entry using an accessory
27th week of pregnancy. She was asymptomatic and unaware
pathway as the mechanism, and the absence of deeply
of her tachycardia. Left ventricular function was normal.
inverted P-waves in the inferior leads excludes AV nodal re-
There were times when the tachycardia persisted despite tran-
entry. Therefore, the mechanism of the tachycardia must be
sient AV block. Transesophageal pacing failed to restore
atrial. The P-wave morphology is clearly inverted in lead I
sinus rhythm. She was treated with quinidine and a meto-
and upright in lead aVR. This suggests a left atrial origin,
prolol. The tachycardia did not terminate, but the atrial rate
away from the septum. In addition, the P-wave is mostly
slowed to 130 bpm. An electrocardiogram is shown in
upright in the inferior leads, but the axis is not inferior enough
Fig.150.1 after the initiation of antiarrhythmic drug therapy.
to be consistent with an upper pulmonary vein origin. It is
She underwent uneventful delivery of her child and was
most likely that the rhythm is arising from the mitral annulus
subsequently referred for further management. The tachy-
or left atrial appendage.
cardia was still present, but the heart rate was around 100
A catheter ablation procedure was performed. Activation
bpm. What is the best approach to this patient at this stage?
mapping was performed in the left atrium and an electroana-
Based on the electrocardiogram, what is the mechanism of
tomic map was created (Fig.150.2). The view is right ante-
the tachycardia and from where is it arising?
rior oblique, with the right-sided pulmonary veins to the left
and the mitral annulus to the right. The color coding is from
red to purple and represents the relative timing at each
recorded site during an activation map. The earliest activa-
Case Discussion tion is depicted as red and is located near the base of the left
atrial appendage. Delivery of radiofrequency energy at this
This is a patient with an incessant tachycardia who was site terminated the tachycardia and rendered it noninducible.
treated medically at the time of presentation, because she Although focal atrial tachycardias can arise anywhere in
was pregnant and asymptomatic. Although catheter ablation the atria, there is a characteristic anatomic distribution.1
can be performed safely during pregnancy, medical therapy Common sites include the crista terminalis, the pulmonary
was reasonable as the initial treatment in this patient. After veins, and the left atrial appendage.2 Catheter ablation for
the patient has delivered her child, the question is whether to focal atrial tachycardias is associated with a high success
continue medical therapy or recommend ablative therapy. rate.

B.P. Knight
Division of Cardiology, Northwestern Medical Center,
676 N. St. Clair, Suite 600, Chicago, IL 60611, USA
e-mail: bknight@nmff.org

A. Natale et al. (eds.), Cardiac Electrophysiology, 575


DOI: 10.1007/978-1-84996-390-9_150, Springer-Verlag London Limited 2011
576 B.P. Knight

Fig.150.1 Electrocardiogram
obtained during tachycardia

Fig. 150.2 Electroanatomic map of the left atrium during an atrial


tachycardia

References

1. Roberts-Thomson KC, Kistler PM, Kalman JM. Focal atrial tachy-


cardia I: clinical features, diagnosis, mechanisms, and anatomic
location. Pacing Clin Electrophysiol. 2006;29:643-652.
2. Kato M, Adachi M, Yano A, etal. Radiofrequency catheter ablation
for atrial tachycardia originating from the left atrial appendage.
J Int Card Electrophys. 2007;19:45-48.
Case 151

Bradley P. Knight

Case Summary without atrial fibrillation and a 5-beat run of nonsustained


monomorphic VT. He was doing well without complaints
A 37-year-old man is referred for evaluation of atrial fibril- and his physical examination was unremarkable. He denied
lation. He presented with his first episode of atrial fibrilla- syncope. He had not history of hypertension or other medi-
tion 6 months earlier. A nuclear perfusion stress test was cal problems. However, he did recall that when he was a
reportedly normal. He was treated with atenolol and warfa- child, his mother was found dead at home unexpectedly
rin, and underwent electrical cardioversion. He had a sec- atage 41.
ond episode of atrial fibrillation 2 months ago and was An electrocardiogram was obtained and is shown in
started on oral amiodarone after another electrical cardio- Fig.151.1. What is the most likely diagnosis? How should
version. A recent Holter monitor showed sinus rhythm he be managed?

Fig. 151.1 A 12-lead electrocardiogram of a young patient with recurrent atrial fibrillation

B.P. Knight
Division of Cardiology, Northwestern Medical Center,
676 N. St. Clair, Suite 600, Chicago, IL 60611, USA
e-mail: bknight@nmff.org

A. Natale et al. (eds.), Cardiac Electrophysiology, 577


DOI: 10.1007/978-1-84996-390-9_151, Springer-Verlag London Limited 2011
578 B.P. Knight

Case Discussion Table151.1 Potential risk factors for sudden cardiac death in patients
with hypertrophic cardiomyopathy
History of cardiac arrest
This case is a young patient with atrial fibrillation and a Family history of premature death
family history of premature sudden death. In such cases, it is History of syncope
important to thoroughly evaluate the patient for possible Presence of nonsustained ventricular tachycardia Holter
heritable diseases, including structural heart diseases and Early onset of disease
channelopathies. The electrocardiogram shows sinus rhythm Magnitude of left ventricular hypertrophy
with biatrial enlargement, left ventricular hypertrophy with Abnormal blood pressure response to exercise
Myocardial ischemia on perfusion tomography
marked repolarization abnormalities. Given the absence of
Extent of myocyte disarray or interstitial fibrosis
hypertension and his malignant family history, the most Malignant causal mutations or modifier genes
likely diagnosis is hypertrophic cardiomyopathy. The diag- Adapted from1
nosis was confirmed by echocardiography. He had severe
concentric left ventricular hypertrophy, but no outflow tract
gradient. References
This case emphasizes the importance of a careful search
for secondary causes of atrial fibrillation, even in patients 1. Marian AJ. On predictors of sudden cardiac death in hypertrophic
who are referred several months after their initial presenta- cardiomyopathy. J Am Coll Cardiol. 2003;41:994-996.
tion. Patients with hypertrophic cardiomyopathy are at risk 2. Maron BJ, Shen WK, Link MS, et al. Efficacy of implantable
cardioverter-defibrillators for the prevention of sudden death in
for cardiac arrest. Various clinical and echocardiographic patients with hypertrophic cardiomyopathy. N Engl J Med.
markers have been identified that appear to increase the risk 2000;342:365-373.
of sudden death (Table151.1).1 Given this patients family 3. Maron BJ, Spirito P, Shen WK, et al. Implantable cardioverter-
history and the presence of nonsustained ventricular tachy- defibrillators and prevention of sudden cardiac death in hypertro-
phic cardiomyopathy. JAMA. 2007;298:405-412.
cardia, he underwent defibrillator implantation.2,3 Because 4. Knight BP, Gersh BJ, Carlson MD, etal. Role of permanent pacing
he had recurrent atrial fibrillation, a dual-chamber device to prevent atrial fibrillation: science advisory from the American
with algorithms that help manage his atrial fibrillation was Heart Association Council on Clinical Cardiology. Circulation.
implanted.4 2005;111:240-243.
Case 152

Andrew D. Krahn

Case Summary Case Discussion

A 39-year-old athletic male presented to the emergency Syncope affects 40% of the population at one point in their
room after an episode of syncope that occurred within a min- life, and is most often explained by vasovagal syncope in the
ute of completing a 90-minute training session that included absence of underlying heart disease. A careful history and
sprints and distance running. He described relatively abrupt exclusion of heart disease by physical examination, resting
onset of lightheadedness with imminent loss of conscious- ECG and an echocardiogram is often sufficient to arrive at a
ness, after which he lay down. He lost consciousness briefly, clinical diagnosis and exclude more sinister causes.
and awakened fatigued but oriented. Because of the concern In the current case, syncope occurred in the context of
of his fellow running club members, he was brought to the exercise, which should always raise concern that cardiac
emergency room where an ECG was performed (Fig.152.1). obstruction or an arrhythmia may have been induced by
He was monitored in the emergency room for several hours, exercise. In this patient, a careful history indicated that the
and discharged with arrangements made to obtain an urgent episode was immediately after exercise and not during exer-
out patient echocardiogram, external loop recorder and tion, a much less sinister presentation. Venous return is depen-
Cardiology follow-up. dent on venous pressure and the skeletal muscle pump. When
Cardiology review found that he had fainted once at age exercise is stopped abruptly, venous return may fall precipi-
14 at his cousins wedding on a hot day. His uncle died tously, leading to a fall in blood pressure and presyncope or
suddenly of a heart attack at age 42. His echocardiogram syncope. This is seen frequently when patients undergo exer-
showed mild concentric left ventricular hypertrophy in keep- cise testing, and are unsteady when the treadmill or bicycle is
ing with an athletic heart. An external loop recorder worn for stopped. Cool down exercise typically attenuates symptoms.
2 weeks captured the cardiac rhythm during presyncope, Follow-up assessment in the current case demonstrated no
which was different in character than the index syncopal evidence of obstruction or tachyarrhythmia. The resting ECG
episode (Fig.152.2). Because of concern regarding the cir- shows sinus arrhythmia, which is typical in athletes. Importantly,
cumstances of syncope and his family history, an implanted the ECG did not show evidence of a repolarization abnormality
loop recorder (ILR) was inserted. Syncope of similar charac- such as a prolonged QT interval. An exercise test to screen for
ter recurred 2 months later while standing awaiting a ride catecholaminergic ventricular tachycardia (CPVT) or other
several minutes after a 90-minute training run (Fig.152.3). benign exercise induced arrhythmia would have been reason-
What is the interpretation of the findings seen in Figs.152.2 able. The external loop recorder again demonstrated sinus
and 152.3? Will this patient benefit from pacemaker arrhythmia during presyncope, suggesting high vagal tone sup-
implantation? porting the clinical diagnosis of probable vasovagal syncope.
Finally, the ILR captured an episode of post exercise
syncope that demonstrated sinus rhythm with premature
atrial beats, followed by dramatic sinus slowing and junc-
tional rhythm, followed by a 21-second pause. Note the
baseline artifact during the pause that likely represents the
motion artifact from striking the ground during syncope.
There is also sinus acceleration and tachycardia after the
episode. The gradual slowing preceding the pause suggests
A.D. Krahn
Division of Cardiology, University of Western Ontario,
a profound cardioinhibitory form of vasovagal syncope.
339 Windermere Road, London, ON, N6A 5A5, Canada Interrogation of implantable loop recorders after a recur-
e-mail: akrahn@uwo.ca rence of syncope will often demonstrate rhythm findings

A. Natale et al. (eds.), Cardiac Electrophysiology, 579


DOI: 10.1007/978-1-84996-390-9_152, Springer-Verlag London Limited 2011
580 A.D. Krahn

Fig.152.1 Resting ECG

Fig.152.2 External loop


recorder rhythm strip during
presyncope
Case 152 581

Fig.152.3 Interrogation of the


implantable loop recorder after
an episode of loss of consious-
ness. See text for discussion

that require clinical correlation and considerable judgment off, and outcome will be assessed. This study promises to be
to interpret. an important trial that uses ILR based bradycardia instead of
Even with documented asystole, the need for permanent tilt testing to identify vasovagal syncope patients that may
pacemaker therapy in presumed vasobagal syncope is con- benefit from pacing.
troversial. The ISSUE investigators have proposed a useful
classification system for symptomatic events in loop record-
ers applied to patients with syncope.1 This is often helpful in
References
assigning a probable mechanism of syncope, and is likely to
be adopted for research purposes to create consistency in end
1. Brignole M, Moya A, Menozzi C, Garcia-Civera R, Sutton R.
points. Despite the dramatic documented pause, pacemaker
Proposed electrocardiographic classification of spontaneous
implantation for what is presumed to be vasovagal syncope syncope documented by an implantable loop recorder. Europace.
is a IIa indication for pacing. Since the guidelines were pub- 2005;7:14-18.
lished in 2002, two additional randomized double blinded 2. Connolly SJ, Sheldon R, Thorpe KE, etal. Pacemaker therapy for
prevention of syncope in patients with recurrent severe vasovagal
studies have not found benefit from pacing, so that pacing in
syncope: second Vasovagal Pacemaker Study (VPS II): a random-
this population is seldom performed.2, 3 The currently enroll- ized trial. JAMA. 2003;289:2224-2229.
ing ISSUE3 study is enrolling patients with vasovagal 3. Raviele A, Giada F, Menozzi C, etal. A randomized, double-blind,
syncope that undergo ILR implantation, and pacemaker placebo-controlled study of permanent cardiac pacing for the treat-
ment of recurrent tilt-induced vasovagal syncope. The vasovagal
implantation in those with documented asystole such as the
syncope and pacing trial (SYNPACE). Eur Heart J. 2004;25:
current patient. The pacemaker is then randomized to on or 1741-1748.

Case 153

Byron K. Lee, Melvin M. Scheinman, and Zian H. Tseng

Case Summary a characteristic of block in the AV node while Type 2 block is


more common with block below or within the bundle of His.
In stable second degree AV block with 2:1 conduction, the
A 71-year-old woman with a history of chronic obstructive
Mobitz classification scheme cannot be applied since every
pulmonary disease was admitted with worsening dyspnea on
other P wave is blocked. PR prolongation cant be measured.
exertion. On evaluation, she was found to be bradycardic
Therefore, the site of block cannot be inferred. Determination
with a ventricular rate of 35bpm. Telemetry showed that she
of the site of block, however, is an important factor for clinical
was in sinus rhythm with 2:1 conduction to the ventricle
decision making and several maneuvers may be helpful.
(Fig.153.1). Her 12 lead ECG revealed right bundle branch
Patients with AV nodal block are often strongly affected
block and left anterior fascicular block.
by changes in vagal tone. These patients typically do not fre-
It was suspected that the patients dyspnea on exertion
quently progress to complete heart block and therefore will
was at least partially due to AV nodal conduction disease.
rarely require a pacemaker. Patients with block in the His
However, she was unable get out of bed and exercise to allow
Purkinje system typically have structural degeneration of the
assessment of AV node conduction at higher heart rates.
conduction system that is strongly rate dependent and is not
Therefore, carotid sinus massage (CSM) was performed
affected by changes in vagal tone. Block in the HisPurkinje
(Fig.153.2). What does the reaction to CSM tell you about
system does frequently progress to complete heart block, and
AV nodal function?
a pacemaker is usually necessary.12
We can often determine the site of second degree AV block
by simply exercising patients to increase their sinus rate. When
second degree AV block occurs in the AV node, you should
Case Discussion
see improvement of conduction when vagal tone is withdrawn
and worsening if vagal tone is augmented. If the site of the
Second degree AV block is often classified as either Mobitz second degree AV block is below the node, you should see
Type 1 or Mobitz Type 2. In Type 1 block there is gradual PR worsening of conduction with a higher ratio of blocked beats
interval prolongation in successive beats before block occurs. at faster sinus rates and improvement if the sinus rate slows.
In Type 2 block, the PR interval in the preceding beats is con- In this patient, exercise could not be performed because she
stant before a P wave is dropped. Type 1 block is thought to be was initially unable to get out of bed. Therefore, CSM was per-
formed, CSM typically causes a temporary increase in centrally
mediated vagal tone which causes the sinus rate to slow. Since
B.K. Lee (*) second degree AV block in the AV node is mainly caused by
Division of Cardiology, University of California,
500 Parnassus Avenue, San Francisco, CA 94143, USA excess vagal tone, you may see the AV block worsen with CSM
e-mail: leeb@medicine.ucsf.edu in these patients. In contrast, increasing vagal tone with CSM in
M.M. Scheinman
patients with second degree AV block in the HisPurkinje sys-
Department of Cardiac Electrophysiology, tem may paradoxically improve conduction. As the sinus rate
University of California, 500 Parnassus Avenue, MUE 436, slows there is more time between beats for recovery in the infra-
San Francisco, CA 94143-1354, USA nodal conduction system and 1:1 conduction may be restored.
e-mail: scheinman@medicine.ucsf.edu
In this patient, CSM improved AV conduction to 1:1 as
Z.H. Tseng the sinus rate slowed from 72 to 50beats per min (Fig.153.2).
Cardiac Electrophysiology Section, Cardiology Division,
Therefore, block in the HisPurkinje system is the likely
University of California, San Francisco, 500 Parnassus Avenue,
MUE, RM 433, San Francisco, CA 94143, USA diagnosis. The patient had a dual chamber PPM implanted.
e-mail: zhtseng@medicine.ucsf.edu Her dyspnea on exertion subsequently improved.

A. Natale et al. (eds.), Cardiac Electrophysiology, 583


DOI: 10.1007/978-1-84996-390-9_153, Springer-Verlag London Limited 2011
584 B.K. Lee et al.

Fig.153.1 Baseline telemetry strip shows 2:1 AV conduction with an underlying sinus cycle length of 840ms

Fig.153.2 Telemetry during and after carotid sinus massage. The sinus cycle length has prolonged to 1,200ms

References 2. Saoudi N, Rinaldfi JP, Imianitoff M, et al. Atrioventricular and


intraventricular conduction disorders. In: Crawford MH, DiMarco
JP, Paulus WJ, eds. Cardiology. 2nd ed. London: Mosby; 2004:
1. Wallick DW, Stuesse SL, Levy MN. Mechanisms of vagosympa- 645-658.
thetic control of the atrial-AV nodal conduction. In: Mazgalev TN,
Tchou PJ, eds. Electrophysiology: A View from the Millenium.
Armonk, NY: Futura; 2000:133-135.
Case 154

Srijoy Mahapatra

Case Summary His echocardiogram the next morning revealed an EF of 45%


with an inferior wall motion abnormality. Coronary angiogra-
A 55-year-old man with a history of an inferior myocardial phy showed a patent right coronary stent and no disease in the
infarction treated with primary angioplasty with stenting 4h left coronary system. Despite oral loading with 10g of amio-
after the onset of pain presented to an outside hospital with darone over the next several days he continue to have symp-
lightheadness. His blood pressure was 90/60 and his heart rate tomatic runs of the same wide complex tachycardia.
was 190bpm. His ECG revealed a wide complex tachycardia He underwent an EP study which confirmed the diag-
(Fig.154.1). He was cardioverted and started on amiodarone. nosis of ventricular tachycardia (VT) and an attempted

Fig.154.1 ECG obtained in the emergency room showing a wide QRS tachycardia at 200bpm

S. Mahapatra
Department of Biomedical Engineering, University of Virginia,
1215 Lee Street, Charlottesville, VA 22908, USA
e-mail: srijoysm@gmail.com

A. Natale et al. (eds.), Cardiac Electrophysiology, 585


DOI: 10.1007/978-1-84996-390-9_154, Springer-Verlag London Limited 2011
586 S. Mahapatra

endocardial ablation at the other hospital. The ablation Table154.1 Clinical and ECG findings associated with epicardial VT
attempt was unsuccessful and the operator reported he did Clinical factors
not find any early endocardial points. He was then trans- 1. Inferior, as opposed to anterior, myocardial scar
ferred for further evaluation. 2. Non-ischemic cardiomyopathies
What do you think is the mechanism for the VT? Can you 3. Prior failed endocardial ablation
predict the site of origin? What should be your next step? ECG (Fig. 154.2) findings
1. VT QRS >198ms
2.A slurred QRS onset with a time from start of Q to peak of
R>0.55 of the total QRS duration using the precordial lead with
Case Discussion the lowest ratio
3.Intrinsicoid time >85ms (time from earliest Q or R deflection to
This patients ECG and the clinical history of a failed endo- nadir of S in V1V6)
cardial ablation attempt suggest that this VT has an epicar- 4.Pseduodelta >34ms (time from first V deflection to first rapid V
deflection in one lead)
dial site of origin or exit. In sinus rhythm and many VTs,
depolarization occurs from the endocardium to the epicar-
dium. If there is an epicardial VT focus (or exit), this sequence
is reversed. Since the focus is now further from the conduc- This patient underwent both an endocardial and epicar-
tion system, the QRS width is relatively wide and the first dial EP study. The latter was via a percutanous subxiphoid
part of the QRS often has a slower upstroke than the second puncture as described by Sosa and collegues. VT was easily
part. Although there are no large, prospective series, clinical inducible but, was not hemodynamically stable. Thus, a volt-
and ECG criteria proposed that have been associated with an age map was created. (Figs. 154.3a, b) It showed multiple
epicardial focus or exit are listed below (Table154.1). scars both endocardially and epicaridially but one area in the
The diagnosis of an epicardial VT can be confirmed at EP epicardium showed a narrow isthmus that could support VT.
study with mapping. In an endocardial VT, the EP study will Furthermore, pacing in this zone yielded a 12/12 pace map.
show an earliest spot (focal VT) or an area where early- Thus, a substrate based ablation was undertaken to produce a
meets-late (circuit). The lack of either suggests an epicar- line of block in this isthmus after a coronary angiogram
dial focus. With epicardial VT the endocardial activation showed that there were no major epicardial vessels in the
map will show a zone of earliest activation but there is often area. At the end of the study, no VT was inducible despite
no endocardial point activated before the onset of the surface isoproterenol and epinephrine infusions and programmed
QRS. Another clue is that the endocardial voltage at that stimulation. After 9months of follow-up he has not experi-
point may be normal, although this is rare. enced any recurrent episodes of VT.

Fig.154.2 ECG findings that


suggested an epicardial VT
Case 154 587

A B

Fig.154.3 Endocardial (Panel A) and epicardial (Panel B) voltage maps. The narrow isthmus that was the critical part of the epicardial circuit is
seen at about 3 oclock in panel B

Bibliography Marchlinski FE, Callans DJ, Gottlieb CD, etal. Linear Ablation lesions
for control of unmappable ventricular tachycardia in patients with
ischemic and nonischemic cardiomyopathy. Circulation. 2000;101:
Berruezo A, Mont L, Nava S, etal. Electrocardiographic recognition of 1268-1296.
the epicardial origin of ventricular tachycardias. Circulation. 2004; Sosa E, Scanavacca M, dAvilla A, Pillegi F. A new technique to per-
109:1842-1847. form epicardial mapping in the electrophysiology laboratory.
Daniels D, Lu YY, Morton JB, etal. Idiopathic epicardial left ventricu- JCardiovasc Electrophys. 1996;7:531-536.
lar tachycardia originiating remote from the sinus of valsalva.
Circulation. 2006;113:1659-1666.

Case 155

Pamela K. Mason

Case Summary conduction and the left bundle for retrograde conduction, the
QRS morphology in the tachycardia is similar, though often
not identical, to that in sinus rhythm.
A 65-year-old man presents to the emergency department
The patient described here demonstrates many of the
complaining of palpitations and dizziness. The patient was
usual electrophysiologic characteristics of patients with bun-
found to be in a wide-complex tachycardia (Fig.155.1) and
dle branch reentry. The QRS morphology of the tachycardia
hypotensive. He was electrically cardioverted with return to
is similar to that in the baseline ECG. A stable His electro-
normal sinus rhythm (Fig. 155.2). His past medical history
gram is present before each QRS complex during the tachy-
was significant for an aortic valve replacement with a single
cardia, and the HV interval in tachycardia is prolonged over
vessel right coronary bypass 8weeks prior to this presentation
baseline. In addition, with further study it was noted that
and chronic left bundle branch block. After cardioversion, an
changes in the RR interval were preceded by changes in the
echocardiogram showed a normally functioning aortic pros-
HH interval, and the tachycardia terminated with block in
thetic valve, moderate left ventricular dyssynchrony, and a
the HisPurkinje system. Ablation of the right bundle should
left ventricular ejection fraction of 40%. He was referred for
eliminate this form of VT since the right bundle forms the
electrophysiology study. Figure155.3 shows the intracardiac
anterograde limb of the circuit. Most patients, however, will
electrograms obtained during the tachycardia. What is the
have delayed anterograde conduction over the left bundle
mechanism of the tachycardia? How should it be treated?
rather than complete block, so the ECG if the right bundle is
ablated will show a right bundle branch block configuration
(Fig.155.4).
Ablation of the left bundle using a retrograde aortic
Case Discussion
approach is also possible and effective. Many patients with
bundle branch reentry will have other clinical or stimulation-
Reentry in the HisPurkinje system is a well known form of induced arrhythmias, so an ICD is often indicated in patients
ventricular tachycardia (VT) that accounts for 6% of cases with ventricular dysfunction even if they have a successful
of VT with a left bundle branch block morphology in patients catheter ablation of the right bundle. Late AV block and wors-
with ischemic heart disease but up to 40% of cases in patients ened heart failure are potential complications of a right bun-
with non-ischemic cardiomyopathies. It has also been found dle ablation for bundle branch reentry and these constitute
in patients without structural heart disease, with the univer- additional reasons to consider an ICD and resynchronization
sal commonality between all patients who develop bundle therapy. However, an ICD alone with ablation may not be an
branch reentry the presence of an abnormal conduction in optimal approach. Even if the patient is going to be scheduled
the HisPurkinje system. This is demonstrated by an intra- for an ICD, a presentation with VT with a left bundle branch
ventricular conduction delay on the baseline ECG, most block morphology, should make one consider an electrophys-
commonly an incomplete or complete left bundle branch iologic study since elimination of this arrhythmia by ablation
block, and a prolonged HV interval at electrophysiologic may decrease the frequency of shocks in the future. This is
stuidy. Because the mechanism of the tachycardia is a mac- particularly likely to be true in patient with valvular disease,
roreentrant circuit using the right bundle for anterograde nonischemic cardiomyopathies and ECG evidence of delayed
conduction in the left bundle at baseline.
This patient had no other inducible arrhythmias and did
not want either a pacemaker of ICD unless absolutely neces-
P.K. Mason
Department of Internal Medicine, University of Virginia, 800158,
sary. Even though his post ablation HV interval was 86 ms,
Charlottesville, VA 22908, USA he has not developed heart block or ventricular arrhythmias
e-mail: pkm5f@virginia.edu during follow-up.
A. Natale et al. (eds.), Cardiac Electrophysiology, 589
DOI: 10.1007/978-1-84996-390-9_155, Springer-Verlag London Limited 2011
590 P.K. Mason

Fig.155.1 12-Lead ECG of the patients presenting rhythm

Fig.155.2 12-Lead ECG after electrical cardioversion


Case 155 591

Fig.155.3 Intracardiac
electrograms during tachycardia.
At this point in time there is I
variable VA block. A His
potential can be seen before each
QRS complex II

AVF

V1

HRA

HBED

RVA

0 1000 2000

d: 100 mm/sec Time: 00: 04: 38 Study #1 Protocol #9 Protocol Name: RRP

Fig.155.4 Sinus rhythm with a right bundle branch block pattern QRS complex after ablation

Bibliography results and long-term follow-up. J Am Coll Cardiol. 1991;18:


1767-1773.
Eckart RE, Hruczkowski TW, Tedrow UB, Koplan BA, Epstein LM,
Becker R, Melkumov M, Senges-Becker JC, etal. Are electrophysio- Stevenson WG. Sustained ventricular tachycardia associated with
logic studies needed prior to defibrillator implantation? PACE. valve surgery. Circulation. 2007;116:2005-2011.
2003;26:1715-1721. Narasimhan C, Jazayeri MR, Sra J, etal. Ventricular tachycardia in val-
Caceres J, Jazayeri M, MacKinnie J, et al. Sustained bundle branch vular heart disease. Facilitation of sustained bundle-branch reentry
reentry as a mechanism of clinical tachycardia. Circulation. by valve surgery. Circulation. 1997;96:4307-4313.
1989;79:256-270.
Cohen TJ, Chien WW, Lurie KG, etal. Radiofrequency catheter abla-
tion for treatment of bundle branch reentrant ventricular tachycardia:

Case 156

Pamela K. Mason

Case Summary A recent echocardiogram showed normal left ventricular


function, normal chamber sizes, and no valvular abnormali-
A 71-year-old man was admitted to the cardiology service ties. During stress testing, he achieved ten METS, reached
after experiencing a syncopal spell while he was driving. He his target heart rate, and developed no angina or electrocar-
sustained only minor injuries after his motor vehicle acci- diographic abnormalities. Nuclear imaging was normal. He
dent. This was the fifth episode of frank syncope that he had had also worn an ambulatory ECG monitor several times.
experienced within the past 6months, several of which had These were all non-diagnostic but he had remained free of
resulted in injury and hospitalization. symptoms during the recordings.
His past medical history included mild hypertension and Physical examination performed at the time of this admis-
hyperlipidemia but he had no history of angina or prior myo- sion was normal as was his ECG. The monitor strip in
cardial infarction. There was no family history of fainting or Fig.156.1 was obtained during a bedside diagnostic maneu-
arrhythmias. Because of his history of multiple syncopal ver. What was this diagnostic maneuver? What treatment
spells, he had already had an extensive cardiac work-up. does this patient require?

Fig.156.1 Rhythm strip during


right carotid sinus massage in the
emergency room

P.K. Mason
Department of Internal Medicine, University of Virginia, 800158,
Charlottesville, VA 22908, USA
e-mail: pkm5f@virginia.edu

A. Natale et al. (eds.), Cardiac Electrophysiology, 593


DOI: 10.1007/978-1-84996-390-9_156, Springer-Verlag London Limited 2011
594 P.K. Mason

Case Discussion manifest an abnormal respone yet never develop symptoms


is unknown.
Diagnosing carotid sinus hypersensitivity over other
The tracings shows a 6.2 s period of asystole associated
forms of neurocardiogenic syncope is important in that it can
with right carotid sinus massage. Despite multiple admis-
alter management. Patients with recurrent, unexplained syn-
sions and evaluations for syncope prior to this episode,
cope with a cardioinhibitory response to carotid sinus mas-
carotid sinus massage had not previously been performed
sage have been shown to benefit from pacemaker placement.
in this patient. Inclusion of carotid sinus massage as a rou-
This is particularly true in elderly patients in whom no other
tine part of the evaluation of elderly patients with syncope
cause of syncope is suggested after their initial evaluation.
can make their evaluation much more efficient. In some
Benefits from pacing are less likely to be found in patients
series, up to 1/3 of elderly patient who present with syn-
with tilt-induced neurocardiac syncope. The latter syndome
cope or falls will have a positive response to carotid sinus
often has both marked cardioinhibitory and vasodepressor
massage.
components and the vasodepressr component often offsets
Carotid sinus hypersensitivity is a form of neurally medi-
any benefits from cardiac pacing. Indeed, carotid sinus
ated syncope. In this syndrome, the carotid sinus barorecep-
hypersensitivity is currently the only form of neurally-
tor is excessively responsive to application of extrinsic
mediated syncope that is a class I indication for pacemaker
pressure. This may be produced by tight clothing, adjacent
placement.
tumors or scarring or manual pressure but many patients will
This patient had undergone a thorough evaluation and no
not report any obvious precipitating factor. Performing
other cause for his recurrent syncope could be identified. He
carotid sinus massage requires ECG and blood pressure
received a dual chamber pacemaker during this hospitaliza-
monitoring. The carotid arteries should be checked for bruits,
tion. He has not had any recurrent syncope or presyncope
which are a contraindication, prior to applying pressure.
since his pacemaker was placed.
Pressure should then be applied to the fusiform portion of the
carotid sinus, the site of the greatest arterial pulsation, for
510s. A positive response is defined as a pause of greater
than 3 s or a systolic blood pressure drop of greater than
50 mmHg. If carotid message is negative on one side, the Bibliography
contralateral side can be checked after waiting for 30120s.
Sensitivity is enhanced if the maneuver is performed in both Hamill SC. Value and limitations of noninvasive assessment of syn-
cope. Cardiol Clin. 1997;15:196-218.
supine and upright positions. Kenny RAM, Richardson DA, Steen N, Bexton RS, Shaw FE, Bond J.
Syncope due to CSH is a disease of the elderly, virtually Carotid sinus syndrome: a modifiable risk factor for nonaccidental falls
never being seen in patients less than 50 years of age. in older adults (SAFE PACE). J Am Coll Cardiol. 2001;38:1491-1496.
However, it must be remembered that a hypersensitive Kerr SR, Pearce MS, Braye C, Davis RJ, Kenny RA. Carotid sinus
hypersensitivity in older persons: implicationsfor diagnosis of syn-
carotid sinus massage response may be only a nonspecific cope and falls. Archiv Intern Med. 2006;166:515-520.
finding since it can be observed in 1030% of asymp Maggi R, Menozzi C, Brignole M, etal. Cardioinhibitory carotid sinus
tomatic, elderly individuals. The postulated mechanism is hypersensitivity predicts an asystolic mechanism of spontaneous
an up-regulation of the alpha-2 adrenoreceptors in the neurally mediated syncope. Europace. 2007;9:563-567.
Mallet M. Carotid sinus syndrome. Hosp Med. 2003;64:92-95.
baroreceptor pathway such that stimulation results in an Shen WK, Decker WW, Smars PA, et al. Syncope Evaluation in the
overshoot baroreflex efferent response that results in bra- Emergency Department Study (SEEDS): a multidisciplinary approach
dycardia and hypotension. Why some patients will to syncope management. Circulation. 2004;110:3636-3645.
Case 157

Pamela K. Mason

Case Summary history of chest pain or shortness of breath. She did report one
previous episode of palpitations associated with near syncope.
Her family history is notable for a cousin with a heart rhythm
A 21-year-old woman is referred to cardiology clinic for eval-
problem. What further testing does this patient require?
uation. She was a passenger in a motor vehicle accident, and
when she was evaluated in the emergency department, she
was incidentally noted to have a loud heart murmur on exam
and mild cardiomegaly on a chest radiograph. An ECG was Case Discussion
obtained during her clinic visit and is shown in Fig. 157.1.
Physical exam was notable for splitting of the first and second This patient demonstrates WolffParkinsonWhite syndrome
heart sounds and a holosystolic murmur at the left sternal bor- (WPW) on ECG and has a murmur consistent with tricuspid
der that increased with inspiration. The patient denied any regurgitation. With this combination of findings, Ebsteins

Fig.157.1 Baseline 12-lead ECG obtained in clinic

P.K. Mason
Department of Internal Medicine, University of Virginia, 800158,
Charlottesville, VA 22908, USA
e-mail: pkm5f@virginia.edu

A. Natale et al. (eds.), Cardiac Electrophysiology, 595


DOI: 10.1007/978-1-84996-390-9_157, Springer-Verlag London Limited 2011
596 P.K. Mason

anomaly is a likely diagnosis. In Ebsteins anomaly, the pos- of symptomatic palpitations. Most patients with Ebsteins
terior and septal tricuspid leaflets and their chordal attach- anomaly and WPW have right-sided or posteroseptal APs
ments are abnormal and displaced downward into the right and multiple different APs are often found. Associated ana-
ventricle. The proximal portion of the right ventricle is atri- tomic abnormalities make it difficult to use standard algo-
alized. Twenty to thirty-three percent of patients with rithms to locate the AP based upon the surface ECG. In
Ebsteins anomaly have accessory pathways (APs). Patients addition, while WPW is the most common supraventricular
with Ebsteins anomaly may present at any age. Those with arrhythmia found in Ebstein patients, other supraventricular
marked hemodynamic abnormalities usually present as arrhythmias are also more frequently seen than in the gen-
infants or young children, but, if the valve displacement is eral population. Catheter ablation of the right-sided APs
not severe, they may first present with arrhythmias. may be challenging due to the antatomic abnormalities.
This patient needs to have an echocardiogram to evaluate Surgical ablation can be considered in patients who require
her tricuspid valve. There is a great deal of variability in the valve surgery.
degree of tricuspid valve incompetence among patients with
this condition, and other abnormalities, such as atrial septal
defects, are common. Some patients require only medical
management, while others will need surgical treatment. Bibliography
While this patient has no symptoms of heart failure, the car-
diomegaly present on her chest radiograph is concerning. HareVan GF. Radiofrequency ablation of accessory pathways associ-
She will also require electrophysiologic evaluation. She ated with congenital heart disease. Pacing Clin Electrophysiol.
has evidence of preexcitation on her ECG and a history 1997;20:2077-2081.
Case 158

Pamela K. Mason

Case Summary syncopal spell. Interrogation of her device showed several


brief episodes of rapid ventricular rate. She was admitted for
A 33-year-old woman presented to her doctor complaining further monitoring. A repeat echocardiogram now showed
of fatigue and exercise intolerance. Her baseline ECG is moderate left ventricular dysfunction. She also related that
shown in Fig.158.1. She underwent ambulatory ECG moni- two members of her family had developed cardiac problems.
toring and was found to have occasional episodes of second Her 40-year-old brother had recently died suddenly and
and third degree heart block scattered throughout the day. An without explanation. An autopsy revealed only cardiomeg-
echocardiogram was normal. She was referred for a dual aly. Her 31-year-old sister was undergoing an evaluation in
chamber pacemaker. This was implanted without complica- another city because of palpitations and frequents premature
tions and her symptoms improved. However, 3 years later ventricular beats. The patient was referred for upgrade of her
she returned to the emergency department after a witnessed device to a dual chamber defibrillator. Several months after

Fig.158.1 Baseline 12-lead ECG

P.K. Mason
Department of Internal Medicine, University of Virginia, 800158,
Charlottesville, VA 22908, USA
e-mail: pkm5f@virginia.edu

A. Natale et al. (eds.), Cardiac Electrophysiology, 597


DOI: 10.1007/978-1-84996-390-9_158, Springer-Verlag London Limited 2011
598 P.K. Mason

Fig.158.2 ICD Interrogation after the patient received a shock

this, she again returned after receiving a shock from her defi- arrhythmias. Her brother died suddenly at age 40 with no
brillator for ventricular tachycardia (Fig.158.2). Subsequently, previous cardiac symptoms or evaluation. Her younger sister
the patient has continued to have worsening left ventricular has ventricular ectopy but her ejection fraction is still nor-
function, increasing heart failure symptoms, and has received mal. Although the onset of symptomatic disease may be
several additional shocks from her defibrillator for ventricu- delayed in patients with lamin A/C defects, disease progres-
lar tachycardia despite antiarrhythmic therapy. A second sion can be rapid. Patients with lamin A/C deficiency have a
device upgrade with addition of a left ventricular lead was worse prognosis compared to patients with non-inherited
performed with some improvement in her symptoms. What forms of cardiomyopathy. These patients are more likely to
genetic condition would be a likely cause of the syndrome experience sudden cardiac death, severe congestive heart
observed in this patient? failure, and require heart transplantaion. Recognizing this
condition is important for prognostic reasons, as well as for
family counseling.
Case Discussion The mechanism by which lamin A/C mutation causes
DCM and conduction system disease is unclear. Lamin
Approximately one-third of cases of dilated cardiomyopathy A and lamin C are both transcribed from the LMNA gene,
(DCM) are familial. Different inheritance patterns have been and both are usually affected by a gene mutation. They are
found, including X-linked, mitochondrial, and autosomal filamentous proteins found in the nuclear envelope, and mul-
recessive, but the majority of congenital cases are inherited in tiple different mutations have been described. Four different
an autosomal dominant manner. The autosomal dominant diseases have been associated with lamin A/C mutations: (1)
cases can be separated into those that cause conduction sys- DCM with conduction system disease; (2) limb girdle mus-
tem disease as well as DCM, and those that only cause DCM. cular dystrophy; (3) autosomal dominant variant of Emery
There is genetic heterogeneity among these groups. In patients Dreifuss muscular dystrophy; (4)autosomal dominant partial
with pure autosomal dominant DCM, a number of genes have lipodystrophy. While these are generally considered to be
been implicated, including actin, desmin, b-sarcoglycan, d- separate entities, elevations in creatinine phosphokinase have
sarcoglycan, troponin T, a-tropomyosin, and b-myosin heavy been found in patients with DCM suggestive of skeletal mus-
chain. Four genetic loci have been implicated in cases of cle involvement, and cardiac rhythm abnormalities have been
DCM associated with conduction system disease, but the found in patients with muscular dystrophy demonstrating
only identified gene, thus far, is lamin A/C, which accounts that these are likely a spectrum of illnesses.
for approximately one-third of cases of DCM with conduc-
tion system disease.
This patient has a history that is strongly suggestive of
lamin A/C mutation. Patients with lamin A/C related cardio- Bibliography
myopathies usually present with arrhythmias as young adults
and then later develop ventricular dysfunction. She fits this Arbustini E, Pilotto A, Repetto A, etal. Autosomal dominant dilated
pattern in that she presented initially with high-grade con- cardiomyopathy with atrioventricular block: a lamin A/C defect-
related disease. J Am Coll Cardiol. 2002;39:981-990.
duction system disease requiring pacing at a point where her Grunig E, Tasman JA, Kucherer H, Franz W, Kubler W, Katusw HA.
ventricular function was normal. It was several years later Frequency and phenotypes of familial dilated cardiomyopathy.
that she progressed to a severe DCM, with ventricular JAm Coll Cardiol. 1998;31:186-194.
Case 159

Lisa G. Umphrey and John Paul Mounsey

Case Summary Case Discussion

A 50-year-old female with a past medical history of severe Myotonic muscular dystrophy also known as Steinerts dis-
type 1 myotonic muscular dystrophy (MD1), was referred ease is the most common muscular dystrophy in Caucasian
from neuromuscular clinic for evaluation. She was diagnosed adults occurring with a prevalence of 35 per 100,000 adults.
with this condition13years previously and had experienced It is inherited in an autosomal dominant disorder localized
progression of her neuromuscular weakness. Most notably she to the 3-untranslated region of the dystrophica myotonica
stated that she had bilateral hand weakness, loss of dexterity, protein kinase (DMPK) gene on chromosome 19q13.3. The
difficulty in communication with slurring of speech and lower genetic abnormality is a CTG repeat sequence, the length of
extremity weakness when standing. She denied syncope or which has been previously shown to correlate with the extent
pre-syncope, palpitations, chest pain exertional dyspnea, lower and severity of disease.1,2 Individuals without disease have
extremity edema, orthopnea or paroxysmal nocturnal dyspnea. 530 repeats where individuals with DM have 502,000 CTG
She had no family history of sudden cardiac death or prema- repeats.3 The disease exhibits anticipation. This means that in
ture coronary artery disease. She had a child who also suffered successive generations the size of the repeat increases, result-
from myotonic muscular dystrophy who passed away at the ing in earlier onset and severity of clinical manifestations as
age of 12 due to the severe juvenile form of the disease. was seen in this case. Noncardiac manifestations of the dis-
An EKG performed demonstrated first degree AV conduc- ease include myotonia affecting the face and distal limbs,
tion delay with a PR interval measuring 208ms and a nonspe- progressive muscle wasting, gonadal atrophy, frontal bald-
cific intraventricular conduction delay with a QRS duration of ing, cataracts, endocrinopathies and cognitive impairment.
124ms. T wave inversions were present in leads III, III, aVF, Cardiac manifestations include sinus node dysfunction, con-
and V2 through V6 (see Fig.159.1). Adenosine SESTAMIBI duction defects, atrial and ventricular tachyarrhythmias, and
perfusion imaging showed no evidence of ischemia or prior cardiomyopathy. Patients with larger expansions of CTG
infarction. A transthoracic echocardiogram demonstrated nor- repeats have been shown to be at increased risk of intraven-
mal LV size and systolic function. Holter monitoring demon- tricular conduction delay at baseline and rapid progression of
strated previously identified first degree AV conduction delay conduction defects.4
and nonspecific intraventricular conduction delay. There were Conduction abnormalities are common in patients with
>4000 ventricular ectopic beats with some short bursts of non- DM. First degree AV conduction delay is the most common
sustained ventricular tachycardia (see Fig.159.2a). There was ECG abnormality found in 2040% of patients. Left anterior
one episode of 2:1 sinus exit block (see Fig.159.2b). Within fascicular block as presented here, right and left bundle
1years time frame she had worsening neuromuscular weak- branch block manifesting in a widened QRS complex occur
ness and evidence of progression of her sinoatrial node dys- in 525% of patients. Prolongation of the HV interval has
function with a resting pulse of 44bpm that had dropped from been demonstrated in half of patients. The progression of
56 on previous evaluation, and left bundle branch block. What conduction system disease is typically slow but can be unpre-
treatment would you recommend? dictable. Thus in this patient population it is recommended
to repeat the ECG at least yearly as the progression of con-
duction defects can progress faster than the neuromuscular
manifestations.5 This patient received a pacemaker due to
L.G. Umphrey and J.P. Mounsey (*)
Division of Cardiology, University of North Carolina,
bradycardia and signs of progressive conduction delay.
160 Dental Circle, CB 7075, Chapel Hill, NC 27599, USA Supraventricular tachyarrhythmias consisting of atrial
e-mail: lumphrey@unch.unc.edu; pmounsey@med.unc.edu fibrillation and flutter are seen in 25% of patients with DM1

A. Natale et al. (eds.), Cardiac Electrophysiology, 599


DOI: 10.1007/978-1-84996-390-9_159, Springer-Verlag London Limited 2011
600 L.G. Umphrey and J.P. Mounsey

Fig159.1 Baseline 12 Lead ECG demonstrating first-degree and nonspecific intraventricular conduction delay

Fig159.2 (a) Holter monitor strip demonstrating a short burst of non-sustained ventricular tachycardia. (b) Holter monitor strip demonstrating an
episode of 2:1 sinus exit block
Case 159 601

and have been shown to be inducible on EPS study in the consistent with a bundle branch reentrant tachycardia abla-
absence of a previously documented arrhythmia.6 Stroke pre- tion of one bundle branch is curative and implantation of an
vention with systemic anticoagulation can be a challenging ICD is not warranted. Symptoms of cardiomyopathy in
proposition in this patient population that is predisposed to DM patients can be masked by limited exertion. A low
falls due to neuromuscular weakness and must be evaluated threshold for echocardiographic study should be main-
on a case by case basis. tained in this patient population.
Ventricular arrhythmias have also been documented in In conclusion, we present a case of myotonic muscular
this patient population. Both sustained and non-sustained dystrophy with a classic presentation of sinus node dysfunc-
VT, as seen here, have been reported. On EPS study of tion and conduction system defects. This patient received
patients referred with conduction abnormalities 18% had a dual chamber pacemaker and did not require an internal
inducible VT.7 Monomorphic and polymorphic VT has been cardiac defibrillator due to the absence of syncope, near
reported. Bundle branch reentry ventricular tachycardia has syncope or sustained VT.
been shown to be one of the mechanisms for sustained mon-
omorphic ventricular tachycardia.8 The conduction defects
present in this disease provide the necessary substrate for
References
reentry to occur. A premature ventricular impulse can find
one bundle branch refractory but travel in a retrograde fash-
1. Jaspert A, Fahsold R, Grehl H, Claus D. Myotonic dystrophy: cor-
ion up the other bundle to the His and then travel antegrade relation of clinical symptoms with the size of the CTG trinucleotide
over the recovered branch to the ventricles creating a macro- repeat. J Neurol. 1995;242:99-104.
reentry circuit. Identification of this reentry circuit is essen- 2. Groh WJ, Lowe MR, Zipes DP. Severity of cardiac conduction
tial as it can be cured with radiofrequency ablation in one of involvement and arrhythmias in myotonic dystrophy type 1 corre-
lates with age and CTG repeat length. J Cardiovasc Electrophysiol.
the limbs, usually the right bundle of the circuit negating the 2002;13:444-448.
need for implantable cardiac defibrillator placement. 3. Sovari AA, Bodine CK, Farokhi F. Cardiovascular manifestations
Clinical symptoms of cardiomyopathy, systolic and of myotonic dystrophy. Cardiol Rev. 2007;15:191-193.
diastolic occur in approximately 2% of patients. It can be 4. Clarke NR, Kelion AD, Nixon J, Hilton-Rase D, Forfar JC. Does
cytosine-thymine-guanine (CTG) expansion size predict cardiac
difficult to detect as the neuromuscular disease often limits events and electrocardiographic progression in myotonic dystro-
exertion. Cardiac imaging in 96 patients with the disease phy? Heart. 2001;86:411-416.
demonstrated left ventricular hypertrophy in 19.8%, left 5. Mammarella A, Paradiso M, Basili S, DeMatteis A, Cardarello LM,
ventricular dilation 18.6% and systolic dysfunction in etal. Natural history of cardiac involvement in myotonic dystrophy
(Steinerts disease): a 13 year follow-up study. Adv Therapy.
14%.9 Echocardiographic studies have also shown diastolic 2000;17:238-251.
dysfunction.10 6. Phillips MF, Harper PS. Cardiac disease in myotonic dystrophy.
Recommendations for evaluation and monitoring of DM Cardiovasc Res. 1997;33:13-22.
1 patients have been previously published.3 A baseline 7. Lazarus A, Varin J, Ounnoughene Z, Radvanyi H, Junien C, etal.
Relationships among electrophysiological findings and clinical
ECG with repeat monitoring every 612months is recom- status, heart function, and extent of DNA mutation in myotonic
mended. Patients with a first-degree conduction delay dystrophy. Circulation. 1999;99:1041-1046.
should be closely monitored for progression of conduction 8. Merino JL, Carmona JR, Fernandez-Lozano I, Peinado R, Basterra N,
abnormalities. Patients with additional ECG abnormalities Sobrino JA. Mechanisms of sustained ventricular tachycardia in
myotonic dystrophy. Circulation. 1998;98:541-546.
or symptoms should undergo EPS. It has been previously 9. Bhakta D, Lowe MR, Groh WJ. Prevalence of structural cardiac
recommended that patients with an HV interval of >70ms abnormalities in patients with myotonic dystrophy type 1. Am Heart J.
should undergo pacemaker placement.11 Individuals with 2004;147:224-227.
symptoms suggesting arrhythmia such as syncope, near 10. Fragola PV, Luzi M, Calo L, Antonini G, Borzi M, etal. Doppler
echocardioghraphic assessment of left ventricular diastolic dys-
syncope or increasing dyspnea without ECG changes function in myotonic dystrophy. Cardiology. 1997;88:498-502.
should undergo further evaluation with Holter montoring, 11. Lazarus A, Varin J, Duboc D. Final results of the French diagnostic
SAECG or EPS. If monomorphic VT is reproduced and is pacemaker study in myotonic dystrophy. PACE. 2002;25:599.

Case 160

James A. Reiffel

Case Summary intravenous ibutilide. Figure 160.1 shows the ECG 30 s


before conversion; note the prolonged QT interval that devel-
oped with ibutilide. The following day, recurrent episodes of
A 57-year-old caucasian male has a history of tobacco abuse
paroxysmal AF developed, lasting 2090min. Blood pres-
(stopped 1 year ago), hyperlipidemia (under good control
sure was stable and the pericardial rub was softer. What
with atorvastatin and omega-3 fish oil), hypertension (man-
change(s) in therapy should be initiated, if any?
aged with trandolapril and indepamide), and angina (treated
with carvedilol, isosorbide, and aspirin). His anginal history
dates to the onset of exertional angina walking up hill to
work approximately 8months ago. A stress echocardiogram
Case Discussion
study suggested single vessel right coronary artery (RCA)
disease and his symptoms were managed medically very Post operative AF is very common, occurring in more than
successfully for 7 months with carvedilol. Over the past 30% of untreated patients and multiple agents have been used
3weeks his exertional angina increased despite the addition to suppress AF in the post-op setting.1-6 Beta blockers are very
of isosorbide. The patient was admitted to the hospital fol- useful in reducing the incidence of AF following cardiac sur-
lowing one episode of rest pain that awakened him from gery and are the standard of care in many institutions. When
sleep, associated with new T wave inversions anteriorly they cannot be used due to hemodynamic compromise, amio-
without any troponin or CK enzyme rise. Coronary angiog- darone is commonly given. In some centers amiodarone is the
raphy was performed and revealed a long left anterior first line prophylactic agent chosen, but it is fraught with so
descending (LAD) stenosis with an irregular filling defect many drug interactions, and, when given IV, with hypotension,
plus an 85% RCA stenosis. Both stenotic regions were heav- that it is usually better considered as a second-line alternative.
ily calcified. The patient underwent coronary artery bypass Also useful is changing the standard beta blocker to sotalol,
surgery with a left internal mammary artery graft to the LAD which, in numerous comparative trials in the post-op setting,
and a saphenous vein graft to the RCA. has been more effective. Careful attention to electrolyte status,
Six hours postoperatively, while still intubated, off pres- renal function, and QT interval response is needed when sotalol
sors, frequent atrial ectopy began to appear. Over the next is used. Dofetilide is a much less well studied alternative in this
2 h, the patient awoke fully and was extubated. Carvedilol circumstance. Propafenone has also been used effectively
and aspirin were reinitiated. Four hours later the patient when the LV is normal and no residual ischemic risk exists.
developed atrial fibrillation (AF). His beta blocker dose was Importantly, the incidence of AF after cardiac surgery has
adjusted to optimize rate control. He had no complaint except been reported to be increased in patients who were taking
palpitations. Blood pressure remained normal. Arterial blood beta blockers and/or ACE-inhibitors prior to surgery when
gases and serum chemistries were unremarkable. A new these agents were not restarted post operatively. In addition,
prominent pericardial friction rub was present on examina- AF after cardiac surgery has also been reduced by the admin-
tion. There was no history of AF at any time previously in his istration of statin therapy and by omega-3 fish oil.
history. When the AF persisted 2 h, it was converted with In this patient, trandolapril, atorvastatin, and omega-3 fish
oil were restarted when PAF recurred on post-op day number
2, and his carvedilol was changed to sotalol, 120mg bid on
day 3 after two more episodes. Figure160.2 shows the time
J.A. Reiffel course of his post-op AF and the therapies used. Figure160.3
Department of Medicine, Division of Cardiology,
Columbia University, 161 Fort Washington Ave,
reviews this patients medications and others in respect to
New York, NY 10032, USA suppressing post-op AF. On this regimen, no further AF
e-mail: jar2@columbia.edu occurred. The patient was discharged home 5 days post

A. Natale et al. (eds.), Cardiac Electrophysiology, 603


DOI: 10.1007/978-1-84996-390-9_160, Springer-Verlag London Limited 2011
604 J.A. Reiffel

Fig.160.1 ECG demonstrating AF, as well as QT prolongation that developed on ibutilide

Fig. 160.2 The time course of this patients AF post-op and the
therapies employed

Fig. 160.3 A review of this patients medications with respect to


operatively, His pericardial rub was gone. Since he had a suppression of postoperative AF as well as other agents that could have
CHADS2 score of only 1, warfarin was not instituted. At out- been utilized
patient visits 3 and 6weeks later, the patient reported no pal-
pitations and his vital signs were normal. ECGs at these References
visits showed sinus rhythm. Sotalol was then tapered off.
Now, 23months later, no AF has recurred and the patient is 1. Echahidi N, Pibarot P, OHara G, Mathieu P. Mechanisms, preven-
doing well, although indepamide was restarted for recurrent tion, and treatment of atrial fibrillation after cardiac surgery. J Am
mild hypertension. The absence of recurrent AF late after Coll Cardiol. 2008;51(8):793-801.
2. Tselentakis, Woodford E, Chandy J, Gaudette GR, Saltman AE.
cardiac surgery (more than 23months post-op) is the rule
Inflammation effects on the electrical properties of atrial tissue and
rather than the exception when the AF is only a perioperative inducibility of post-operative atrial fibrillation. J Surg Res. 2006;
phenomenon and was not part of the preoperative history. 135:68-75.
Case 160 605

3. Fuster V, Ryden LE, Cannom DS, et al. ACC/AHA/ESC 2006 5. Patti G, Chello M, Candura D, etal. Randomized trial of atorvasta-
guidelines for the management of patients with atrial fibrillation tin for reduction of post-operative atrial fibrillation in patients
executive summary: a report of the American College of Cardiology/ undergoing cardiac surgery: results of the ARMYDA-3 (Atorvastatin
American Heart Association Task Force on Practice Guidelines and for Reduction of MYocardial Dysrhythmia After cardiac surgery)
the European Society of Cardiology Committee for Practice study. Circulation. 2006;114:1455-1461.
Guidelines (writing committee to revise the 2001 Guidelines for the 6. Calo L, Bianconi L, Colivicchi F, et al. N-3 Fatty acids for the
Management of Patients with Atrial Fibrillation). J Am Coll Cardiol. prevention of atrial fibrillation after coronary artery bypass sur-
2006;48:854-906. gery: a randomized, controlled trial. J Am Coll Cardiol. 2005;45:
4. Burgess DC, Kilborn MJ, Keech AC. Interventions for prevention 1723-1728.
of post-operative atrial fibrillation and its complications after car-
diac surgery: a meta-analysis. Eur Heart J. 2006;27:2846-2857.

Case 161

James A. Reiffel

Case Summary MAZE surgery was not performed at that time at his hos-
pital. His angina and congestive symptoms resolve, but his
A 60-year-old Caucasian male with a background of PAF continues. Between 1997 and 2003 he suffers symp-
hypertension, rheumatic mitral insufficiency, and occasional tomatic AF 620 times/year despite trials of metoprolol,
paroxysmal atrial fibrillation (PAF) develops angina and con- verapamil, and sotalol, given by his cardiologist. In addition,
gestive symptoms in 1996 and undergoes triple vessel coro- a new RBBB is noted on his ECG. Figure161.1 shows his
nary artery bypass grafting and mitral valve replacement. ECG with sinus rhythm in 2001 prior to RBBB; Fig.161.2

Fig.161.1 ECG during sinus rhythm in 2001

J.A. Reiffel
Department of Medicine, Division of Cardiology,
Columbia University, 161 Fort Washington Ave,
New York, NY 10032, USA
e-mail: jar2@columbia.edu

A. Natale et al. (eds.), Cardiac Electrophysiology, 607


DOI: 10.1007/978-1-84996-390-9_161, Springer-Verlag London Limited 2011
608 J.A. Reiffel

Fig.161.2 ECG during AF in 2003

shows his ECG in 2003 in AF with RBBB. In June 2003 hypertension. Postoperative AF in this patient was not just a
amiodarone (Pacerone) 200 mg/day was begun in addition to perioperative phenomenon but rather, it was part of a longer
the beta blocker being used for his hypertension and over the term AF story. Little has been written about antiarrhythmic
next 5 months he has 15 more episodes all with good rate drug combinations for the suppression of AF (in contrast to
control and less awareness of palpitations. An amiodarone frequent drug combinations for ventricular rate control and
serum level was 0.6. In November 2003, his amiodarone is occasional antiarrhythmic drug combinations for refractory
changed to another amiodarone product, Cordarone, 200 ventricular tachycardia or fibrillation).1-7 However, in this
mg/day. The serum concentration rises to 1.0. The dose is patient, at this point in time, having refused non-pharma-
increased to 300 mg/day. Between 11/03 and 3/04, only two cological treatment, it was felt prudent to try to combine
episodes are detected; however, from 3/04 to 11/06, 31 addi- another agent with his amiodarone. Amiodarone has signif-
tional episodes of PAF are noted by the patient each of icant class III antiarrhythmic drug actions; it also has class
which leaves him feeling a little fatigued. He seeks consulta- IV actions and weak sodium channel blocking and sym-
tion with an electrophysiologist and is offered ablation, but patholytic actions. Increasing the effects of these less potent
he refuses as a neighbor of his had a stroke during an abla- actions may improve antiarrhythmic efficacy. Beta blockers
tive procedure. Purified omega-3 fish oil is added following have been known to improve efficacy with amiodarone for
reports that it may reduce AF. years. Adding sodium channel blockade to increase refrac-
Between 11/06 and 3/07 the patient suffers 12 more epi- toriness by an additional mechanism could serve to further
sodes and revisits the electrophysiologist. He is again offered impair macro- or micro-reentrant mechanisms, and sodium
ablation, and again refuses. What else might be tried? channel blockade could also reduce automatic triggers.
Over 80% of the patients PAF episodes were nocturnal
and disopyramide was added. Despite no prior history of
symptoms of prostatism, disopyramide had to be discon-
Case Discussion tinued in 2 weeks because of urinary obstruction. It was
replaced with sustained release propafenone (Rythmol SR)
This patient appears to have recurrent, resistant PAF on 225 mg/bid. Over the next 2 months, only one short episode
a background of structural heart disease, in the form of occurred and there were no adverse additional conduction
prior rheumatic mitral insufficiency, prior angina, and changes on the ECG. However, the patient complained of
Case 161 609

vivid dreams. The propafenone was changed to flecainide carvedilol, omega-3 fish oil) the patient had one brief epi-
(Tambocor) 100 mg bid. Figure161.3 shows his ECG on this sode in 8/07 and none from 9/07 to 4/08 at the time of this
combination. The PR interval and RBBB have lengthened report. Figure 161.4 summarizes his events and therapies.
minimally. On this combination (amiodarone, flecainide, This case demonstrates that in resistant AF, antiarrhythmic

Fig.161.3 ECG during sinus rhythm on amiodarone, flecainide, beta blocker, and fish oil

Fig.161.4 Plot of average event


rates per 3 months along with
summary of therapy
610 J.A. Reiffel

drug combination therapy may be effective. However, when 3. Korantzopoulos P, Kolettis TM, Papathanasiou A, etal. Propafenone
adding any agent to amiodarone, because of its proclivity added to ibutilide increases conversion rates of persistent atrial
fibrillation. Heart. 2006;92:631-634.
for drug interactions with all antiarrhythmic drugs studied 4. Wadhani N, Singh BN. Prolongation of repolarization as antifibril-
to date, the maximal dose should be limited to no more than latory action revisited: drug combination therapy in atrial fibrilla-
half of the usual maximal dose. tion. J Cardiovasc Pharmacol Ther. 2005;10:149-152.
5. Fujiki A, Tsuneda T, Sakabe M, etal. Maintenance of sinus rhythm
and recovery of atrial mechanical function after cardioversion with
bepridil or in combination with aprindine in long-lasting persistent
References atrial fibrillation. Circulation. 2004;68:834-839.
6. Auer J, Weber T, Berent R, etal. Study of prevention of postopera-
tive atrial fibrillation. A comparison between oral antiarrhythmic
1. Ross DL, Sze DY, Keefe DL, etal. Antiarrhythmic drug combina- drugs in the prevention of atrial fibrillation after cardiac surgery:
tions in the treatment of ventricular tachycardia. Circulation. the pilot study of prevention of postoperative atrial fibrillation
1982;66:1205-1210. (SPPAF), a randomized, placebo-controlled trial. Am Heart J. 2004;
2. Kanna B, Gopalam M. Prospective randomized study comparing 147:636-643.
amiodarone vs. amiodarone plus losartan vs. amiodarone plus per- 7. Nagai S, Takeishi Y, Kubota I. Low-dose flecainide infusion fol-
indopril for the prevention of atrial fibrillation recurrence in patients lowed by oral pilsicainide is highly effective and safe for paroxys-
with lone paroxysmal atrial fibrillation. Eur Heart J. 2007;28:381. mal atrial fibrillation. Cardiovasc Drugs Ther. 2003;17:95-97.
Case 162

Jens Seiler and William G. Stevenson

1.Diagnosis: Arrhythmogenic map of the right ventricle was also obtained (Fig. 162.1.3).
Right Ventricular Dysplasia Based on these findings what is the most likely diagnosis?
The findings suggest scar-related right ventricular tachy-
A 47-year-old female presented with exercise wide QRS cardia. Arrhythmogenic right ventricular dysplasia/cardio-
tachycardia at the age of 32 years. Coronary angiography myopathy (ARVC), cardiac sarcoidosis, and idiopathic
and echocardiography were reportedly normal. An ICD was cardiomyopathy with predominant right ventricular involve-
implanted. Ventricular tachycardia (VT) has become recur- ment are the major diagnostic considerations.
rent despite therapy with sotalol. The 12-lead ECG is shown Figure 162.1.1 shows the electrocardiogram during sinus
in Fig. 162.1.1. At electrophysiology study, four morpholo- rhythm. Several findings that are consistent with ARVC are
gies of sustained VT were induced (Fig. 162.1.2). A voltage present. Incomplete right bundle branch block (RBBB),

200 ms

II

III

aVR

aVL

aVF

V1

V2

V3
Fig. 162.1.1 Sinus rhythm
ECG is shown. Sinus bradycar-
dia of 54 bpm, incomplete right V4
bundle branch block, localized
QRS prolongation of 110 ms
and epsilon waves (arrows) in V5
V1 and V2, delayed S-wave
upstroke in V2, and T-wave V6
inversion in V1V3

J. Seiler and W.G. Stevenson(*)


The Arrhythmia Service, Cardiovascular Division,
Brigham and Womens Hospital, Boston, MA
e-mail: wstevenson@partners.org

A. Natale et al. (eds.), Cardiac Electrophysiology, 611


DOI: 10.1007/978-1-84996-390-9_162, Springer-Verlag London Limited 2011
612 J. Seiler and W.G. Stevenson

Fig. 162.1.2 Surface ECG of a b c d


four different sustained 200 ms
monomorphic VTs. All VTs have
a left bundle branch block I
configuration, suggesting the
origin in the right ventricle: (a) II
VT #1 cycle length (CL) of
III
290ms, inferior QRS axis; (b)
VT #2, CL 230 ms, inferior QRS aVR
axis; (c) VT #3, CL 300 ms,
inferior QRS axis; (d) VT #4, CL aVL
230 ms, superior QRS axis. VTs
#13 with an inferior QRS axis aVF
have their origin in the right V1
ventricular outflow tract; VT #4
has its origin in the base of the V2
right ventricle
V3

V4

V5

V6

10.42 mV
Bipolar

1-SR endo > 102 Points

1.51 mV
0.49 mV
0.09 mV

Fig. 162.1.3 Electroanatomic


endocardial voltage map of the R
right ventricle in sinus rhythm.
Purple denotes normal amplitude
bipolar electrograms (> 1.5 mV) 1.48 cm
with progressive decrease in
amplitude with progression from
blue to green to yellow to red.
Itis important to exclude poor
contact as a cause of low-voltage
regions. Gray indicates
unexcitable scarred tissue that
has a pacing threshold > 10 mA.
Blue dots indicate sites with
double, fractionated potentials
Case 162 613

localized widening of the QRS complex of 110 ms and epsi- repolarization abnormalities in V1V3 in absence of RBBB,
lon waves (fractionated terminal QRS deflections) in V1 and late potentials, sustained LBBB type VT, frequent PVCs,
V2, delayed S-wave upstroke in V2 and T-wave inversions in and family history of arrhythmias without confirmation of
V1V3 are present. Inverted T-waves in leads V1V3 are ARVC. This patient has LBBB VT and a sinus rhythm ECG
seen in more than 80% of patients with ARVC. Localized consistent with the diagnosis. It is well recognized, however,
widening of the QRS duration to >110 ms in V1V3 is seen that these criteria are imperfect and are not met by some
in >60% of patients. A prolonged S-wave upstroke of >55 ms patients with the disease, while other causes of RV cardio-
in V1V3 is seen in >90% of patients, and epsilon waves are myopathy, notably sarcoidosis, can cause similar findings.
visible in approximately a third of patients. This patient did not have a family history of the disease or
Figure 162.1.2 shows four different monomorphic VTs. of sudden death, but this absence does not exclude the diag-
All of these VTs had a dominant S-wave in V1, giving V1 a nosis, as spontaneous mutations occur. Magnetic resonance
left bundle branch block (LBBB)-like configuration. This (MR) imaging may show RV wall motion abnormalities and
indicates that the exit for the VT is likely to be in the right intramyocardial fat, which by itself is not sufficiently spe-
ventricle (RV) or the septum. VT #1 had an inferior axis rais- cific for the disease. Areas of delayed gadolinium enhance-
ing the possibility of an RV outflow tract location that is ment, consistent with scar are more specific, particularly if
common for idiopathic VT. The transition from dominant present in the RV. MR imaging was not done in this patient
S- to R-wave in the precordial leads occurs relatively late, due to presence of an ICD.
however, at V3V4. In addition, the QRS complexes are Cardiac sarcoidosis can mimic the clinical features of
somewhat slurred, particularly in the inferior leads (II, III, ARVC. Absence of evidence of pulmonary sarcoid does not
aVF) that usually show tall peaked complexes in idiopathic exclude the diagnosis. Endomyocardial biopsy that reveals
RV outflow tract VT. The presence of multiple morphologies non-caseating granulomas without other pathogens confirms
of monomorphic VT makes idiopathic VT without structural the diagnosis, but biopsies may miss areas of involvement
heart disease, extremely unlikely. Multiple morphologies of due to the patchy nature of the disease. Endomyocardial
VT are common, however, in scar-related VTs. biopsy of the RV septum showed only nonspecific hypertro-
Figure 162.1.3 shows the bipolar right ventricular voltage phy in this patient. Biopsy of the septum often misses abnor-
map. Areas of normal voltage > 1.5 mV are purple. Electrogram malities in ARVC that are more common in the free wall of
amplitude progressively decreases from blue to green to yel- the ventricles. Cardiac positron emission tomography can be
low to red. Areas of low amplitude, consistent with scar are helpful showing areas of FDG uptake indicating inflamma-
present in the free wall of the RV outflow tract, and at the tion in sarcoidosis.
inferior aspect of the RV, extending along the tricuspid annu- Other causes of RV scar include prior surgery, such as
lus as well. These findings are consistent with scar-related RV repair of Tetralogy of Fallot, right ventricular infarction, and
tachycardia. idiopathic cardiomyopathies. The latter remains a possibility
in this patient.

Discussion
Bibliography
Arrhythmogenic right ventricular dysplasia is the most likely
diagnosis. This is a heterogeneous genetic disease with more Calkins H. Arrhythmogenic right-ventricular dysplasia/cardiomyopathy.
than eight causative mutations identified. Most, but not all, Curr Opin Cardiol. 2006;21:5563.
mutations involve proteins coding for parts of desmosomes Sen-Chowdhry S, Syrris P, Ward D, etal. Clinical and genetic charac-
that are involved in cell-to-cell adhesion, including plakophi- terization of families with arrhythmogenic right ventricular dyspla-
sia/cardiomyopathy provides novel insights into patterns of disease
lin, plakoglobin, desmoglein, and desmocollin. It is hypoth- expression. Circulation. 2007;115:17101720.
esized sufficient wall stress to compromise cell adhesion Sen-Chowdhry S, Syrris P, McKenna WJ. Role of genetic analysis in
in certain areas, such as along the tricuspid and pulmonary the management of patients with arrhythmogenic right ventricular
valve annulus leads to cell death and fibrofatty replacement. dysplasia/cardiomyopathy. J Am Coll Cardiol. 2007;50:
18131821.
LV involvement is present in more than 30% of patients and Tandri H, Macedo R, Calkins H, et al. Role of magnetic resonance
predominates in some patients. The 1994 task force docu- imaging in arrhythmogenic right ventricular dysplasia: insights
ment defining the disease listed major diagnostic criteria from the North American arrhythmogenic right ventricular dyspla-
as severely abnormal RV wall motion or localized definite sia (ARVD/C) study. Am Heart J. 2008;155:147153.
Corrado D, Basso C, Leoni L, etal. Three-dimensional electroanatomic
aneurysms, epsilon waves or localized delayed depolar- voltage mapping increases accuracy of diagnosing arrhythmogenic
ization in V1V3, or confirmed familial disease. Minor right ventricular cardiomyopathy/dysplasia. Circulation. 2005;111:
criteria included less severe RV wall motion abnormalities, 30423050.
614 J. Seiler and W.G. Stevenson

Koplan BA, Soejima K, Baughman K, et al. Refractory ventricular 200 ms


tachycardia secondary to cardiac sarcoid: electrophysiologic
characteristics, mapping, and ablation. Heart Rhythm. 2006;3: I
924929.
Marchlinski FE, Zado E, Dixit S, etal. Electroanatomic substrate and II
outcome of catheter ablative therapy for ventricular tachycardia in
setting of right ventricular cardiomyopathy. Circulation. 2004;110: III
22932298.
Tadamura E, Yamamuro M, Kubo S, et al. Images in cardiovascular aVR
medicine. Multimodality imaging of cardiac sarcoidosis before and
after steroid therapy. Circulation. 2006;113:e771773.
aVL

aVF

2.Diagnosis: Tetralogy of Fallot Presenting V1


with Wide QRS Tachycardia
V2

This 49-year-old male with Tetralogy of Fallot (TOF) V3


repaired 40 years ago including placement of a patch repair
of the ventricular septal defect (VSD), presented with hemo- V4
dynamically stable wide QRS tachycardia that was termi-
V5
nated by cardioversion. The 12-lead ECG during sinus
rhythm is shown in Fig. 162.2.1. At electrophysiology study, V6
the tachycardia shown in Fig. 162.2.2 was induced. What is
the most likely mechanism of tachycardia?
Atrial tachycardias are the most common arrhythmia Fig. 162.2.2 Surface ECG of monomorphic VT induced with pro-
grammed ventricular stimulation (VT #1), CL 300 ms, left bundle
branch block morphology, QRS axis +10, R/S transition V5

800 ms

I encountered late after repair of TOF. Ventricular tachycardia


is also an important cause of morbidity and sudden death.
II During sinus rhythm (Fig. 162.2.1) right bundle branch block
(RBBB) is present, as is usually the case after repair of TOF.
III The tachycardia, however, has a left bundle branch block
(LBBB) configuration with a markedly negative deflection in
aVR
V1, consistent with early depolarization of the RV or sep-
tum. Thus, the most likely diagnosis is ventricular tachycar-
aVL
dia. This was confirmed at electrophysiology study and
aVF catheter ablation attempted (Case 163.2). The QRS morphol-
ogy with an inferiorly directed frontal plane axis suggests
V1 origin in the superior aspect of the RV, potentially involving
160 ms the outflow tract repair.
V2

V3
Discussion
V4

V5 VT is an important cause of sudden death late after repair of


TOF. In a multicenter cohort study of 793 patients with TOF
V6 after surgical repair, 33 patients developed sustained VT and
16 died suddenly within a 10-year observation period. In
addition to clinical variables (higher age at repair and need
Fig. 162.2.1 Baseline 12-lead ECG. The patient presented at baseline
in sinus rhythm. Right bundle branch block with QRS width of 160 ms for transannular patch repair were predictors of sudden
was present death), QRS duration of 180 ms or more and a faster increase
Case 162 615

of QRS duration were identified as predictors of VT and Bibliography


sudden death in a multivariate analysis. The most common
wide QRS tachycardia encountered, however, is macroreen- Gatzoulis MA, Balaji S, Webber SA, etal. Risk factors for arrhythmia
trant atrial flutter. Since the vast majority of patients have and sudden cardiac death late after repair of tetralogy of Fallot:
RBBB after repair, atrial arrhythmias present with RBBB. amulticentre study. Lancet. 2000;356:975981.
Walsh EP. Interventional electrophysiology in patients with congenital
In contrast, because the VT is due to reentry through the RV
heart disease. Circulation. 2007;115:32243234.
areas of scar, VT often has an LBBB configuration. Zeppenfeld K, Schalij MJ, Bartelings MM, etal. Catheter ablation of
Occasionally, VT with an exit toward the septum will have ventricular tachycardia after repair of congenital heart disease: elec-
an RBBB-type configuration in V1. ICD implantation is troanatomic identification of the critical right ventricular isthmus.
Circulation. 2007;116:22412252.
recommended. Programmed electrical stimulation does not
reliably predict risk.

Case 163

Jens Seiler and William G. Stevenson

1.Ablation: Arrhythmogenic voltage map (Fig. 163.1.1, panel a) showed areas of abnor-
Right Ventricular Dysplasia mal low-amplitude signals (<1.5 mV), consistent with scar
in the free wall of the right ventricular outflow tract (RVOT),
and at the base of the right ventricle, consistent with ARVC.
This 47-year-old female (see diagnostic 162.1) with arrhyth- Pace mapping at the inferior aspect of the RVOT free wall
mogenic right ventricular cardiomyopathy (ARVC) is referred low-voltage area produced a QRS morphology similar to
for recurrent episodes of ventricular tachycardia (VT) caus- VT #1, suggesting that this was a possible exit region. The
ing ICD shocks despite therapy with sotalol. In addition to mapping catheter was positioned at this site and programmed
the history noted in Case 162.1, this patient had a prior epi- stimulation performed to induce VT. An LBBB inferior axis
cardial ICD placed that became infected and was removed VT was induced that had slightly presystolic activation of
via a median sternotomy several years previously. An abla- 10 ms at the inferior margin of the RVOT scar. Pacing at that
tion procedure had been performed 10 months prior to refer- site showed entrainment with QRS fusion and a post-pacing
ral, but VT recurred 3 months later. interval that was 30 ms longer than the VT cycle length
The electrophysiology study proceeded in steps. After (Fig.163.1.2), suggesting that this was in or near an outer
insertion of catheters in the right atrium, His bundle position, loop site, close to the circuit, but not in an isthmus. No ear-
and right ventricular (RV) apex, programmed ventricular lier sites were identified and pacing at adjacent sites failed
stimulation was performed, confirming inducible sustained to identify endocardial isthmus sites. RF ablation using a
VTs. The initial VT (Fig. 162.1.2 (a), panel a in Case 162.1) saline-irrigated catheter at this region was performed (brown
had a left bundle branch block (LBBB) configuration and tags in Fig. 163.1.1), but VT remained inducible. Another
inferiorly directed frontal plane axis, consistent with an ori- VT that had a superior frontal plane axis demonstrated dia-
gin in the right ventricle. VT was rapid, necessitating termi- stolic activity and isthmus sites at the inferior aspect of the
nation for further mapping. Repeated programmed tricuspid annulus (Fig. 163.1.3). RF lesions from the tricus-
stimulation and pacing during VT for attempted termination pid annulus through that region abolished that VT, but the
revealed three more monomorphic VTs, with cycle lengths inferior axis VT remained inducible.
(CL) ranging from 230 to 300 ms. All VTs had an a LBBB
morphology, consistent with an origin in the right ventricle.
Three had an axis directed inferiorly, consistent with an ori-
gin in the outflow tract region; one had an axis directed supe- What Should Be Done Next?
riorly indicating an exit located in the inferior portion of the
ventricle. These VTs were not hemodynamically tolerated, Failure of ablation can be due to inadequate mapping, inabil-
allowing only limited mapping during VT. ity to create an adequate ablation lesion, or inability to reach
Therefore, sinus rhythm mapping was begun in the RV the arrhythmogenic substrate. Although mapping was lim-
using an electroanatomic mapping system (Carto) to cre- ited by hemodynamic intolerance, limited mapping during
ate an endocardial voltage map (displaying peak-to-peak VT demonstrated that moving away from the region initially
bipolar electrogram amplitudes) in the right ventricle. The identified moved further from the reentry circuit, evident
from longer post-pacing intervals and later activation relative
to the QRS onset during VT. These observations suggest that
the reentry circuit was not endocardial. Although the right
J. Seiler and W.G. Stevenson(*)
ventricle is generally viewed as thin-walled, reentry circuits
The Arrhythmia Service, Cardiovascular Division,
Brigham and Womens Hospital, Boston, MA that cannot be ablated from the endocardium, even with large
e-mail: wstevenson@partners.org tip or irrigated electrodes do occur.

A. Natale et al. (eds.), Cardiac Electrophysiology, 617


DOI: 10.1007/978-1-84996-390-9_163, Springer-Verlag London Limited 2011
618 J. Seiler and W.G. Stevenson

a b
10.42 mV 11.81 mV
Bipolar Bipolar

1-SR endo > 127 Points 3-SR epi > 100 Points

1.52 mV 1.52 mV
0.49 mV 0.49 mV
0.09 mV 0.08 mV

R
R

1.49 cm 1.71 cm

Fig. 163.1.1 Electroanatomic endocardial and epicardial voltage map is likely to be normal tissue; red <0.49 mV; gray, unexcitable scarred
of the right ventricle in sinus rhythm: (a) endocardial voltage map, tissue; respectively. Brown dots represent ablation lesions in the RVOT,
RAO view, voltage color-coded; (b) epicardial voltage map, AP view, right ventricular free wall, and right ventricular base. Blue dots indi-
voltage color-coded. Purple denotes electrogram voltage >1.5 mV that cate sites with double, fractionated potentials

a b 200 ms

II

III

V1

V5

Abl d

Abl m
240 290 260
Abl p ms ms ms

RVa

S S S

Fig. 163.1.2 Endocardial entrainment and activation mapping of VT are low-amplitude fragmented potentials preceding the QRS complex
#2, CL 260 ms. (a) During entrainment pacing that is performed at a CL by 10 ms (arrows) recorded at the distal and middle electrode pairs of
of 240 ms from the tip of the ablation catheter (Abl d), the post-pacing the ablation catheter (Abl d and Abl m). The findings are consistent with
interval is 290 ms and minimal QRS fusion is evident, particularly with a pacing location close to an outer loop in the tachycardia circuit. Abl p
the change in QRS configuration in V5. (b) The electrograms at this site proximal ablation catheter, RVa right ventricular apex, S stimulus
Case 163 619

Fig. 163.1.3 Surface ECG and a b


intracardiac recordings during 200 ms 800 ms
VT #4, CL 230 ms, superior
QRS axis. (a) A fractionated I I
diastolic potential preceding the
QRS onset by 60 ms (arrow) is II
clearly visible at the ablation II
catheter (Abl), at the inferior III
aspect of the tricuspid annulus.
(b) RF energy application at that III
aVR
site converted VT #4 (superior
axis) to VT #2 (inferior axis).
Note the abrupt shift of the QRS V1 aVL
axis with a slight increase of CL
from 230 to 250 ms. Abl d distal aVF
ablation catheter, Abl m middle V5
ablation catheter, Abl p proximal V1
ablation catheter, RVa right
ventricular apex Abl d V2

V3
Abl m

V4
Abl p
V5

RVa V6

Prior cardiac surgery often results in pericardial adhe- Discussion


sions that are a barrier to percutaneous epicardial access.
In this case, the pericardial space was approached with a In ARVC, RV regions of scar are characterized by significantly
micropuncture needle and a few milliliters of contrast lower amplitudes than in unaffected RV regions. In a study of
injected, which was observed to disperse in the pericardial seven patients, affected RV regions had an endocardial bipolar
space. Pericardial access was then successfully obtained voltage of 0.5 0.1 mV compared to unaffected regions with
with the method described by Sosa and colleagues. a voltage of 4.6 0.2 (mean SEM). An endocardial voltage
Epicardial mapping was possible with the exception of the of <1.5 mV for bipolar electrograms is generally accepted to
region of the right ventricle immediately beneath the indicate abnormal tissue. It is extremely important to consider
sternum, caudal to the low-voltage endocardial region. the possibility that low amplitude at a site is due to poor cath-
The epicardial voltage map also revealed a free wall low- eter contact. The presence of normal fat in the epicardium is
voltage region with late potentials (Fig. 163.1.1, panel b) also a potential source of low-voltage regions. We would not
and pace mapping consistent with the inferior axis VTs expect fat to cause fractionated or delayed potentials, or long
(Fig. 163.1.4). Following mapping, VTs were no longer S-QRS delays during pacing (Fig. 163.1.4) and rely on these
reliably inducible, suggesting a mechanical, traumatic observations to help identify abnormal areas. We use assess-
effect. Coronary angiography was performed prior to epi- ment of capture during pacing at the site as well as assessment
cardial RF ablation to assure a safe distance between abla- of catheter stability and position relative to the plane of the
tion lesions and epicardial vessels (the left anterior tissue surface to assess this possibility. These low-voltage
descending coronary artery) (Fig. 163.1.5). A series of RF areas often extend from the pulmonic or tricuspid annulus
lesions were then placed through the region guided by involving the RV free wall. VTs originate from these abnormal
pace mapping and abnormal fractionated electrograms. areas. The approach to mapping and ablation is as for other
Repeat programmed stimulation during isoproterenol scar-related VTs. Techniques include activation, entrainment,
infusion induced VT with an inferior axis that was slower and pace mapping, depending on hemodynamic stability. In
than prior VTs and often terminated spontaneously. our experience, epicardial mapping and ablation is not uncom-
Additional epicardial and endocardial lesions were placed monly required.
through this region rendering the pacing threshold at these VTs in an ARVC are amenable to RF catheter ablation,
sites >10 mA. No VTs were then inducible with pro- but recurrences during long-term follow-up are observed in
grammed stimulation during isoproterenol infusion. 1985% of patients when follow-up exceeds 2 years. In this
620 J. Seiler and W.G. Stevenson

Fig. 163.1.4 Epicardial pace a b


map at the RVOT low-voltage 200 ms
area. Unipolar pacing (panel a)
produces a variable QRS I
morphology similar to one of
the sustained VTs with inferior II
QRS axis (panel b). The delay
from the stimulus (S) to QRS III
and the changing QRS
morphology suggests pacing in aVR
an area of abnormal, slow
conduction. The local epicardial aVL
electrogram at that site in sinus
rhythm is abnormal fractionated aVF
(arrow, panel c). Abl d distal
electrode pair of the ablation V1
catheter
V2

V3

V4

V5

V6

S S

c
100 ms

V5

Abl d

a b

Fig. 163.1.5 Angiography of the


left coronary system during
epicardial mapping of the RVOT:
(a) LAO 60; (b) RAO 30
projection. The ablation catheter
(1) is in the pericardial space
overlying the RVOT. The left
main coronary artery (2) and left
anterior descending artery (3) are
sufficiently distant to allow
ablation at this site and regions to
the right of the ablation catheter
Case 163 621

patient, acute success was achieved by a combined endo- and 2.Ablation: Tetralogy of Fallot
epicardial RF ablation approach. After a follow-up of Presenting with Ventricular Tachycardia
8months, there have been no recurrences of VT.

Catheter ablation of VT is attempted in a 49-year-old male


patient with Tetralogy of Fallot (TOF) and patch repair of
Bibliography the ventricular septal defect (VSD) (patient discussed in
Case 162.2). At electrophysiologic study, sinus rhythm was
Borger van der Burg AE, de Groot NM, van Erven L, etal. Long-term
present with a cycle length (CL) of 760 ms, right bundle
follow-up after radiofrequency catheter ablation of ventricular branch block with a QRS width of 160 ms, and a slightly
tachycardia: a successful approach? J Cardiovasc Electrophysiol. prolonged HV interval of 60 ms. Programmed ventricular
2002;13:417423. stimulation easily induced wide QRS tachycardia with a CL
Boulos M, Lashevsky I, Reisner S, etal. Electroanatomic mapping of
arrhythmogenic right ventricular dysplasia. J Am Coll Cardiol.
of 300 ms, left bundle branch block morphology and a QRS
2001;38:20202027. axis of +10. AV dissociation was present and His bundle
Dalal D, Jain R, Tandri H, etal. Long-term efficacy of catheter ablation deflections were not identifiable during the tachycardia, con-
of ventricular tachycardia in patients with arrhythmogenic right sistent with VT as the diagnosis (Fig. 163.2.1). The QRS
ventricular dysplasia/cardiomyopathy. J Am Coll Cardiol. 2007;50:
432440.
morphology of this tachycardia (VT #1) was different from
Marchlinski FE, Zado E, Dixit S, etal. Electroanatomic substrate and the tachycardia that had occurred spontaneously, which had
outcome of catheter ablative therapy for ventricular tachycardia in a more vertical QRS axis. The tachycardia was hemodynam-
setting of right ventricular cardiomyopathy. Circulation. 2001;110: ically tolerated, allowing mapping.
22932298.
Sosa E, Scanavacca M, dAvila A. A new technique to perform epicar-
Based on the QRS morphology that suggested a superi-
dial mapping in the electrophysiology laboratory. J Cardiovasc orly located right ventricular exit site, and the known asso-
Electrophysiol. 1996;7:531536. ciation of the region between the tricuspid annulus and

a b
400 ms

II

III

aVR

aVL

aVF

V1

V2

V3

Fig. 163.2.1 Surface ECG


during sinus rhythm and V4
ventricular tachycardia at
electrophysiologic study V5
(VT#1). (a) sinus rhythm and
right bundle branch block at
baseline, (b) VT #1, CL 300 ms, V6
left bundle branch block
morphology, QRS axis +10
622 J. Seiler and W.G. Stevenson

right ventricular outflow tract (RVOT) patch to be involved Mapping during sinus rhythm showed low-voltage/scarred
in these tachycardias, that area was explored first. Activation areas in the free wall of the outflow tract at the site of the
at the first site in that region was diastolic. Pacing at that RVOT patch and in the region of the VSD patch. An isthmus
site accelerated all electrograms and QRS complexes to the between the RVOT patch and tricuspid annulus was discern-
pacing rate with no change in the QRS morphology, consis- ible. Pace mapping at that isthmus site matched VT #1. Pace
tent with entrainment with concealed fusion. The post-pac- mapping at the septal side of the RVOT patch matched the
ing interval (PPI) from the last stimulus to the next activation spontaneous VT (Fig. 163.2.3).
at the site was 310 ms, consistent with a reentry circuit site. The findings are consistent with two different VT circuits
These findings suggest the region between RVOT patch and that share a common isthmus but have different directions
tricuspid annulus is a protected isthmus site and critical and exits (Fig. 163.2.4): one at the RVOT free wall, produc-
part of the tachycardia circuit (Fig. 163.2.2). Pacing at the ing VT #1 that was induced at electrophysiologic study and
septal side of the RVOT accelerated all electrograms to the the other at the septal side, producing the VT that had been
pacing CL, and there was a change in the QRS morphology documented to occur spontaneously. RF lesions were applied
consistent with constant fusion. The PPI was 25 ms longer between the RVOT patch and the tricuspid annulus and at the
than the tachycardia CL, again consistent with a proximity free wall side of the RVOT. Energy was deployed until the
to the reentry circuit, but in an outer loop rather than an targeted regions were unexcitable to pacing stimuli at 10 mA.
isthmus. VT terminated during catheter manipulation for At the end of the procedure, no VT was inducible with up to
mapping. three extrastimuli and burst pacing from two sites in the RV.

200 ms

II

III

V1

V5

280 ms 310 ms 300 ms 15 ms

Fig. 163.2.2 Entrainment of Abl d


VT#1, CL 300 ms, from the region
between tricuspid annulus and
RVOT patch. Stimulation with Abl m
themapping and ablation catheter
(Abl d) at that site with a CL of 280
ms produces no change in QRS
Abl p
morphology and results in a PPI of
310 ms, indicating this site being
within the VT circuit in a protected
isthmus. Local activation at that HRA
site precedes the onset of the
QRS complex by 15 ms. Abl d
distal ablation catheter, Abl m middle
ablation catheter, Abl p proximal RVa
ablation catheter, HRA high right
atrium, RVa right ventricular apex,
S stimulus S S
Case 163 623

Fig. 163.2.3 Pace mapping. a b c 200 ms


(a)surface ECG during VT #1,
(b) ventricular stimulation with a
I
CL of 600 ms at the isthmus
between tricuspid annulus and
II
RVOT patch. The paced QRS
complexes match that of VT #1 III
in all 12 leads. (c) Ventricular
stimulation with a CL of 600 ms aVR
at the septal side of the RVOT
produces QRS complexes with
aVL
left bundle branch block
morphology and inferior axis
matching the QRS morphology aVF
of the spontaneous VT. S denotes V1
stimulus
V2

V3

V4

V5

V6

S S

PV
Discussion
VT after repair of tetralogy of Fallot is usually due to reentry
involving areas with scar in the right ventricular outflow tract
P and/or septum. The potential isthmuses available for reentry
are related to the type of surgical repair. The most common
2 isthmus location is between the tricuspid valve annulus and the
area of dense scar or patch in the free wall of the RVOT. Other
isthmuses can be present between the pulmonary annulus and
RV free wall scar, or between the septal patch and tricuspid
1
annulus or free wall scar. Mapping and ablation is done as for
TV
other scar-related VTs, using activation mapping and entrain-
ment when possible. When VT is unstable, potential isthmuses
for ablation can be identified from voltage maps combined
with pace mapping. Ablation is most commonly used to reduce
symptomatic VT episodes in patients with ICDs. Ablation has
been used either as a primary treatment strategy in selected
Fig. 163.2.4 Schematic drawing of the right ventricle (RAO view) patients with hemodynamically stable arrhythmias.
depicting the presumed tachycardia mechanisms. The RVOT patch (P)
and tricuspid valve (TV) annulus create a protected isthmus of slow
conduction (five-point star). Pace mapping at that site produces QRS
complexes matching VT #1, entrainment mapping at that site reveals a Bibliography
protected isthmus site of the reentry circuit. Pacing mapping at the sep-
tal side of the RVOT patch (four-point star) produces QRS complexes
matching the clinical VT, entrainment mapping at that site reveals an Walsh EP. Interventional electrophysiology in patients with congenital
outer loop site of VT #1. Deployment of ablation lesions at the described heart disease. Circulation. 2007;115:32243234.
isthmus (dotted line) and at the right ventricular free wall rendered Zeppenfeld K, Schalij MJ, Bartelings MM, etal. Catheter ablation of
the patient non-inducible. The findings are consistent with two VT ventricular tachycardia after repair of congenital heart disease: elec-
circuits with different exits and directions, sharing a common isthmus troanatomic identification of the critical right ventricular isthmus.
(arrow #1, VT #1; arrow #2, spontaneous VT). PV pulmonary valve Circulation. 2007;116:22412252.

Case 164

Jens Seiler and William G. Stevenson

Case Summary Postoperatively, recurrent episodes of monomorphic VT


failed to respond to intravenous amiodarone.
A 59-year-old male with recurrent monomorphic ventricular At electrophysiology study he is in sinus rhythm with first-
tachycardia (VT) was referred for electrophysiologic study. degree AV block (PR interval 255 ms) and premature ven-
He has an old inferior posterior myocardial infarction and tricular contractions. The HV interval is prolonged to 75 ms
a left ventricular ejection fraction of 25%. Sustained VT (Fig.164.1). Programmed ventricular stimulation induced a
had occurred during exercise testing leading to coronary wide QRS tachycardia with a left bundle branch block
artery bypass surgery and mitral valve repair 7 days ago. (LBBB) configuration (Fig.164.2). A His bundle deflection

a 800 ms b 100 ms

I I

II
II
III
III
aVR
V1
aVL

aVF V5
Fig.164.1 Baseline surface ECG
and EGM. (a) Surface 12-lead V1 HRA 75 ms
ECG showing sinus rhythm,
first-degree AV block (PQ 255 ms) V2 HIS d
and one PVC with right bundle
branch block morphology. V3
HIS m
(b) Intracardiac readings during
sinus rhythm show a significant HV V4
prolongation to 75 ms. HRA HIS p
denotes high right atrium, HIS d, V5
HIS m, and HIS p distal middle and RVa
proximal His catheter, RVa right V6
ventricular apex, respectively

J. Seiler(*)
Department of Cardiology, University Hospital,
Freiburgstrasse, Bern 3010, Switzerland
e-mail: jens-seiler@gmx.net
W.G. Stevenson
Cardiovascular Division, Harvard Medical School,
Brigham and Womens Hospital, 75 Francis Street,
Boston, MA 02115, USA
e-mail: wstevenson@partners.org

A. Natale et al. (eds.), Cardiac Electrophysiology, 625


DOI: 10.1007/978-1-84996-390-9_164, Springer-Verlag London Limited 2011
626 J. Seiler and W.G. Stevenson

100 ms 200 ms
I

II I
III II
V1
III
V5

HRA V1

V5
HIS p
360 ms 345 ms
HRA

RVa
360 ms 345 ms

HIS m
Fig.164.2 Bundle branch reentry tachycardia (VT #1). Oscillations of
the His to His intervals precede identical changes in ventricular cycle
length, which is consistent with bundle branch reentry tachycardia.
Arrowheads indicate His deflections. HRA high right atrium, HIS p His 340 ms 380 ms 360 ms
proximal, RVa right ventricular apex, respectively RVa

S S

is present preceding the QRS onset. What is the mechanism Fig.164.3 Entrainment of bundle branch reentry tachycardia (VT #1)
of the tachycardia? from the right ventricular apex. The CL of VT #1 is 360 ms. Entrainment
from the right ventricular apex is performed at a cycle length of 340 ms
and shows QRS fusion. The post-pacing interval (PPI) of the return
cycle is 380 ms. The difference of PPI VT CL=20 ms reveals the right
ventricular apex is in or near the circuit. QRS fusion indicates pacing in
Case Discussion an outer loop type of site rather than a protected isthmus. These findings
are consistent with bundle branch reentry tachycardia. HRA denotes
high right atrium, HIS m middle pair of electrodes on the His bundle
The differential diagnosis of this tachycardia is supraven- catheter, RVa right ventricular apex, respectively
tricular tachycardia with aberrancy, ventricular tachycardia
with passive retrograde activation of the HisPurkinje
system, bundle branch macroreentry, and participation of a was part of the circuit, although this finding would also be
portion of the Purkinje system in a scar-related VT. AV expected if VT originated from the apical septum or RV.
dissociation is present excluding atrial tachycardia or AV In addition, placement of an ablation catheter along the
reentry using an accessory pathway. AV nodal reentry with His bundle and right bundle shows that the right bundle
VA block or junctional ectopic tachycardia are possible, but is activated in the anterograde direction (Fig. 164.4), as
are rare. The latter is generally not induced by programmed expected for bundle branch reentry that has a LBBB configu-
stimulation. ration. Thus, a diagnosis of bundle branch reentry using the
Analysis of oscillations in the HH and VV interval left bundle in the retrograde direction and the right bundle in
indicate that the Purkinje system is closely linked to the the anterograde direction was made.
tachycardia. As shown in Fig.164.2, although the His bundle The right bundle branch was targeted for ablation by
deflection is low amplitude, the decrease in HH from 360 to placing the ablation catheter along the right bundle, at the
345 ms anticipates the same decrease in VV interval, sup- location where the RB-V was shorter than the His-V by
porting bundle branch reentry rather than passive retrograde approximately 15 ms. RF ablation at that site during ongoing
activation of the Purkinje system during VT. bundle branch reentry tachycardia terminated the arrhythmia
Entrainment of the tachycardia from the right ventricular and produced right bundle branch block (RBBB), (Fig.164.4).
apex revealed that this pacing site is close to the circuit, which The HV interval increased further to 85 ms.
is also consistent with bundle branch reentry tachycardia Many patients with bundle branch reentry will have other
(Fig.164.3). The VT cycle length is 360 ms. Pacing at a cycle forms of inducible VT. In this patient, following the initial
length of 340 ms changes the QRS morphology, indicating ablation, VTs #2 and #3 were induced. These had a RBBB
fusion, but the PPI is 380 ms, indicating close proximity to the configuration with superior or inferior axis (Fig.164.5). In
reentry circuit, as would be expected if the right bundle branch both, mapping along the left side of the LV septum
Case 164 627

200 ms a b 200 ms
I I

II II
III
III
V1
V1
V5
V5
HRA
Abl d
Abl d
Abl m Abl m
Abl p
Abl p
HIS d
HIS m
15 ms HIS p
RVa
RVa

Fig. 164.4 Ablation of bundle branch reentry tachycardia (VT #1). Fig.164.6 Interfascicular reentry tachycardia (VT #2) after RF ablation
The ablation catheter (Abl) is positioned at the right bundle. Ongoing of the right bundle. The ablation catheter (Abl) is positioned at the left
bundle branch reentry tachycardia (first two QRS complexes) with right fascicle, and catheter position is unchanged during VT (a) and sinus
bundle branch potentials (arrowheads, visible on the ablation catheter, rhythm (b). (a) A sharp high frequency left fascicle potential (arrows)
activation from proximal to distal) are preceded by a His potential precedes the ventricular activation during VT. (b) A His potential (arrow-
(arrow, visible at His middle (HIS m)) by 15 ms. Termination of the head) and left fascicle potential (arrows) are visible during sinus rhythm,
tachycardia after 4 s of RF energy delivery. Note the RBBB after termi- indicating the potentials in (a) being true Purkinje/left fascicle potentials.
nation of the tachycardia and the increase of the PQ interval to 300 ms. A clearly discernable His potential is only visible during sinus rhythm.
The increase of the HV interval to 85 ms is not visible on this tracing. Note the similar QRS morphology during VT and sinus rhythm after
Abl d denotes distal ablation catheter, Abl m middle ablation catheter, right bundle ablation. HRA denotes high right atrium, Abl d, Abl m, Abl p
Abl p proximal ablation catheter, RVa right ventricular apex, distal, middle, and proximal ablation catheter, HIS d and HIS p distal and
respectively proximal His catheter, RVa right ventricular apex, respectively

demonstrated Purkinje potentials preceding the QRS. AV retrograde activation of the Purkinje system less likely. These
dissociation was present, making VT most likely. findings are consistent with interfascicular reentry with ret-
Furthermore, proximal to distal activation of the left bundle rograde activation of the left posterior fascicle and antegrade
was demonstrable during VT (Fig. 164.6), making passive activation of the left anterior fascicle in VT #2 and vice versa
in VT #3. Passive retrograde activation of the left bundle
branch from a scar-related LV tachycardia is also possible.
a b c 200 ms Left ventricular RF applications at the inferior septum
I
including portions of the posterior fascicle abolished these
II VTs and produced intermittent 2:1 infranodal block
III (Fig.164.7). Frequent PVCs with early activation in the infe-
aVR rior septum originating from the distal Purkinje system were
aVL also ablated. At the end of the procedure, no VT was induc-
aVF ible with programmed ventricular stimulation.
V1 The following day VT #3 which had a RBBB superior
V2 axis recurred and became incessant. A repeat electrophysio-
V3 logic study was performed. AV conduction was present in
V4 sinus rhythm with prolonged AH and HV intervals, RBBB,
V5 and frequent salvos of nonsustained VT. An ablation catheter
V6 was advanced into the left ventricle via a retrograde aortic
approach and placed in the vicinity of the left anterior fasci-
Fig.164.5 Three different VTs with observed HisPurkinje potentials cle guided by electrogram and anatomical landmarks.
preceding ventricular activation. (a) VT #1, bundle branch reentry, Intracardiac electrograms during VT demonstrated left ante-
cycle length (CL) 365 ms, LBBB morphology, QRS axis +60, precor-
rior fascicle potentials preceding the His deflection, consis-
dial R/S transition V5. (b) VT #2, interfascicular reentry, CL 395 ms,
RBBB morphology, QRS axis +120. (c) VT #3, interfascicular reentry, tent with retrograde conduction up the left anterior fascicle
CL 320 ms, RBBB morphology, QRS axis 100 and antegrade conduction down the left posterior fascicle
628 J. Seiler and W.G. Stevenson

400 ms 200 ms

I I
II
II
III
V1 III

V5
V1
HRA
V5

Abl d

Abl m

Abl p
HIS m

HIS

Fig. 164.8 Interfascicular reentry tachycardia (VT #3). The surface


ECG leads show RBBB morphology and superior QRS axis. The abla-
tion catheter is positioned at the left bundle. This portion of the left
RVa bundle is activated from proximal to distal (arrows) and precedes acti-
vation of the His bundle (arrowhead). These characteristics are consis-
tent with interfascicular reentry tachycardia with retrograde activation
Fig.164.7 Transient infranodal block after ablation of the right bundle up the left anterior fascicle and antegrade activation down the left
and the left posterior fascicle. Arrowheads indicate His deflections. posterior fascicle. Abl d denotes distal ablation catheter, Abl m middle
HRA denotes high right atrium, HIS m His middle, RVa right ventricular ablation catheter, Abl p proximal ablation catheter, HIS His catheter,
apex, respectively respectively

(Fig.164.8). RF energy was applied in the region of the left observed. Other scar-related reentrant tachycardias were
anterior fascicle and a septal line was created. Because of the present in 60% of patients who had VTs involving the
persistence of interfascicular reentry beats after left anterior Purkinje system.
fascicle ablation, RF energy was applied in at the left poste- A recent study also emphasizes the potential involvement
rior fascicle position, guided by electrogram and anatomical of the Purkinje system in forming one limb of a reentry
landmarks. A line of ablation was created in the mid septal circuit involving regions of scar. Previous studies have shown
region which abolished ectopy. The HV interval prolonged bundle branch reentry as an important cause of sustained
to 128 ms. monomorphic VT in patients with cardiomyopathy and
Following administration of isoproterenol, programmed valvular heart disease.
stimulation induced sustained monomorphic VT with no When the Purkinje system is involved in causing VT, evi-
HisPurkinje potentials preceding ventricular activation and dence of infranodal conduction delay is usually present dur-
a cycle length of 330 ms that was attributed to scar-related ing sinus rhythm. The VTs often have a typical LBBB, and
reentry. A line of ablation lesions was created through the less frequently RBBB configuration, that can mimic the sinus
low voltage inferior wall scar connecting the mid inferior left rhythm QRS when intraventricular conduction delay is pres-
ventricle with the septum. During follow-up the patient has ent. Distinction from supraventricular tachycardias is usually
been free of VT. possible by dissociating atrial activity. AV nodal reentry with
Of 234 consecutive patients with structural heart disease persistent AV dissociation is rare. Automatic junctional
referred to our laboratory for VT ablation, involvement of tachycardias with AV dissociation are not generally paroxys-
the HisPurkinje system in one or more VTs was observed in mal or inducible with programmed stimulation. Distinguishing
20 patients (8.5%). The frequency of Purkinje system bundle branch and interfascicular reentry from a scar-related
involvement in VT was similar in ischemic and nonischemic VT with passive retrograde activation of the Purkinje system
heart disease. Bundle branch macroreentry was the most relies on demonstrating linking of the Purkinje system to the
common form, but interfascicular reentry and automaticity VT circuit and appropriate activation of the bundle branches
in the distal fascicles of the Purkinje system were also for the QRS morphology observed.
Case 164 629

Bibliography Eckart RE, Hruczkowski TW, Tedrow UB. etal. Sustained ventricular
tachycardia associated with corrective valve surgery. Circulation.
2007;116:2005-2011.
Bogun F, Good E, Reich S, etal. Role of Purkinje fibers in post-infarction Lopera G, Stevenson WG, Soejima K, etal. Identification and ablation
ventricular tachycardia. J Am Coll Cardiol. 2006;48:2500-2507. of three types of ventricular tachycardia involving the His-Purkinje
Caceres J, Jazayeri M, McKinnie J, etal. Sustained bundle branch reentry system in patients with heart disease. J Cadiovasc Electrophysiol.
as a mechanism of clinical tachycardia. Circulation. 1989;79:256-270. 2004;15:52-58.

Case 165

J. Jason West and John Paul Mounsey

Case Summary infusion of isoproterenol at 1 mcg/m2/min, the characteris-


tics of the accessory pathway changed significantly. The
accessory pathway then supported 1:1 antegrade conduc-
An 18-year-old man presented to the electrophysiology
tion during atrial pacing to 230 ms (see Fig. 165.3).
clinic for evaluation of an abnormal electrocardiogram. He
Retrograde conduction properties also improved. Atrial
was a freshman participating in the varsity wrestling pro-
fibrillation was not induced but short runs of AV reentrant
gram at a major university. He denied ever experiencing
tachycardia could be initiated with atrial stimulation. What
palpitations, syncope/pre-syncope, chest pain, or light-
would you do now?
headedness. As per the standard screening protocol of ath-
letes at this institution, a pre-participation electrocardiogram
had been obtained upon his arrival at school, and demon-
strated a short PR interval and a delta wave (see Fig.165.1).
His work-up included a normal echocardiogram without
Case Discussion
evidence of Ebsteins anomaly. He was referred for elec-
trophysiologic study for risk stratification of his accessory In this young man who participates regularly in vigorous ath-
pathway and possible catheter ablation. During the course letics, we were concerned by the change in the properties
of his preoperative work-up, he was noted to have lost pre- ofthe accessory pathway with isoproterenol and decided to
excitation on his electrocardiogram, a finding thought to proceed with ablation. Using a transseptal approach, radiof-
suggest a low risk for sudden death (see Fig.165.2). At the requency energy was applied to the lateral wall at the site of
time of electrophysiologic study under conscious sedation, continuous atrial and ventricular signals. Pathway conduc-
he was noted to have regained preexcitation. Baseline elec- tion was terminated within two beats (see Fig.165.4).
trophysiologic intervals revealed PR=100, QRS=116, Asymptomatic ventricular preexcitation is an uncommon
QT=302, AH=83, HV=3 ms. With atrial pacing, he yet vexing finding in competitive athletes. In the general
demonstrated further preexcitation with earliest ventricu- adult population, it is generally accepted that the risk of sud-
lar activation consistent with a left lateral pathway. The den death in a patient with a previously asymptomatic acces-
pathway lost antegrade conduction with atrial pacing at a sory pathway is quite low. However, athletes are potentially
cycle length of 520 ms, and retrograde conduction with at higher risk for sudden death due to their propensity for
ventricular pacing at a cycle length of 500 ms. With the higher adrenergic states and a higher incidence of atrial
fibrillation. To date, there have been no major trials examin-
ing this issue in athletes. We must, therefore, extrapolate
from the literature focused upon the general population.
Multiple population-based studies have shown a low risk
J.J. West
Bend Memorial Clinic, of arrhythmia in individuals with asymptomatic preexcita-
1501 N.E Medical Center Drive, tion, and an even lower risk of sudden death.3,5 These risks
Bend, OR 97701, are potentially underestimated by the older age of many indi-
e-mail: johnjasonwest@hotmail.com viduals in these cohorts. Studies involving younger subjects
J.P. Mounsey (*) suggest a somewhat higher risk.6,7
Department of Cardiology, Athletes have two features that potentially increase
University of North Carolina,
160 Dental Circle, CB 7075, Chapel Hill,
their risk from bypass pathways relative to the general popu-
NC 27599, USA lation. First, athletes exhibit a high resting vagal tone. This
e-mail: pmounsey@med.unc.edu vagal influence results in an increased incidence of atrial

A. Natale et al. (eds.), Cardiac Electrophysiology, 631


DOI: 10.1007/978-1-84996-390-9_165, Springer-Verlag London Limited 2011
632 J.J. West and J.P. Mounsey

Fig.165.1 Baseline 12-lead electrocardiogram demonstrating a short PR interval and delta wave consistent with a left lateral accessory pathway

Fig.165.2 Repeat 12-lead electrocardiogram obtained on hospital admission demonstrating the absence of ventricular preexcitation
Case 165 633

Fig. 165.4 Intracardiac electrograms demonstrating radiofrequency


Fig.165.3 Intracardiac electrograms demonstrating rapid atrial pacing
ablation of the left lateral pathway. Within 2 s, the pathway is elimi-
during infusion of isoproterenol resulting in maximal ventricular preex-
nated and ventricular preexcitation resolves
citation. The earliest ventricular signal is in lead CSd consistent with a
left lateral pathway. The pathway blocks at a pacing rate of 230ms

fibrillation.4 Rapid antegrade conduction during atrial fibril- References


lation is hypothesized as the mediator for sudden death in
individuals with an accessory pathway. Second, despite this 1. Calkins H, Yong P, Miller JM, etal. Catheter ablation of accessory
relatively high resting vagal tone, athletes are often produc- pathways, atrioventricular nodal reentrant tachycardia, and the atrio-
ing high-adrenergic states during the course of training and ventricular junction: final results of a prospective, multicenter clini-
cal trial. The Atakr Multicenter Investigators Group. Circulation.
competition. This adrenergic surge can alter the conduction
1999;99:262-270.
of an accessory pathway, rendering an otherwise low-risk 2. German LD, Gallagher JJ, Broughton A, Guarnieri T, Trantham JL.
pathway into a more dangerous one.2 While the specific role Effects of exercise and isoproterenol during atrial fibrillation in
of isoproterenol in the evaluation of accessory pathways is patients with Wolff-Parkinson-White syndrome. Am J Cardiol.
1983;51:1203-1206.
controversial, the characteristics of the adrenergically stimu-
3. Goudevenos JA, Katsouras CS, Graekas G, Argiri O, Giogiakas V,
lated pathway are potentially much more in line with what Sideris DA. Ventricular pre-excitation in the general population:
athletes regularly experience. a study on the mode of presentation and clinical course. Heart.
These risks must then be balanced by the risks associated 2000;83:29-34.
4. Mont L, Sambola A, Brugada J, etal. Long-lasting sport practice and
with evaluation and treatment of the accessory pathway.
lone atrial fibrillation. Eur Heart J. 2002;23:477-482.
While noninvasive testing probably still has some role in the 5. Munger TM, Packer DL, Hammill SC, etal. A population study of
identification of low-risk accessory pathways, the electro- the natural history of Wolff-Parkinson-White syndrome in Olmsted
physiologic study has become the gold standard in the evalu- County, Minnesota, 1953-1989. Circulation. 1993;87:866-873.
6. Pappone C, Manguso F, Santinelli R, etal. Radiofrequency ablation
ation of this condition. The safety of these studies and
in children with asymptomatic Wolff-Parkinson-White syndrome.
radiofrequency catheter ablation for accessory pathways has NEngl J Med. 2004;351:1197-1205.
been repeatedly supported in large-scale ablation registries.1,8 7. Sarubbi B, Scognamiglio G, Limongelli G, etal. Asymptomatic ven-
Thus, the identification of athletes with asymptomatic preex- tricular pre-excitation in children and adolescents: a 15year follow
up study. Heart. 2003;89:215-217.
citation should prompt further work-up and potential cura-
8. Scheinman MM, Huang S. The 1998 NASPE prospective catheter
tive therapy. ablation registry. Pacing Clin Electrophysiol. 2000;23:1020-1028.

Case 166

Darren Traub, James P. Daubert, and Spencer Rosero

Case Summary corrected for heart rate measures 520 ms.1 Multiple other
ECGs from this patient demonstrated a prolonged QTc
greater than 500 ms. The patient was started on atenolol
A 6-ft 3-in., 16-year-old male who was a high-school varsity
50mg daily and referred to the electrophysiology service for
soccer and basketball player was referred to pediatric cardi-
evaluation.
ology to rule out Marfans syndrome based on his tall slim
During outpatient clinical evaluation, the patients symp-
body habitus and occasional joint aches. The patient denied
toms of lightheadedness were reproduced with jumping
any history of palpitations, shortness of breath, or chest dis-
jacks. Telemetry monitoring while performing jumping
comfort with exertion. He did mention that occasionally he
jacks is displayed in Fig.166.2. The underlying rhythm is
became slightly lightheaded or dizzy at the end of a game,
sinus with ventricular bigeminy and a short burst of poly-
but would always feel better after a few minutes of rest.
morphic ventricular tachycardia. Short bursts of polymor-
These occasional episodes of lightheadedness had not caused
phic ventricular tachycardia occurred multiple times while
him or his family enough concern to seek medical attention.
performing exercise commensurate with the patients light-
The patients family history was unremarkable for sudden
headedness. He was admitted to the hospital for further test-
cardiac death, with the exception of his maternal great-
ing and possible placement of an implantable cardioverter
grandmother who had a seizure disorder and died at the
defibrillator. The evening of his admission he was awakened
age of 36 of unknown causes. Physical exam was normal. An
by a CCU resident. (Fig. 166.3) demonstrates telemetry
echocardiogram showed no structural heart disease and no
monitoring when he was startled by this unexpected inter-
anomalous coronary origin. The patients electrocardiogram
ruption of his sleep.
(ECG) is shown below (Fig.166.1). What is the likely diag-
This patients clinical presentation and electrocardiogram
nosis and what is the best management option for this
are consistent with the long-QT syndrome (LQTS). In the
patient?
Pediatric Electrophysiology Societys International Registry
of 287 patients with the long QT syndrome, 61% of patients
were symptomatic at presentation. Serious symptoms were
present in 45% of patients including cardiac arrest (9%),
Case Discussion syncope (26%), and seizures (10%). Six percent of patients
presented with pre-syncope or palpitations. Among the
This ECG demonstrates sinus bradycardia at a rate of 51 symptomatic patients, 67% had symptoms related to exer-
beats per minute. The T waves are broad and prolonged. The cise, 18% with exercise and emotion, and 7% with emotion
QT interval in lead II measures 560 ms. The QT interval alone.2
LQTS is thought to affect approximately 1 in 2,500 peo-
ple and be responsible for up to 2,0003,000 sudden deaths
in children to young adults each year.3,4 Currently, more than
D. Traub(*) 150 different mutations have been identified in ten genes
St. Lukes Hospital and Health Network, linked to the long-QT syndrome. LQT1, LQT2, and LQT3
801 Ostrum Street, Bethlehem, PA 18015, USA account for 95% of the identified mutations.4-7 An abnormal
e-mail: traubd@slhn.org
electrocardiogram demonstrating a prolonged QT interval
J.P. Daubert and S. Rosero corrected for heart rate (QTc) is the key to establishing the
Cardiac Electrophysiology, Cardiology Division,
diagnosis. The QTc is prolonged when it is greater than
Duke University Health System, DUMC Box3174,
Duke Hospital 7451H, Durham, NC 27710, USA 440 ms for men and 460 ms for women and children.5,6
e-mail: james.daubert@duke.edu The QTc interval can be variable, necessitating frequent

A. Natale et al. (eds.), Cardiac Electrophysiology, 635


DOI: 10.1007/978-1-84996-390-9_166, Springer-Verlag London Limited 2011
636 D. Traub et al.

Fig.166.1 Baseline ECG showing sinus bradycardia at 51 beats per minute, a QT interval of 640ms, and a QTc of 592ms

Fig.166.2 Telemetry monitoring while performing jumping jacks. The rhythm is sinus with ventricular bigeminy and short burst of polymorphic
ventricular tachycardia
Case 166 637

Fig.166.3 Telemetry monitoring when patient was startled by abrupt awakening

follow-up ECGs when long-QT syndrome is suspected but patients with mutation confirmed LQTS.10,11 Female gender,
not confirmed during initial evaluation or an ECG demon- LQT2 channel mutations, and a history of a syncopal event
strates borderline prolongation.5 have also been associated with increased risk for aborted car-
Exercise testing is frequently performed in the evaluation diac arrest or sudden cardiac death. Beta-blockers provide an
of patients with suspected long QT syndrome. The exercise approximately 60% reduction in the risk cardiac events and
test is particularly useful if it precipitates a diagnostic life threatening cardiac events.11 Patients with LQT1 obtain
arrhythmia as occurred in our patient or if the QTc interval the greatest benefit from beta-blockade, with less of a thera-
becomes excessively prolonged during the recovery phase.59 peutic response in patients with LQT2 and unproven benefit
As the heart rate increases during exercise, the normal among those with LQT3. Implantation of an ICD should be
response of the QT interval is to shorten. In patients with considered among those patients considered to be at high
long QT syndrome, the QT interval may fail to shorten or risk for sudden cardiac death including survivors of cardiac
may paradoxically prolong with exercise or during the recov- arrest, patients with syncope while receiving beta-blockers,
ery phase. Exercise testing does have limitations. The QTc and possibly those with an extremely prolonged QT
may be difficult to measure accurately during exercise due to interval.6,12
motion artifact and fusion of the T and P waves.8,9 Variations Because our patient demonstrated high-risk features con-
in the response of different types of ion channel defects also sisting of a QTc interval of over 500ms and symptoms with
limit the diagnostic utility of exercise testing for establishing demonstrated polymorphic VT despite beta-blocker therapy,
a LQTS diagnosis. LQT1 in particular shows a failure of the an ICD was placed. Screening electrocardiograms of the
QTC to shorten with exercise, while in LQT2 and LQT3 the patients family revealed that his mother and one of his two
response of the QTc to exercise is normal and supranormal, sisters had QTc intervals of approximately 460465 ms.
respectively.6,9 Thus exercise testing may give a suggestion Genetic testing is still in progress.
to the LQTS subtype.
Once the diagnosis of LQTS is established in a patient,
ECGs should be obtained on all first-degree family members
to determine whether others are affected. Unexplained sud- References
den death in a young individual should also prompt an evalu-
ation for the presence of LQTS in the family. Occasionally 1. Bazett HC. An analysis of the time relations of electrocardiograms.
Heart. 1918;7:353-370.
an asymptomatic individual may be noted to have QTc pro- 2. Garson A Jr, Dick M II, Fournier A, etal. The long QT syndrome in
longation based on an ECG obtained for another reason. children: an international study of 287 patients. Circulation.
While the absence of symptoms does not rule out familial 1993;87:1866-1872.
LQTS, a careful medication history is mandated.5,6 Genetic 3. Etheridge SP, Sanatani S, Cohen MI, etal. Long QT Syndrome in
Children in the Era of Implantable Defibrillators. J Am Coll Cardiol.
testing can identify a mutation in up to 75% of probands in 2007;50:1335-1340.
whom the diagnosis of LQTS is relatively certain based on 4. Vincent MG. The long QT syndrome. Indian Pacing Electrophysiol J.
ECG and clinical presentation. Establishing the particular 2002;2(4):127.
genetic mutation of a proband is useful for ruling out the 5. Moss AJ. Long QT syndrome. JAMA. 2003;289:2041-2044.
6. Roden DM. Long-QT syndrome. N Engl J Med. 2008;358:169-176.
diagnosis in family members. Additionally, the prognosis of 7. Morita H, Wu J, Zipes DP. The QT syndromes: long and short.
LQTS patients and response to medical therapy appear to be Lancet. 2008;372(9640):750-763.
gene specific.4-7,10,11 8. Dillenburg RF, Hamilton RM. Is exercise testing useful in identify-
The QTc duration is a powerful predictor of risk for syn- ing congenital long QT syndrome? Am J Cardiol. 2002;89:233-236.
9. Connuck DM. The role of exercise stress testing in pediatric patients
cope, cardiac arrest, or sudden death in patients with with heart disease. Progr Pediatr Cardiol. 2005;20:45-52.
LQTS.6,10,11 A QTc >500 ms has been associated with 10. Priori SG, Schwartz PJ, Napolitano C, etal. Risk stratification in the
increased risk for cardiac events in two separate analysis of long-QT syndrome. N Engl J Med. 2003;348:1866-1874.
638 D. Traub et al.

11. Sauer AJ, Moss AJ, McNitt S, etal. Long QT syndrome in adults. the European Society of Cardiology Committee for Practice
J Am Coll Cardiol. 2007;49:327-337. Guidelines (writing committee to develop Guidelines for
12. Zipes DP, Camm AJ, Borggrefe M, et al. ACC/AHA/ESC 2006 Management of Patients with Ventricular Arrhythmias and the
guidelines for management of patients with ventricular arrhythmias Prevention of Sudden Cardiac Death): developed in collaboration
and the prevention of sudden cardiac death: a report of the American with the European Heart Rhythm Association and the Heart Rhythm
College of Cardiology/American Heart Association Task Force and Society. Circulation. 2006;114:e385-e484.
Case 167

Vikas P. Kuriachan and George D. Veenhuyzen

Case Summary interval, QRS duration, and HV interval), initiating a supraven-


tricular tachycardia (SVT) with a 1:1 AV and HV relationship.
What structures are responsible for this girls tachycardia?
A 12-year-old girl with WolffParkinsonWhite Syndrome
underwent catheter ablation. (Fig.167.1) shows programmed
atrial stimulation from the proximal electrode pair of a multi-
polar catheter, approximately 2cm inside the coronary sinus Case Discussion
(CS). The last two beats of the pacing train (A1) are preex-
cited (short PR interval, delta wave with QRS widening, no At first glance, this resembles a usual induction of orthodro-
visible His potential). The extra stimulus (A2) blocks in the mic atrioventricular reciprocating tachycardia (AVRT) by
accessory pathway (AP) and conducts to the ventricles exclu- programmed atrial stimulation: The premature beat blocks in
sively over the normal AV conduction system (normal PR the AP and while traveling to the ventricle via the normal AV

S1: 500
200 ms
I S2: 310

II
aVF
aVL

V1
V3

V6
HRA
HIS
CS proximal

CS mid
Fig.167.1 Initiation of
tachycardia with a single CS distal
atrial extrastimulus.
(A1A2=500/310ms) RV

V.P. Kuriachan (*)


Department of Cardiac Electrophysiology,
University of Calgary, Foothills Medical Centre,
1403-29th St. NW, Calgary, Alberta T2N 2T9, Canada
e-mail: akuriachan@me.com
G.D. Veenhuyzen
Libin Cardiovascular Institute of Alberta,
University of Calgary, Foothills Hospital,
1403-29th Street NW, Calgary, Alberta T2N 2T9, Canada
e-mail: george.veenhuyzen@albertahealthservices.ca

A. Natale et al. (eds.), Cardiac Electrophysiology, 639


DOI: 10.1007/978-1-84996-390-9_167, Springer-Verlag London Limited 2011
640 V.P. Kuriachan and G.D. Veenhuyzen

conduction system, there is sufficient AV delay for the AP to Figure167.2 shows programmed ventricular stimulation.
recover and conduct the impulse retrogradely, inducing cir- At a coupling interval of 320ms, the retrograde atrial activa-
cus movement tachycardia. Indeed, a His-synchronous ven- tion sequence is crescentic (with collision at the mid
tricular premature beat terminated this tachycardia without CS electrode pair) and appears non-decremental. This is con-
conduction to the atrium, proving that it was AVRT. sistent with retrograde conduction over both the right- and
The earliest atrial electrogram during the SVT was left-sided APs (a component of conduction also via the nor-
recorded by the distal CS electrode pair, identifying the loca- mal AV conduction system cannot be excluded). At a cou-
tion of the operative AP as along the free wall of the mitral pling interval of 310ms, retrograde conduction over the right
annulus. However, the delta wave vector based on the first few sided AP blocks, revealing conduction over the left free wall
preexcited beats is consistent with a right sided AP. Indeed, AP (Fig. 167.3). The induction of orthodromic AVRT
the earliest ventricular electrogram during ventricular preex- employing the left free wall AP was facilitated by pro-
citation is recorded by the electrode at the His bundle region. grammed atrial stimulation from the left atrium, and induc-
Could these findings could be explained by a single AP? tion of orthodromic AVRT was facilitated by programmed
While the earliest atrial electrogram in AVRT is recorded at atrial stimulation from the right atrium.
the distal CS, the ventricular electrogram recorded by this Multiple accessory pathways have been reported in
electrode pair during atrial pacing with ventricular preexcita- 520% of patients with preexcitation.1 This case demon-
tion is inscribed at the end of those QRS complexes, indicat- strates the importance of matching the delta wave vector dur-
ing that the left free wall AP could not possibly have ing preexcitation with the earliest atrial electrogram during
meaningfully contributed to ventricular preexcitation: There AVRT, and studying atrial activation during programmed
must be two APs. ventricular stimulation carefully.

S1: 500
S2: 320
200 ms
V1

V6

HRA

HIS C

CS proximal

CS mid

Fig.167.2 Fusion of
CS distal
retrograde conduction over
different pathways after a
ventricular extrastimulus RV
(V1V2=500/320ms)
Case 167 641

Fig.167.3 Change in atrial S1: 500


S2: 310
200 ms
activation sequence in the V1
coronary sinus with a shorter
interval (V1V2=500/310ms)
V6

HRA

HIS C

CS proximal

CS mid

CS distal

RV

Reference

1. Zipes DP, Jaliffe J. Cardiac Electrophysiology: From Cell to Bedside.


4th ed. Philadelphia, PA: Saunders; 2004:870.

Case 168

Vikas P. Kuriachan and George D. Veenhuyzen

Case Summary conduction is occurring mostly over the AP. Thus, this
patients ventricular response to AF is determined by the
electrophysiologic properties of his AP, not those of his
A previously healthy 40-year-old male called paramedics
normal AV conduction system.
after experiencing palpitations, weakness, and presyncope.
In Fig.168.2, a flurry of extremely rapid preexcited beats
In the emergency room, the 12-lead ECG shown in Fig.168.1
(*) results in repeated ventricular stimulation at very short
was recorded. What rhythm is shown on the ECG? What is
cycle lengths (CL) and is followed by the development of
the best approach for acute therapy?
ventricular fibrillation (VF; cycle length=100 ms). This is
analogous to performing programmed ventricular stimula-
tion with multiple extrastimuli at very short coupling inter-
vals, which is capable of inducing VF in healthy hearts.
Case Discussion
Thus, Fig. 168.2 illustrates the mechanism by which
WolffParkinsonWhite (WPW) syndrome is associated
This wide complex, irregularly irregular tachycardia has an with a small risk of sudden death. Two hits are required:
average ventricular rate of 250 bpm, with RR intervals (1) the development of an extremely rapid atrial tachyar-
varying from 160 to 360 ms. Despite the marked beat to rhythmia (usually AF) and (2) an AP with a very short ante-
beat variation in rate, the QRS complex morphologies are grade refractory period. Therapy should be directed
all similar (but not the same). The irregularity should accordingly: (1) cardioversion and (2) slowing the ventricu-
prompt consideration of atrial fibrillation (AF) as the lar response by prolonging the refractory period of the AP. It
underlying rhythm. Atrioventricular (AV) conduction inter- should be no surprise that agents that only slow conduction
vals <250ms are usually not possible through the normal over the normal AV conduction system can precipitate sud-
AV conduction system, and should prompt consideration of den death (and are absolutely contraindicated) since they do
AV conduction over an accessory AV pathway (AP). The nothing to help the patient (and thereby allow a medical
QRS complexes result from variable fusion of conduction emergency to continue unabated) and may cause hypoten-
over the normal AV conduction system and the AP, result- sion or myocardial depression.
ing in subtle beat to beat variation in QRS complex mor- While which of the two therapies described above should
phology. The dominant feature of the QRS complexes is be applied is often debated, we advocate a simple approach,
that they are wide, slurred, and bizarre, indicating that AV particularly in patients with RR intervals <250ms (a risk fac-
tor for sudden death in individuals with WPW syndrome),
which is to perform both as quickly as possible. Because elec-
V.P. Kuriachan (*) trical cardioversion requires sedation and other preparation
Department of Cardiac Electrophysiology, that can be time consuming, it is logical to concurrently and
University of Calgary, Foothills Medical Centre, quickly provide an intravenous antiarrhythmic agent that will
1403-29 St. NW, Calgary, Alberta T2N 2T9, Canada slow the ventricular response in the interim. Current guidelines
e-mail: akuriachan@me.com
recommend the use of flecainide, ibutilide, and procainamide.1
G.D. Veenhuyzen This may improve hemodynamics and facilitate the adminis-
Libin Cardiovascular Institute of Alberta,
tration of adequate doses of sedation. Furthermore, the drug
University of Calgary, Foothills Hospital,
1403-29th Street NW, Calgary, Alberta T2N 2T9, Canada could result in pharmacologic cardioversion and/or help to pre-
e-mail: george.veenhuyzen@albertahealthservices.ca vent early recurrences of AF after electrical cardioversion.

A. Natale et al. (eds.), Cardiac Electrophysiology, 643


DOI: 10.1007/978-1-84996-390-9_168, Springer-Verlag London Limited 2011
644 V.P. Kuriachan and G.D. Veenhuyzen

Fig.168.1 Tachycardia on presentation to the emergency room

Fig.168.2 Degeneration of
preexcited atrial fibrillation to
ventricular fibrillation in another
patient

Figure168.3 shows a 12-lead ECG recorded after electri- (Fig.168.4). These features include: (1) a short PR interval
cal cardioversion. The features of ventricular preexcitation (120 ms in lead V5), (2) a delta wave (best appreciated in
are subtle (indicating that in sinus rhythm, ventricular activa- lead V5), and (3) an early R/S precordial transition in lead
tion is mostly over the normal AV conduction system) and V2. Because the clues to the presence of a potentially life
become more clear after comparison with the ECG recorded threatening AP are so subtle in sinus rhythm, this truly is a
after successful catheter ablation of a left free wall AP Wolff in sheeps clothing!
Case 168 645

Fig.168.3 ECG post-cardioversion: a Wolff in sheeps clothing. Delta waves are visible but are quite subtle

Fig.168.4 ECG post-catheter ablation of the left-sided accessory pathway. Note the absence of delta waves now in V1V3
646 V.P. Kuriachan and G.D. Veenhuyzen

Reference Cardiology/American Heart Association Task Force and the


European Society of Cardiology Committee for Practice Guidelines.
Bethesda, MD: American College of Cardiology Foundation;
1. Blomstrom-Lundqvist C, Scheinman MM, Aliot EM, et al. ACC/ 2003.
AHA/ESC Guidelines for the Management of Patients with
Supraventricular Arrhythmias. A Report of the American College of
Case 169

Jeffrey D. Booker and George D. Veenhuyzen

Case Summary Case Discussion

A 25-year-old female runner who had recently started train- Since her symptoms seemed to be progressing rapidly, she
ing for a competitive marathon was admitted to hospital after was admitted to the hospital for observation. While under-
complaining of recurrent episodes consisting of the sudden going continuous telemetry monitoring in hospital, initial
onset of extreme fatigue, lightheadedness, and transient loss intermittent 2:1 AV block and AV Wenckebach resolved
of vision lasting a few seconds. over several days of observation. She completed 24 METS
As a result of these symptoms, she stopped training, and on an accelerated Bruce protocol with no evidence of AV
her symptoms abated. Upon this improvement, she began block and appropriate shortening of her PR and QT inter-
training again, only to have her symptoms return. She kept a vals. She achieved a heart rate of 181 beats per minute with
training diary that revealed a clear repeating pattern over the a physiologic blood pressure rise. No AV block or dysrhyth-
weeks preceding her admission to hospital: intense running mias were observed.
for several days followed by several days where she was Cardiac MRI revealed no evidence of myocarditis or
unable to train at all because of the above mentioned symp- regional fibrosis that may involve the conduction system.
toms. This was followed by improvement and several more Lyme disease serology was negative.
days of training. Her symptoms were most pronounced at rest The final diagnosis was inappropriately high resting vagal
and were improved by mild activity (like walking), which led tone secondary to high intensity physical training resulting in
her to try exercising again. With continued exercise, her symptomatic AV block. She was discharged home with
symptom frequency progressed to daily episodes that occa- instructions to discontinue high intensity training. Her symp-
sionally included recurrent night sweats and one episode of toms have subsequently resolved and she has been well dur-
nocturnal enuresis. She decided to stop driving her car. ing over 1year of follow up.
Twenty-four hour ambulatory ECG monitoring during Resting bradycardia in athletes has been traditionally
symptoms and during waking hours (Fig.169.1) revealed high attributed to enhanced vagal tone, but there may also be
grade atrioventricular (AV) block with ventricular asystole up intrinsic changes in physiology of the SA and AV nodes.
to 4 s in duration. In Panel B, lengthening of the PR interval Athletes commonly have sinus pauses, first degree AV block,
prior to the first nonconducted P-wave (AV Wenckebach) is and AV Wenckebach, but more advanced AV block is
most consistent with block at the level of the AV node. uncommon.14 Because this patient had marked high grade
Physical examination was unremarkable. Her resting AV block, numerous investigations directed at identifying
heart rate was 50bpm. A 12-lead ECG showed only sinus underlying structural or conduction disease were performed.
bradycardia. In a series of athletes with both symptomatic and asymp-
An echocardiogram was normal. What would you recom- tomatic AV block, both symptoms and AV block resolved
mend now? with cessation of high intensity exercise training. There
were no adverse outcomes with long-term follow-up.46 This
condition is important to identify because the inappropriate
implantation of a permanent pacemaker in a young other-
J.D. Booker (*) and G.D. Veenhuyzen
Libin Cardiovascular Institute, University of Calgary, wise healthy person can have devastating complications
1403 29th St. NW, Calgary, Alberta T2N2T9, Canada over the course of a lifetime.
e-mail: booker.jeff@gmail.com;
george.veenhuyzen@albertahealthservices.ca

A. Natale et al. (eds.), Cardiac Electrophysiology, 647


DOI: 10.1007/978-1-84996-390-9_169, Springer-Verlag London Limited 2011
648 J.D. Booker and G.D. Veenhuyzen

Fig.169.1 (a, b) ECG from


24-h ambulatory monitoring

References 4. Zehender M, Meinertz T, Keul J, Just H. ECG variants and cardiac


arrhythmias in athletes: clinical relevance and prognostic impor-
tance. Am Heart J. 1990;119:1378-1391.
1. Bjrnstad H, Storstein L, Meen HD, Hals O. Ambulatory electro- 5. DiNardo-Ekery D, Abedin Z. High degree atrioventricular block
cardiographic findings in top athletes, athletic students and control in a marathoner with 5-year follow-up. Am Heart J. 1987;113:
subjects. Cardiology. 1994;84:42-50. 834-837.
2. Link MS, Homoud MK, Wang PJ, Estes NA 3rd. Cardiac arrhyth- 6. Meytes I, Kaplinsky E, Yahini JH, Hanne-Paparo N, Neufeld HN.
mias in the athlete. Cardiol Rev. 2001;9:21-30. Wenckebach A-V block: a frequent feature following heavy physi-
3. Talan DA, Bauernfeind RA, Ashley WW, Kanakis C Jr, Rosen KM. cal training. Am Heart J. 1975;90:426-430.
Twenty-four hour continuous ECG recordings in long-distance run-
ners. Chest. 1982;82:19-24.
Case 170

M. Eyman Mortada, Jasbir S. Sra, and Masood Akhtar

Case Summary Case Discussion

A 35-year-old male had a history of end-stage renal disease. The first rhythm strip shows initiation of the tachycardia. On
Three months prior to presentation, he received a renal trans- the left, the patients rhythm is sinus rhythm with first-degree
plant. Other medical problems include: diabetes mellitus, AV block and long QT (>500ms). During sinus rhythm, the
hypertension, and bipolar disorder. He had normal coronaries patient was hemodynamically stable (illustrated at the bot-
and left ventricular function by cardiac catheterization. tom of Fig. 170.1). The third beat is a premature ventricular
The patient was admitted to the hospital due to acute renal beat, which appears at the end of the T-wave without causing
failure from graft rejection, respiratory failure with intuba- dysarrhythmia. Another premature ventricular beat occurs
tion, and systemic infection. During the course of his hospi- after the ninth beat. It occurs at the end of a long QT interval
talization he was on fluconazole, levofloxacin, azithromycin, and initiates the polymorphic ventricular tachycardia. Note
haloperidol, valganciclovir, nephro-Vite, insulin, calcium the change of blood pressure during the tachycardia.
carbonate, para calcitrol, and epoetin alfa. After a few days, Baseline 12-lead ECG demonstrated sinus tachycar-
he suddenly became hemodynamically unstable and pre- diawith normal QT interval (QTc 450ms). The later 12-lead
sented with the following rhythm, which was terminated by ECG confirms the new onset of prolonged QT (QTc
cardioversion (Fig.170.1). 616ms).
His baseline 12-lead ECG a few days prior to the incident The causes of polymorphic ventricular tachycardia
is shown in Fig. 170.2. (PMVT) include: myocardial ischemia, bradyarrhythmia, or
His 12-lead ECG post cardioversion is shown in Fig.170.3. long QT. This patient was young, with normal coronaries by
At that time, his electrolytes were normal (potassium, 4.5; cardiac catheterization done a few months prior to this hos-
sodium, 140; magnesium, 2.1), his cardiac markers were pitalization. His 12-lead ECG is not suggestive of acute cor-
normal, and his head CT scan was normal. onary artery closure by thrombus or embolus (no ST elevation
What was the rhythm when he was hemodynamically was observed). Moreover, cardiac markers were normal, rul-
unstable? What is the etiology? ing out myocardial infarction. The patient is in tachycardia
rather than bradycardia, both on the rhythm strip and his
12-lead ECG. Therefore, the only possible cause for the
PMVT is long QT.
M.E. Mortada(*), J.S. Sra, and M. Akhtar
The causes of long QT include: congenital abnormalities,
Department of Electrophysiology, Aurora Cardiovascular Services,
Aurora Sinai/Aurora St. Lukes Medical Centers, University of metabolic disorders, bradyarrhythmia, drugs, and intracra-
Wisconsin School of Medicine and Public Health, nial disease. The only possible etiology for the long QT
2801 W. Kinnickinnic River Parkway, #777, Milwaukee, interval in this scenario is drugs, specifically, the combina-
WI 53215, USA
tion of fluconazole, levofloxacin, and haloperidol. Once the
e-mail: publishing4@aurora.org;
publishing1@aurora.org; drugs were discontinued, the QT prolongation disappeared
publishing@aurora.org and the symptoms relating to the PMVT subsided.1

A. Natale et al. (eds.), Cardiac Electrophysiology, 649


DOI: 10.1007/978-1-84996-390-9_170, Springer-Verlag London Limited 2011
650 M.E. Mortada et al.

Fig.170.1 After becoming hemodynamically unstable, the patient presented with this rhythm, which was terminated by cardioversion. (Continuous
rhythm strip of [from top to bottom]: Lead II, Lead V1, pulse oximetry SPO2, and arterial blood pressure)

Fig.170.2 Twelve-lead ECG a few days prior to the incident


Case 170 651

Fig.170.3 Twelve-lead ECG after cardioversion

Reference

1. A list of drugs causing long QT is available at: http://www.azcert.org/


medical-pros/drug-lists/pubMed-drug-list.cfm (Accessed November,
2008).

Case 171

M. Eyman Mortada, Jasbir S. Sra, and Masood Akhtar

Case Summary Additional rhythm strips are shown in Fig.171.2.


What is the most likely diagnosis?
A 62-year-old female was admitted to the hospital for elec-
tive cardiac catheterization secondary to abnormal stress test
and preserved left ventricular ejection fraction. The catheter- Case Discussion
ization revealed 80% stenosis of the mid left anterior descend-
ing (LAD) coronary artery, where an angioplasty and stenting The possibility exists that any patient receiving coronary
was performed. The patient did well until 5 h postprocedure, artery revascularization could experience dysrhythmia
at which time she had asymptomatic intermittent runs of within several hours postprocedure. This is due either to rep-
abnormal rhythm. The following rhythm was seen on the erfusion of the ischemic cardiac cells or dislodgement of
rhythm strips lasting 20 s, the terminal part is shown small particles from the atherosclerotic plaque into the
(Fig. 171.1). microvasculature, causing distal ischemia. Therefore, it is

Fig.171.1 The patient had


asymptomatic intermittent runs
of abnormal rhythm at five hours
post procedure. This rhythm, as
seen in Panels A and B, lasted 20
seconds; the terminal part is
shown. (Continuous rhythm strip
of three leads: [from top to
bottom] Leads II, III, and V1.)

M.E. Mortada (*), J.S. Sra,and M. Akhtar


Department of Electrophysiology, Aurora Cardiovascular Services,
Aurora Sinai/Aurora St. Lukes Medical Centers,University of
Wisconsin School of Medicine and Public Health,
2801 W. Kinnickinnic River Parkway, #777, Milwaukee,
WI 53215, USA
e-mail: publishing4@aurora.org

A. Natale et al. (eds.), Cardiac Electrophysiology, 653


DOI: 10.1007/978-1-84996-390-9_171, Springer-Verlag London Limited 2011
654 M.E. Mortada et al.

Fig.171.2 The dotted lines A


(Panel A) and arrows (Panel B)
demonstrate the sharp spikes
that represent the sinus QRS.
The rest are artifact

essential to monitor all patients who undergo revasculariza- irregular polymorphic wide complexes with regular march-
tion closely. ing narrow complexes or spikes. These narrow complexes
In this patient, wide complex rhythm developed 5 h post- march well with the sinus QRS complexes (marked by dotted
coronary angioplasty. Once the patient is stabilized, careful lines in the top and arrows in the bottom tracing). They are
analysis of the rhythm strips is essential prior to proceeding the clue for the diagnosis an artifact.
with management options. The second rhythm strip reveals
Index

A left bundle branch block, 617


Ablation catheter pace mapping, 619, 620
atrial activation, 69, 70 pericardial access, 619
circular mapping catheter, 69 repeated programmed stimulation and pacing, 617
3D mapping system, 69 sotalol, 617
posterior LA, 36, 37 tricuspid annulus, 617, 619
Ablation site amiodarone, 562
AT mapping, 29 baseline ECG, 560
conduction around mitral annulus, 29 diagnosis
early recurrence, AT, 29 bipolar right ventricular voltage map, 612613
entrainment, arrhythmia, 29 cardiac sarcoidosis, 613
extensive LA, 29 ECG during sinus rhythm, 611, 613
NavX activation map, 29, 32 endomyocardial biopsy, 613
PV isolation, 29 heterogeneous genetic disease, 613
restored sinus rhythm, 29, 32 LBBB VT, 613
RFAC and decapolar catheter insertion, CS monomorphic VT, 612613
anterior LA, 29, 30 sotalol, 611
complex fractionated electrograms site, 29, 31 diagnostic criteria, 562
Accessory pathway (AP), 57 differential diagnosis, 559
Acute coronary syndrome, revascularization, 337, 339 disease-causing genes, 561562
Alternating bundle branch block, 496 exercise stress test, 559, 560
Amiodarone-induced proarrhythmia, 568569. histologic features, 561
See also Long QT syndrome ICD
Amiodarone pulmonary toxicity burst pacing, 561
chest x-ray, 529 dual-chamber, 561
clinical presentations, 529530 recommended guidelines, 562
corticosteroid therapy, 530 inheritance pattern, 561562
diagnosis, 530 monomorphic VT with left bundle morphology, 561, 562
incidence, 529 prevalence, 562
prednisone, 530 RV angiogram, 559, 561
Anterior interventricular vein (AIV), 267269 RV endomyocardial biopsy, 559, 561
Anteroseptal accessory pathway, 21 Sotalol 160 mg, 561, 562
Antiarrhythmic medications, 51, 403 Aspirin, 3
Antidromic AVRT, 258 Asymptomatic atrial fibrillation
Antitachycardia pacing (ATP), 403 anticoagulation, 353
atrial arrhythmia (see Atrial arrhythmia) antitachycardia pacing, 353
intracardiac electrogram, 383, 385 pacemaker interrogation, 353, 354
termination, 481, 482 radiofrequency catheter ablation, 353, 355
Aortic sinuses of valsalva (ASOV), 267 sotalol therapy, 353
Arrhythmia, in post-cardiac transplant patients, 63 successfully treated episode, 353355
Arrhythmogenic right ventricular cardiomyopathy/ Athletic heart with left ventricular hypertrophy, 579
dysplasia (ARVC/D) Atrial arrhythmia
ablation ATP therapy
catheter contact, 619 atrial and ventricular marker channels, 357, 359
combined endo-and epicardial RF ablation approach, 619, 621 cycle lengths, 357
coronary angiography, left coronary system, 619, 620 interval plot, 357, 358
electroanatomic voltage map, 617619 symptomatic atrial arrhythmia, 357
endocardial entrainment and activation mapping, 617, 618 tachyarrhythmia
epicardial ICD infection, 617 biventricular pacing, 483
ICD shocks, 617 DDI pacing, 482
isoproterenol infusion induced VT, 619 device interrogation, 482, 483

A. Natale et al. (eds.), Cardiac Electrophysiology,


DOI: 10.1007/978-1-84996-390-9, Springer-Verlag London Limited 2011 655
656 Index

PR interval, 482 macro re-entrant AT, 25


treatment, 482 mapping area, 25
Atrial capture, 331. See also Ventricular pacing organized atrial activity, V1, 23
Atrial depolarization, 53 patients history, 39
Atrial ectopy permanent pacemaker
endocardial tracing, LSPV, 8 electrocautery, recommendations, 343, 344
P-wave morphology, 8 lead/connector problems, 343
reverse activation pattern, 8 oversensing (Vs), 343
Atrial fibrillation (AF), 256 ventricular electrograms, 344
ablation, 39 quadripolar catheter, CS, 23
ablation catheter positioning R-R intervals, 321
anterior and posterior, LA, 23, 25 septal-to-lateral activation sequence, 23, 24
anterior and septal, 23, 26 suppression, tachycardia
amiodarone pulmonary toxicity, 529530 atrium pacing, 375
area scanning, 24, 26 pacemaker-repetitive nonreentrant ventriculoatrial synchronous
arterial hypertension rhythm, 375
AF attacks, 565 premature ventricular captures, 375377
clinical examination, 565 programmable algorithm, 375, 377
DC cardioversion, 565 PVARP shortening, 375377
flecainide, 565, 566 VIP mode, 377
intermittent antiarrhythmic drug therapy, 565566 SVT with severe rheumatoid arthritis, 539, 541
pill-in-the-pocket approach, 565566 symptomatic paroxysmal
propafenone, 565, 566 ablation, left inferior pulmonary vein and mitral annulus,
ramipril 5 mg, 565 185, 188
athletes, accessory pathway atrial flutter, 185186
adrenergic surge, 633 catheter-based radiofrequency ablation, 185
1:1 antegrade conduction, 631 intracardiac recordings, 185, 186
asymptomatic ventricular preexcitation, 631 pulmonary vein isolation, 185
baseline ECG, short PR interval and delta wave, 631, 632 surgical maze procedure, 185
isoproterenol infusion, 631, 633 three-dimensional voltage map, 185, 187
lost antegrade conduction with atrial pacing, 631 symptomatic persistent
lost ventricular preexcitation, 631, 632 atrial flutter, 157
radiofrequency ablation, left lateral pathway, 631, 633 persistent left SVC ablation (see Persistent left superior vena cava)
resting vagal tone, 631 pulmonary vein isolation, 157158
risks associated, 633 quinidine and diltiazem, 157
atrial arrhythmia, 325 repeat ablation, 157
catheter positioning, 39 ventricular capture, 321
conduction block, roofline, 24 ventricular pacing, exceeding rate limit (see Ventricular pacing)
conventional quadripolar ablation catheter mapping, 24 ventricular rhythm, 477
coronary artery bypass surgery Atrial Fibrillation-Congestive Heart Failure Study (AF-CHF), 527
AF time course post-op, 604 Atrial fibrillation cycle length (AFCL)
amiodarone, 603 AF ablation
atorvastatin, 603 active vs. passive pattern, 16
beta blockers, 603 fractionated electrical activity, anterseptal LA, 16, 18
carvedilol, 603 impact, 16
incidence, 603 inferior left atrium, 16, 17
indepamide, 604 posterior part, LAA, 16, 17
omega-3 fish oil, 603 prolongation of, 16
patients medications vs. postoperative AF suppression, 604 roof, LA, 16, 17
propafenone, 603 custom analysis software, 16
QT prolongation, 604 electrogram annotation, 15, 16
trandolapril, 603 measurement, 15
focal re-entrant AT, 25 PV isolation, 15, 16
hypertension, 527528 Atrial flutter (AFL)
hypertrophic cardiomyopathy ablation
amiodarone, 577 ATP, 403
atenolol and warfarin, 577 catheter, 230
defibrillator implantation, 578 conducted atrial flutter, 403
echocardiography, 578 postcardiac surgery patients, 4
electrical cardioversion, 577 shock, 403404
electrocardiogram, 577 ablation catheter
risk factors, 578 left atrium, 12, 13
ICD discharges, 477 mitral valve, 12
lasso catheter, LSPV and RSPV, 39, 40 atrial EGM, 347, 348
12-lead EKG, 321, 322 atrial electrical signals, 325, 326
localized source, discrete point, 24 canine model, 11
Index 657

cardiac tracing, 51 bradycardia settings, 392


cavo-caval anastomosis, 63 cross talk
cavotricuspid isthmus, 230, 231 Safety Pacing, 393
conduction, 4 ventricular lead, 391, 393
dextrocardia ventricular sensed event filling, 393
catheter ablation, 123124 cross-talk window, 456
counterclockwise AFL, 123124 device settings, 351
duodecapolar catheter, 123 ICD status report, 456
duodecapolar recording integrated dual sensors, 351
donor atrium, 8384 lower rate limit
SVT, OHT patients, 83 12-lead ECG, 351, 352
earliest atrial activation, 51 verification, intracardiac electrogram, 352
electroanatomic mapping minute ventilation sensor, 351
atrio-atrial anastamosis, 84 telemetry recordings, 455
recurrent cavotricuspid isthmus dependent atrial ventricular sensing, 456, 457
flutter, 63, 64 Atrial premature depolarization (APD), 115, 119
right atrium, 5152 Atrial programmed stimulation, tachycardia, 57, 59
entrainment mapping, 230 Atrial stimulus delivery, 317
entrainment pacing, 51 differential diagnosis, 361, 362
left atrium, 12, 13 lead misconnection diagnosis, 361
right atrium, 12 QRS complex, 361
fluoroscopic RAO view, 83 ventricular depolarization, 361
intracardiac activation sequence, 83 ventricular pacing, 361
isthmus-dependent, 34 Atrial tachycardia (AT). See also Ectopic atrial tachycardia
12-lead ECG AF ablation, 36
atrial arrhythmia, 3 CARTO map
tachycardia, 229 electrical information, 127129
mode switch function, 325 inter-atrial connections, 127, 130
orthotopic heart transplantation intra-cardiac electrograms, 125127
activation map, 102 and propagation maps, 127128
intracardiac recording, 101 catheter ablation, AF
pacemaker function focal point tachycardia, 36
manufacturer-specific algorithms, 348 macro re-entry, 36
mode switch failure, 348 context of AF ablation, 36
programmed parameters, 347, 348 cycle length, 3536
PVAB and PVARP, 348 early recurrence, AF, 35
patients complaint, 11 EKG
patients history, 3 emergency room, 125
postpacing interval, 230 pre-vs. post-ablation p waves morphology, 131132
primary pulmonary hypertension electrogram-based ablation, RA1, 35
counterclockwise and clockwise AFL, 217218 localized re-entrant, 36
dyspnea, 217 mapped activity spanning, entire CL, 36
organized atrial activity with variable flutter wave past medical history, 125
morphology, 217 PV isolation, 35
positive and negative F waves in inferior leads, 219 radiofrequency ablation, 131
radiofrequency ablation, 218219 second ablation, 35
surface and intracardiac recordings, 218 spontaneous changes, electrical rhythm, 125, 126
right atrial dilation, 1112 three close posterior spot mapping, 36, 37
right side vs. left side, 12 Atrial Therapy Efficacy and Safety Trial (ATTEST), 353
SVT initiation, 236 Atriofascicular fiber, 522
symptomatic persistent atrial fibrillation Atrioventricular nodal reentrant tachycardia (AVNRT), 233, 234, 246,
CT scan, 158 371372
recurrence prevention, left SVC ablation, 157158 ablation site, 71, 77
TV-IVC isthmus atrial overdrive pacing, 189190
ablation catheter positioning, 4 atrial premature depolarization
RF application, 4, 5 cycle length, 182183
twelve-lead ECG, 11 RF energy delivery, 181
typical counterclockwise, 199200 atypical
Atrial high rate episode clinical presentation, 111, 112
DDI mode, 483 differential diagnoses, 111
stored electrograms intracardiac tracing, 111, 113
EGM and marker intervals, 319, 320 spontaneous premature ventricular capture, 111, 113
initiation, 320 surface ECG, 111, 112
Atrial pacing event ventricular entrainment, 111, 113
atrioventricular interval, 456 1:1 AV conduction, 189190
biventricular pacing, 456, 457 2:1 AV conduction
658 Index

adenosine IV bolus, 159 programmed atrial stimulation, 639640


2:1 AV conduction to 1:1 AV conduction, 162 programmed ventricular stimulation, 640
baseline 12-lead ECG, 160 preexcited tachycardia, 253
b-blockers/calcium channel blockers, 159 Atrioventricular reentry tachycardia (AVRT)
electrophysiology evaluation, 161 ablation catheter positioning, 47, 49
hemodynamic stability, 161 ablation of mitral annulus, 103, 106
His-Purkinje system block, 159 accessory pathway and SVT, 47
mechanism, 162 accessory pathway location, 506, 507
continuous atrial stimulation (see Continuous atrial stimulation) adenozine, verapamil and diltiazem, 506
coronary artery disease antidromic, 61
bypass surgery, clinical history, 555 AV nodal block, 506
cardioversion, 555 cryoablation, 506, 507
ECG and rhythm strip, 555, 556 eccentric atrial activation, 61
left bundle branch block, 555, 557 ECG, induced tachycardia, 47, 48
normal QRS, 555, 558 initial presentation, emergency room, 505
slow pathway ablation, 555 left-sided accessory pathway
distal coronary sinus, 81, 82 ECG, 147, 148
four surface ECG leads, 71, 72 RF energy application, 147, 149
ischemic heart disease with reduced LVEF VA block, 147, 150
first-degree AV block, 191, 192 long RP tachycardia, 144145
intracardiac recording, programmed atrial stimulation, 192 orthodromic (see Orthodromic AV reentry tachycardia)
junctional rhythm, ablation, 191, 193 QRS morphology, 103, 105
retrograde fast pathway, Kochs triangle, 191 right-sided AP, 47
left-sided conducting accessory pathway sinus rhythm with preexcitation, 505, 506
distal mapping catheter, 89 slowly conducting anterograde accessory pathway, 103
intracardiac recording at baseline, 88 AV block
normal PQ interval, 92 dual chamber pacing system implantation
programmed atrial stimulation, 92 pacemaker interrogation, 341
programmed ventricular stimulation, 89, 91 PVARP, 342
RF energy delivery, 9091 real time electrogram and marker channel, 341, 342
short PQ interval with ventricular pre-exitation, 87 telemetry
sinus rhythm, 90 AAI/DDD pacing mode, 335
SVT induced with catheter manipulation, 88 device settings, 335
mapping catheter, RAO and LAO, 71, 79 ventricular oversensing, 335, 336
pace mapping, Kochs triangle AV nodal conduction
anteroseptal region, 71, 76 body surface ECG, 349, 350
midseptal region, 71, 76 disease/block
posteroseptal region, 71, 77 carotid sinus massage, 583584
palpitations and dyspnea 2:1 conduction, 583, 584
initial presentation, emergency room, 513, 514 dyspnea, 583
initiation of tachycardia, 513, 514 His-Purkinje system, 583
isoproterenol, 513 Mobitz classification, 583
spontaneous 2:1 AV block, 513, 515 pacemaker, 583
programmed atrial stimulation (see Programmed atrial stimulation) vagal tone, 583
RF delivery, slow junctional beat induction, 71, 78 pacing system implantation
slow-fast type, 81 atrial pacing output, 349
accelerated junctional rhythm, 222 DOO mode, 349
atrial electrogram, 222 V pace event, 349
left-sided slow AV nodal pathway ablation, 221222
retrograde aortic approach, 221 B
right-sided vs. left-sided approach, 221 Bachmanns bundle, 52
typical Balloon angioplasty, pulmonary vein stenosis/occlusion, 135136
atrial premature depolarization, 115, 119 Beta blockade therapy, 327
diagnosis, 115, 116 Bradycardia pacing
diagnostic maneuver, 118 antitachycardia pacing, 451, 452
intracardiac recording, 115, 120 ICD programming, 451
left bundle branch block morphology, 115, 117 noncompetitive atrial pacing feature, 451, 452
RV pacing, 121 post-ventricular atrial refractory period, 451, 452
Wolff-Parkinson-White syndrome (WPW), 517518 sinus tachycardia, 451
Atrioventricular reciprocating tachycardia (AVRT) Bundle branch block, 81, 247
orthodromic Bundle branch reentry ventricular tachycardia, 589591
accessory pathways, 640 ablation, right bundle branch, 626, 627
atrial activation sequence, 640, 641 baseline ECG, 625
delta wave vector, 640 differential diagnosis, 626
fusion of retrograde conduction, 639640 entrainment, 626
initiation of tachycardia, 639 exercise testing, 625
Index 659

first-degree AV block and premature ventricular isoproterenol IV infusion, 71, 79


contraction, 625 non-sustained AVNRT induction, 71, 75
HisPurkinje potentials, 626627 Conventional mapping technique, 69
interfascicular reentry tachycardia, 627628 Coronary artery vasculopathy, 63
scar-related reentry, 628 Coronary sinus (CS) activation, 47
transient infranodal block, 627, 628 Counterclockwise atrial flutter, 84
Cryoablation
C anteroseptal accessory pathway, 506507
Cardiac arrest junctional tachycardia, 523, 525
QTc interval prolongation vs. radiofrequency ablation, 153
chronotropic agents, 487 septal atrial tachycardia, 546, 549
drug induced torsade de pointes, 487 WolffParkinsonWhite syndrome, 153155
intravenous magnesium sulfate, 487 Cumulative stenosis index (CSI), 136
methadone and moxifloxacin, 487 Custom analysis software, 16
sinus arrhythmia, 487, 488 Cycle length (CL), 81
ventricular bigeminy, 487, 488
ventricular fibrillation, 489 D
Cardiac catheterization, 63, 83 Decapolar catheter, CS, 7, 29, 35
Cardiac CompassT trend, 433, 436 Defibrillation threshold test, 373
Cardiac resynchronization pacing, 472 Delta waves, 19
patient related causes, 471 Dextrocardia, 123124
programmed parameters, 471 Diaphragmatic myopotential oversensing, 380
system related causes, 471 Dilated cardiomyopathy (DCM)
T wave oversensing, 471, 472 autosomal dominant DCM, 598
ventricular oversensing, 471 baseline ECG, 597
Cardiac sarcoidosis ICD
diagnostic criteria, 499 dual chamber defibrillator, 597
differential diagnosis, 497 interrogation, 597598
granuloma, 497498 lamin A/C mutation, 598
granulomatous infiltration, 499 ventricular dysfunction, 597
therapy, 499 Distal and proximal bipoles, conduction delay, 23
ventricular dysrhythmias, 498499 DOO mode, 349
ventricular tachyarrhythmias, 498499 Double tachycardia
Cardiac tarcing, 51 arrhythmia termination, 411, 414
Cardiomyopathy atrial and ventricular EGMs, 411, 412
ischemic, 51 device settings, 411
R-waves, 465 first shock, 411
stored EGM, 465, 466 interval plot, 411, 413
T-wave oversensing Dual chamber pacing system implantation, 341342
fifth ventricular beat, 466 Duodecapolar catheter, 83
real-time EGM, 465, 466
sensitivity, 465, 467 E
viral, 83 Ebsteins anomaly, 595596
CareLink remote monitoring network Ectopic atrial tachycardia
baseline artifact, 432 carvedilol and enalapril, 571
dislodgement, 432 catheter ablation, 573
electrograms transmitted remotely, 431 first degree AV block, 571, 572
far-field tracings, 432 initial hospitalization, 572
real-time electrogram post-shock, 432 initial presentation, 571
Carotid sinus hypersensitivity, 593594 vs. sinus tachycardia, 573
CARTO activation map Electrocautery, recommendations, 343, 344
electrical information, 127129 Electromagnetic interference (EMI), 389390
inter-atrial connections, 127, 130 characteristics, 447
and intra-cardiac electrograms, 127 extra-cardiac noise oversensing, 447, 449
and propagation maps, 127, 128 integrated bipolar leads, 447
Cavotricuspid isthmus, 63, 230, 231 myopotential oversensing, 447, 449
Complex fractionated atrial electrogram (CFAE), 15 pectoralis oversensing, 447, 449
Concealed retrograde septal accessory pathway shock delivery, ICD, 448
vs. AV nodal pathway, 531 Epicardial ventricular tachycardia, 586
ventricular pacing, highly symptomatic SVT
ablation site, 531, 533 F
ECGs, 531, 533 FlashbackT intervals, 453, 454
initiation, 533 Flecainide, 39
Para-Hisian pacing, 531, 534 Focal atrial tachycardia, 36
ventricular pacing, 531 12-lead ECG, 3
Continuous atrial stimulation pregnancy
660 Index

catheter ablation, left atrium, 575 J


electroanatomic map, 575576 Junctional tachycardia
electrocardiogram, 575, 576 cryoablation, 523, 525
mechanism of tachycardia, 575 nonparoxysmal, 523
quinidine and metoprolol, 575 VA block, 523

H L
Heart failure Lasso catheter, 39
AV block Lead fracture, 387388
cardiac sarcoidosis (see Cardiac sarcoidosis) Left anterior descending (LAD) coronary artery
differential diagnosis, 497 angioplasty and stenting, 653654
first-degree AV block and left bundle branch block, asymptomatic intermittent runs, abnormal rhythm, 653
497, 498 dysrhythmia, 653
high grade AV block, 497, 498 sinus QRS complex, 654
refractory heart failure (see Refractory heart failure) Left atrial myxoma, 151152
His bundle potential, 44 Left atrial tachycardia
Hyperkeratosis, 561 ablation of the cavotricuspid isthmus, 199200
Hypertension, 39, 71 entrainment mapping, 200
metoprolol, 527528 organized atrial rhythm with irregular ventricular
patients age and history, 527 rhythm, 199
rate vs. rhythm control, 527 typical counterclockwise activation of right atrium, 199200
warfarin therapy, 527 Left coronary cusp (LCC), 267, 268
Hyperthyroidism with severe rheumatoid arthritis, Left-sided septal slow pathway ablation
539, 542 mitral annulus, 221
Hypertrophic cardiomyopathy (HCM) RF current application, junctional rhythm acceleration,
amiodarone, 577 221, 222
atenolol and warfarin, 577 site identification, 221, 222
clinical presentation, 502 Left ventricular ejection fraction (LVEF), 39, 51
defibrillator implantation, 578 Left ventricular hypertrophy, 39
diagnostic work up, 502 Left ventricular non-compaction (LVNC)
ECG, 501 cardiac magnetic resonance, diastole, 492
echocardiography, 578 clinical presentations, 492
electrical cardioversion, 577 ECG, 491
electrocardiogram, 577 inheritance pattern, 492
parasternel long axis image, 502 neuromuscular disorders, 492
physical findings, 502 prevalence, 492
risk stratification, 502503, 578 prognosis, 492493
treatment, 503 survival rate, 493
transthoracic echocardiogram, 491, 492
I treatment recommendations, 493
ICD-detected VT Linear lesions, 29
amiodarone, 453 Long-QT syndrome (LQTS)
flashbackT intervals, 453, 454 atenolol 50 mg, 635
interval plot, 453, 454 baseline ECG, sinus bradycardia, 635, 636
stored electrogram, 453, 454 beta-blocker therapy and ICD implantation, 637
Implantable cardioverter-defibrillator therapy, 275 drug induced torsades de pointes
Induced tachycardia, 47, 48 amiodarone-induced proarrhythmia, 568569
Inferior vena cava (IVC) baseline ECG, 568
isthmus-dependent atrial flutter, 34, 63 gene mutation associated with, 567568
tricuspid isthmus ablation, 84 microvolt T-wave alternans test, 567569
InSync SentryT 7299 CRT-D defibrillator implantation reduced repolarization reserve, 567, 568
provocative maneuvers, 443, 445 resting ECG, 567, 569
shock risk factors for, 569
lead impedance values, 444 tachycardia initiation, 568
telemetry, 443, 444 visible TWA, 568
VT/VF episode report, 443, 444 exercise testing, 636
Inter-atrial connections gene mutations, 635
Bachmann Bundle, 130 genetic testing, 637
fossa ovalis, 130 QTc interval, 635, 637
passive activation, 127, 130 risk for, 637
Interfascicular reentry tachycardia, 627628 symptoms, 635
Intermittent AV block, 495496 telemetry monitoring
Intra-His conduction block, 496 abrupt awakening, 635, 637
Intrinsic atrial activation, 317 short bursts of polymorphic ventricular tachycardia,
Ischemic cardiomyopathy, 51 635636
Isoproterenol, 39 sinus with ventricular bigeminy, 635636
Index 661

M His spikes, 523, 524


Mahaim tract tachycardia, 522 junctional tachycardia, 523, 525
Medtronic Kappa 400 pacemaker, 351, 352 NavX activation map, 29, 32
Metal ion oxidation (MIO) induced damage, 416 NavX fractionation map, 15, 16
Minute ventilation (MV) Naxos disease, 561562
impedance measurements, 363, 367 Noise oversensing, RV lead
sensor-driven tachycardia, 363 arrhythmia logbook, 399, 400
Mitral annulus, conduction, 29 bradycardia, ventricular pacing inhibition, 399
Mitral regurgitation diaphragmatic myopotentials, 399
pacemaker syndrome, 327 shock delivery, 401
ventricular pacing, reprogramming tachycardia zone redetection, 402
abnormal cardiac activation, 370 Noncompetitive atrial pacing (NCAP) feature, 451, 452
biventricular pacing, 370 Nonparoxysmal junctional tachycardia, 523
echocardiogram, 369 Non-ventricular tachyarrhythmias, ICD discharges, 477, 478
heart failure, 369370 NSVT episode, 287, 289
Monomorphic PVCs, 295, 296
Monomorphic ventricular tachycardia (MMVT) O
Fidelis failures, 459, 462 Orthodromic AV reentry tachycardia, 246
high-voltage impedance, 459 diagnosis, 140, 141
PVCs radiofrequency ablation, mitral valve annulus, 140
activation and pacemapping, 295, 297 WolffParkinsonWhite syndrome (see WolffParkinsonWhite
bipolar voltage map, 295, 297 syndrome)
monomorphic PVCs, 295, 296 Orthotopic heart transplantation (OHT), 63, 83
VT initiation, 295, 296 Outflow-type tachycardia, 267
R wave amplitude and impedance measurements, 461
stored electrogram, 459, 460 P
Multielectrode circular mapping catheter, 69 Pacemaker
Multiple shocks artifacts, 317318
etiologies, 416 pacemaker syndrome
first episode, 416 baseline rhythm strip, 329
RV EGM, 417, 418 pathophysiology, 327
second episode, 416, 417 prevalence, 327
VT/VF therapy delivery, 416417 single-chamber ventricular pacemaker, 327, 328
Myotonic muscular dystrophy vague symptoms, 327
autosomal dominant disorder, 599 settings, 317
bundle branch reentry ventricular tachycardia, 601 Pacemaker and Beta-Blocker Therapy after Myocardial Infarction
first degree AV conduction delay, 599 (PACE-MI) Trial, 327328
baseline ECG, 600 Pacemaker atrial refractory period (PVARP)
dual chamber pacemaker, 601 atrial fibrillation suppression, 375
evaluation and monitoring, 601 atrial sensed signals, 348
Holter monitor strip, 600 AV block, 342
monomorphic ventricular tachycardia, 601 bradycardia pacing, 451
noncardiac and cardiac manifestations, 599 premature atrial contraction, 371
prevalence, 599 tachycardia, 375, 376
supraventricular tachyarrhythmia, 599, 601 ventricular pacing, 345346
Pacemaker-repetitive nonreentrant ventriculoatrial synchronous
N rhythm, 375, 377
Narrow complex tachycardia, 41, 57, 58, 71 Palpitation, 41, 57, 65, 71
emergency room Para-Hisian pacing, 2021, 95, 96, 250251
ablation, septal atrial tachycardia, 546, 549 Paroxysmal atrial fibrillation (PAF), 39
adenosine, 545546 ablation
AV dissociation, 546, 548 dissociation, venous potential, 9, 10
baseline electrocardiogram, 546 earliest venous potential, 9
cryoablation, 546, 549 left inferior pulmonary vein, 89
ECG, 545 in right veins, 9
initiation with single atrial extrastimulus, 545546 amiodarone, 608
ventricular pacing, 546, 547 arrhythmia recurrence, 8
QRS cordarone, 200 mg, 608
ablation of AP, 96, 99 disopyramide, 608
left bundle branch block, 96, 98 drug combination therapy, 609610
para-Hisian pacing, 95, 96 ECG
right bundle branch block, 96, 97 during AF with RBBB, 608
ventricular pacing, 95 during sinus rhythm, 607
VA block endocardial tracing, LSPV, 8
carvedilol, ECG, 523, 524 events and therapies, 609
differential diagnosis, 523 flecainide, 609
662 Index

monomorphic atrial ectopy, 7 NSVT episode, 287, 289


omega-3 fish oil, 608 pre-QRS activation time, 267, 268
propafenone, 608609 Purkinje system, 279, 281
reverse activation sequence, 8 qrS pattern, 271
rheumatic mitral insufficiency, 608 right bundle, 281
Paroxysmal supraventricular tachycardia (PSVT) RV outflow tract pacemaps, 267, 268
adenosine trigeminal pattern, 287, 288
after administration, 509510 ventricular bigeminy, 12-lead ECG, 271, 272
atrial fibrillation induction, 509, 510 ventricular outflow tract, 271
effect, EP study, 510 ventricular unipolar and bipolar activation mapping,
initial presentation, emergency room, 509 271, 272
recurrent, 201202 Programmed atrial stimulation
Permanent form of reciprocating tachycardia (PJRT), 552 atrial ERP, 71, 74
12-lead ECG, 133 coupling intervals, 71, 73
long RP tachycardia, 144145 isoproterenol IV infusion
posterior septum, accessory pathway, 133134 AVNRT induction, 71, 75
retrograde activation, lateral mitral annulus, 134 slow anterograde conduction, 71, 78
Persistent left superior vena cava, 157158 retrograde conduction, 71, 74
Polymorphic ventricular tachycardia (PMVT), 279, 280, 287 Programmed ventricular stimulation
end-stage renal disease atrial activation, 640
baseline ECG, 649, 650 AVNRT
cardioversion, 649, 650 decremental VA conduction, 91
causes of, 649 SVT induction, 89
ECG post cardioversion, 649, 651 inducible sustained VT, 617
first-degree AV block, 649 response of, 263
nonsustained and sustained, 280 tachycardia, 57, 59
sinus bradycardia, 635, 636 wide QRS tachycardia, 621, 625
torsades de pointes, 567 Proton pump inhibitor (PPI), 29
ventricular fibrillation, 287 Provocative maneuvers, 443, 445
Polyurethane insulation, ICD, 416 Pseudopseudo-fusion complex, 331, 332
Post-pacing interval (PPI), 312 Pulmonary toxicity. See Amiodarone pulmonary toxicity
Post-pacing T-wave oversensing, 429 Pulmonary vein isolation (PVI), 35
Post-ventricular atrial blanking period (PVAB), 348 atrial fibrillation, 23, 24, 29, 35
Post-ventricular atrial refractory period (PVARP), 345346, 348, pulmonary vein stenosis/occlusion, 135
375377, 451, 452 symptomatic persistent atrial fibrillation, 157
Preexcited tachycardia Pulmonary vein (PV) stenosis/occlusion
accessory pathway, 253 asymptomatic patient, 136
atrial fibrillation, 256 balloon angioplasty and stenting, 135136
atrial stimulation, 255 CT scan/MRI, 135, 136
baseline ECG, 254 cumulative stenosis index, 136
baseline interval, 254 dyspnea, 135
differential diagnosis, 253 incidence, 135136
post-ablation ECG, 261 misdiagnosis, 136
post-ablation intervals, 260 pathogenesis, 136
RF energy, 259 pulmonary artery dilatation, 136
shortest preexcited RR interval, 256 pulmonary hypertension, 135, 136
VA block, 257 PV isolation, 135
ventricular pacing, 257 ventilation/perfusion scan, 136
Premature atrial capture, 140, 141 P wave oversensing
Premature atrial complex, 81 accelerated junctional rhythm, 337
Premature ventricular capture, 111, 113, 140, 141, 375377 RV lead positioning, 337
Premature ventricular contractions (PVC)
ablation, premature Purkinje potentials, 287, 289 Q
ablation site, 271, 273 QRS complex, 21
AIV pacemaps, 267269 Quadripolar catheter, 23
cardiomyopathy
ablation, 284 R
aortic cusp, pacemapping, 284, 285 Radiofrequency ablation catheter (RFAC), 30
intracardiac echo, aortic valve, 284 Radiofrequency distal (RFD), 47
spontaneous VPC vs. pacemap, 285 Radiofrequency (RF) energy application
VPC, 12-lead ECG, 283, 284 anteroseptal pathway, 21
left bundle, 280 arrhythmia, 25, 36
left coronary cusp, 267, 268 intracardiac recording, ablation site, 45
LV activation mapping, 287, 288 TV-IVC area, 5
morphology, 287 Recurrent PSVT, slow pathway ablation procedure, 233
nonsustained and sustained polymorphic VT, 279, 280 Recurrent tachycardia, 517518
Index 663

Refractory heart failure Sinus bradycardia


ablation, left atrial appendage, 552553 ambulatory ECG monitoring, 648
ACE inhibitor, diuretic, and beta blocker, AV Wenckebach, 647
551, 552 symptomatic AV block, 647
cardiomegaly and pulmonary congestion, 551 Sinus tachycardia, 442
dyspnea, 551 Automatic Sensitivity ControlT, 441
physical examination, initial presentation, 551 morphology mismatch, 441
resolution of tachycardia induced, 552 template verification, 441
sinus tachycardia, initial diagnosis of, 551, 552 Situs inversus, 123124
tachycardia during sleep, 552 Slow pathway ablation procedure
Resting bradycardia, 647 inferior vena cava, 233234
Retrograde atrial activation sequence recurrent PSVT, 233
induced SVT, 20 vascular access, 233
loss of His bundle capture, 21 Sotalol, 39
Para-Hisian pacing, 2021 Spontaneous rhythm T-wave oversensing, 429430
RF application, 20 Steinerts disease. See Myotonic muscular dystrophy
second SVT, 20 Stimulus-QRS (S-QRS) interval, 313
sinus rhythm, ventricular pre-excitation, 19 Sudden cardiac death, 502, 503
ventricular pacing, 19, 20 Supraventricular tachycardia (SVT). See also Atrioventricular nodal
Retrograde conduction, ventricular programmed reentrant tachycardia (AVNRT)
stimulation ablation, left-sided AP, 65, 6768
atrioventricular node, first atrial beat, 264 adenosine response, 140
ECG, 263 antitachycardic pacing, intracardiac electrogram, 383, 385
electrical activation, 264 atrial activation sequence, 65
fourth ventricular complex, 264265 atrial sensed events, 406
HV duration, 265 AV association, 383
left free-wall accessory pathway, 264 A-waves, 81
sinus node high right atrium, 263, 264 device parameters, 384
sixth ventricular complex, 265 duodecapolar catheter, 83, 84
Right anterior oblique (RAO), 83 echocardiography evaluation, 41
Right bundle branch block (RBBB), 57 episode list, 384
Right superior pulmonary vein (RSPV) four surface ECG leads, 41, 43, 65, 66
intracardiac activation, 108, 110 induced, 20
isolation and cavotricuspid isthmus ablation, 108 intracardiac recording, 40, 43
Right-to left atrial conduction block, 51, 52 sinus rhythm restoration, 65, 67
RV lead, nonphysiologic noise tachycardia termination, 65, 67
diaphragmatic myopotentials, oversensing, 419 ventricular stimulation, 65, 68
first episode initiation, 420 12-lead ECG
ICD atrial tachycardia, 41, 42
helix stabilizing post, 421 heart rate, 71, 72
pin/device header connection, 421 sinus rhythm, 71
VT zone episode detection, 419, 421 long RP tachycardia
shock electrograms, 419, 420 AVRT, 144145
RV outflow tract (RVOT) pacemaps, 267, 268 intracardiac recordings, 143, 144
12-lead ECG, 143
S VPC, 143144
Safety Pacing, 393 mental retardation, 53
Sarcoidosis granuloma, 497498 narrow complex tachycardia, 41
Scar-related VT normal sinus rhythm, 41
bipolar voltage map, 305 orthotopic heart transplantation
common isthmus, 303 activation map, 102
first VT, 303, 304 intracardiac recording, 101
second VT, 303, 304 premature atrial complex, 81
septal scar, 303, 305 recurrent
VT morphologies, 303 intracardiac tracing during ablation, 86
Sensor-driven tachycardia, 363 left bundle branch block, 85
Septal atrial tachycardia, 546, 549 orthodromic reentrant tachycardia, 85
Shock recurrent paroxysmal
atrial and ventricular channels, 389 differential diagnosis, 201
EGM characteristics, 389 intracardiac tracing, 202
electromagnetic interference, 389390 termination of tachycardia with spontaneous AV block,
episode electrogram, 387 201202
lead fracture severe rheumatoid arthritis
impedance rise, 387 ablation, coronary sinus os, 543544
lead extraction, 388 adenosine, 539
lead performance report, 388 after orthopedic surgery, 539
664 Index

atrial fibrillation, 539, 541 AVRT, 235


baseline recordings, 539540 A-waves, CS, 81
fast pathway ablation, 541 bundle branch block, 247
flecainide, 539 catheter positions, fluoroscopic left anterior oblique views,
hyperthyroidism, 539 242, 243
left sided AV nodal fibers, 543 cavotricuspid isthmus, 230, 231, 243
pseudo VAAV response, 542 constant VA interval, 53
slow vs. fast pathway ablation, 541 device programmed parameter, 363, 365
tachycardia initiation, 540541 differential diagnosis, 481
ventricular pacing, 540, 542 3D mapping system, 69
ventricular premature stimulus, 540, 543 electroanatomical mapping
verapamil, 539, 540, 542 arrhythmia origin, 41, 43
slow pathway ablation CARTO, 41, 44
mitral annulus, 221 electroanatomic mapping, 230
RF current application, junctional rhythm acceleration, induced, 47, 48, 57, 58
221, 222 induced heart failure (see Refractory heart failure)
site identification, 221, 222 induced narrow complex, 65
symptomatic, 199200 induced SVT, 248
1:1 tachycardia intracardiac electrogram, 481, 482
beta-blocker therapy, 409 intracardiac recordings, 81, 82
1:1 conduction, 407 intracardiac signals, 249
reconfirmation algorithm, 409 intracardiac tracings, 242, 243
twelve-lead ECG, 65, 66 laboratory considerations
ventricular extrastimuli, 61 arrhythmia inducibility, sedative medications, 299
VPD precordial morphology, 299, 300
activation sequence, 195, 197 pre-QRS activation time, 299
initiation, 195, 196 PVC morphology, 299, 301
ventricular pacing, 195, 198 right inferior axis ventricular ectopy, 300
VT zone and shock, 383 12-lead EKG, 363, 364
Symptomatic atrial arrhythmia, 357 macroreentrant right atrial arrhythmia, 243
Syncope. See also Vasovagal syncope minute ventilation
carotid sinus hypersensitivity impedance measurements, 363, 367
carotid sinus massage response, 594 sensor-driven tachycardia, 363
dual chamber pacemaker, 594 multipolar deflectable catheter, 242, 243
ECG, 593 narrow complex, 41, 57, 58, 71
elderly patient, 594 orthodromic and antidromic
hypertension and hyperlipidemia, 593 ablation site, 173174
device interval plot, 395 atrial pacing, 165
device reprogramming, 397 baseline intracardiac recordings, 166
double counting, 395, 397 bypass tract, 163
stored electrograms, 395, 396 12-lead resting ECG, 164
programmed atrial stimulation, 168169, 172
T programmed ventricular stimulation, 170171, 177178
Tachyarrhythmia radiofrequency delivery, 175176
adenosine infusion, AV block, 107, 108 RAO and LAO views, proximal coronary sinus, 180
cavotricuspid isthmus ablation, 108 ventricular pacing, 167
ECG palpitation, 57, 58
intracardiac activation, 108110 Parahisian pacing, 250251
isoproterenol infusion, 108, 109 premature atrial beat, sinus rhythm, 53, 55
falling, VT (See VT) programmed ventricular stimulation, 57, 59
recurrent palpitations, 107 P wave morphology, 235, 239
right superior pulmonary vein recurrent, verapamil, 147
right superior pulmonary vein isolation, 108 RF application, 41, 45
Tachycardia. See also Atrial tachycardia; rhythm change, 57, 58
Ventricular tachycardia right bundle branch block, 81
ablation site, 41, 45 sinus rhythm, pacemaker interrogation
activation sequence, 61 AV reentry tachycardia, 372373
anti-tachycardia pacing (ATP) termination, 481, 482 premature atrial contraction, 371
AP-mediated, 61 RP interval, 371
atrial extrastimuli, 371, 372 slow/fast AVNRT, 81
atrial programmed stimulation, 57, 59 surface electrocardiogram, 241
atrial S2, 259 SVT induction, single atrial extrastimulation, 53, 54
AVNRT mechanism three ventricular extrastimulation, 57, 60
SVT, 223, 224 two ventricular extrastimulation, 57, 60
tachycardia initiation, 225226 ventricular entrainment, 371, 372
typical AVNRT, 227 ventricular overdrive pacing
Index 665

atrial tachycardia, 246 stored electrogram, 439, 440


AV nodal dependent tachycardia, 246 SVT, 439
cessation, 245 VF detection, 439
supraventricular tachycardia, 246 VT with 1:1 VA conduction, 439
V pacing, 235, 238239 Ventricular fibrillation (VF)
VPC delivery, 237 cardiac arrest, 489
VT induction, 61 ICD implantation
wide complex, 57, 81, 82, 258 chest X-ray, biventricular system, 374
1:1 Tachycardia defibrillation threshold testing, 373
episode electrograms and marker channels, 408 device settings, 373
shock, interval plot, 407 multiple maximum energy shock, 373, 374
supraventricular tachycardia (SVT) polymorphic ventricular tachycardia, 287
beta-blocker therapy, 409 ventricular electrogram morphology discriminator, 439
1:1 conduction, 407 Ventricular Intrinsic Preference (VIP) mode, 377
reconfirmation algorithm, 409 Ventricular long term histogram, 328
Telemetry findings Ventricular pacing, 19, 20, 327
atrial capture, 331 atrial capture, new lead implantation
AV block bipolar and integrated lead designs, 424
AAI/DDD pacing mode, 335 chest x-ray, 424
device settings, 335 lead characteristics, 424
ventricular oversensing, 335, 336 pacing threshold test, 423
AV delay, 391, 393 tachycardia event, 424, 425
posteroanterior and lateral chest radiographs, 331, 332 atrial stimulus delivery, 361
pseudopseudo-fusion complex, 331, 332 exceeding rate limit, 473
QRS complex, 391 non-tracking mode, 345
reprogramming, 332333 PMT termination, 346
safety pacing, 331332 post-ventricular atrial refractory period, 345346
spontaneous junctional complex, 332 reprogramming, mitral regurgitation
Tetralogy of Fallot (TOF) abnormal cardiac activation, 370
ablation biventricular pacing, 370
activation mapping and entrainment, 623 echocardiogram, 369
entrainment of VT, 621622 heart failure, 369370
pace mapping, 622623 retrograde atrial activation, 345
potential isthmuses, reentry, 623 telemetry recording, 474
surface ECG during sinus rhythm and ventricular upper tracking rate, 345
tachycardia, 621 ventricular rate regularization
tachycardia mechanism, 622623 device reprogramming, 474
ventricular septal defect, 614 telemetry recording, 474475
wide QRS tachycardia V-V cycle length variation, 473474
baseline ECG, 614 Ventricular premature contraction (VPC), 319320
ICD implantation, 615 first ablation, 275, 277
left bundle branch block, 614, 615 LVEF, cardioverter-defibrillator insertion, 275
monomorphic VT with programmed ventricular LVOT, 275
stimulation, 614 multiple ablations, temperature and power limitation,
sudden death, 614615 275, 276
Three ventricular extrastimuli, CL, 61 pacemapping, 275
Trending Plot/Sensor Replay, 363, 366 presystolic activation, 275, 276
Tricuspid valve-inferior vena cava (TV-IVC) isthmus, 34 Ventricular rate/sense lead, 381
Typical AVNRT, 223, 227 diaphragmatic myopotential oversensing, 380
header torque wrench insertion, 380
V vs. shock coil EGMs, 380
VA conduction, atrial rhythm dependent on, 53 stored EGM, 379, 380
Vasovagal syncope V sensed event
clinical diagnosis, 579 P wave oversensing, 379381
cool down exercise, 579 QRS complexes, 379
external loop recorder, 579560 Ventricular refractory period (VRP), 321, 323
implanted loop recorder, 561, 579 Ventricular sensed events
permanent pacemaker therapy, 561 EGM and marker channel, 339
resting ECG, 560, 579 programmed parameters, 337, 338
venous return, 579 P wave oversensing
Ventricular capture, 321 accelerated junctional rhythm, 337
Ventricular depolarization, 53 RV lead positioning, 337
Ventricular dysrhythmias, 498499 ventricular histogram, 337, 338
Ventricular electrogram morphology discriminator (MDT) Ventricular sensing, 319
diagnostic and therapeutic considerations, 439 Ventricular stimulus, 317
ICD reprogramming, 439 Ventricular tachycardia (VT), 57
666 Index

adenosine response, 140 device interval plot, 395


biventricular voltage maps, 291, 292 device reprogramming, 397
bundle branch reentry double counting, 395, 397
ablation, right/left bundle, 589, 591 stored electrograms, 395, 396
aortic valve replacement, 589 tachyarrhythmia falling
electrical cardioversion, 589, 590 atrial and ventricular EGMs, 405406
His-Purkinje system, 589 atrial sensed events, 405
ICD and resynchronization therapy, 589 device settings, 405
intracardiac electrograms, 589, 591 differential diagnosis, 406
12-lead ECG, 589, 590 therapeutic options, 433
macroreentrant circuit, 589 ventricular vs. atrial flutter morphology, 437
Cardiac Compass trend, 433, 436 ventriculoatrial, 57
ECG and intracardiac recordings Ventriculo-atrial (VA) conduction, 57, 9596
pace termination, 311 Ventriculoatrial (VA) interval, 53
postpacing interval (PPI), 312 Viral cardiomyopathy, 83
electroanatomical map VT/VF episode list, 428
entrainment response, 311, 312 T-wave oversensing
isthmus site, mapping and targeting, 314 Medtronic defibrillators, reprogramming sensitivity, 430
stimulus-QRS (S-QRS) interval, 313 post-pacing, 429
ICD reprogramming, 433 spontaneous rhythm, 429430
linear lesions, 292293 V-V intervals, 429
mid-diastolic potentials
cycle length, 307, 309 W
ECG and intracardiac recordings, 307, 308 Wenckebach AV block, 53
entrainment response, 307, 308 Wide complex tachycardia, 57, 81, 82, 258
non-propagated extrastimulus, 307 adenosine, 140, 522
misdiagnosis, sinus tachycardia ambulatory event recorder, 103, 104
1:1 AV relationship and cycle length, 469 antiarrhythmic agent, slow ventricular response, 643
dual chamber onset algorithm, 469 atriofascicular pathways, 522
ICD programming, 469, 470 atrioventricular conduction system, 643
managed ventricular pacing, 469 AV nodal reentry tachycardia, 140, 141
morphology, 291 at 115 beats per minute, 151
multiple shocks catheter ablation, left-sided accessory pathway, 644645
electrical noise, 416 ECG, emergency room, 644
etiologies, 416 ECG post-cardioversion, 644, 645
first episode, 416 electrical cardioversion, 643644
ICD, polyurethane insulation, 416 left atrial myxoma
rate/sense and marker channels, atriotomy scar, 152
415416 intracardiac recordings, 152
second episode, 416, 417 surface electrocardiogram, 151
therapy delivery, 416417 left bundle branch block, 521
myocardial infarction mild hypertension
amiodarone, 585 AV nodal blocking agents, 512
cardioversion, 585 flecainide, 511512
ECG, emergency room, 585 initial presentation, emergency room, 511
endocardial ablation, 585586 lisonopril, 511
epicardial VT, 586587 metoprolol, 512
substrate based ablation, 586 paroxysmal atrial fibrillation, 511512
overdrive pacing, 311314 proarrhythmia, 512
vs. pacemap morphology, 292, 293 orthodromic AV reentry tachycardia
programmed settings, 434 diagnosis, 140, 141
PVCs radiofrequency ablation, mitral valve annulus, 140
activation and pacemapping, 295, 297 preexcited atrial fibrillation degeneration, 644
bipolar voltage map, 295, 297 premature atrial capture, 140, 141
monomorphic PVCs, 295, 296 premature ventricular capture, 140, 141
VT initiation, 295, 296 QRS morphology, 103, 105
QRS fusion, 312313 sinus rhythm, 140
shock, 395397 stable vitals and abnormal rhythm, 139
antiarrhythmic medications, 403 ventricular vs. supraventricular tachycardia, 139140
ATP, 403 Wide QRS complex tachycardia with left bundle branch block
atrial flutter ablation, 403404 antidromic tachycardia with retrograde conduction,
conducted atrial flutter, 403 210211
sinus rhythm QRS complex, 291 arrhythmia circuit, 215
stored electrograms, 433, 435 atrial stimulation
substrate-based ablation, 291292 atrial pacing, 208
syncope ventricular preexcitation, 207
Index 667

atriofascicular pathway, 210 Ebsteins anomaly


induced tachycardia accessory pathways, 596
12-lead ECG, 210 baseline 12-lead ECG, 595
programmed atrial stimulation, 209 catheter/surgical ablation, 596
intracardiac recordings at baseline, 204 tricuspid valve incompetence, 596
normal PR interval, 203 ECG, 535536
programmed atrial stimulation family history, 535
A-H interval and H-V interval, 205 left ventricular hypertrophy, 535
atrioventricular node duality, 206 pacemakers, 535
radiofrequency ablation, 214 PRKAG2 gene defect, 535
tricuspid annulus mapping during sinus rhythm recurrent tachycardia
mechanical block, 213 AV nodal reentry and accessory pathway,
M potential, 212 519
WolffParkinsonWhite syndrome (WPW), 19 initiation, 518
ablation therapy, 535 preexcitation, retrograde P wave, 517
autosomal dominant inheritance, 535 slow pathway ablation, 519
cryoenergy ablation termination, 12 lead ECG, 518
early para-Hisian activation, 153, 154 VA interval, 519
His bundle appearance after rewarming, 153, 155 signs and symptoms, 535
inadvertent AV block, 153, 154 symptomatic, 153
ventricular preexcitation, 153, 155

Potrebbero piacerti anche